You are on page 1of 283

www.luyenthi24h.

com
www.luyenthi24h.com

TRN ANH TUN


TRNG I HC THNG MI

Cc chuyn

LUYN THI I HC
WWW.VNMATH.COM

H NI - 2011

www.luyenthi24h.com
www.luyenthi24h.com

www.luyenthi24h.com
www.luyenthi24h.com

Mc lc
WWW.VNMATH.COM
I i s - Lng gic - Gii tch
Chng 1

Phng trnh, bt phng trnh, h i s

11

1.1. Phng trnh, bt phng trnh a thc . . . . . . . . . . . . . . . . . . . . . . . . . . . . . . . . . . . . . 11


1.1.1. Phng trnh, bt phng trnh bc hai . . . . . . . . . . . . . . . . . . . . . . . . . . . . . . . . . 11
1.1.2. Phng trnh trnh bc ba . . . . . . . . . . . . . . . . . . . . . . . . . . . . . . . . . . . . . . . . 13
1.1.3. Phng trnh, bt phng trnh bc bn . . . . . . . . . . . . . . . . . . . . . . . . . . . . . . . . 13
1.2. Phng trnh, bt phng trnh cha gi tr tuyt i . . . . . . . . . . . . . . . . . . . . . . . . . . . . . . 14
1.3. Phng trnh, bt phng trnh cha cn . . . . . . . . . . . . . . . . . . . . . . . . . . . . . . . . . . . . 16
Vn 1 : Phng trnh, bt phng trnh c bn . . . . . . . . . . . . . . . . . . . . . . . . . . . . . . . 16
Vn 2 : Phng php t n ph . . . . . . . . . . . . . . . . . . . . . . . . . . . . . . . . . . . . . . 17
Vn 3 : Phng php nhn lin hp . . . . . . . . . . . . . . . . . . . . . . . . . . . . . . . . . . . . . 19
Vn 4 : Phng php nh gi . . . . . . . . . . . . . . . . . . . . . . . . . . . . . . . . . . . . . . . 19
Vn 5 : Phng trnh, bt phng trnh c tham s . . . . . . . . . . . . . . . . . . . . . . . . . . . . . 20
1.4. H phng trnh . . . . . . . . . . . . . . . . . . . . . . . . . . . . . . . . . . . . . . . . . . . . . . . . . 23
1.4.1. Phng php th . . . . . . . . . . . . . . . . . . . . . . . . . . . . . . . . . . . . . . . . . . . . 23
1.4.2. Phng php phn tch thnh nhn t hoc coi mt phng trnh l phng trnh bc hai (ba) theo
mt n

. . . . . . . . . . . . . . . . . . . . . . . . . . . . . . . . . . . . . . . . . . . . . . . . . 24

1.4.3. Phng php t n ph . . . . . . . . . . . . . . . . . . . . . . . . . . . . . . . . . . . . . . . . 24


1.4.4. Phng php hm s . . . . . . . . . . . . . . . . . . . . . . . . . . . . . . . . . . . . . . . . . . 27
1.4.5. Phng php nh gi . . . . . . . . . . . . . . . . . . . . . . . . . . . . . . . . . . . . . . . . . 27
1.5. S nghim ca phng trnh, h phng trnh . . . . . . . . . . . . . . . . . . . . . . . . . . . . . . . . . 28
Vn 1 : Chng minh phng trnh c nghim duy nht . . . . . . . . . . . . . . . . . . . . . . . . . . . 28
Vn 2 : Chng minh phng trnh c ng hai nghim phn bit . . . . . . . . . . . . . . . . . . . . . . 28
Vn 3 : Chng minh phng trnh c ng ba nghim phn bit . . . . . . . . . . . . . . . . . . . . . . 29
1.6. Phng trnh, bt phng trnh, h i s trong cc k thi tuyn sinh H . . . . . . . . . . . . . . . . . . . 29
1.7. Bi tp tng hp . . . . . . . . . . . . . . . . . . . . . . . . . . . . . . . . . . . . . . . . . . . . . . . . . 31
Chng 2

Bt ng thc

37

2.1. Phng php s dng bt ng thc Cauchy . . . . . . . . . . . . . . . . . . . . . . . . . . . . . . . . . . 37


2.1.1. Bt ng thc Cauchy - So snh gia tng v tch . . . . . . . . . . . . . . . . . . . . . . . . . . . 37
2.1.2. Mt s h qu trc tip . . . . . . . . . . . . . . . . . . . . . . . . . . . . . . . . . . . . . . . . . 37
2.1.3. Bi tp ngh . . . . . . . . . . . . . . . . . . . . . . . . . . . . . . . . . . . . . . . . . . . . . 37
2.2. Bt ng thc hnh hc . . . . . . . . . . . . . . . . . . . . . . . . . . . . . . . . . . . . . . . . . . . . . 42
2.3. Phng php s dng iu kin c nghim ca phng trnh hoc h phng trnh . . . . . . . . . . . . . . 44
3

www.luyenthi24h.com
www.luyenthi24h.com
2.4. Bt ng thc trong cc k thi tuyn sinh H . . . . . . . . . . . . . . . . . . . . . . . . . . . . . . . . . . 44
2.5. Bi tp tng hp . . . . . . . . . . . . . . . . . . . . . . . . . . . . . . . . . . . . . . . . . . . . . . . . . 46
Chng 3

Lng gic

51

3.1. Phng trnh c bn . . . . . . . . . . . . . . . . . . . . . . . . . . . . . . . . . . . . . . . . . . . . . . . 51


3.2. Phng trnh dng a sin x + b cos x = c . . . . . . . . . . . . . . . . . . . . . . . . . . . . . . . . . . . . . 52
3.3. Phng php t n ph . . . . . . . . . . . . . . . . . . . . . . . . . . . . . . . . . . . . . . . . . . . . . 53
3.4. a phng trnh v dng tch . . . . . . . . . . . . . . . . . . . . . . . . . . . . . . . . . . . . . . . . . 60
3.5. Phng php nh gi v phng php hm s . . . . . . . . . . . . . . . . . . . . . . . . . . . . . . . . . 62
3.6. Gi tr ln nht v nh nht ca biu thc lng gic . . . . . . . . . . . . . . . . . . . . . . . . . . . . . 63
3.7. Lng gic trong cc k thi tuyn sinh H . . . . . . . . . . . . . . . . . . . . . . . . . . . . . . . . . . . 63
3.8. Bi tp tng hp . . . . . . . . . . . . . . . . . . . . . . . . . . . . . . . . . . . . . . . . . . . . . . . . . 64
Chng 4

T hp

69

4.1. Cc quy tc m. T hp, chnh hp, hon v . . . . . . . . . . . . . . . . . . . . . . . . . . . . . . . . . . 69


4.2. Gii phng trnh, bt phng trnh, h . . . . . . . . . . . . . . . . . . . . . . . . . . . . . . . . . . . . 74
4.3. H s ca xk trong khai trin . . . . . . . . . . . . . . . . . . . . . . . . . . . . . . . . . . . . . . . . . . 76
4.4. H s ca xk trong khai trin nh thc (a + b)n . . . . . . . . . . . . . . . . . . . . . . . . . . . . . . . . . 76
4.5. H s ca xk trong khai trin (a + b)n (c + d)m . . . . . . . . . . . . . . . . . . . . . . . . . . . . . . . . . 77
4.6. H s ca xk trong khai trin (a + b + c)n . . . . . . . . . . . . . . . . . . . . . . . . . . . . . . . . . . . . 77
4.7. Tnh tng cc h s t hp :

n
P

k=0

ak Cnk . . . . . . . . . . . . . . . . . . . . . . . . . . . . . . . . . . . . . . 77

4.8. Phng php c bn vi ak ch l hm s m theo bin k . . . . . . . . . . . . . . . . . . . . . . . . . . . 77


4.9. Phng php o hm vi ak l tch hm s m v a thc theo k . . . . . . . . . . . . . . . . . . . . . . . 78
4.10. Phng php tch phn vi ak l tch hm s m v phn thc theo k . . . . . . . . . . . . . . . . . . . . . 79
4.11. Bi tp tng hp . . . . . . . . . . . . . . . . . . . . . . . . . . . . . . . . . . . . . . . . . . . . . . . . . 80
Chng 5

Hm s

83

5.1. Tnh n iu . . . . . . . . . . . . . . . . . . . . . . . . . . . . . . . . . . . . . . . . . . . . . . . . . . 83
Vn 1 : Xt chiu bin thin ca hm s . . . . . . . . . . . . . . . . . . . . . . . . . . . . . . . . . . . 83
Vn 2 : Tm iu kin tham s hm s n iu trn mt min . . . . . . . . . . . . . . . . . . . . . . 84
Vn 3 : Gi tr ln nht, gi tr nh nht ca hm mt bin s . . . . . . . . . . . . . . . . . . . . . . . 87
Vn 4 : S dng tnh n iu chng minh bt ng thc . . . . . . . . . . . . . . . . . . . . . . . . . . 89
Vn 5 : ng dng s bin thin vo vic gii phng trnh, bt phng trnh, h . . . . . . . . . . . . . 91
Vn 6 : ng dng s bin thin vo bi ton s nghim phng trnh c tham s . . . . . . . . . . . . . 92
5.2. Cc tr ca hm s . . . . . . . . . . . . . . . . . . . . . . . . . . . . . . . . . . . . . . . . . . . . . . . 93
Vn 1 : S dng du hiu 1 v du hiu 2 xc nh cc im cc tr ca hm s . . . . . . . . . . . . 94
Vn 2 : iu kin ca tham s hm s t cc tr (cc i hoc cc tiu) ti x = x0 hoc th hm
s t cc tr ti im (x0 ; y0 ) . . . . . . . . . . . . . . . . . . . . . . . . . . . . . . . . . . . . . 94
Vn 3 : Tm iu kin hm s c cc tr v tha mn mt vi iu kin . . . . . . . . . . . . . . . . . 95
5.3. Tim cn . . . . . . . . . . . . . . . . . . . . . . . . . . . . . . . . . . . . . . . . . . . . . . . . . . . . 100
Vn 1 : Tm tim cn ca th hm s . . . . . . . . . . . . . . . . . . . . . . . . . . . . . . . . . . . 100
Vn 2 : Cc bi ton v tim cn c tham s . . . . . . . . . . . . . . . . . . . . . . . . . . . . . . . . 101
5.4. Tm i xng v trc i xng. im thuc th . . . . . . . . . . . . . . . . . . . . . . . . . . . . . . . 102

www.luyenthi24h.com
www.luyenthi24h.com
Vn 1 : Tm i xng, trc i xng . . . . . . . . . . . . . . . . . . . . . . . . . . . . . . . . . . . . . 102
Vn 2 : Khong cch . . . . . . . . . . . . . . . . . . . . . . . . . . . . . . . . . . . . . . . . . . . . . 102
5.5. Bin lun s nghim ca phng trnh, bt phng trnh bng phng php th . . . . . . . . . . . . . . 103
5.6. Bi ton v s tng giao . . . . . . . . . . . . . . . . . . . . . . . . . . . . . . . . . . . . . . . . . . . . 108
5.7. S tip xc ca hai ng cong v tip tuyn . . . . . . . . . . . . . . . . . . . . . . . . . . . . . . . . . . 109
Vn 1 : Vit phng trnh tip tuyn bit tip im . . . . . . . . . . . . . . . . . . . . . . . . . . . . . 109
Vn 2 : Hai ng cong tip xc . . . . . . . . . . . . . . . . . . . . . . . . . . . . . . . . . . . . . . . 111
Vn 3 : Tip tuyn i qua mt im . . . . . . . . . . . . . . . . . . . . . . . . . . . . . . . . . . . . . 112
Vn 4 : Tip tuyn c h s gc cho trc . . . . . . . . . . . . . . . . . . . . . . . . . . . . . . . . . . 113
5.8. Hm s trong cc k thi tuyn sinh H . . . . . . . . . . . . . . . . . . . . . . . . . . . . . . . . . . . . . 114
5.9. Bi tp tng hp . . . . . . . . . . . . . . . . . . . . . . . . . . . . . . . . . . . . . . . . . . . . . . . . . 121
Chng 6

M v lgart

127

6.1. Hm s m, hm s ly tha . . . . . . . . . . . . . . . . . . . . . . . . . . . . . . . . . . . . . . . . . . 127


6.2. Hm s logarit . . . . . . . . . . . . . . . . . . . . . . . . . . . . . . . . . . . . . . . . . . . . . . . . . . 127
6.3. Phng trnh m v logarit . . . . . . . . . . . . . . . . . . . . . . . . . . . . . . . . . . . . . . . . . . . 129
Vn 1 : Phng trnh c bn . . . . . . . . . . . . . . . . . . . . . . . . . . . . . . . . . . . . . . . . . 129
Vn 2 : Phng php logarit hai v . . . . . . . . . . . . . . . . . . . . . . . . . . . . . . . . . . . . . 130
Vn 3 : Phng php t n ph . . . . . . . . . . . . . . . . . . . . . . . . . . . . . . . . . . . . . . 130
Vn 4 : Phng php phn tch thnh nhn t . . . . . . . . . . . . . . . . . . . . . . . . . . . . . . . 131
Vn 5 : Phng php nh gi . . . . . . . . . . . . . . . . . . . . . . . . . . . . . . . . . . . . . . . 131
6.4. Bt phng trnh m v logarit . . . . . . . . . . . . . . . . . . . . . . . . . . . . . . . . . . . . . . . . . 132
Vn 1 : Bt phng trnh c bn . . . . . . . . . . . . . . . . . . . . . . . . . . . . . . . . . . . . . . . 132
Vn 2 : Phng php t n ph . . . . . . . . . . . . . . . . . . . . . . . . . . . . . . . . . . . . . . 133
Vn 3 : Phng php phn tch thnh nhn t . . . . . . . . . . . . . . . . . . . . . . . . . . . . . . . 134
6.5. H phng trnh . . . . . . . . . . . . . . . . . . . . . . . . . . . . . . . . . . . . . . . . . . . . . . . . . 134
6.6. Phng trnh m v lgarit trong cc k thi tuyn sinh H . . . . . . . . . . . . . . . . . . . . . . . . . . . 135
6.7. Bi tp tng hp . . . . . . . . . . . . . . . . . . . . . . . . . . . . . . . . . . . . . . . . . . . . . . . . . 136
Chng 7

Tch phn

149

7.1. Cc dng ton c bn v nguyn hm . . . . . . . . . . . . . . . . . . . . . . . . . . . . . . . . . . . . . . 149


Vn 1 : Chng minh mt hm s F(x) l mt nguyn hm ca hm s f (x) . . . . . . . . . . . . . . . . 149
Vn 2 : S dng bng nguyn hm c bn . . . . . . . . . . . . . . . . . . . . . . . . . . . . . . . . . . 149
Vn 3 : Tm hng s C . . . . . . . . . . . . . . . . . . . . . . . . . . . . . . . . . . . . . . . . . . . . 150
Vn 4 : Phng php nguyn hm tng phn . . . . . . . . . . . . . . . . . . . . . . . . . . . . . . . . 150
Vn 5 : Phng php i bin s . . . . . . . . . . . . . . . . . . . . . . . . . . . . . . . . . . . . . . 151
7.2. Cc dng ton tch phn . . . . . . . . . . . . . . . . . . . . . . . . . . . . . . . . . . . . . . . . . . . . . 152
Vn 1 : S dng tch phn c bn . . . . . . . . . . . . . . . . . . . . . . . . . . . . . . . . . . . . . . 152
Vn 2 : Tch phn hm cha du tr tuyt i . . . . . . . . . . . . . . . . . . . . . . . . . . . . . . . . 152
Vn 3 : Phng php tch phn tng phn

. . . . . . . . . . . . . . . . . . . . . . . . . . . . . . . . . 153

Vn 4 : Phng php i bin s . . . . . . . . . . . . . . . . . . . . . . . . . . . . . . . . . . . . . . 154


Vn 5 : Tch phn cc hm hu t . . . . . . . . . . . . . . . . . . . . . . . . . . . . . . . . . . . . . . 157
Vn 6 : Tch phn mt s hm c bit . . . . . . . . . . . . . . . . . . . . . . . . . . . . . . . . . . . 159

www.luyenthi24h.com
www.luyenthi24h.com
7.3. ng dng tch phn tnh din tch hnh phng . . . . . . . . . . . . . . . . . . . . . . . . . . . . . . . . 161
7.4. ng dng tch phn tnh th tch vt th trn xoay . . . . . . . . . . . . . . . . . . . . . . . . . . . . . . . 162
7.5. Tch phn trong cc k thi H . . . . . . . . . . . . . . . . . . . . . . . . . . . . . . . . . . . . . . . . . 163
7.6. Bi tp tng hp . . . . . . . . . . . . . . . . . . . . . . . . . . . . . . . . . . . . . . . . . . . . . . . . . 164
Chng 8

S phc

II Hnh hc
Chng 9

167

173
Phng php ta trong trong mt phng

175

9.1. Phng php ta trong mt phng . . . . . . . . . . . . . . . . . . . . . . . . . . . . . . . . . . . . . . 175


9.2. Phng trnh ca ng thng . . . . . . . . . . . . . . . . . . . . . . . . . . . . . . . . . . . . . . . . . 176
9.2.1. Cc bi ton thit lp phng trnh ng thng . . . . . . . . . . . . . . . . . . . . . . . . . . . . 176
9.2.2. Cc bi ton lin quan n vic s dng phng trnh ng thng . . . . . . . . . . . . . . . . . . 176
9.2.3. Bi tp tng hp . . . . . . . . . . . . . . . . . . . . . . . . . . . . . . . . . . . . . . . . . . . . 177
9.3. ng trn . . . . . . . . . . . . . . . . . . . . . . . . . . . . . . . . . . . . . . . . . . . . . . . . . . . 180
9.4. ng elip . . . . . . . . . . . . . . . . . . . . . . . . . . . . . . . . . . . . . . . . . . . . . . . . . . . 183
9.5. ng hypebol . . . . . . . . . . . . . . . . . . . . . . . . . . . . . . . . . . . . . . . . . . . . . . . . . 184
9.6. ng parabol . . . . . . . . . . . . . . . . . . . . . . . . . . . . . . . . . . . . . . . . . . . . . . . . . 186
9.7. Phng php ta trong mt phng qua cc k thi tuyn sinh H . . . . . . . . . . . . . . . . . . . . . . 187
9.8. Bi tp tng hp . . . . . . . . . . . . . . . . . . . . . . . . . . . . . . . . . . . . . . . . . . . . . . . . . 188
Chng 10

M u v hnh hc khng gian. Quan h song song

191

10.1. i cng v ng thng v mt phng . . . . . . . . . . . . . . . . . . . . . . . . . . . . . . . . . . . . 192


Vn 1 : Xc nh giao tuyn ca hai mt phng . . . . . . . . . . . . . . . . . . . . . . . . . . . . . . . 192
Vn 2 : Xc nh giao im ca ng thng a v mt phng (P) . . . . . . . . . . . . . . . . . . . . . 192
Vn 3 : Phng php chng minh ba im thng hng v ba ng thng ng quy . . . . . . . . . . . 193
Vn 4 : Tm thit din ca hnh chp ct bi mt phng . . . . . . . . . . . . . . . . . . . . . . . . . . . 193
10.2. Hai ng thng song song . . . . . . . . . . . . . . . . . . . . . . . . . . . . . . . . . . . . . . . . . . . 195
Vn 1 : Tm giao tuyn ca hai mt phng (dng quan h song song) . . . . . . . . . . . . . . . . . . . 195
Vn 2 : Chng minh hai ng thng song song . . . . . . . . . . . . . . . . . . . . . . . . . . . . . . 196
Vn 3 : Chng minh hai ng thng cho nhau . . . . . . . . . . . . . . . . . . . . . . . . . . . . . . 196
10.3. ng thng v mt phng song song . . . . . . . . . . . . . . . . . . . . . . . . . . . . . . . . . . . . . 197
Vn 1 : Chng minh ng thng song song vi mt phng . . . . . . . . . . . . . . . . . . . . . . . . 197
Vn 2 : Tm giao tuyn ca hai mt phng. Dng thit din song song vi mt ng thng

. . . . . . . 197

Vn 3 : Dng mt mt phng cha mt ng thng v song song vi ng thng khc


Xc nh giao im ca ng thng vi mt phng . . . . . . . . . . . . . . . . . . . . . . . . . . 198
10.4. Hai mt phng song song . . . . . . . . . . . . . . . . . . . . . . . . . . . . . . . . . . . . . . . . . . . . 199
Vn 1 : Chng minh hai mt phng song song . . . . . . . . . . . . . . . . . . . . . . . . . . . . . . . 199
Vn 2 : Tm giao tuyn ca hai mt phng
Thit din ct bi mt mt phng song song vi mt mt phng cho trc . . . . . . . . . . . . . . 199

www.luyenthi24h.com
www.luyenthi24h.com
Chng 11

Vect trong khng gian. Quan h vung gc

201

11.1. Vect trong khng gian. S ng phng ca cc vect . . . . . . . . . . . . . . . . . . . . . . . . . . . . . 202


Vn 1 : Biu th mt vect qua ba vect khng ng phng . . . . . . . . . . . . . . . . . . . . . . . . . 202
Vn 2 : Chng minh cc ng thc vect . . . . . . . . . . . . . . . . . . . . . . . . . . . . . . . . . . 203
Vn 3 : Chng minh cc im thng hng v quan h song song . . . . . . . . . . . . . . . . . . . . . . 203
Vn 4 : Chng minh cc vect ng phng . . . . . . . . . . . . . . . . . . . . . . . . . . . . . . . . . 204
11.2. Hai ng thng vung gc . . . . . . . . . . . . . . . . . . . . . . . . . . . . . . . . . . . . . . . . . . . 205
Vn 1 : Tnh gc gia hai vect . . . . . . . . . . . . . . . . . . . . . . . . . . . . . . . . . . . . . . . 205
Vn 2 : Tnh gc gia hai ng thng a v b . . . . . . . . . . . . . . . . . . . . . . . . . . . . . . . . 206
Vn 3 : Chng minh hai ng thng vung gc . . . . . . . . . . . . . . . . . . . . . . . . . . . . . . 207
11.3. ng thng vung gc vi mt phng . . . . . . . . . . . . . . . . . . . . . . . . . . . . . . . . . . . . . 207
Vn 1 : Chng minh ng thng a vung gc vi mt phng (P) . . . . . . . . . . . . . . . . . . . . . 207
Vn 2 : Chng minh hai ng thng vung gc vi nhau . . . . . . . . . . . . . . . . . . . . . . . . . 208
Vn 3 : Xc nh gc gia ng thng a v mt phng (P) . . . . . . . . . . . . . . . . . . . . . . . . 210
Vn 4 : Dng mt phng qua im M cho trc v vung gc vi mt ng thng d cho trc . . . . . 211
11.4. Hai mt phng vung gc . . . . . . . . . . . . . . . . . . . . . . . . . . . . . . . . . . . . . . . . . . . . 213
Vn 1 : Xc nh gc gia hai mt phng . . . . . . . . . . . . . . . . . . . . . . . . . . . . . . . . . . 213
Vn 2 : Chng minh hai mt phng (P) v (Q) vung gc . . . . . . . . . . . . . . . . . . . . . . . . . 214
Vn 3 : Chng minh ng thng a vung gc vi mt phng (P) . . . . . . . . . . . . . . . . . . . . . 215
Vn 4 : Dng mt phng (Q) cha a v vung gc vi (P) (gi thit a khng vung gc vi (P)) . . . . . 216
11.5. Khong cch . . . . . . . . . . . . . . . . . . . . . . . . . . . . . . . . . . . . . . . . . . . . . . . . . . 217
Vn 1 : Tnh khong cch t im M n ng thng cho trc . . . . . . . . . . . . . . . . . . . . 217
Vn 2 : Dng ng thng i qua mt im A cho trc v vung gc vi mt mt phng (P) cho trc.
Khong cch t im A n mt phng (P) . . . . . . . . . . . . . . . . . . . . . . . . . . . . . . 217
Vn 3 : on vung gc chung v khong cch gia hai ng thng cho nhau . . . . . . . . . . . . . 219
11.6. Khi a din v th tch khi a din . . . . . . . . . . . . . . . . . . . . . . . . . . . . . . . . . . . . . . 222
Vn 1 : Phng php trc tip tm th tch khi chp . . . . . . . . . . . . . . . . . . . . . . . . . . . . 222
Vn 2 : Tnh th tch hnh chp mt cch gin tip . . . . . . . . . . . . . . . . . . . . . . . . . . . . . 227
Vn 3 : Dng cng thc th tch gii mt s bi ton hnh hc . . . . . . . . . . . . . . . . . . . . . 228
11.7. Phn loi mt s hnh khi a din . . . . . . . . . . . . . . . . . . . . . . . . . . . . . . . . . . . . . . . 230
11.7.1. Hnh chp c cnh bn vung gc vi y . . . . . . . . . . . . . . . . . . . . . . . . . . . . . . . 230
11.7.2. Hnh chp u . . . . . . . . . . . . . . . . . . . . . . . . . . . . . . . . . . . . . . . . . . . . . 231
11.7.3. Hnh chp c mt bn vung gc vi y . . . . . . . . . . . . . . . . . . . . . . . . . . . . . . . 232
11.7.4. Hnh chp c hai mt vung gc vi y . . . . . . . . . . . . . . . . . . . . . . . . . . . . . . . . 233
11.7.5. Hnh chp c cc cnh bn bng nhau hoc cc cnh bn cng to vi y nhng gc bng nhau . . 233
11.7.6. Hnh hp - Hnh lng tr . . . . . . . . . . . . . . . . . . . . . . . . . . . . . . . . . . . . . . . . 234
11.8. Bi tp tng hp . . . . . . . . . . . . . . . . . . . . . . . . . . . . . . . . . . . . . . . . . . . . . . . . . 235
Chng 12

Mt cu v khi trn xoay

239

12.1. Mt cu, khi cu . . . . . . . . . . . . . . . . . . . . . . . . . . . . . . . . . . . . . . . . . . . . . . . . 239


12.2. Mt trn xoay. Mt tr, hnh tr v khi tr . . . . . . . . . . . . . . . . . . . . . . . . . . . . . . . . . . 243

www.luyenthi24h.com
www.luyenthi24h.com
Chng 13

Phng php khng gian to trong khng gian

249

13.1. H to trong khng gian . . . . . . . . . . . . . . . . . . . . . . . . . . . . . . . . . . . . . . . . . . . 249


Vn 1 : Tm ta ca mt vect v cc yu t lin quan n vect tha mn mt s iu kin cho trc . 249
Vn 2 : ng dng ca tch v hng v tch c hng . . . . . . . . . . . . . . . . . . . . . . . . . . . 249
Vn 3 : Lp phng trnh ca mt cu . . . . . . . . . . . . . . . . . . . . . . . . . . . . . . . . . . . 252
Vn 4 : Phng php ta gii hnh hc khng gian . . . . . . . . . . . . . . . . . . . . . . . . . . . 253
13.2. Phng trnh mt phng . . . . . . . . . . . . . . . . . . . . . . . . . . . . . . . . . . . . . . . . . . . . . 254
Vn 1 : Vit phng trnh mt phng i qua mt im v c mt vect php tuyn cho trc . . . . . . . 254
Vn 2 : V tr tng i ca hai mt phng . . . . . . . . . . . . . . . . . . . . . . . . . . . . . . . . . 255
Vn 3 : Khong cch t mt im n mt phng . . . . . . . . . . . . . . . . . . . . . . . . . . . . . . 256
Vn 4 : Gc gia hai mt phng . . . . . . . . . . . . . . . . . . . . . . . . . . . . . . . . . . . . . . . 258
Vn 5 : V tr tng i gia mt phng v mt cu . . . . . . . . . . . . . . . . . . . . . . . . . . . . . 258
13.3. Phng trnh ng thng . . . . . . . . . . . . . . . . . . . . . . . . . . . . . . . . . . . . . . . . . . . 260
Vn 1 : Phng trnh tham s v phng trnh chnh tc ca ng thng . . . . . . . . . . . . . . . . . 260
Vn 2 : Tm im trn ng thng tha mn iu kin cho trc . . . . . . . . . . . . . . . . . . . . . 260
Vn 3 : V tr tng i ca hai ng thng v trong khng gian . . . . . . . . . . . . . . . . . . 261
Vn 4 : V tr tng i gia ng thng v mt phng (P) . . . . . . . . . . . . . . . . . . . . . . . 262
Vn 5 : Khong cch t mt im n ng thng . . . . . . . . . . . . . . . . . . . . . . . . . . . . . 263
Vn 6 : V tr tng i gia ng thng v mt cu . . . . . . . . . . . . . . . . . . . . . . . . . . . . 264
Vn 7 : Gc gia hai ng thng ; gc gia ng thng v mt phng . . . . . . . . . . . . . . . . . 266
Vn 8 : Phng trnh ng thng bit ng thng song song, hoc vung gc vi ng thng
hoc mt phng khc, hoc nm trn mt phng khc . . . . . . . . . . . . . . . . . . . . . . . . . 267
Vn 9 : Phng trnh ng thng bit ct . . . . . . . . . . . . . . . . . . . . . . . . . . . . . 268
Vn 10 : Hnh chiu v tnh i xng . . . . . . . . . . . . . . . . . . . . . . . . . . . . . . . . . . . . 270
Vn 11 : Bi ton cc tr . . . . . . . . . . . . . . . . . . . . . . . . . . . . . . . . . . . . . . . . . . . 271
13.4. Hnh hc khng gian trong cc k thi tuyn sinh H . . . . . . . . . . . . . . . . . . . . . . . . . . . . . . 273
13.5. Bi tp tng hp . . . . . . . . . . . . . . . . . . . . . . . . . . . . . . . . . . . . . . . . . . . . . . . . . 278

III Hng dn v p s

287

www.luyenthi24h.com
www.luyenthi24h.com
www.VNMATH.com

www.VNMATH.com

Phn I

i s - Lng gic - Gii tch

www.luyenthi24h.com
www.luyenthi24h.com

WWW.VNMATH.COM

WWW.VNMATH.COM

www.luyenthi24h.com
www.luyenthi24h.com

Chng 1

Phng trnh, bt phng trnh, h i s


1.1 Phng trnh, bt phng trnh a thc
1.1.1 Phng trnh, bt phng trnh bc hai
Bi 1.1 : Gii v bin lun cc phng trnh sau :
1. (m 2)x2 2mx + m + 1 = 0 ;

2.

Bi 1.2 : Cho phng trnh :

a
1
+
= 2.
x1 xa

(m2 4)x2 + 2(m + 2)x + 1 = 0.


1. Tm m phng trnh c hai nghim phn bit.
2. Tm m phng trnh c nghim duy nht.
Bi 1.3 : Gi a, b, c l di ba cnh ca mt tam gic. Chng minh phng trnh sau v nghim :
c2 x2 + (a2 b2 c2 )x + b2 = 0.
Bi 1.4 : Cho phng trnh :
x2 (2m + 3)x + m2 + 2m + 2 = 0.
1. Tm m phng trnh c hai nghim x1 , x2 .
2. Vit phng trnh bc hai c hai nghim

1 1
, .
x1 x2

3. Tm h thc gia x1 , x2 c lp vi tham s m.


4. Tm m phng trnh c hai nghim x1 , x2 tha mn x1 = 2x2 .
Bi 1.5 : Cho phng trnh : x2 cos a.x + sin a 1 = 0.
1. Chng minh rng phng trnh lun c hai nghim x1 , x2 vi mi a.
2. Tm h thc gia x1 , x2 c lp vi a.
3. Tm gi tr ln nht, gi tr nh nht ca E = (x1 + x2 )2 + x21 x22 .
Bi 1.6 : Cho phng trnh :
mx2 2(m 2)x + m 3 = 0.
Tm m phng trnh c :
11

www.VNMATH.com

CHUYN LUYN THI I HC

1. hai nghim tri du ;

2. hai nghim dng phn bit ;

www.luyenthi24h.com
www.luyenthi24h.com
www.VNMATH.com

3. ng mt nghim m.

Bi 1.7 : Gii cc bt phng trnh sau :


1.

x2 4x + 3
<1x;
3 2x

3.

2
1
4
+
;
x + 2 2 x2 + 2x

4. x2 + (x + 1)2

2. (x2 + 3x 2)(x2 5x + 6) 0 ;

15
;
x2 + x + 1

Bi 1.8 : Gii v bin lun cc bt phng trnh sau :


1. x2 mx + m + 3 > 0 ;
Bi 1.9 : Gii h bt phng trnh sau :
Bi 1.10 : Tm m :
1. x2 mx + m + 3 0, x R ;

8
< x2 7x + 6 0

2. (m + 1)x2 2(m 1)x + 3m 3 0 ;

: 2
x 8x + 15 0

2. mx2 + 4x + m > 0, x R ;

3. mx2 mx 5 < 0, x R.

Bi 1.11 : Tm m cc hm s sau xc nh vi mi x R :
1. y =

m(m + 2)x2 + 2mx + 2 ;

2. y =

1
(1 m)x2 2mx + 5 9m

Bi 1.12 : Cho f (x) = (m + 1)x2 2(m 1)x + 3m 3. Tm m bt phng trnh :


1. f (x) < 0 v nghim.

2. f (x) 0 c nghim.

Bi 1.13 : Tm m cc bt phng trnh sau c tp nghim l R :


3x2 mx + 5
1. 1
<6;
2x2 x + 1



x2 + mx + 1


2.
<2;

x2 + 1

Bi 1.14 : Cho bt phng trnh : x2 + 6x + 7 + m 0. Tm m bt phng trnh :


1. v nghim.
2. c ng mt nghim.
3. c min nghim l mt on trn trc s c di bng 1.
Bi 1.15 : Tm m f (x) = mx2 4x + 3m + 1 > 0 vi mi x > 0.
Bi 1.16 : Tm m f (x) = 2x2 + mx + 3 0 vi mi x [1; 1].
Bi 1.17 : Tm m f (x) = x2 2mx m 0 vi mi x > 0.
Bi 1.18 : Tm m f (x) = mx2 2(m + 1)x m + 5 > 0 vi mi x < 1.
Bi 1.19 : Tm m f (x) = 2x2 (3m + 1)x (3m + 9) 0 vi mi x [2; 1].

TRN ANH TUN - 0974 396 391 - (04) 66 515 343

WWW.VNMATH.COM

Trang 12

www.VNMATH.com

CHUYN LUYN THI I HC

www.luyenthi24h.com
www.luyenthi24h.com
www.VNMATH.com

1.1.2 Phng trnh trnh bc ba


Bi 1.20 : Cho phng trnh :
x3 (m2 m + 7)x (3m2 + m 6) = 0.
1. Tm m phng trnh c mt nghim l 1.
2. Vi m > 0 tm c cu trn, hy gii phng trnh .
Bi 1.21 : Gii cc phng trnh sau :
1. x3 6x2 + 11x 6 = 0 ;

3. x3 5x2 + 7x 2 = 0 ;

4. x3 3 3x2 + 7x 3 = 0 ;

2. 2x3 + x + 3 = 0 ;

Bi 1.22 : Tm m cc phng trnh sau c ba nghim phn bit :


1. x3 (2m + 1)x2 + 3(m + 4)x m 12 = 0 ;

2. mx3 2mx2 (2m 1)x + m + 1 = 0 ;

Bi 1.23 : Tm m phng trnh :


mx3 (3m 4)x2 + (3m 7)x m + 3 = 0
c ba nghim dng phn bit.

1.1.3 Phng trnh, bt phng trnh bc bn


Bi 1.24 : Gii cc phng trnh sau :
1. x4 3x2 + 4 = 0 ;

6. 6x4 35x3 + 62x2 35 + 6 = 0 ;

2. (x 1)(x + 5)(x 3)(x + 7) = 297 ;

7. x4 + x3 4x2 + x + 1 = 0 ;

3. (x + 2)(x 3)(x + 1)(x + 6) = 36 ;

8. x4 5x3 + 10x2 10x + 4 = 0 ;

4. x4 + (x 1)4 = 97 ;

9. x4 x2 + 6x 9 = 0 ;
10. 2x4 x3 15x2 x + 3 = 0.

5. (x + 3)4 + (x + 5)4 = 16 ;
Bi 1.25 : Tm cc gi tr ca m sao cho phng trnh

x4 + (1 2m)x2 + m2 1 = 0.
1. V nghim ;

2. C hai nghim phn bit ;

3. C bn nghim phn bit.

Bi 1.26 : Tm cc gi tr ca a sao cho phng trnh


(a 1)x4 ax2 + a2 1 = 0
c ba nghim phn bit.
Bi 1.27 : Cho phng trnh :
(m 1)x4 + 2(m 3)x2 + m + 3 = 0.
Tm m phng trnh trn v nghim.

TRN ANH TUN - 0974 396 391 - (04) 66 515 343

WWW.VNMATH.COM

Trang 13

www.VNMATH.com

CHUYN LUYN THI I HC

www.luyenthi24h.com
www.luyenthi24h.com
www.VNMATH.com

Bi 1.28 : Cho phng trnh :


x4 (2m + 1)x2 + m + 3 = 0.
Tm m phng trnh trn c bn nghim phn bit, trong mt nghim b hn 2 v ba nghim cn li ln hn 1.

Bi 1.29 : Tm h phng trnh sau y c khng t hn hai nghim m khc nhau :


x4 + hx3 + x2 + hx + 1 = 0.
Bi 1.30 : Cho phng trnh :
(x + 1)(x + 2)(x + 3)(x + 4) = m.
Tm m phng trnh c bn nghim phn bit.

1.2 Phng trnh, bt phng trnh cha gi tr tuyt i


1. Phng trnh (bt phng trnh) | f (x)| + g(x) < 0 (hoc = , hoc > , hoc , hoc ) tng ng vi
8
< f (x) 0
:

f (x) + g(x) < 0

hoc

8
< f (x) < 0
:

f (x) + g(x) < 0.

Mt s phng trnh hoc bt phng trnh cha nhiu hn mt du gi tr tuyt i th vic ph du gi tr tuyt i
s phc tp hn nhiu, phi chia thnh nhiu trng hp bng cch lp bng xt du cc biu thc trong du gi tr
tuyt i.
2. Phng trnh (bt phng trnh) | f (x)| < |g(x)| (hoc = , hoc > , hoc , hoc ) phng php n gin l bnh
phng hai v, chuyn v, phn tch thnh nhn t.

3. Mt s phng trnh v bt phng trnh thng dng (gi s a > 0).

|x| = a x = a hoc x = a.
|x| < a a < x < a.
|x| a a x a.
|x| > a x < a hoc x > a.
|x| a x a hoc x a.
Bi 1.31 : Gii phng trnh |x2 8x + 15| = x 3.

Bi 1.32 : Gii cc phng trnh v bt phng trnh sau :


1. |x2 5x + 4| = x2 + 6x + 5;

3. | x2 + x 1| 2x + 5;

2. |x 1| = 2x 1;

4. |x2 x| |x2 1|.

Bi 1.33 : Gii cc phng trnh v bt phng trnh sau :




x2 2


1.
= 2;
x+1



3x + 4

3;
2.
x2



2x 3

1;
3.
x3

4. |2x + 3| = |4 3x|.

Bi 1.34 : Gii cc bt phng trnh sau :

TRN ANH TUN - 0974 396 391 - (04) 66 515 343

WWW.VNMATH.COM

Trang 14

CHUYN LUYN THI I HC

www.VNMATH.com
1. |x2 5x + 4| x2 + 6x + 5;

www.luyenthi24h.com
www.luyenthi24h.com
www.VNMATH.com

2. 4x2 + 4x |2x + 1| 5.

Bi 1.35 : Gii cc bt phng trnh sau :



1. 1

|x| 1
;
1 + |x| 2

2. log5 log

x 4|x|

|x| 7

6. ||x| 1| < 1 x ;

7. |x2 3x 7| + 2x 1 < x2 8x 5 ;

0;

8. x2 |x2 3x 5| 5 < x + 1 ;

3. |x2 2x 8| > 2x ;

9. |x 1| + |x 2| > 3 + x ;
|x2 4x| + 3
0;
x2 + |x 5|

4. |x3 7x 3| < x3 + x2 + 3 ;

10. log3

5. |x3 x2 + 4| + x3 x2 2x 2 0 ;

11. ||3x + 4x 9| 8| 3x 4x 1 ;

Bi 1.36 : Gii cc bt phng trnh sau :


1. |3x + 2| + |2x 3| < 11 ;

3. |x 1| + |2 x| > 3 + x ;

2. |x2 3x 7| + |2x2 x 9| + |3x2 7x 5| < x + 15 ;

4. |x2 3x 17| |x2 5x 7| > 3.

Bi 1.37 : Tm m bt phng trnh : x2 + |x + m| < 2 c t nht mt nghim m.

Bi 1.38 : Gii v bin lun bt phng trnh sau theo tham s p :

2|x p| + 5|x 3p| + 4x + 6p + 12 0.


Bi 1.39 : Gii v bin lun bt phng trnh sau theo tham s p :
|2x + 21p| 2|2x 21p| < x 21p.
Bi 1.40 : Tm tt c cc gi tr thc ca tham s a sao cho bt phng trnh
x2 |x a| |x 1| + 3 0
ng vi mi x R.

Bi 1.41 : Tm tt c cc gi tr ca a sao cho gi tr nh nht ca hm s


y = x2 + 2x 1 + |x a|
ln hn 2.
Bi 1.42 : Tm tt c cc gi tr ca a sao cho gi tr nh nht ca hm s
y = x2 + |x a| + |x 1|
ln hn 2.
Bi 1.43 : Tm tt c cc gi tr ca a sao cho gi tr nh nht ca hm s
y = ax + |x2 4x + 3|
ln hn 1.
Bi 1.44 : Tm tt c cc gi tr ca a sao cho gi tr ln nht ca hm s
y = 4x x2 + |x m|
nh hn 4.

TRN ANH TUN - 0974 396 391 - (04) 66 515 343

WWW.VNMATH.COM

Trang 15

CHUYN LUYN THI I HC

www.VNMATH.com

www.luyenthi24h.com
www.luyenthi24h.com
www.VNMATH.com

1.3 Phng trnh, bt phng trnh cha cn


Vn 1 : Phng trnh, bt phng trnh c bn

Phng php chung l tm cch bnh phng hai v ( gim s cn, hoc mt cn) vi iu kin l hai v ca phng trnh
phi khng m.
1. Phng trnh

2. Phng trnh

f (x) =

g(x)

f (x) = g(x)

3. Bt phng trnh

f (x) >

8
< f (x) 0 (hoc cng c th xt g(x) 0)
:

f (x) = g(x).

8
<g(x) 0
:

f (x) = (g(x))2 .

g(x) (hoc ) tng ng vi

4. Bt phng trnh

f (x) < g(x) (hoc ) tng ng vi

5. Bt phng trnh

f (x) > g(x) (hoc ) tng ng vi


(I)

8
< f (x) 0
:

g(x) < 0

Bi 1.45 : Gii phng trnh

hoc (II)

8
<g(x) 0
:

f (x) > g(x).

8
>
>
f (x) 0
>
<

g(x) 0

>
>
>
: f (x) < (g(x))2 .

8
<g(x) 0
:

f (x) > (g(x))2 .

x2 + 56x + 80 = x + 20.

Bi 1.46 : Gii bt phng trnh x2 2x 15 < x 3.

Bi 1.47 : Gii bt phng trnh x2 1 > x + 2.


Bi 1.48 : Gii cc phng trnh sau :
1.

2.

2x2 + 4x 1 = x + 1;

3.

4x2 + 101x + 64 = 2(x + 10);

4.

x2 + x 6 < x 1;

3.

2x 1 2x 3;

4.

x2 + 2x = 2x2 4x + 3;
(x + 1)(x + 2) = x2 + 3x 4.

Bi 1.49 : Gii cc bt phng trnh:


1.

2.

2x2 1 > 1 x;
x2 5x 14 2x 1.

Bi 1.50 : Tm tp xc nh ca mi hm s sau :

TRN ANH TUN - 0974 396 391 - (04) 66 515 343

WWW.VNMATH.COM

Trang 16

CHUYN LUYN THI I HC

www.VNMATH.com
1. y =



x2 + 3x 4 x + 8;

2. y =

3. y =

x2 + x + 1
;
|2x 1| x 2

4. y =

x2

www.luyenthi24h.com
www.luyenthi24h.com
www.VNMATH.com

1
1
2
;
7x + 5 x + 2x + 5

x2 5x 14 x + 3.

Bi 1.51 : Gii cc phng trnh sau :

1.

5x2 6x 4 = 2(x 1);

2.

4.

5.

x2 + 3x + 12 = x2 + 3x.

Bi 1.52 : Gii cc bt phng trnh sau :

1.

x2 + 6x + 8 2x + 3;

2.

2x 4

x2 x 12 x 1;

x2 4x 12 > 2x + 3;

x+5
6.
< 1.
1x

> 1;

x2 3x 10

3. 6 (x 2)(x 3) x2 34x + 48 ;

Vn 2 : Phng php t n ph

Chng ta thng s dng mt s quy tc t n ph nh sau :


1. Nu phng trnh cha hai loi cn, c th
(a) t u =

ax + b, rt x, th vo phng trnh c phng trnh n u.

(b) Hoc cng c th t u = n u(x), v = m v(x), ly tha rt ra rng buc gia u v v c 1 phng trnh
theo u, v. Kt hp vi phng trnh ban u, ta c h hai n u, v.
2. t u =

u(x), ly tha hai v c phng trnh cha u, x. Kt hp vi phng trnh ban u, ta c h hai n

u, x.Gii phng trnh bc hai (c l bnh phng mt s).


3. t n ph khng hon ton, t u =
4. Nu phng trnh cha

b v

u(x), a v phng trnh bc hai theo u vi x coi nh l tham s.

ab ta thng t u =

b.

5. phng trnh ng cp, chng hn ng cp bc 2 : A.x2 + B.xy + C.y2 = 0. C cch gii nh sau :
(a) Xt y = 0, rt c x;
x
(b) Xt y , 0, chia c hai v cho y2 , t u = , a c v phng trnh bc hai theo u.
y

Bi 1.53 : Gii cc phng trnh sau :

1. 3x2 + 21x + 18 + 2 x2 + 7x + 7 = 2 ;
2. x2 +

x+1 =1;

3. 2(x2 + 2) = 5(x3 + 1) ;

TRN ANH TUN - 0974 396 391 - (04) 66 515 343

4. 2x2 3x + 2 = x 3x 2 ;

5. 6x2 10x + 5 (4x 1) 6x2 6x + 5 = 0 ;

6.

97 x + 4 x = 5 ;

WWW.VNMATH.COM

Trang 17

www.luyenthi24h.com
www.luyenthi24h.com
www.VNMATH.com

CHUYN LUYN THI I HC

www.VNMATH.com

Bi 1.54 : Gii cc phng trnh sau :

3x + 1 = 2 x + 2x + 2 ;

4
2. 2x2 + x + 6 + x2 + x + 2 = x + ;
x

1.

x+3+

1
= 3x + 1 ;
x
4
4

4. 4 x + x + 1 = 2 2x + 1 ;

5. x2 + 4x + 3 + x2 + x = 3x2 + 4x + 1 ;

6. 3 x + 5 x 3 ;
3. x2 + 2x

7.

8.

9.

10.

11.

2x2 + x + 9 + 2x2 x + 1 = x + 4.

x1+
x+

3
x+1 = x 2;

x 16 =

2x3 1 +

x8;

1 x3 = x ;

x2 x + 1 +

x2 + x + 1 = 2 ;

Bi 1.55 : Gii cc phng trnh sau :


1.

1x+

2. 2x +

1 + x + 2 1 x2 = 4 ;

x+1+

x + 2 x2 + x = 1 ;

3. x2 + 2x +

x + 3 + 2x x + 3 = 9 ;

4. 2x2 + x +

x2 + 3 + 2x x2 + 3 = 9 ;

Bi 1.56 : Gii cc phng trnh sau :

x + 2 + x 2x + 1

4.
= x+2;
x + 2x + 1

1. 2x2 + x + 3 = 3x x + 3 ;
2.

x+8=

3.

x2 + x + 2 =

3x2 + 7x + 8
;
4x + 2
3x2 + 3x + 2
;
3x + 1

5. ( x + 3 x + 1)(x2 + x2 + 4x + 3) = 2x.

Bi 1.57 : Gii cc phng trnh sau :


3

1.

x+1+

x+2 =1+

2.

x+1+

x2 =

3.

x+1+

4.

x + 3 + 2x x + 1 = 2x + x2 + 4x + 3 ;

5.

x3 + x2 + 3x + 3 +

x+

x =1+

x2 + 3x + 2 ;

6.

x2 + x ;

x3 + x2 ;

2x =

x2 + 3 +

2x2 + 2x ;

x+3+

4x
=4 x;
x+3

3
7. 4 x + 3 = 1 + 4x + ;
x

8. 2 x + 3 = 9x2 x 4 ;

9. 12 x + 2 x 1 = 3x + 9 ;

Bi 1.58 : Gii cc phng trnh sau :


1.

x+3+

2.

x+

x=3;

x1=

x+1;

3. 2 x2 = (2 x)2 ;

4. 2x + 1 + x x2 + 2 + (x + 1) x2 + 2x + 3 = 0 ;

5. x2 x + (x 5)2 5 x = 11( x + 5 x) ;
r

6.

2x3

=1+

x+1
;
2

Bi 1.59 : Gii cc phng trnh sau :


1.

1x+

x=1;

2. 2 x + 1 2x = 1 ;

TRN ANH TUN - 0974 396 391 - (04) 66 515 343

x+4+ x+ 1x=3;

2+ x

4.
= x+ 1x;
3+ 1x
3.

WWW.VNMATH.COM

Trang 18

CHUYN LUYN THI I HC

www.VNMATH.com

www.luyenthi24h.com
www.luyenthi24h.com
www.VNMATH.com

Vn 3 : Phng php nhn lin hp


Dng 1 : Phng trnh dng

u(x)

v(x) = f (x), trong f (x) v u(x) v(x) c cng nghim x = x0 .

u(x) v(x)

= f (x).
u(x) v(x)
(b) Chuyn v, t (x x0 ) lm nhn t chung.
(a) Phng trnh tr thnh

Dng 2 : Phng trnh dng ( n u1 (x) n v1 (x)) + ( m u2 (x) m v2 (x)) = f (x), trong f (x); u1 (x) v1 (x); u2 (x) v2 (x) c
cng nghim x = x0 ( y f (x) c th ng nht bng 0).

Phng php gii loi ny l chng ta nhn lin hp theo tng cm, t (x x0 ) lm nhn t chung.

Bi 1.60 : Gii cc phng trnh, cc bt phng trnh sau :


1. 3(2 +

x 2) = 2x +

6. x2 + x 1 = (x + 2) x2 2x + 2;

x + 6;

x2
2 > x 4;

1+ 1+x

3. x 2 + 4 x = x2 6x + 11;

4. x 2 + 4 x = 2x2 5x 1;
2.

5.

7.

x + 24 +

12 x = 6;

8. 2 x2 7x + 10 = x + x2 12x + 20;
3
9. 2x2 11x + 21 = 3 4x 4;

1 x 2x + x2
=
;
x
1 + x2

10.

5x 1 +

9 x = 2x2 + 3x 1.

Bi 1.61 : Gii cc phng trnh sau :


1.

x+4

2. x +

2x + 3 = x 1 ;

5. 2 +

1
+
x

6. 1 +

2x =

x+

1
;
x

3. (x 1) x + 1 + 2x + 1 = x + 2 ;

7.

4.

8.

1
1
+ x + 5 = + 2x + 4 ;
2
x
x

x+6 =

2x + 5 +

x+3= x+

x+3;

2x ;

x+2+

x+6 =

2x + 5 +

x+8+

x+4 =

2x + 3 +

3x

2x + 1 ;

Vn 4 : Phng php nh gi

C s ca phng php ny l chng ta s dng bt ng thc hoc phng php hm s nh gi.


Cch 1 : C s nhn dng :
(a) Nu hm s y = f (x) ng bin trn (a; b) v hm s y = g(x) nghch bin trn (a; b) th phng trnh
f (x) = g(x) nu c nghim th nghim l duy nht.
(b) Nu hm s y = f (x) ng bin (hoc nghch bin) trn (a; b) th phng trnh f (x) = c (vi c l hng s)
nu c nghim th nghim l duy nht.

TRN ANH TUN - 0974 396 391 - (04) 66 515 343

WWW.VNMATH.COM

Trang 19

CHUYN LUYN THI I HC

www.VNMATH.com

www.luyenthi24h.com
www.luyenthi24h.com
www.VNMATH.com

Phng php gii l :


(a) Nhn thy x = x0 l mt nghim ca phng trnh cho.
(b) Nu x > x0 , ta suy ra v tri ln hn v phi hoc ngc li.
(c) Nu x < x0 , ta suy ra v tri ln hn v phi hoc ngc li.
(d) Kt lun phng trnh cho c nghim duy nht x = x0 .
Cch 2 : Nu hm s y = f (x) ng bin (hoc nghch bin) trn (a; b) th phng trnh f (u) = f (v) tng ng vi
u = v.
Cch 3 : Nu hm s y = f (x) tha mn f (x) = 0 c nhiu hn 1 nghim th chng ta lp bng bin thin suy ra
phng trnh c ti ta bao nhiu nghim, ri nhm s nghim , dn n l tt c cc nghim ca phng
trnh.
Cch 4 : Nu f (x) c v g(x) c th phng trnh f (x) = g(x) tng ng vi

8
< f (x) = c
:

g(x) = c.

Bi 1.62 : Gii cc phng trnh sau :

x=3;

2. x + 3 + x + x + 8 = 4 ;

3. x2 x + 1 + x2 + 7x + 1 = 4 x ;

1.

x+3+

4.

x+3

+ 2x 1 = 2 ;
1+ 2x

5.

x2 x + 4 +

2x 1 = 5 ;

Vn 5 : Phng trnh, bt phng trnh c tham s


1. S dng phng trnh, bt phng trnh c bn;
2. S dng t n ph, v t iu kin "cht" cho n;
3. S dng iu kin c nghim ca phng trnh bc hai;
4. S dng phng php hm s ch ra iu kin c nghim.

Bi 1.63 : Tm iu kin ca m phng trnh


1. c nghim thc ;

x2 + 2x m = 2x 1 :

2. c ng mt nghim thc ;

Bi 1.64 : Tm iu kin ca m phng trnh x +


Bi 1.65 : Tm iu kin ca m phng trnh

1
x+ +
2

16 x2

TRN ANH TUN - 0974 396 391 - (04) 66 515 343

x+

3. c hai nghim thc phn bit.

1
= m c nghim thc.
4

m
16 x2

4 = 0 c nghim thc.
WWW.VNMATH.COM

Trang 20

www.VNMATH.com

CHUYN LUYN THI I HC


r

www.luyenthi24h.com
www.luyenthi24h.com
www.VNMATH.com

x1
x+2
m
+ 2 = 0 c nghim thc.
Bi 1.66 : Tm iu kin ca m phng trnh
x+2
x1
4

Bi 1.67 : Tm iu kin ca m phng trnh x + 1 m x 1 + 2 x2 1 = 0 c nghim thc.

Bi 1.68 : Tm iu kin ca m phng trnh x2 2x 3 = x + m

TRN ANH TUN - 0974 396 391 - (04) 66 515 343

WWW.VNMATH.COM

Trang 21

CHUYN LUYN THI I HC

www.VNMATH.com
1. c nghim thc ;

www.luyenthi24h.com
www.luyenthi24h.com
www.VNMATH.com

2. c hai nghim thc phn bit.

Bi 1.69 : Bin lun theo m s nghim thc ca phng trnh x + 1 + 1 x = m.

Bi 1.70 : Tm iu kin m phng trnh x + 9 x = x2 + 9x + m c nghim thc.

Bi 1.71 : Tm iu kin m phng trnh x + 4 x 4 + x + x 4 = m c nghim thc.

x+m
Bi 1.72 : Tm iu kin m phng trnh x + 6 x 9 + x 6 x 9 =
c nghim thc.
6

4
Bi 1.73 : Tm m phng trnh x4 + 4x + m + x4 + 4x + m = 6 c nghim thc.

3
Bi 1.74 : Tm iu kin ca m phng trnh 1 x2 + 2 1 x2 = m :
1. c nghim thc duy nht ;

2. c nghim thc.

3x2 1
= 2x 1 + mx lun c nghim thc vi mi gi tr ca m.
Bi 1.75 : Chng t rng phng trnh
2x 1
r
x+1
= m c nghim thc.
Bi 1.76 : Tm m phng trnh (x 3)(x + 1) + 4(x 3)
x3
3
3
Bi 1.77 : Tm m phng trnh 1 x + 1 + x = m c nghim thc.

Bi 1.78 : Bin lun theo m s nghim thc ca phng trnh m x2 + 2 = x + m.

Bi 1.79 : Tm m phng trnh x2 2x 3 = mx + m c nghim thc x , 1.


Bi 1.80 : Tm m phng trnh sau c nghim thc :
x+

x+

Bi 1.81 : Tm m phng trnh

1 x + 2m

x(1 x) 2

x(1 x) = m.

x2 x + 1 = m c nghim thc.

Bi 1.82 : Tm m cc phng trnh sau c nghim thc :


1.

x2 + x + 1

x2 x + 1 = m ;

2.

x2 + 1

x = m.

Bi 1.83 : Tm m phng trnh sau c nghim thc :

x x + x + 12 = m
5x+ 4x .

Bi 1.84 : Tm m phng trnh sau c nghim thc :


m


4
4

x 2 + 2 x2 4 x + 2 = 2 x2 4.

Bi 1.85 : Tm m phng trnh sau c nghim thc :

(4m 3) x + 3 + (3m 4) 1 x + m 1 = 0.
Bi 1.86 : Tm m phng trnh sau c 4 nghim thc phn bit :
m

1 + x2

1 x2 + 2 = 2 1 x4 + 1 + x2 1 x2 .

Bi 1.87 : Tm m phng trnh sau c nghim duy nht :

x2 2x =

TRN ANH TUN - 0974 396 391 - (04) 66 515 343

mx + 1.
WWW.VNMATH.COM

Trang 22

CHUYN LUYN THI I HC

www.VNMATH.com

www.luyenthi24h.com
www.luyenthi24h.com
www.VNMATH.com

Bi 1.88 : Tm m phng trnh sau c nghim :


x+
Bi 1.89 : Cho phng trnh :
x2 + 2x + 4

1 x2 = m.

(3 x)(x + 1) = m 3.

1. Tm m phng trnh c nghim.


2. Tm m phng trnh c ng hai nghim phn bit.
Bi 1.90 : Tm m phng trnh sau c nghim :

Bi 1.91 : Cho phng trnh :

x+1+

3x

(x + 1)(3 x) = m.

|x + 1| + m|x 1| = (m + 1) x2 1.

1. Gii phng trnh khi m = 2 ;


2. Tm m phng trnh trn c nghim.
Bi 1.92 : Tm m cc bt phng trnh sau c nghim :
1.

4x+

x+5 m;

2. mx

x 3 m + 1.

Bi 1.93 : Tm m bt phng trnh


m

c nghim trong on 0; 1 +


3 .

Bi 1.94 : Tm m bt phng trnh

x2 2x + 2 + 1 + x(2 x) 0

(4 + x)(6 x) x2 2x + m nghim ng vi mi x [4; 6].

1.4 H phng trnh


1.4.1 Phng php th
Bi 1.95 : Gii cc h phng trnh sau :

1.

2.

3.

8
< x2 (y + 1)(x + y + 1) = 3x2 4x + 1
:

xy + x + 1 = x2

8
< x3 y = 16
:

3x + y = 8

8
<y(1 + x2 ) = x(1 + y2 )
: 2
x + 3y2 = 1

TRN ANH TUN - 0974 396 391 - (04) 66 515 343

4.

5.

8
>
<

1
1
=y
x
y

>
:2y = x3 + 1

8
3
< x + y =
x+y
:

xy =

x y 12

8
< x + y x y = 2
6.

:
2
2
2
2

x +y +

x y =4

WWW.VNMATH.COM

Trang 23

www.VNMATH.com
7.

8
< x3 8x = y3 + 2y

8.

8
<|x2 2x| + y = 1

9.

10.


8

1
>
=2
< 3x 1 +
x + y
11. 

1
>
: 7y 1
=4 2

: 2
x 3 = 3(y2 + 1)

x+y

12.

: 2
x + |y| = 1

8
2xy
>
< x2 + y2 +
=1

13.

x+y

>
: x + y = x2 y
8

< 7x + y + 2x + y = 5
:

www.luyenthi24h.com
www.luyenthi24h.com
www.VNMATH.com

CHUYN LUYN THI I HC

14.

2x + y + x y = 2

8
< x3 + 3xy2 = 49

: 2
x 8xy + y2 = 8y 17x
8

< y( x + x + 3) = 3
:

x+

>
<

x+

>
:

xy +

y= x+1

1+

1
=
y

y+1+

x
y

1x=1

1.4.2 Phng php phn tch thnh nhn t hoc coi mt phng trnh l phng trnh bc hai (ba)
theo mt n
Bi 1.96 : Gii cc h phng trnh sau :

1.

2.

8
< x3 + 3y2 x = 4
: 3
y + 3x2 y = 4;
8
< x2 + y + 1 = 0
:

3.

8
<3x3 = x2 + 2y2
:

3y3

4.

x + y2 + 1 = 0;

y2

5.

2x2 ;

8
4y
>
< x 3y =

4x
>
:y 3x =
;
y

8
< x3 = 3x + 8y

6.

: 3
y = 3y + 8x;

8
< x3 = 5x + y
: 3
y = 5y + x.

Bi 1.97 : Gii cc h phng trnh sau :

1.

2.

3.

4.

8
< x2 = 3x + 2y

5.

: 2
y = 3y + 2x
8
< x2 2y2 = 2x + y

6.

: 2
y 2x2 = 2y + x
8
< x3 = 2x + y

7.

: 3
y = 2y + x
8
< xy + x + y = x2 2y2

8.

x 2y y x 1 = 2x 2y

8
<y2 = (5x + 4)(4 x)

: 2
y 5x2 4xy + 16x 8y + 16 = 0
8
< x3 + 1 = 2y
: 3
y + 1 = 2x
8

< x + y + x y = 1 +
x2 y2
:

x+

y=1

8
< x2 y + 2x + 3y = 6
:

3xy + x + y = 5

1.4.3 Phng php t n ph


Bi 1.98 : Gii cc h phng trnh sau :

1.

8
< x + xy + y = 11
:

x xy + y = 1;

TRN ANH TUN - 0974 396 391 - (04) 66 515 343

WWW.VNMATH.COM

Trang 24

CHUYN LUYN THI I HC

www.VNMATH.com

2.

8
>
< x2 y + xy2 = 20

3.

4.

5.

6.

1 1 5
>
: + = ;
y

8
< x2 + y2 = 2x2 y2
:

x + y + 1 = 3xy;

7.

8
< x y + xy = 1

8
x
>
<

y 26
=
x
5

9.

>
: x2 y2 = 24;

xy3 + yx3 = 2;

(x y)(1 + xy) = 1 xy;

8
x y
>
< + =2

x + y + x2 y2 = 3xy
10. 1 1
>
: + xy = 1;
x y
11.

8.

x2 y2
>
:x + y +
+
= 4;
y

: 2
x + y2 = 2;

8
< x2 + y2 + x2 y2 = 1 + 2xy

8
x y
>
<x + y + + = 4

8
>
<

8
< x2 + y2 + xy = 3
:

www.luyenthi24h.com
www.luyenthi24h.com
www.VNMATH.com

12.

1 1
>
: + + x + y = 4;

8
< x2 + y2 + xy = 3x2 y2
: 2
x + y2 xy = x2 y2 ;
8
< x + xy + y = 7

: 2
x + xy + y2 = 13;

Bi 1.99 : Gii cc h phng trnh sau :

1.

2.

3.

4.

8
< x + y + xy = 5

5.

: 2
x + y2 + xy = 7
8
x
>
<

y 13
=
x
6

6.

>
:x + y = 5

>
:2x +

7.

3
=7
(x + y)2

1
=3
x+y

9.

8
< x2 3xy + y2 = 1
:

3x2

8
< x2 + xy + y2 = 1
:

8
2
2
>
<4xy + 4(x + y ) +

xy +

3y2

10.

= 13

8
<2x2 4xy + y2 = 1

11.

3x2 + 2xy + 2y2 = 7

x y xy = 3

8
< x2 + 1 + y(x + y) = 4y

8.

: 2
(x + 1)(y + x 2) = y

8
<y2 3xy = 4

12.

: 2
x 4xy + y2 = 1

8
< x2 + y2 = 1
:

x+y+

xy =2

8
< x2 + xy + y2 = 19(x y)2
: 2
x xy + y2 = 7(x y)
8

< x y + y x = 30
:

x x + y y = 35

< x2 + y2 + 2xy = 8 2
:

x+

y=4

Bi 1.100 : Gii cc h phng trnh sau :


8

< x2 + x + y + 1 + x +
y2 + x + y + 1 + y = 18
1.

:
2
2

x +x+y+1x+

2.

3.

4.

5.

8r

x
y
7
>
<
+
=

xy

+1

>
: x xy + y xy = 78;
8

< x2 + y2 + 2xy = 8 2
:

x+

6.

y + x + y + 1 y = 2;

: 2
x + y2 = 128;
8

< x + y = 1
:

|x| + |y| = 1;

TRN ANH TUN - 0974 396 391 - (04) 66 515 343

x+5+

y + 5 = 6;

7.

8
< x + y xy = 7

8.

8
<(x y)(x2 y2 ) = 7

y = 4;

8
< x + y + x y = 4

< x + y = 4

9.

10.

: 2
x + y2 + xy = 133;

(x + y)(x2 + y2 ) = 175;

< x x + y y = 2 xy
:

x+

y = 2;

< x + y + x y = 1 +
x2 y2
:

x+

y = 1;

WWW.VNMATH.COM

Trang 25

11.

12.

13.

14.

15.

8r
x
>
<

8
< 3 + 2x2 y x4 y2 + x4 (2 2x2 ) = y4
17.

:
1 + 1 + (x y)2 = x3 (x3 x + 2y2 );

y 5
=
x 2

>
: x + y = 10

< x y + y x = 30
:

18.

x x + y y = 35;

8



<2(x + y) = 3 3 x2 y + 3 xy2
:
3

x+

8
6x
>
<
>
:

y=6

x+y

x+y

19.

x+y 5
=
6x
2

2+

>
: x xy + y xy = 7;

< x + y = 10
:

x+6+

y + 6 = 14

8
x
+
x2 y2 9x
>
>

=
<

x2 y2

>
x
5 + 3x
>
: =

xy = 9;

8r

x
y
>
<
+
=

www.luyenthi24h.com
www.luyenthi24h.com
www.VNMATH.com

CHUYN LUYN THI I HC

www.VNMATH.com

30 6y

<x + y +
x2 y2 = 12
20.

:
2
2

xy

x y = 12.

8r
20y

>
= x+y+ xy
<
x
21. 16x

>
:


8

5
>
2y = 4
< 3
y + 42x 
16. 

5
>
: 3+
x=2

5y

y + 42x

x+y

xy

Bi 1.101 : Gii cc h phng trnh sau :

1.

2.

3.

8
<2x + x2 y2 = 3
: 2
x + y2 = 1

8
1
>
< x2 + y2 =

2
>
:2x3 + 6y2 x = 1
8
< x3 + 3y2 x = y
: 2
x + 3y2 = 1
8
xy
1 3x
>
=
<

3x
4. 1x+ xy
y
1 2y
>
:
=
1 + xy
2y
5.

7.

8
<(x + y)(1 + xy) = 18xy
: 2
(x + y2 )(1 + x2 y2 ) = 208x2 y2


8
1
>
(x
+
y)
1
+
=4
<

xy
6.
2 + y2
1
x
>
: xy +
+
=4
xy
xy

8.

8
>
<

y(x2

+ 1) =


2x(y2

+ 1)

1
>
:(x2 + y2 ) 1 + 2 2
x y

= 24

8
>
<

1
=4
x
12.
1
>
: x2 + xy + = 3
x
2x + y +



8
1
>
=5
<(x + y) 1 +

1
>
: xy +
=4

xy

13.

xy

1
1
x
>
: 2 +
+ =3
x



8
1
>
=6
<(x + y) 1 +
xy 

9.
1 2
>
:(x2 + y2 ) 1 +
= 18

14.

xy

8


>
1 2
2
2
>
=9
<(x + y ) 1 +
xy 

10.
>
1 3
>
= 27
:(x3 + y3 ) 1 +

15.

xy

8

x y
>
>
+
(x + y) = 15
<
y x
11. x2 y2
>
>
+ 2 (x2 + y2 ) = 85
:
2

8
>
< x2 y + 2y + x = 4xy

16.

xy

8
>
< x2 y + y = 2

1
>
: x2 +
+ x2 y2 = 3;
x2
8
>
< x2 + y2 + x + y = 4xy

1 1
y
x
>
: + + 2 + 2 = 4;
x y x
y
8
<2y(x2 y2 ) = 3x
:

x(x2 + y2 ) = 10y.

Bi 1.102 : Gii cc h phng trnh :

1.

8
< x2 + y2 3x + 4y = 1
:

3x2 2y2 9x 8y = 3

TRN ANH TUN - 0974 396 391 - (04) 66 515 343

WWW.VNMATH.COM

Trang 26

CHUYN LUYN THI I HC

www.VNMATH.com


8
1
9
>
=
<(x + y) 2
xy  2

2.
1
5
>
:(x y) 2 +
=

xy

3.

4.

Bi 1.103 : Cho h phng trnh :

8
< x xy y = 1

www.luyenthi24h.com
www.luyenthi24h.com
www.VNMATH.com

5.

: 2
x y xy2 = 6

8
<y + xy2 = 6x2
:

1 + x2 y2 = 5x2 ;

8
< x(x + 2)(2x + y) = 9

6.

: 2
x + 4x + y = 6

8
<1 + x3 y3 = 19x3
:

y + xy2 = 6x2 .

8
< x + xy + y = a + 1
: 2
x y + xy2 = a.

Tm a h c t nht mt nghim (x; y) tha mn : x > 0 v y > 0.


Bi 1.104 : Cho h phng trnh :
8

< x+1+ y+1 =3

x y + 1 + y x + 1 + y + 1 + x + 1 = m.

1. Gii h phng trnh vi m = 6.


2. Tm m h phng trnh trn c nghim.

1.4.4 Phng php hm s


Bi 1.105 : Gii cc h phng trnh sau :

1.

8
< x3 5x = y3 5y

4.

: 8
x + y4 = 1

< x + x2 2x + 2 = 3y1 + 1
2.

:
x1
2

y+

3.

y 2y + 2 = 3

5.

+1

< x2 = y 1 + 2x 1

: 2
y = x 1 + 2y 1

6.

< x+1+ 7y=4


:

y+1+

7x=4

< x + x + 3 = 3 y
:

y+

y+3=3 x

8
< x3 3x = y3 3y
: 6
x + y6 = 1

7.

8
>
<e x ey = x y

8.

8
<ln(1 + x) ln(1 + y) = x y

x
>
:log2 + log 4y3 = 10
2
2

9.

2x2 5xy + y2 = 0

< x+ 2y= 2
:

y+

2x=

2.

Bi 1.106 : Tm m h phng trnh sau c nghim :


8

< x+1+ 3y=m


:

y+1+

3x=m

Bi 1.107 : Chng minh rng vi mi m > 0, h phng trnh sau c nghim duy nht :
8
<3x2 y 2y2 m = 0
:

3y2 x 2x2 m = 0

1.4.5 Phng php nh gi


Bi 1.108 : Gii cc h phng trnh sau :

TRN ANH TUN - 0974 396 391 - (04) 66 515 343

WWW.VNMATH.COM

Trang 27

1.

<x + x + y + 1 = 1
:

y+

y+

x+1=1

8
2xy
>
>
= x2 + y
3
<x +
2
x 2x + 9
2.
2xy
>
y+
= y2 + x
>
:
3
2

4.

5.

y 2y + 9

3.

8
<y = x3 + 3x + 4
:

www.luyenthi24h.com
www.luyenthi24h.com
www.VNMATH.com

CHUYN LUYN THI I HC

www.VNMATH.com

6.

x = 2y3 6y 2

8
1 1
>
<x + y + + = 4

7.

1
1
>
: x2 + y2 +
+ 2 =4
2
x
y
8
< x2 + 2y2 = 3

8.

: 2 2
x (y + 1) = 4
8
< x3 y3 = 7
:

9.

xy(x y) = 2

3y = 1
< 3x+
:
4

x+

y=1

<x +
2 y2 = 2
:

y+

2 x2 = 2;

< x + 4 32 x y2 = 3
:
4

x+

32 x + 6y = 24.

1.5 S nghim ca phng trnh, h phng trnh


Bi ton : Chng minh rng phng trnh f (x) = 0 c ng k nghim thc phn bit trong min D.1

Vn 1 : Chng minh phng trnh c nghim duy nht


Cch 1 : Lp bng bin thin ca hm s y = f (x) vi x D (tnh y cc gi tr ti u v cui mi tn), t suy ra
c s nghim ca phng trnh.

Cch 2 : Da vo hai nh l :
nh l 1 : Nu hm s y = f (x) lun ng bin hoc nghch bin trn (a; b) th phng trnh f (x) = 0 c ti a
mt nghim trong khong (a; b).
nh l 2 : Nu hm s y = f (x) lin tc trn [a; b] v f (a). f (b) < 0 th phng trnh f (x) = 0 c t nht mt
nghim trong khong (a; b).

Bi 1.109 : Chng minh rng cc phng trnh sau c nghim duy nht :
1. x5 + x4 + 2x3 + 2x2 + x + 1 = 0;

2. ex (x2 + 1) 4 = 0.

Bi 1.110 : Chng minh rng phng trnh : x3 +

x 1 = 0 c nghim duy nht.

Bi 1.111 : Chng minh rng phng trnh xx+1 = (x + 1) x c mt nghim dng duy nht.
Bi 1.112 : Chng minh rng vi mi a > 0, h phng trnh sau c nghim duy nht :
8
<e x ey = ln(1 + x) ln(1 + y)
:

yx=a

Vn 2 : Chng minh phng trnh c ng hai nghim phn bit


1

Nu k = 0 tc l phng trnh v nghim

TRN ANH TUN - 0974 396 391 - (04) 66 515 343

WWW.VNMATH.COM

Trang 28

www.VNMATH.com

CHUYN LUYN THI I HC

www.luyenthi24h.com
www.luyenthi24h.com
www.VNMATH.com

Cch 1 : Lp bng bin thin ca hm s y = f (x) vi x D (tnh y cc gi tr ti u v cui mi tn), t suy ra


c s nghim ca phng trnh.

Cch 2 : Ch lp bng bin thin nhng khng tnh c ht tt c cc u mt (lc ny y=0 c nghim duy nht). T
suy ra c phng trnh c ti a 2 nghim. Kt hp vi nh l 1 ta cng ch ra c phng trnh c t nht 2
nghim.

Bi 1.113 : Chng minh rng cc phng trnh sau c ng hai nghim thc phn bit :
1. x4 x2 2x 1 = 0;

3. 3x4 4x3 6x2 + 12x 20 = 0;

2. x4 3x3 1 = 0;

4. x3 2x

x + 1 = 0.

Vn 3 : Chng minh phng trnh c ng ba nghim phn bit


Cch 1 : Lp bng bin thin ca hm s y = f (x) vi x D (tnh y cc gi tr ti u v cui mi tn), t suy ra
c s nghim ca phng trnh.

Cch 2 : Ch lp bng bin thin nhng khng tnh c ht tt c cc u mt (lc ny y=0 c ng 2 nghim). T
suy ra c phng trnh c ti a 3 nghim. Kt hp vi nh l 1 ta cng ch ra c phng trnh c t nht 3
nghim.

Bi 1.114 : Chng minh rng cc phng trnh sau c ng ba nghim thc phn bit :
1. sin x

x
= 0;
2

2. 4x (4x2 + 1) = 1.

1.6 Phng trnh, bt phng trnh, h i s trong cc k thi tuyn sinh H


Bi 1.115 (C08) : Tm gi tr ca tham s m h phng trnh

8
< x my = 1
:

c nghim (x; y) tha mn xy < 0.

mx + y = 3

x + 1 + 2 x 2 5x + 1.
8

<2 2x + y = 3 2x y
Bi 1.117 (C10) : Gii h phng trnh
(x, y R).
: 2
x 2xy y2 = 2
Bi 1.116 (C09) : Gii bt phng trnh

Bi 1.118 (A03) : Gii h phng trnh :

8
>
<

1
1
=y
x
y

>
:2y = x3 + 1.

Bi 1.119 (A04) : Gii bt phng trnh :

2(x2 16)
7x

+ x3 >
.
x3
x3

TRN ANH TUN - 0974 396 391 - (04) 66 515 343

WWW.VNMATH.COM

Trang 29

CHUYN LUYN THI I HC

www.VNMATH.com

5x 1 x 1 > 2x 4.
8
< x + y xy = 3
(x, y R).
Gii h phng trnh :

:
x+1+ y+1 =4
4

Tm m phng trnh sau c nghim thc : 3 x 1 + m x + 1 = 2 x2 1.


8
5
>
< x2 + y + x3 y + xy2 + xy =
4 (x, y R).
Gii h phng trnh :
5
>
: x4 + y2 + xy(1 + 2x) =
4
Tm cc gi tr ca tham s m phng trnh sau c ng hai nghim thc phn bit :

Bi 1.120 (A05) : Gii bt phng trnh :


Bi 1.121 (A06) :
Bi 1.122 (A07) :
Bi 1.123 (A08) :
Bi 1.124 (A08) :

www.luyenthi24h.com
www.luyenthi24h.com
www.VNMATH.com

2x +

2x + 2 6 x + 2 6 x = m (m R).

Bi 1.125 (A09) : Gii phng trnh 2 3x 2 + 3 6 5x 8 = 0.

x x

1.
Bi 1.126 (A10) : Gii bt phng trnh
1 2(x2 x + 1)
8

<(4x2 + 1)x + (y 3) 5 2y = 0
Bi 1.127 (A10) : Gii h phng trnh

: 2
4x + y2 + 2 3 4x = 7
Bi 1.128 (B02) : Gii h phng trnh :

Bi 1.129 (B03) : Gii h phng trnh :

(x, y R).

3
<
xy= xy
:

x+y =

x + y + 2.

8
y2 + 2
>
<3y =
2

x2 + 2
>
:3x =
.
y2

Bi 1.130 (B04) : Xc nh m phng trnh sau c nghim :


m

1 + x2

1 x2 + 2 = 2 1 x4 + 1 + x2 1 x2 .

Bi 1.131 (B06) : Tm m phng trnh sau c hai nghim thc phn bit :

x2 + mx + 2 = 2x + 1.

Bi 1.132 (B07) : Chng minh rng vi mi gi tr dng ca tham s m, phng trnh sau c hai nghim thc phn bit :
x2 + 2x 8 =

Bi 1.133 (B08) : Gii h phng trnh :

Bi 1.134 (B09) : Gii h phng trnh


Bi 1.135 (B10) : Gii phng trnh

m(x 2).

8
< x4 + 2x3 y + x2 y2 = 2x + 9

: 2
x + 2xy = 6x + 6
8
< xy + x + 1 = 7y

(x, y R).

: 2 2
x y + xy + 1 = 13y2 .

6 x + 3x2 14x 8 = 0 (x R).

Bi 1.136 (D02) : Gii bt phng trnh : (x2 3x) 2x2 3x 2 0.


3x + 1

Bi 1.137 (D02) : Gii h phng trnh :

8
>
<23x = 5y2 4y

4x + 2x+1
>
:
= y.
x
2 +2

Bi 1.138 (D04) : Tm m h phng trnh sau c nghim :

TRN ANH TUN - 0974 396 391 - (04) 66 515 343

< x + y = 1
:

x x + y y = 1 3m.
WWW.VNMATH.COM

Trang 30

CHUYN LUYN THI I HC

www.VNMATH.com

www.luyenthi24h.com
www.luyenthi24h.com
www.VNMATH.com

Bi 1.139 (D04) : Chng minh rng phng trnh sau c ng mt nghim :


x5 x2 2x 1 = 0.

Bi 1.140 (D05) : Gii phng trnh : 2 x + 2 + 2 x + 1 x + 1 = 4.

Bi 1.141 (D06) : Gii phng trnh : 2x 1 + x2 3x + 1 = 0 (x R).

Bi 1.142 (D07) : Tm cc gi tr ca tham s m h phng trnh sau c nghim thc :


8
1
1
>
<x + + y + = 5

1
1
>
: x3 +
+ y3 + 3 = 15m 10.
3
x
y
Bi 1.143 (D08) : Gii h phng trnh :

Bi 1.144 (D09) : Gii h phng trnh

8
< xy + x + y = x2 2y2

(x, y R).

x 2y y x 1 = 2x 2y

8
>
<

x(x + y + 1) 3 = 0
5
>
:(x + y)2
+ 1 = 0.
x2

1.7 Bi tp tng hp

x + 4 + x 4 = 2x 12 + 2 x2 16.

Bi 1.146 : Gii bt phng trnh : x + 12 x 3 + 2x + 1.


Bi 1.145 : Gii phng trnh :

Bi 1.147 : Gii h phng trnh :

Bi 1.148 : Gii h phng trnh :

8
< x2 + y2 + x + y = 4
:

x(x + y + 1) + y(y + 1) = 2.

< 2x + y + 1 x + y = 1
:

3x + 2y = 4.

Bi 1.149 : Gii bt phng trnh : 8x2 6x + 1 4x + 1 0.

Bi 1.150 : Gii bt phung trnh : 2x + 7 5 x 3x 2.


Bi 1.151 : Tm m h phng trnh sau c nghim :

Bi 1.152 : Gii h phng trnh :

Bi 1.153 : Gii h phng trnh :

<72x+ x+1 72+ x+1 + 2005x 2005


: 2
x (m + 2)x + 2m + 3 0.

8
<(x2 + 1) + y(y + x) = 4y
: 2
(x + 1)(y + x 2) = y
8
< x3 8x = y3 + 2y

(x, y R).

(x, y R).

Bi 1.154 : Gii h phng trnh :

: 2
x 3 = 3(y2 + 1)
8
<(x y)(x2 + y2 ) = 13

Bi 1.156 : Gii h phng trnh :

8
< x2 xy + y2 = 3(x y)

(x, y R).
(x + y)(x2 y2 ) = 25

Bi 1.155 : Gii phng trnh : 3x 2 + x 1 = 4x 9 + 2 3x2 5x + 2, x R.


:

: 2
x + xy + y2 = 7(x y)3

TRN ANH TUN - 0974 396 391 - (04) 66 515 343

(x, y R).
WWW.VNMATH.COM

Trang 31

CHUYN LUYN THI I HC

www.VNMATH.com

www.luyenthi24h.com
www.luyenthi24h.com
www.VNMATH.com

Bi 1.157 : Gii phng trnh : x + 2 7 x = 2 x 1 + x2 + 8x 7 + 1, x R.


Bi 1.158 : Tm m phng trnh :

c nghim thuc on 0; 1 +


3 .

Bi 1.159 : Gii h phng trnh :

x2 2x + 2 + 1 + x(2 x) 0

8
< x4 x3 y + x2 y2 = 1
: 3
x y x2 + xy = 1.

Bi 1.160 : Tm m phng trnh :

Bi 1.161 : Tm m phng trnh :

x2 + 1

x = m c nghim.

x4 13x + m + x 1 = 0 c ng mt nghim.

Bi 1.162 : Tm m phng trnh : x 3 2 x 4 + x 6 x 4 + 5 = m c ng hai nghim thc.


Bi 1.163 : Tm m h phng trnh :

8
<2x y m = 0
:

x+

c nghim duy nht.

xy = 1

Bi 1.164 : Vi gi tr no ca a th h c t nht mt nghim tha mn x, y > 0. Vi cc gi tr a tm c hy tm tt c


cc nghim ca h cho :

8
1 1
>
<x + y + + = 4

1
1
>
: x2 + y2 +
+
=
2 a2 +
x2 y2
Bi 1.165 : Gii h phng trnh :

Bi 1.166 : Cho h phng trnh :

1
a2 + 1
+
.
a2
a

8
<y3 + y2 x + 3x 6y = 0
: 2
x + xy = 3.
8
< x2 + y2 = m
:

x + y = 6.

1. Gii h phng trnh vi m = 26 ;

3. Tm m h c nghim duy nht ;

2. Tm m h v nghim ;

4. Tm m h hai nghim phn bit.

Bi 1.167 : Cho h phng trnh :

8
< x + xy + y = m + 2
: 2
x y + xy2 = m + 1.

1. Gii h phng trnh vi m = 3 ;


Bi 1.168 : Cho h phng trnh :

8
<(x 2)2 + y2 = m

Bi 1.169 : Cho h phng trnh :

8
< x = y2 y + m

: 2
x + (y 2)2 = m.

2. Xc nh m h c nghim duy nht.


Tm m h c nghim duy nht.

y = x2 x + m.

1. Gii h phng trnh vi m = 0 ;


2. Tm m h phng trnh c nghim ;
3. Tm m h phng trnh c nghim duy nht.

TRN ANH TUN - 0974 396 391 - (04) 66 515 343

WWW.VNMATH.COM

Trang 32

CHUYN LUYN THI I HC

www.VNMATH.com

Bi 1.170 : Tm a h sau c nghim duy nht :

Bi 1.171 : Tm a h sau c nghim :

www.luyenthi24h.com
www.luyenthi24h.com
www.VNMATH.com

8
<2|x| + |x| = y + x2 + a
: 2
x + y2 = 1.

8
1a
>
< x2 + 2xy 7y2

1+a

>
:3x2 + 10xy 5y2 2.

Bi 1.172 : Tm m phng trnh sau c nghim duy nht :

4x+

x + 5 = m.

Bi 1.173 : Tm m phng trnh sau c nghim duy nht :


4

x+

1x+

x+

1 x = m.

Bi 1.174 : Tm a h sau c nghim :


8
< x2 2xy 3y2 = 8
:

2x2 + 4xy + 5y2 = a4 4a3 + 4a2 12 +

Bi 1.175 : Cho phng trnh x +

105.

17 x2 + x 17 x2 = m.

1. Gii phng trnh khi m = 9;


2. Tm m phng trnh c nghim thc;
3. Tm m phng trnh c nghim thc duy nht.

x2 3
6 0.
x
Bi 1.177 : Chng t rng vi mi s m khng m th phng trnh sau lun c nghim thc
Bi 1.176 : Gii bt phng trnh 2x2 5x 3x

3x2 + (3m2 7) x2 + 4 m3 + 6 = 0.

Bi 1.178 : Gii h phng trnh

Bi 1.179 : Gii h phng trnh

< x2 + 2 +
y2 + 3 + x + y = 5

: 2
x + 2 + 2 + 3 x y = 2.
8
< x2 + y3 = 2y2
:

x + y3 = 2y.

Bi 1.180 : Gii bt phng trnh 2 x 1 x + 2 > x 2.

Bi 1.181 : Gii bt phng trnh 3x + 7 2x + 3 > x + 2.


Bi 1.182 : Gii h phng trnh

Bi 1.183 : Gii h phng trnh

Bi 1.184 : Gii h phng trnh

8
<2x2 + x + y2 = 7
:

xy x + y = 3.

<(x + 3) 2x 1 + (y + 3) 2y 1 = 2 (x + 3)(y + 3)
:

x + y = 2xy.

8
3x y
>
=3
<x + 2
2

x +y

x + 3y
>
:y
= 0.
2
2
x +y

TRN ANH TUN - 0974 396 391 - (04) 66 515 343

WWW.VNMATH.COM

Trang 33

CHUYN LUYN THI I HC

www.VNMATH.com

www.luyenthi24h.com
www.luyenthi24h.com
www.VNMATH.com

(x + 2)(2x 1) 3 x + 6 = 4 (x + 6)(2x 1) + 3 x + 2.
8

< xy x+y =2
Bi 1.186 : Gii h phng trnh

:
x2 + y2 + x2 y2 = 4.
Bi 1.185 : Gii phng trnh

Bi 1.187 : Gii h phng trnh

8
< x2 + xy + y2 = 7(x y)2
: 2
x xy + y2 = 3(x y).

<x + y +
x2 y2 = 12
Bi 1.188 : Gii h phng trnh

:
2
2

x y = 12.

Bi 1.189 : Gii h phng trnh

8
<(2x + 1)2 + y2 + y = 2x + 3
:

xy + x = 1.

Bi 1.190 : Gii phng trnh (x2 + 1)2 = 5 x 2x2 + 4.


Bi 1.191 : Gii h phng trnh

8
< x3 y3 + 2 = 0

: 2
x + y2 + x y = 0.

Bi 1.192 : Gii phng trnh |x +


Bi 1.193 : Gii h phng trnh

1 x2 | =

2(1 2x2 ).

8
< x2 + 6y = y + 3
:

x+y+

x y = 4.

Bi 1.194 : Tm m phng trnh sau c nghim thc

x3 + x2 + x m(x2 + 1)2 = 0.
Bi 1.195 : Gii bt phng trnh
Bi 1.196 : Tm m phng trnh
Bi 1.197 : Gii h phng trnh

Bi 1.198 : Gii h phng trnh

1
2x2

+ 3x 5

2x2

8
2x
>
<
>
:

>

1
.
2x 1

mx + 13 = x 2 c nghim thc.
r

2y
=3
x

x y + xy = 3.

< x+1+ y1=4


:

x+6+

y + 4 = 6.

Bi 1.199 : Tm cc gi tr ca tham s m h phng trnh sau c nghim thc


8
< x2 + y2 + 2(x + y) = 2
:

xy(x + 2)(y + 2) = 2m (2m+1 1).

Bi 1.200 : Chng minh rng vi mi m 2010 h phng trnh sau c khng qu mt nghim thc
8

< x + 27 y + 1 = (m 2010)y + 1
:

Bi 1.201 : Gii phng trnh

y + 27

x + 1 + 1 = 4x2 +

Bi 1.202 : Gii h phng trnh

x + 1 = (m 2010)x + 1.

3x.

8
< x3 y(1 + y) + x2 y2 (2 + y) + xy3 30 = 0
: 2
x y + x(1 + y + y2 ) + y 11 = 0.

TRN ANH TUN - 0974 396 391 - (04) 66 515 343

WWW.VNMATH.COM

Trang 34

CHUYN LUYN THI I HC

www.VNMATH.com
Bi 1.203 : Gii h phng trnh

www.luyenthi24h.com
www.luyenthi24h.com
www.VNMATH.com

8
< x3 + 4y = y3 + 16x
:

1 + y2 = 5(1 + x2 ).

Bi 1.204 : Gii h phng trnh

8
>
<

2 + 6y =

x
x 2y
y

>
: x + x 2y = x + 3y 2.

Bi 1.205 : Gii bt phng trnh x 2 x x2 x 2 2 x.


Bi 1.206 : Gii h phng trnh

8
< x3 + y3 = 1

: 2
x y + 2xy2 + y3 = 2.

Bi 1.207 : Tm cc gi tr ca tham s m bt phng trnh


x(4 x) + m
nghim ng vi mi gi tr ca x [2; 2 +
Bi 1.208 : Gii h phng trnh

Bi 1.209 : Gii h phng trnh

Bi 1.210 : Gii h phng trnh

x2 4x + 5 + 2 0

3].

8
<2x2 y + y3 = 2x4 + x6

(x + 2) y + 1 = (x + 1)2 .

8
< x 2y xy = 0
:

x1+

4y 1 = 2.

< x x 8 y = x + y y
:

x y = 5.

8
< x3 y3 + 3y2 3x 2 = 0
Bi 1.211 : Tm m h phng trnh

: 2
x + 1 x2 3 2y y2 + m = 0

c nghim thc.

x + 3 + 2x x + 1 = 2x + x2 + 4x + 3.

Bi 1.213 : Xc nh cc gi tr ca tham s m phng trnh 2x2 + 2mx + m + 1 = 1 x c ng mt nghim thc

Bi 1.212 : Gii phng trnh

dng.

Bi 1.214 : Gii h phng trnh


Bi 1.215 :
Bi 1.216 :
Bi 1.217 :
Bi 1.218 :

8
< x2 + y2 + x2 y2 = 1 + 2xy
:

x + x2 y + xy = xy2 + y + 1.

Gii phng trnh 2x2 + 3x + 1 2x2 2 = x + 1.


6

Xc nh cc gi tr ca tham s m phng trnh x 1 m x + x3 x2 = 0 c nghim thc.


8
x
y
1
>
+
=
<
4
4
3
+
x
3
+
y
8
Tm m h phng trnh sau c nghim thc
xy
>
:
= m.
4
9 + 3x + 3y4 + x4 y4
1
1
Gii phng trnh +
= 2.
x
2 x2

Bi 1.219 : Gii h phng trnh

Bi 1.220 : Gii h phng trnh

8
< x2 + y2 = 5
:

y 1(x + y 1) = (y 2) x + y.

8
>
< x2 + 1 + y2 + xy = 4y
>
:x + y 2 =

x2

y
.
+1

TRN ANH TUN - 0974 396 391 - (04) 66 515 343

WWW.VNMATH.COM

Trang 35

www.VNMATH.com

CHUYN LUYN THI I HC

Bi 1.221 : Tm m phng trnh


Bi 1.222 : Gii h phng trnh

Bi 1.223 : Gii h phng trnh

x+

www.luyenthi24h.com
www.luyenthi24h.com
www.VNMATH.com

x + 4 m 4 x = 3m c nghim thc.

8
< x3 y = 24

3
: 3
2 x + y = 6 3.
8

< x1+ y1=3

x + y (x 1)(y 1) = 5.


4
4
Bi 1.224 : Tm m phng trnh m
x 2 + 2 x2 4 x + 2 = 2 x2 4 c nghim.
Bi 1.225 : Gii h phng trnh

Bi 1.226 : Gii h phng trnh

Bi 1.227 : Gii h phng trnh

8
< x2 + y2 + x + y = 18
:

x(x + 1)y(y + 1) = 72.

< 7x + y + 2x + y = 5
:

2x + y + 20x + 5y = 38.

8
< xy + x2 = 1 + y
:

xy + y2 = 1 + x.

Bi 1.228 : Tm cc gi tr ca tham s m phng trnh m +

2
x x2 = x + 1 x c nghim.
3

5
1
Bi 1.229 : Gii bt phng trnh 5 x + 2x +
+ 5.
2x
2 x

WWW.VNMATH.COM

TRN ANH TUN - 0974 396 391 - (04) 66 515 343

WWW.VNMATH.COM

Trang 36

www.luyenthi24h.com
www.luyenthi24h.com

Chng 2

Bt ng thc
2.1 Phng php s dng bt ng thc Cauchy
2.1.1 Bt ng thc Cauchy - So snh gia tng v tch
Cho ba s khng m a, b, c, ta c :
a+b
ab, du bng xy ra khi a = b ;
2
a + b + c 3
2.
abc, du bng xy ra khi a = b = c.
3

1.

2.1.2 Mt s h qu trc tip


H qu 1 : So snh gia tng nghch o v tng.
Cho ba s dng a, b, c c :
1.

1 1
4
+
;
a b a+b

2.

1 1 1
9
+ +
.
a b c a+b+c

H qu 2 : So snh gia tng bnh phng v tng.


Cho ba s thc a, b, c c :
1. 2(a2 + b2 ) (a + b)2 ;

2. 3(a2 + b2 + c2 ) (a + b + c).

H qu 3 : So snh gia tng, tng bnh phng v tch.


Cho ba s thc a, b, c c :
1. (a + b + c)2 3(ab + bc + ca) ;

2. a2 + b2 + c2 ab + bc + ca.

2.1.3 Bi tp ngh
Bi 2.1 : Cho a, b, > 0. Chng minh rng :

ab(a + b)
a+b

2
2

(a + b)(a2 + ab + b2 ) a3 + b3 (a2 + b2 )3

.
6
2
(a + b)3

Bi 2.2 : Cho a, b > 0 v a + b 1. Chng minh rng :

37

WWW.VNMATH.COM

CHUYN LUYN THI I HC

www.VNMATH.com
1.

1 1
+ 4;
a b

2.

www.luyenthi24h.com
www.luyenthi24h.com
www.VNMATH.com

1 1
+ + a + b 5.
a b

Bi 2.3 : Cho cc s khng m a, b, c c a + b + c 3. Chng minh rng :


1. a + b + c ab + bc + ca ;

2.

a + b + c ab + bc + ca.

Bi 2.4 : Cho x, y > 0. Chng minh rng : (1 + x)(1 + y) (1 + xy)2 .

1 1

Bi 2.5 : Cho x, y > 0. Chng minh rng : x2 + y2 + + 2( x + y).


x y
1
1
+ .
Bi 2.6 : Cho x, y > 0 v x + y = 1. Tm gi tr nh nht ca P = 2
2
x +y
xy
x
y
z
Bi 2.7 : Cho x, y, z > 0 v x + y + z = 1. Tm gi tr ln nht ca P =
+
+
.
x+1 y+1 z+1
b2
1
a2
+
.
a+1 b+1 3
Bi 2.9 : Cho cc s thc dng a, b, c. Chng minh rng :
Bi 2.8 : Cho a, b > 0 v a + b = 1. Chng minh rng :

1
1
1
1
1
1
+
+

+
+
.
a + 3b b + 3c c + 3a 2a + b + c 2b + c + a 2c + a + b
Bi 2.10 : Chng minh rng vi mi a, b, c > 0 u c :
1.

1
1
1
27
+
+

;
a(b + c) b(c + a) c(a + b) 2(a + b + c)2

2.

1
1
1
27
+
+

.
a(a + b) b(b + c) c(c + a) 2(a + b + c)2

1
.
ab

a+b
ab
Bi 2.12 : Cho a, b > 0. Tm gi tr nh nht ca biu thc S = +
.
ab a + b
3
1 1 1
Bi 2.13 : Cho a, b, c > 0 v a + b + c . Tm gi tr nh nht ca biu thc S = a + b + c + + + .
2
a b c

2
2
2
Bi 2.14 : Chng minh rng vi mi s dng x, y, z u c : x + y + z 2(xy + yz).
Bi 2.11 : Cho a, b > 0 v a + b 1. Tm gi tr nh nht ca S = ab +

Bi 2.15 : Cho a, b, c > 0 v a + b + c = 4. Chng minh rng :

ab
bc
ca
+
+
1.
a + b + 2c b + c + 2a c + a + 2b
Bi 2.16 : Cho a, b, c > 0. Chng minh rng :
ab
bc
ca
a+b+c
+
+

.
a + 3b + 2c b + 3c + 2a c + 3a + 2b
6
Bi 2.17 : Cho a, b, c > 0. Chng minh rng :
1.

a+b b+c c+a


+
+
6;
c
a
b

3.

a2
b2
c2
a+b+c
+
+

;
b+c c+a a+b
2

2.

a
b
c
3
+
+
;
b+c c+a a+b 2

4.

a3
b3
c3
a2 + b2 + c2
+
+

.
b+c c+a a+b
2

Bi 2.18 : Cho a, b, c > 0 v abc = 1. Tm gi tr nh nht ca cc biu thc sau :


a2
b2
c2
1. P =
+
+
;
b+c c+a a+b
a3
b3
c3
+
+
;
b+c c+a a+b
TRN ANH TUN - 0974 396 391 - (04) 66 515 343
2. Q =

a2 a b2 b c2 c
3. R =
+
+
;
b+c
c+a a+b
bc
ca
ab
4. S = 2
+ 2
+ 2
;
2
2
a b + a c b c + b a c a + c2 b
WWW.VNMATH.COM

Trang 38

www.VNMATH.com

CHUYN LUYN THI I HC

www.luyenthi24h.com
www.luyenthi24h.com
www.VNMATH.com

Bi 2.19 : Cho x, y, z, t > 0 v xyzt = 1. Tm gi tr nh nht ca biu thc :


P=

x3 (yz

1
1
1
1
+ 3
+ 3
+ 3
.
+ zt + ty) y (zt + tx + xz) z (tx + xy + yt) t (xy + yz + zx)

Bi 2.20 : Cho a, b, c > 0. Tm gi tr nh nht ca biu thc sau :


1. P =

b
c
a
+
+
.
b + 2c c + 2a a + 2b

2. Q =

a
b
c
+
+
, m N, m > 2.1
b + mc c + ma a + mb

Bi 2.21 : Cho a, b, c > 0. Chng minh rng :


1. (a + b)(b + c)(c + a) 8abc ;

2.

bc ca ba
+
+
a + b + c.
a
b
c

Bi 2.22 : Cho a, b, c l di ba cnh ca mt tam gic. Chng minh rng :


1.

a
b
c
+
+
3;
b+ca c+ab a+bc

Bi 2.23 :

2.

a2
b2
c2
+
+
a + b + c.
b+ca c+ab a+bc

1. Cho a, b, c l di ba cnh ca mt tam gic, p l na chu vi ca tam gic. Chng minh rng :
(p a)(p b)(p c)

abc
.
8

2. Cho tam gic ABC c chu vi bng 3 v di ba cnh ca tam gic l a, b, c. Chng minh rng :
4(a3 + b3 + c3 ) + 15abc 27.
Bi 2.24 : Cho a, b, c, d > 0 v a + b + c + d = 1. Chng minh rng :

1
1
1
1
1
1
1
1 81.
a
b
c
d

Bi 2.25 : Cho a, b 1. Chng minh rng : a b 1 + b a 1 ab.

Bi 2.26 : Cho a, b, c 0 v a + b + c = 1. Chng minh rng : ab + bc + ca + abc

2
1 1
1

+
.
2
a + bc 2 ab ac
3
2
Cho a, b > 0 v a + b = 1. Chng minh rng :
+ 2
16.
ab a + b2
1
1
1
1
Cho a, b, c > 0 v
+
+
2. Chng minh rng : abc .
1+a 1+b 1+c
8
2
2

a +b
Cho a > b > 0 v ab = 1. Chng minh rng :
2 2.
ab



1
1
Tm gi tr nh nht ca A = (1 + x) 1 +
+ (1 + y) 1 +
vi x, y > 0 tha mn x2 + y2 = 1.
y
x

Bi 2.27 : Cho a, b, c > 0. Chng minh rng :


Bi 2.28 :
Bi 2.29 :
Bi 2.30 :
Bi 2.31 :

10
.
27

Bi 2.32 : Cho x, y, z > 1 tha mn x + y + z = xyz. Tm gi tr nh nht ca :


P=

y2 z2 x2
+ 2 + 2 .
x2
y
z

Bi 2.33 : Cho a, b, c > 1. Chng minh rng :


3
alogb c + blogc a + cloga b 3 abc.
1

Mt cch tng qut, tm gi tr nh nht ca R =

a
b
c
+
+
vi a, b, c, x, y l nhng s dng
xb + yc xc + ya xa + yb

TRN ANH TUN - 0974 396 391 - (04) 66 515 343

WWW.VNMATH.COM

Trang 39

CHUYN LUYN THI I HC

www.VNMATH.com

www.luyenthi24h.com
www.luyenthi24h.com
www.VNMATH.com

Bi 2.34 : Cho a, b, c > 0 v a + b + c = 1. Chng minh rng :

1
1+
a

Bi 2.35 : Cho a, b > 0. Chng minh rng : (a +

1
1+
b

b)2

Bi 2.36 : Cho a, b, c > 0. Chng minh rng :

1
1+
64.
c

1 1
+
+
a b

8.

bc
ca
ab
1 1 1 1
+ 2
+ 2

+ +
.
2
2
2
2
a b+a c b c+b a c a+c b 2 a b c
Bi 2.37 : Cho a, b, c > 0. Chng minh rng :
bc
ca
a+b+c
ab
+
+

.
a+b b+c c+a
2
1
Bi 2.38 : Cho a 3. Tm gi tr nh nht ca biu thc S = a + .
a
1
Bi 2.39 : Cho a 2. Tm gi tr nh nht ca biu thc S = a + 2 .
a
Bi 2.40 : Cho a, b, c 0 tha mn a2 + b2 + c2 = 1. Tm gi tr nh nht ca biu thc
S =a+b+c+

1
.
abc

Bi 2.41 : Cho x, y > 0 v x + y = 1. Tm gi tr nh nht ca biu thc S =


Bi 2.42 : Cho a, b, c 0 v a + b + c = 1. Tm gi tr ln nht ca biu thc
S =

a+b+

b+c+

x
y
+
.
1y
1x

c + a.

Bi 2.43 : Cho a, b, c > 0 v a + b + c = 3. Tm gi tr ln nht ca biu thc

S =

a(b + 2c) +

b(c + 2a) +

c(a + 2b).

Bi 2.44 : Cho a 2; b 6; c 12. Tm gi tr ln nht ca biu thc

3
4
bc a 2 + ca b 6 + ab c 12
S =
.
abc

a b c 2 3 a+b b+c c+a


+ +

+
+
Bi 2.45 : Chng minh rng :
vi mi a, b, c > 0.
b c a
2
c
a
b
Bi 2.46 : Cho a, b, c > 0 v a + b + c = 3. Chng minh rng :
a3
b3
c3
3
+
+
.
(a + b)(a + c) (b + c)(b + a) (c + a)(c + b) 4
Bi 2.47 : Cho a, b, c > 0 v a + b + c = 3. Chng minh rng :
a3
b3
c3
+
+
1.
b(2c + a) c(2a + b) c(2b + c)
Bi 2.48 : Cho a, b, c > 0 v a2 + b2 + c2 = 1. Chng minh rng :
a3
b3
c3
1
+
+
.
b + 2c c + 2a a + 2b 3
Bi 2.49 : Cho a, b, c > 0 v a2 + b2 + c2 = 1. Chng minh rng :
a3
b3
c3
1
+
+
.
a+b b+c c+a 2

TRN ANH TUN - 0974 396 391 - (04) 66 515 343

WWW.VNMATH.COM

Trang 40

www.VNMATH.com

CHUYN LUYN THI I HC

www.luyenthi24h.com
www.luyenthi24h.com
www.VNMATH.com

Bi 2.50 : Cho a, b, c > 0 v ab + bc + ca = 1. Chng minh rng :

a
1 + a2

b
1 + b2

3
.
1 + c2 2

Bi 2.51 : Cho a, b, c > 0 v ab + bc + ca = 1. Chng minh rng :


1
1
9
1
+
+
.
a(a + b) b(b + c) c(c + a) 2
Bi 2.52 : Cho a, b, c > 0 v a + b + c = 1. Chng minh rng :
b
c
9
a
+
+
.
2
2
2
(b + c)
(c + a)
(a + b)
4
Bi 2.53 : Cho a, b, c > 0 v a2 + b2 + c2 = 3. Chng minh rng :
Bi 2.54 : Cho a, b, c > 0 v a + b + c = 1. Chng minh rng :

ab bc ca
+
+
3.
c
a
b

ca
ab
1
bc
+
+
.
a + bc
b + ca
c + ab 2

Bi 2.55 : Cho a, b, c > 0 v a + b + c = 2. Chng minh rng :

ca
ab
bc
+
+
1.
2a + bc
2b + ca
2c + ab

Bi 2.56 : Cho a, b, c > 0 v abc = 1. Chng minh rng :


a3
b3
c3
3
+
+
.
(1 + b)(1 + c) (1 + c)(1 + a) (1 + a)(1 + b) 4
Bi 2.57 : Cho a, b, c > 0 v abc = 1. Chng minh rng :
1
1
1
3
+
+
.
a3 (b + c) b3 (c + a) c3 (a + b) 2

1 1 1
1
1
1
Bi 2.58 : Cho a, b, c > 0. Chng minh rng : + + 2
+
+
.
a b c
a+b b+c c+a
1
1
1
Bi 2.59 : Cho a, b, c > 0 v a + b + c 1. Chng minh rng : 2
+
+
9.
a + 2bc b2 + 2ca c2 + 2ab
1
1
Bi 2.60 : Cho a, b > 0 v a + b 1. Chng minh rng : 2
+
6.
a + b2 ab
1
1
Bi 2.61 : Cho a, b > 0 v a + b 1. Chng minh rng : 2
+
+ 4ab 7.
2
a +b
ab
Bi 2.62 : Cho a, b, c > 0 v ab + bc + ca = abc. Chng minh rng :
1
1
1
3
+
+
< .
a + 2b + 3c b + 2c + 3a c + 2a + 3b 16
a
b
c
+
+
vi a, b, c > 0 v a + b + c = 1.
1+ba 1+cb 1+ac
Bi 2.64 : Cho x, y, z > 0 v x2 + y2 + z2 = 1. Tm gi tr nh nht ca biu thc :
Bi 2.63 : Tm gi tr nh nht ca : A =

P=

y2

x
y
z
+ 2
+ 2
.
2
2
+z
z +x
x + y2

Bi 2.65 : Cho x, y l hai s thc thay i. Tm gi tr ln nht v gi tr nh nht ca biu thc :


P=

(x + y)(1 xy)
.
(1 + x2 )2 (1 + y2 )2

TRN ANH TUN - 0974 396 391 - (04) 66 515 343

WWW.VNMATH.COM

Trang 41

www.luyenthi24h.com
www.luyenthi24h.com
www.VNMATH.com

CHUYN LUYN THI I HC

www.VNMATH.com

Bi 2.66 : Cho x, y, z l ba s thc tha mn x + y + z = 0. Tm gi tr nh nht ca


P=

2x + 3 + 2y + 3 + 2z + 3.

Bi 2.67 : Cho cc s thc x, y, z tha mn x + y + z = 6. Chng minh rng : 8x + 8y + 8z 4x+1 + 4y+1 + 4z+1 .

Bi 2.68 : Cho 0 < a b c d e v a + b + c + d + e = 1. Chng minh rng :


a(bc + be + cd + de) + cd(b + e a)

1
.
25

Bi 2.69 : Cho a, b, c l ba s dng tha mn iu kin ab + bc + ca = abc. Chng minh rng :


a2
b2
c2
a+b+c
+
+

.
a + bc b + ca c + ab
4
Bi 2.70 : Cho a, b, c l cc s thc dng, chng minh rng :
b+c

a+

4(b3

c3 )

c+a

b+

4(c3

a3 )

a+b

c+

4(a3 + b3 )

2.

Bi 2.71 : Cho a, b, c l cc s thc dng, chng minh rng :


1
1
1
1
+
+

.
a3 + b3 + abc b3 + c3 + abc c3 + a3 + abc abc
Bi 2.72 : Cho a, b, c l cc s thc dng tha mn abc = 1. Chng minh rng :
a3 + b3
b3 + c3
c3 + a3
+
+
2.
a2 + ab + b2 b2 + bc + c2 c2 + ca + a2
Bi 2.73 : Cho ba s thc dng a, b, c. Chng minh rng :

2 a
2 b
2 c
1
1
1
+ 3
+ 3
2 + 2 + 2.
3
2
2
2
a +b
b +c
c +a
a
b
c
Bi 2.74 : Cho a, b, c > 0. Chng minh rng :
1
1
1
a+b+c
+
+

.
a2 + bc b2 + ca c2 + ab
2abc
Bi 2.75 : Cho a, b, c l ba s dng sao cho ab + bc + ca 1. Chng minh rng :

a3
b3
c3
3
+
+

.
b2 + 1 c2 + 1 a2 + 1
4

2.2 Bt ng thc hnh hc


Bi 2.76 : Cho a, b, c R. Chng minh rng :

a2 + b2 + 4c2 + 4ac +

a2 + b2 + 4c2 4ac 2 a2 + b2 .

Bi 2.77 : Vi mi a, b, c, d R. Chng minh rng :

a2 + b2 + c2 + d2 + 2ac + 2bd

a2 + b2 +

c2 + d2 .

Bi 2.78 : Cho x, y, z > 0. Chng minh rng :

x + 2 y + 3 z 14(x + y + z).

TRN ANH TUN - 0974 396 391 - (04) 66 515 343

WWW.VNMATH.COM

Trang 42

CHUYN LUYN THI I HC

www.VNMATH.com

www.luyenthi24h.com
www.luyenthi24h.com
www.VNMATH.com

Bi 2.79 : Cho bn s a, b, c, d R tha mn a2 + b2 = 1 v c + d = 3. Chng minh rng :

9+6 2
.
ac + bd + cd
4

Bi 2.80 : Vi mi a, b, c R. Chng minh rng :

a2 + ab + b2 +

a2 + ac + c2

Bi 2.81 : Vi mi x, y R. Chng minh rng :

4 cos2 x cos2 y + sin2 (x y) +

Bi 2.82 : Vi mi x, y R. Chng minh rng :

4x2 + y2 + 12x + 9 +

b2 + bc + c2 .

4 sin2 x sin2 y + sin2 (x y) 2.

4x2 + y2 4x 6y + 10 5.

Bi 2.83 : Cho a + b + c = 1, ax + by + cz = 4 vi a, b, c , 0. Chng minh rng :

9a2 + a2 x2 +

9b2 + b2 y2 +

9c2 + c2 z2 5.

Bi 2.84 : Cho a, b, c > 0. Chng minh rng :

a2

ab 2 +

b2

b2

bc 3 +

c2

a2 ac

3 + c2 .

Bi 2.85 : Cho a, b, c > 0 v abc + bc + ca = abc. Chng minh rng :

b2 + 2a2
c2 + 2b2
a2 + 2c2
+
+
3.
ab
bc
ac

Bi 2.86 : Cho x2 + y2 = 1. Chng minh rng : x2 5 + 2xy y2 5 6.

Bi 2.87 : Cho

8
< x2 + xy + y2 = 3

: 2
y + yz + z2 = 16

v x, y, z l cc s thc dng. Chng minh rng : xy + yz + zx 8.

Bi 2.88 : Cho x, y, z l nhng s dng. Chng minh rng :

x2 + xy + y2 +

y2 + yz + z2 +

z2 + zx + x2

3(x + y + z).

Bi 2.89 : Cho a + b + c = 12. Chng minh rng :

3a + 2 a + 1 + 3b + 2 b + 1 + 3c + 2 c + 1 3 17.

Bi 2.90 : Cho cc s dng x, y, z v x + y + z 2. Chng minh rng :

4x2

1
+ 2+
x

4y2

1
+ 2+
y

4z2

1
+ 2
z

145
.
2

Bi 2.91 : Gi s x, y, u, v R tha mn : x2 + y2 = 1; u2 + v2 + 16 = 8u + 4v. Tm gi tr ln nht ca biu thc


P = 8u + 4v 2(ux + vy).

Bi 2.92 : Cho x, y, z l cc s dng tha mn xy + yz + zx = 5. Tm gi tr nh nht ca P = 3x2 + 3y2 + z2 .

TRN ANH TUN - 0974 396 391 - (04) 66 515 343

WWW.VNMATH.COM

Trang 43

www.VNMATH.com

CHUYN LUYN THI I HC

www.luyenthi24h.com
www.luyenthi24h.com
www.VNMATH.com

2.3 Phng php s dng iu kin c nghim ca phng trnh hoc h phng trnh
- phng php min gi tr
2x2 + 7x + 23
.
x2 + 2x + 10
x2 (x 4y)2
Bi 2.94 : Tm gi tr ln nht v gi tr nh nht ca biu thc P =
, vi x2 + y2 > 0.
x2 + 4y2

Bi 2.93 : Tm gi tr ln nht v nh nht ca hm s : f (x) =

Bi 2.95 : Cho x l s dng, y l s thc ty . Tm gi tr ln nht, gi tr nh nht (nu c) ca biu thc :


P=

xy2

(x2 + 3y2 ) x +

x2 + 12y2

.

Bi 2.96 : Tm gi tr nh nht ca biu thc P = x2 + y2 , vi 2x2 + y2 + xy 1.



Bi 2.97 : Cho cc s thc x, y tha mn iu kin : 3 x( 3 x 1) + 3 y( 3 y 1) = 3 xy. Tm gi tr ln nht, nh nht ca

biu thc : P = 3 x + 3 y + 3 xy.


Bi 2.98 : Cho x, y tha mn iu kin : x2 xy + y2 = 3. Tm gi tr ln nht v nh nht ca biu thc : P = x2 + xy 2y2 .

Bi 2.99 : Cho hai s thc x, y tha mn iu kin : x 3 x + 1 = 3 y + 2 y. Tm gi tr ln nht, nh nht ca biu


thc P = x + y.

Bi 2.100 : Cho hai s thc x, y tha mn : x2 + y2 = 2(x + y) + 7. Tm gi tr ln nht, gi tr nh nht ca biu thc

P = 3 x(x 2) + 3 y(y 2).


Bi 2.101 : Cho cc s thc x, y tha mn : 4x2 3xy + 3y2 = 6. Tm gi tr ln nht, gi tr nh nht ca biu thc
P = x2 + xy 2y2 .

Bi 2.102 : Cho cc s thc x, y tha mn :

x + 1 + y + 9.

x+

y = 4. Tm gi tr ln nht, gi tr nh nht ca biu thc P =

Bi 2.103 : Cho cc s thc x, y tha mn : xy + x + y = 3. Tm gi tr ln nht, gi tr nh nht ca biu thc P =


3x
3y
+
x2 y2 .
y+1 x+1
Bi 2.104 : Cho a, b 0 v a2 + b2 + ab = 3. Tm gi tr nh nht v gi tr ln nht ca biu thc
P = a4 + b4 + 2ab a5 b5 .
Bi 2.105 : Cho cc s thc x, y tha mn x + y = 2. Tm gi tr ln nht ca P = (x3 + 2)(y3 + 2).

2.4 Bt ng thc trong cc k thi tuyn sinh H


Bi 2.106 (C08) : Cho hai s thc x, y thay i v tho mn x2 + y2 = 2. Tm gi tr ln nht v gi tr nh nht ca biu
thc: P = 2(x3 + y3 ) 3xy.
Bi 2.107 (C10) : Cho hai s thc dng thay i x, y tha mn iu kin 3x + y 1. Tm gi tr nh nht ca biu thc
1
1
A= + .
x
xy
Bi 2.108 (A03) : Cho x, y, z l ba s dng v x + y + z 1. Chng minh rng :

1
x2 + 2 +
x

1
y2 + 2 +
y

TRN ANH TUN - 0974 396 391 - (04) 66 515 343

z2 +

82.
z2
WWW.VNMATH.COM

Trang 44

www.VNMATH.com

CHUYN LUYN THI I HC

Bi 2.109 (A05) : Cho x, y, z l cc s dng tho mn :

www.luyenthi24h.com
www.luyenthi24h.com
www.VNMATH.com

1 1 1
+ + = 4. Chng minh rng :
x y z

1
1
1
+
+
1.
2x + y + z x + 2y + z x + y + 2z
Bi 2.110 (A06) : Cho hai s thc x , 0, y , 0 thay i v tho mn iu kin : (x + y)xy = x2 + y2 xy. Tim gi tr ln
1
1
nht ca biu thc A = 3 + 3 .
x
y
Bi 2.111 (A07) : Cho x, y, z l cc s thc dng thay i v tho mn iu kin xyz = 1. Tm gi tr nh nht ca biu
thc :

x2 (y + z)
y2 (z + x)
z2 (x + y)
+
P=
+
.
y y + 2z z z z + 2x x x x + 2y y

Bi 2.112 (A09) : Chng minh rng vi mi s thc dng x, y, z tha mn x(x + y + z) = 3yz ta c :
(x + y)3 + (x + z)3 + 3(x + y)(y + z)(z + x) 5(y + z)3 .
Bi 2.113 (B05) : Chng minh rng vi mi x R, ta c :

Khi no ng thc xy ra.

12
5

15
4

20
3

33 + 4x + 5x .

Bi 2.114 (B06) : Cho x, y l cc s thc thay i. Tm gi tr nh nht ca biu thc :

A=

(x 1)2 + y2 +

(x + 1)2 + y2 + |y 2|.

Bi 2.115 (B07) : Cho x, y, z l ba s thc dng thay i. Tm gi tr nh nht ca biu thc :




P=x

x 1
+
2 yz

+y

y
z
1
1
+
+z
+
.
2 xz
2 xy

Bi 2.116 (B08) : Cho hai s thc x, y thay i v tho mn h thc x2 + y2 = 1. Tm gi tr ln nht v gi tr nh nht
2(x2 + 6xy)
.
ca biu thc P =
1 + 2xy + 2y2
Bi 2.117 (B09) : Cho cc s thc x, y thay i tha mn (x + y)3 + 4xy 2. Tm gi tr nh nht ca biu thc :
A = 3(x4 + y4 + x2 + y2 ) 2(x2 + y2 ) + 1.
Bi 2.118 (B10) : Cho cc s thc khng m a, b, c tha mn a + b + c = 1. Tm gi tr nh nht ca biu thc M =

3(a2 b2 + b2 c2 + c2 a2 ) + 3(ab + bc + ca) + 2 a2 + b2 + c2 .


Bi 2.119 (D05) : Cho cc s dng x, y, z tho mn xyz = 1. Chng minh rng :

1 + x3 + y3
1 + y3 + z3
1 + z3 + x3
+
+
3 3.
xy
yz
zx

a
1 b
b+ 1

2
.
2a
2b
Bi 2.121 (D08) : Cho x, y l hai s thc khng m thay i. Tm gi tr ln nht v gi tr nh nht ca biu thc :

Bi 2.120 (D07) : Cho a b > 0. Chng minh rng : 2a +

P=

(x y)(1 xy)
.
(1 + x)2 (1 + y)2

Bi 2.122 (D09) : Cho cc s thc khng m x, y thay i v tha mn x + y = 1. Tm gi tr ln nht v gi tr nh nht


ca biu thc :
S = (4x2 + 3y)(4y2 + 3x) + 25xy.

Bi 2.123 (D10) : Tm gi tr nh nht ca hm s y = x2 + 4x + 21 x2 + 3x + 10.

TRN ANH TUN - 0974 396 391 - (04) 66 515 343

WWW.VNMATH.COM

Trang 45

www.luyenthi24h.com
www.luyenthi24h.com
www.VNMATH.com

CHUYN LUYN THI I HC

www.VNMATH.com

2.5 Bi tp tng hp
5
Bi 2.124 : Gi s x, y l hai s dng thay i tho mn iu kin x + y = . Tm gi tr nh nht ca biu thc :
4
1
4
S = + .
x 4y
a c b2 + b + 50
Bi 2.125 : Gi s a, b, c, d l bn s nguyn thay i tho mn 1 a < b < c < d 50. Chng minh +
b d
50b
a c
v tm gi tr nh nht ca biu thc : S = + .
b d
Bi 2.126 : Cho x, y, z l ba s tho mn x + y + z = 0. Chng minh rng :

3 + 4x + 3 + 4y + 3 + 4z 6.
Bi 2.127 : Chng minh rng vi mi x, y > 0 ta c :

y
(1 + x) 1 +
x

9
1+
y

256.

ng thc xy ra khi no.


3
. Chng minh rng :
4
3
3
3
a + 3b + b + 3c + c + 3a 3.

Bi 2.128 : Cho a, b, c l ba s dng tho mn a + b + c =

Khi no ng thc xy ra?

Bi 2.129 : Chng minh rng 0 y x 1 th x y y x . ng thc xy ra khi no ?


4
Bi 2.130 : Cho x, y, z l ba s dng v xyz = 1. Chng minh rng :
x2
y2
z2
3
+
+
.
1+y 1+z 1+x 2
Bi 2.131 : Cho x, y l cc s thc tho mn iu kin x2 + xy + y2 3. Chng minh rng :

4 3 3 x2 xy 3y2 4 3 3.

Bi 2.132 : Cho cc s thc x, y, z tho mn iu kin 3x + 3y + 3z = 1. Chng minh rng :


9x
9y
9z
3x + 3y + 3z
+
+

.
3x + 3y+z 3y + 3z+x 3z + 3x+y
4
Bi 2.133 : Cho hai s dng x, y thay i v tho mn iu kin x + y 4. Tm gi tr nh nht ca biu thc A =
3x2 + 4 2 + y3
+
.
4x
y2
r

11
7
Bi 2.134 : Tm gi tr nh nht ca hm s : y = x +
+ 4 1 + 2 , x > 0.
2x
x
Bi 2.135 : Cho x, y, z l cc s thc dng. Tm gi tr nh nht ca biu thc :

P=

4(x3

y3 ) +

4(y3

z3 ) +

4(z3

x3 ) +

x
y
z
2 2+ 2+ 2 .
y
z
x

Bi 2.136 : Cho a, b l cc s dng tho mn ab + a + b = 3. Chng minh rng :


3a
3b
ab
3
+
+
a2 + b2 + .
b+1 a+1 a+b
2
Bi 2.137 : Cho x, y > 0 v xy = 100. Hy xc nh gi tr nh nht ca biu thc P =

TRN ANH TUN - 0974 396 391 - (04) 66 515 343

x2 + y2
.
xy

WWW.VNMATH.COM

Trang 46

www.VNMATH.com

CHUYN LUYN THI I HC

www.luyenthi24h.com
www.luyenthi24h.com
www.VNMATH.com

Bi 2.138 : Gi s phng trnh ax2 + bx + c c hai nghim thuc on [0; 1]. Xc nh a, b, c biu thc P c gi tr nh
(a b)(2a c)
nht, gi tr ln nht, trong P =
.
a(a b + c)



1
1
2
2
y + 2 .
Bi 2.139 : Cho x, y > 0 tha mn x + y = 1. Tm gi tr nh nht ca biu thc P = x + 2
y
x
Bi 2.140 : Chng minh cc bt ng thc sau vi a, b, c l cc s nguyn khng m :

1+ a 1+ b 1+ c
+
+
3 + a + b + c.
3
1+ b 1+ c 1+ a
Bi 2.141 (*) : Cho 6 s thc x1 , x2 , . . . , x6 [0; 1]. Chng minh rng :
(x1 x2 )(x2 x3 )(x3 x4 )(x4 x5 )(x5 x6 )(x6 x1 )
Bi 2.142 : Cho x, y, z > 0. Chng minh rng :

Bi 2.143 : Cho x1 , x2 , x3 , x4 > 0 tha mn

4
P
i=1

x6

1
.
16

2y
2z
1
1
1
2x
+ 6
+ 6
4 + 4 + 4.
4
4
4
+y
y +z
z +x
x
y
z
4
P
x4

x1 = 1. Hy tm gi tr nh nht ca T =

i=1
4
P
i=1

x3i

Bi 2.144 : Cho x, y l hai s dng thay i tha mn xy = 1. Tm gi tr ln nht ca biu thc A =

x4

x
y
+ 2
.
2
+y
x + y4

Bi 2.145 : Cho hai s thc x, y tha mn x2 + y2 = x + y. Tnh gi tr ln nht, gi tr nh nht ca biu thc
A = x3 + y3 + x2 y + xy2 .
Bi 2.146 : Cho ba s thc dng x, y, z tha mn iu kin x2 + y2 + z2 3. Tm gi tr nh nht ca biu thc :
P=

1
1
1
+
+
.
xy + z2 yz + x2 zx + y2

Bi 2.147 : Cho ba s dng a, b, c tha mn iu kin ab + bc + ca = 2abc. Chng minh rng


1
1
1
1
+
+
.
2
2
2
a(2a 1)
b(2b 1)
c(2c 1)
2
Bi 2.148 : Cho x, y l cc s thc dng thay i tha mn iu kin xy y 1. Tm gi tr nh nht ca biu thc
x2
y3
P = 2 + 9 3.
y
x
Bi 2.149 : Cho cc s thc khng m x, y, z tha mn x2 + y2 + z2 = 3. Tm gi tr ln nht ca biu thc A = xy + yz +
5
zx +
.
x+y+z
Bi 2.150 : Cho cc s thc dng x, y, z tha mn x + y + z = 1. Tm gi tr nh nht ca biu thc P =
z2

z3
.
+ xy

Bi 2.151 : Cho x, y, z > 0 tha mn 13x + 5y + 12z = 9. Tm gi tr ln nht ca biu thc A =

x3
y3
+
+
x2 + yz y2 + zx

xy
3yz
6xz
+
+
.
2x + y 2y + z 2z + x

Bi 2.152 : Cho cc s thc x, y, z tha mn iu kin x2 + y2 + z2 = 3. Tm gi tr ln nht ca A = x3 (y + z) + y3 (z + x) +


z3 (x + y).
Bi 2.153 : Gi s x, y, u, v R tha mn iu kin x2 + y2 = 1, u2 + v2 + 16 = 8u + 4v. Tm gi tr ln nht ca biu thc
M = 8u + 4v 2(ux + vy).

TRN ANH TUN - 0974 396 391 - (04) 66 515 343

WWW.VNMATH.COM

Trang 47

www.luyenthi24h.com
www.luyenthi24h.com
www.VNMATH.com

CHUYN LUYN THI I HC

www.VNMATH.com

Bi 2.154 : Cho a, b, c l cc s thc dng thay i v tha mn iu kin a + b + c =


P=

a2 + ab + b2 +

b2 + bc + c2 +

3. Tnh gi tr nh nht ca

c2 + ca + a2 .

Bi 2.155 : Cho x, y R tha mn x2 + y2 2x 4y + 4 = 0. Chng minh rng

x2 y2 + 2 3xy 2(1 + 2 3)x + (4 2 3)y 5 4 3.

Bi 2.156 : Gi s x, y, z l cc s thc tha mn x + y + z = 6. Chng minh rng


8x + 8y + 8z 4x+1 + 4y+1 + 4z+1 .
Du ng thc xy ra khi no ?
Bi 2.157 : Cho cc s thc dng x, y, z tha mn x + y + z = 1. Tm gi tr nh nht ca biu thc :
P=

x2 (y + z) y2 (z + x) z2 (x + y)
+
+
.
yz
zx
xy

Bi 2.158 : Cho a, b, c l cc s dng tha mn abc = 8. Hy tm gi tr ln nht ca biu thc


P=

1
1
1
+
+
.
2a + b + 6 2b + c + 6 2c + a + 6

Bi 2.159 : Cho x, y > 0 v tha mn x + y = 1. Chng minh rng

x
1 x2

2
.
3
1 y2

Bi 2.160 : Cho hai s thc khng m x, y tha mn x2 + y2 + xy = 3. Tm gi tr ln nht, gi tr nh nht ca biu thc
P = x3 + y3 (x2 + y2 ).
Bi 2.161 : Cho a, b, c l cc s thc khng m, khc nhau tng i mt v tha mn iu kin ab + bc + ca = 4. Chng
minh rng
1
1
1
+
+
1.
2
2
(a b)
(b c)
(c a)2

Bi 2.162 : Cho x, y, z l ba s thc thuc (0; 1]. Chng minh rng

1
1
1
5
+
+

.
xy + 1 yz + 1 zx + 1 x + y + z
Bi 2.163 : Cho a, b, c l di ba cnh ca mt tam gic. Chng minh rng

1
1
2
b
c
a
+
+
+
+
< 2.
3a + b 3a + c 2a + b + c
3a + c 3a + b
Bi 2.164 : Cho a, b, c l nhng s dng tha mn a2 + b2 + c2 = 3. Chng minh rng
1
1
1
4
4
4
+
+

+
+
.
a + b b + c c + a a2 + 7 b2 + 7 c2 + 7
Bi 2.165 : Cho x, y R, chng minh rng

|x y|
|x|
|y|

+
.
1 + |x y| 1 + |x| 1 + |y|

4x + y 2x y
+
.
xy
4
(1 + a)(1 + b)(1 + c)
Bi 2.167 : Cho a, b, c l cc s thc dng tha mn a + b + c = 1. Tm gi tr nh nht ca P =
.
(1 a)(1 b)(1 c)
Bi 2.166 : Cho hai s dng x, y tha mn x + y = 5. Tm gi tr nh nht ca biu thc P =

Bi 2.168 : Cho a, b, c l cc s thc dng tha mn a + b + c = abc. Chng minh rng :

1. c ab 1 + 1 + c2 ;

TRN ANH TUN - 0974 396 391 - (04) 66 515 343

WWW.VNMATH.COM

Trang 48

www.VNMATH.com
2. ab + bc + ca 3 +

CHUYN LUYN THI I HC

a2 + 1 +

b2 + 1 +

www.luyenthi24h.com
www.luyenthi24h.com
www.VNMATH.com

c2 + 1.

Bi 2.169 : Cho a, b, c l cc s thc dng tha mn ab + bc + ca = 3. Chng minh rng :


1
1
1
1
+
+

.
1 + a2 (b + c) 1 + b2 (c + a) 1 + c2 (a + b) abc
Bi 2.170 : Cho x, y, z l cc s thc dng tha mn xyz = 1. Chng minh rng :
1
1
1
+
+
1.
x+y+1 y+z+1 z+x+1

2 y
2 x
2 z
1
1
1
Bi 2.171 : Cho x, y, z > 0. Chng minh rng 3
+ 3
+ 3
2 + 2 + 2.
2
2
2
x +y
y +z
z +x
x
y
z
Bi 2.172 : Cho a, b, c l di ba cnh ca mt tam gic. Chng minh rng :
4a
9b
16c
+
+
26.
b+ca c+ab a+bc

Bi 2.173 : Cho cc s thc khng m a, b. Chng minh rng :

3
a +b+
4
2

3
1
b +a+
2a +
4
2

1
2b +
.
2

Bi 2.174 : Cho cc s thc dng a, b, c thay i tha mn a + b + c = a. Chng minh rng :


a2 + b b2 + c c2 + a
+
+
2.
b+c
c+a
a+b
Bi 2.175 : Cho cc s thc dng x, y, z. Chng minh rng :
x2 xy y2 yz z2 zx
+
+
0.
x+y
y+z
z+x
Bi 2.176 : Cho cc s dng a, b, c tha mn abc = 1. Chng minh rng
1
1
1
+ 2 + 2 + 3 2(a + b + c).
2
a
b
c

yz
2 33
Bi 2.177 : Cho ba s dng x, y, z tha mn x + y + z = . Chng minh rng x
(y + z).
3x
6

Bi 2.178 : Cho cc s thc x, y tha mn 0 x v 0 y . Chng minh rng cos x + cos y 1 + cos(xy).
3
3

Bi 2.179 : Cho s nguyn n (n > 2) v hai s thc khng m x, y. Chng minh rng n xn + yn n+1 xn+1 + yn+1 . ng
thc xy ra khi no?

WWW.VNMATH.COM

TRN ANH TUN - 0974 396 391 - (04) 66 515 343

WWW.VNMATH.COM

Trang 49

www.luyenthi24h.com
www.luyenthi24h.com

Chng 3

Lng gic
3.1 Phng trnh c bn
Bi 3.1 : Gii cc phng trnh sau :

b) 3 cos 2x +

3
a) sin x =
;
2

=1;
6

c) sin 3x = cos 2x ;

Bi 3.2 : Tm tt c cc nghim thuc ; ca phng trnh :


2

1
= .
3
3

tan 3x +
Bi 3.3 : Gii phng trnh : cos (. sin x) = cos

. sin x .
2

sin 2x
= 0.
1 + sin x
cos 2x cos x

Bi 3.5 : Gii phng trnh :


= 0.
cos x
Bi 3.6 : Gii phng trnh : cos x cot 2x = sin x.
Bi 3.4 : Gii phng trnh :

Bi 3.7 : Gii phng trnh :

=0;
4

Bi 3.8 : Gii phng trnh : 32 x2 . cos 2x = 0.


cos 8x
Bi 3.9 : Gii phng trnh :
= 0.
sin 4x
sin 3x
Bi 3.10 : Gii phng trnh :
= 1.
sin 2x
a) cos2 2x + sin2 x +

b) cos 2x. sin x +

=0;
4

3
Bi 3.11 : Gii phng trnh : cos3 x sin 3x + sin3 x cos 3x = .
8
3
3
Bi 3.12 : Gii phng trnh : sin x cos 3x + cos x sin 3x = sin3 4x.
Bi 3.13 : Gii phng trnh : cos 10x + 2 cos2 4x + 6 cos 3x cos x = cos x + 8 cos x cos3 3x.
1
Bi 3.14 : Gii phng trnh : cos x cos 2x cos 4x cos 8x = .
16
2
Bi 3.15 : Gii phng trnh : tan x tan x tan 3x = 2.
11
Bi 3.16 : Gii phng trnh : tan2 x + cot2 x + cot2 2x = .
3
51

WWW.VNMATH.COM

www.VNMATH.comCHUYN LUYN THI I HC

www.luyenthi24h.com
www.luyenthi24h.com
www.VNMATH.com

cot2 x tan2 x
= 16(1 + cos 4x).
cos 2x

7
cot
x .
Bi 3.18 : Gii phng trnh : sin4 x + cos4 x = cot x +
8
3
6
3(sin x + tan x)
Bi 3.19 : Gii phng trnh :
2(1 + cos x) = 0.
tan x sin x
Bi 3.20 : Gii phng trnh : cos 3x tan 5x = sin 7x.
Bi 3.17 : Gii phng trnh :

Bi 3.21 : Gii phng trnh :

sin4 x + cos4 x 1
= (tan x + cot 2x).
sin 2x
2

3.2 Phng trnh dng a sin x + b cos x = c


Bi 3.22 : Tm nghim ca phng trnh :
cos 7x

3 sin 7x = 2

6
2
<x<
5
7

1
Bi 3.23 : Gii phng trnh : 3 sin x + cos x =
.
cos x

Bi 3.24 (C08) : Gii phng trnh : sin 3x 3 cos 3x = 2 sin 2x.

Bi 3.25 : Gii phng trnh : cos x + 3 sin x = 2 cos 2x.

Bi 3.26 : Gii phng trnh : sin 8x cos 6x = 3(sin 6x + cos 8x).

Bi 3.27 : Gii phng trnh : sin x sin 4x = 2 cos


x 3 cos x sin 4x.
6

Bi 3.28 : Gii phng trnh : cos 7x cos 5x 3 sin 2x = 1 sin 7x sin 5x.

x 2
3x
Bi 3.29 : Gii phng trnh : 2 cos

6 sin

= 2 sin
+
2 sin
+
.
5 12
5 12
5
3
5
6
Bi 3.30 : Gii phng trnh : 3 cos2 x = sin2 x + sin 2x.

Bi 3.31 : Gii phng trnh : 4 sin3 x 1 = 3 sin x 3 cos 3x.

Bi 3.32 : Gii phng trnh : 4(sin4 x + cos4 x) + 3 sin 4x = 2.

Bi 3.33 : Gii phng trnh : 2 + cos 2x + 3 sin 2x = sin x 3 cos x.

Bi 3.34 : Gii phng trnh : 3 sin 2x 2 cos2 x = 2 2 + 2 cos 2x.

Bi 3.35 : Gii phng trnh : sin x + 3 cos x + sin x + 3 cos x = 2.

Bi 3.36 : Gii phng trnh : cos 2x 3 sin 2x 3 sin x cos x + 4 = 0.

Bi 3.37 : Gii phng trnh : 3 sin 3x 3 cos 9x = 1 + 4 sin3 3x.

1
Bi 3.38 : Gii phng trnh : tan x sin 2x cos 2x + 2 2 cos x
= 0.
cos x

3
1
Bi 3.39 : Gii phng trnh : 8 sin x =
+
.
cos x sin x
Bi 3.40 : Gii phng trnh : 9 sin x + 6 cos x 3 sin 2x + cos 2x = 8.
tha mn iu kin

Bi 3.41 : Gii phng trnh : sin 2x + 2 cos 2x = 1 + sin x 4 cos x.

Bi 3.42 : Gii phng trnh : 2 sin 2x cos 2x = 7 sin x + 2 cos x 4.

Bi 3.43 : Gii phng trnh : sin 2x cos 2x = 3 sin x + cos x 2.

Bi 3.44 : Gii phng trnh : sin 2x + 3 cos 2x 5 = cos 2x


.
6

TRN ANH TUN - 0974 396 391 - (04) 66 515 343

WWW.VNMATH.COM

Trang 52

www.VNMATH.com

www.luyenthi24h.com
www.luyenthi24h.com
CHUYN LUYN THI I HCwww.VNMATH.com

Bi 3.45 : Gii phng trnh : 2 cos3 x + cos 2x + sin x = 0.


1 cos 2x
Bi 3.46 : Gii phng trnh : 1 + cot 2x =
.
sin2 2x

Bi 3.47 : Gii phng trnh : 4(sin4 x + cos4 x) + 3 sin 4x = 2.


1
Bi 3.48 : Gii phng trnh : 1 + sin3 2x + cos3 2x = sin 4x.
2

Bi 3.49 : Gii phng trnh : tan x 3 cot x = 4 sin x + 3 cos x .

Bi 3.50 : Gii phng trnh : sin3 x + cos3 x = sin x cos x.

1
= .
Bi 3.51 : Gii phng trnh : cos4 x + sin4 x +
4
4

3
3
Bi 3.52 : Gii phng trnh : 4 sin x cos 3x + 4 cos x sin 3x + 3 3 cos 4x = 3.

5
4 sin
+ x sin
+x
6
6
Bi 3.53 : Gii phng trnh :
+ 2 tan x = 0.
cos2 x
Bi 3.54 : Gii phng trnh : 1 + 2(cos 2x tan x sin 2x) cos2 x = cos 2x.

Bi 3.55 : Gii phng trnh : sin x(1 sin x) = cos x(cos x 1).

Bi 3.56 : Gii phng trnh : cos x + sin 2x +


sin 2x
+ 1 = 3(1 + 2 cos x).
6
6

2x
2x
3x
x 2
Bi 3.57 : Gii phng trnh : 2 sin

6 sin
+
= 2 sin

2 cos
+
.
3
3
3
6
2
6
6
3
Bi 3.58 : Gii cc phng trnh sau :
d) 8 sin2 2x cos 2x =

sin 2x
a) 2 cos2 x + = 1 ;
3

b) 4 cos2 x +

3 sin 2x + cos 2x ;

cos x 2 sin x cos x


= 3;
2 cos2 x + sin x 1

f) cos 7x cos 5x 3 sin 2x = 1 sin 7x sin 5x ;

e)

+ sin 2x = 1 ;
3

c) 2 2(sin x + cos x) cos x = 3 + cos 2x ;

g) 4(sin4 x + cos4 x) +

3 sin 4x = 2 ;

Bi 3.59 : Cho phng trnh : 2 sin2 x sin x cos x cos2 x = m.


a) Tm m phng trnh c nghim ;
b) Gii phng trnh khi m = 1.

1
3 sin2 x + sin 2x = m.
2

a) Gii phng trnh khi m = 3 ;

Bi 3.60 : Cho phng trnh :

b) Xc nh m phng trnh c nghim ;


Bi 3.61 : Tm m phng trnh sau y c nghim :
2 sin2 x sin x cos x cos2 x = m.

3.3 Phng php t n ph

+ cos 2x
+ 4 sin x = 2 + 2(1 sin x).
4
4

3 + x
Bi 3.63 : Gii phng trnh : 1 cos( + x) sin
= 0.
2

Bi 3.62 : Gii phng trnh : cos 2x +

TRN ANH TUN - 0974 396 391 - (04) 66 515 343

WWW.VNMATH.COM

Trang 53

www.VNMATH.comCHUYN LUYN THI I HC

www.luyenthi24h.com
www.luyenthi24h.com
www.VNMATH.com

4 sin2 2x + 6 sin2 x 9 3 cos 2x


= 0.
cos x

3
cos 2x 1
Gii phng trnh : cot
+ x tan2 x =
.
2
cos2 x

5
+ 4 cos x
= .
Gii phng trnh : cos 2 x +
3
6
2

cos2 3x 3 cos
Gii phng trnh : cos2 3x +
3x + 2 = 0.
2
2
cos 2x + 3 cot 2x + sin 4x
Gii phng trnh :
= 2.
cot 2x cos 2x

cos x(cos x + 2 sin x) + 3 sin x(sin x + 2)


= 1.
Gii phng trnh :
sin 2x 1
17
Gii phng trnh : sin8 x + cos8 x =
cos2 2x.
16
5x
x
Gii phng trnh : sin
= 5 cos3 x sin
2
2
Gii phng trnh : sin 2x(cot x + tan 2x) = 4 cos2 x.
8x
6x
+ 1 = 3 cos .
Gii phng trnh : 2 cos2
5
5

3
Gii phng trnh : tan x
= tan x 1.
4
sin4 2x + cos4 2x

= cos4 4x.
Gii phng trnh :

tan
x tan
+x
4
4
1
2
Gii phng trnh : 48
2 (1 + cot 2x cot x) = 0.
4
cos x sin x
5
Gii phng trnh : sin8 x + cos8 x = 2(sin10 x + cos10 x) + cos 2x.
4
Gii phng trnh : sin 2x + 2 tan x = 3.
1
Gii phng trnh : 2 tan x + cot 2x = 2 sin 2x +
.
sin 2x

Gii phng trnh : 3 cot2 x + 2 2 sin2 x = (2 + 3 2) cos x.

Bi 3.64 : Gii phng trnh :


Bi 3.65 :
Bi 3.66 :
Bi 3.67 :
Bi 3.68 :
Bi 3.69 :
Bi 3.70 :
Bi 3.71 :
Bi 3.72 :
Bi 3.73 :
Bi 3.74 :
Bi 3.75 :
Bi 3.76 :
Bi 3.77 :
Bi 3.78 :
Bi 3.79 :
Bi 3.80 :

Bi 3.81 : Tm x [; ] tha mn phng trnh :

cos4 x + sin4 x + cos x


Bi 3.82 :
Bi 3.83 :
Bi 3.84 :
Bi 3.85 :
Bi 3.86 :

3
sin 3x
= .
4
4
2

cos x(2 sin x + 3 2) 2 cos2 x 1


= 1.
Gii phng trnh :
1 + sin 2x
x
3x
x
3x 1
Gii phng trnh : cos x cos cos
sin x sin sin
= .
2
2
2
2
2

3
Gii phng trnh : 4 cos x + 3 2 sin 2x = 8 cos x.
1
1
Gii phng trnh : 2 sin 3x
= 2 cos 3x +
.
sin x
cos x
Gii phng trnh : 3 cos 4x 8 cos6 x + 2 cos2 x + 3 = 0.

Bi 3.87 : Gii phng trnh : 3 cos 4x 2 cos2 3x = 1.


3
Bi 3.88 : Gii phng trnh : 1 + sin3 x + cos3 x = sin 2x.
2
Bi 3.89 : Gii phng trnh : sin x sin 2x + sin 3x = 6 cos3 x.
Bi 3.90 : Gii phng trnh : tan x + 2 cot 2x = sin 2x.
Bi 3.91 : Gii phng trnh : 1 + 3 tan x = 2 sin 2x.

TRN ANH TUN - 0974 396 391 - (04) 66 515 343

WWW.VNMATH.COM

Trang 54

www.VNMATH.com

www.luyenthi24h.com
www.luyenthi24h.com
CHUYN LUYN THI I HCwww.VNMATH.com

Bi 3.92 : Gii phng trnh : sin x + cot

x
= 2.
2

Bi 3.93 : Gii phng trnh : sin 2x + 2 sin x


= 1.
4

Bi 3.94 : Gii phng trnh : 2(sin x + cos x) sin x cos x = 1.

Bi 3.95 : Gii phng trnh : sin x cos x + 2 sin x + 2 cos x = 2.

2 3
1 + sin x cos x.
Bi 3.96 : Gii phng trnh : sin x + cos x =
3

Bi 3.97 : Gii phng trnh : (1 + 2)(sin x cos x) + 2 sin x cos x = 1 + 2.


3
Bi 3.98 : Gii phng trnh : 1 + sin3 2x + cos3 2x = sin 4x.
2
2
Bi 3.99 : Gii phng trnh :
+ 2 tan2 x + 5 tan x + 5 cot x + 4 = 0.
sin2 x
Bi 3.100 : Gii phng trnh : 4 sin3 x + 3 cos3 x 3 sin x sin2 x cos x = 0.

Bi 3.101 : Gii phng trnh : sin2 x(tan x + 1) = 3 sin x(cos x sin x) + 3.

2
Bi 3.102 : Gii phng trnh : 2 tan x cot x = 3 +
.
sin 2x

= cos 3x.
Bi 3.103 : Gii phng trnh : 8 cos3 x +
3
3
Bi 3.104 : Gii phng trnh : 1 + sin3 x + cos3 x = sin 2x.
2

Bi 3.105 : Gii phng trnh : 2(sin x + cos x) = tan x + cot x.

3(1 + sin x)
2 x .
Bi 3.106 : Gii phng trnh : 3 tan3 x tan x +
=
8
cos
cos2 x
4 2
3
3
Bi 3.107 : Gii phng trnh : 2 sin x sin x = 2 cos x cos x + cos 2x.

Bi 3.108 : Gii phng trnh : sin x + sin2 x + sin3 x + sin4 x = cos x + cos2 x + cos3 x + cos4 x.

Bi 3.109 : Gii phng trnh : tan2 x(1 sin3 x) + cos3 x 1 = 0.

Bi 3.110 : Gii phng trnh : 3(cot x cos x) 5(tan x sin x) = 2.


Bi 3.111 : Gii phng trnh : 2 sin x + cot x = 2 sin 2x + 1.

Bi 3.112 : Gii phng trnh : cos 2x + 5 = 2(2 cos x)(sin x cos x).
Bi 3.113 : Gii phng trnh : sin3 x + cos3 x = cos 2x.

Bi 3.114 : Gii phng trnh : 3 tan2 x + 4 tan x + 4 cot x + 3 cot2 x + 2 = 0.


Bi 3.115 : Gii phng trnh : tan x + tan2 x + tan3 x + cot x + cot2 x + cot3 x = 6
2
Bi 3.116 : Gii phng trnh :
+ 2 tan2 x + 5 tan x + 5 cot x + 4 = 0.
sin2 x

Bi 3.117 : Gii phng trnh : cos2 x 3 sin 2x = 1 + sin2 x.

2
2 3
Bi 3.118 : Gii phng trnh : 3 sin (3 x) + 2 sin
+ x cos
+ x 5 sin
+ x = 0.
2
2
2
Bi 3.119 : Gii phng trnh : cos3 x 4 sin3 x 3 cos x sin2 x + sin x = 0.
Bi 3.120 : Gii phng trnh : 3 cos4 x 4 sin2 x cos2 x + sin4 x = 0.

Bi 3.121 : Gii phng trnh : sin x sin 2x + sin 3x = 6 cos3 x.

Bi 3.122 : Gii phng trnh : sin 3x + cos 3x + 2 cos x = 0.


4 sin 4x cos x
Bi 3.123 : Gii phng trnh : 6 sin x 2 cos3 x =
.
2 cos 2x
Bi 3.124 : Gii phng trnh : sin x 4 sin3 x + cos x = 0.

TRN ANH TUN - 0974 396 391 - (04) 66 515 343

WWW.VNMATH.COM

Trang 55

www.VNMATH.comCHUYN LUYN THI I HC

www.luyenthi24h.com
www.luyenthi24h.com
www.VNMATH.com

Bi 3.125 : Gii phng trnh : 2 cos3 x = sin 3x.

Bi 3.126 : Gii phng trnh : 2 sin3 x +


= 2 sin x.
4

Bi 3.127 : Gii phng trnh : sin3 x


= 2 sin x.
4

1
3
+
.
Bi 3.128 : Gii phng trnh : 2 sin x + 2 3 cos x =
cos x sin x
Bi 3.129 : Gii cc phng trnh sau :
1. sin x cos 2x = 6 cos x(1 + 2 cos 2x).
2. sin3

x
x
x
x
x
x
sin2 cos 3 sin cos2 + 3 cos3 = 0.
3
3
3
3
3
3

6 cos3 2x + 2 sin3 2x
= cos 4x.
3 cos 2x sin 2x

x
x
40 sin3 cos3
2
2 = sin x.
4.
x
x
16 sin 25 cos
2
2

x + 3 cos2
+ x cos x 5 cos2 x sin
+ x = 0.
5. 2 sin x cos2
2
2
2

x
3
3
6. 3 sin2 cos
2x + 3 sin 2x sin2
+ 2x + 2 cos3 2x = 0.
2
2
2
3.

7.

2(cos3 x + 2 sin3 x)
= sin 2x.
2 sin x + 3 cos x

8. sin3 x + sin x sin 2x 3 cos3 x = 0.


9.
10.
11.
12.
13.

14.

sin3 x + cos3 x
= cos 2x.
2 cos x sin x

sin3 x
+ 3 sin3 x +
= cos x + sin 2x.
6
3

sin 3x +
+ 2 cos 2x
sin x +
2 cos 3x +
= 0.
4
2
2
4
1 + sin 2x cos 2x
8
=
10 cot x.
cos2 x
sin 2x
2 + 5 sin 2x
(sin x 2 cos x)(1 sin 2x cos 2x) = 1
.
1 + cos 2x

3
3

sin x +
+ cos x
2

3
6

=
cos 2x
+ sin 2x +
.
3
6
3
3 sin x + cos x

2
+ cos 3x +
6
3 = sin x + 3 cos x.

15.

cos 2x
sin 2x +
6
3
Bi 3.130 : Gii phng trnh : tan x sin2 x 2 sin2 x = 3(cos 2x + sin x cos x).
sin 3x +

Bi 3.131 : Cho phng trnh :

(4 6m) sin3 x + 3(2m 1) sin x + 2(m 2) sin2 cos x (4m 3) cos x = 0.


a) Gii phng trnh khi m = 2 ;

b) Tm m phng trnh c nghim duy nht trn on 0;

.
4

Bi 3.132 : Gii cc phng trnh sau :

TRN ANH TUN - 0974 396 391 - (04) 66 515 343

WWW.VNMATH.COM

Trang 56

www.luyenthi24h.com
www.luyenthi24h.com
CHUYN LUYN THI I HCwww.VNMATH.com

www.VNMATH.com
1.

1
;
cos x

3 sin x + cos x =

2. 4 sin x + 6 cos x =

1
.
cos x

Bi 3.133 : Gii cc phng trnh sau :


1. 4 sin x cos
2. 2 sin x cos
3.

3
x + 4 sin( + x) cos x + 2 sin
x cos( + x) = 1.
2
2

+ x 3 sin( x) cos x + sin


+ x cos x = 0.
2
2

tan x + cot x
= 6 cos 2x + 4 sin 2x.
cot x tan x

Bi 3.134 : Gii cc phng trnh sau :


a) 8 sin x =

1
3
+
.
cos x sin x

b)

sin3 x + cos3 x
= cos 2x.
2 cos x sin x

Bi 3.135 : Gii cc phng trnh sau :


a) 1 + 3 sin2 2x = 2 tan x ;

e) tan x + tan2 x + tan3 x + cot x + cot2 x + cot3 x = 6 ;

b) tan x + cot x = 2(sin 2x + cos 2x) ;

f) 2 cos3 x = sin 3x ;

c) sin3 x + cos3 x = sin 2x + cos x + sin x + 1 ;

g) 8 cos3 x +

d)

4 sin2 2x + 6 sin2 x 9 3 cos 2x


=0;
cos x

= cos 3x ;
3

h) tan x + 2 sin 2x = 3 ;

Bi 3.136 : Tm a phng trnh sau c nghim :


sin6 x + cos6 x = a| sin 2x|.
Bi 3.137 : Cho phng trnh : cos3 x sin3 x = m.
a) Gii phng trnh khi m = 1 ;


b) Tm m sao cho phng trnh c ng hai nghim trn on ;
4 4

Bi 3.138 : Cho phng trnh :


2 cos 2x + sin2 x cos x + sin x cos2 x = m(sin x + cos x).
a) Gii phng trnh khi m = 2 ;

b) Tm m phng trnh c t nht mt nghim trn on 0;

Bi 3.139 : Cho phng trnh : m(sin x + cos x) = 1 + sin 2x. Tm m phng trnh c nghim thuc on 0;

.
2

Bi 3.140 : Cho phng trnh : cos3 x + sin3 x = m sin x cos x.


a) Gii phng trnh khi m =

2;

b) Tm m phng trnh c nghim.


Bi 3.141 : Cho phng trnh : sin 2x + 4(cos x sin x) = m.
a) Gii phng trnh khi m = 4.

TRN ANH TUN - 0974 396 391 - (04) 66 515 343

WWW.VNMATH.COM

Trang 57

www.VNMATH.comCHUYN LUYN THI I HC

www.luyenthi24h.com
www.luyenthi24h.com
www.VNMATH.com

b) Tm m phng trnh c nghim.


Bi 3.142 : Cho phng trnh :

m(sin x + cos x) + 1 +
a) Gii phng trnh khi m =

1
;
2

b) Tm m phng trnh c nghim trn 0;

1
1
1
tan x + cot x +
= 0.
+
2
sin x cos x

.
2

Bi 3.143 : Cho phng trnh : cos 2x (2m + 1) cos x + m + 1 = 0.


a) Gii phng trnh khi m =

3
;
2

b) Tm m phng trnh c nghim thuc khong

3
;
.
2 2

Bi 3.144 : Cho phng trnh : (cos x + 1)(cos 2x m cos x) = m sin2 x.


a) Gii phng trnh khi m = 2 ;

2
b) Tm m sao cho phng trnh c ng hai nghim trn 0;
.
3
Bi 3.145 : Cho phng trnh (1 a) tan2 x
a) Gii phng trnh khi a =

2
+ 1 + 3a = 0.
cos x

1
;
2

b) Tm a phng trnh c nhiu hn mt nghim trn 0;

.
2

Bi 3.146 : Cho phng trnh : cos 4x + 6 sin x cos x = m.


a) Gii phng trnh khi m = 1 ;

b) Tm m phng trnh c hai nghim phn bit trn 0; .


4
Bi 3.147 : Cho phng trnh : 4 cos5 x sin x 4 sin5 x cos x = sin2 4x + m.
a) Bit x = l mt nghim ca phng trnh trn. Hy gii phng trnh trong trng hp ny ;

b) Cho bit x = l mt nghim ca phng trnh trn. Hy tm tt c cc nghim tha mn x4 3x2 + 2 < 0.
8
Bi 3.148 : Tm a hai phng trnh sau tng ng
2 cos x cos 2x = 1 + cos 2x + cos 3x

(1)

4 cos2 x cos 3x = a cos x + (4 a)(1 + cos 2x)

(2)

Bi 3.149 : Cho phng trnh : cos 4x = cos2 3x + a sin2 x.


a) Gii phng trnh khi a = 1 ;

b) Tm a phng trnh c nghim trn 0;

.
12

Bi 3.150 : Tm m phng trnh sau c nghim :


sin6 x + cos6 x = m| sin 2x|.

TRN ANH TUN - 0974 396 391 - (04) 66 515 343

WWW.VNMATH.COM

Trang 58

www.luyenthi24h.com
www.luyenthi24h.com
CHUYN LUYN THI I HCwww.VNMATH.com

www.VNMATH.com

Bi 3.151 : Tm m phng trnh sau c nghim :


3
+ 3 cot2 x + m(tan x + cot x) 1 = 0.
cos2 x
Bi 3.152 : Tm m phng trnh sau c nghim :
cos4 x 2 sin2 x + m = 0.
Bi 3.153 : Tm m phng trnh sau c nghim :
4(sin4 x + cos4 x) 4(sin6 x + cos6 x) sin2 4x = m.
Bi 3.154 : Tm m phng trnh sau c nghim :
sin6 x + cos6 x
= m tan 2x.
cos2 x sin2 x
Bi 3.155 : Tm m phng trnh sau c nghim :
sin4 x + cos4 x cos 2x +

1 2
sin 2x + m = 0.
4

Bi 3.156 : Cho phng trnh :


1
+ cot2 x + m(tan x + cot x) + 2 = 0.
cos2 x
a) Gii phng trnh khi m =

5
;
2

b) Tm m phng trnh v nghim.


Bi 3.157 : Tm m phng trnh sau c nghim :
sin 2x m(cos x sin x) = m.
Bi 3.158 : Tm a phng trnh :
(1 a) tan2 x

2
+ 1 + 3a = 0
cos x

co nhiu hn mt nghim thuc khong 0;


.
2
Bi 3.159 : Tm m phng trnh :
2 cos x cos 2x cos 3x + m = 7 cos 2x
c nhiu hn 1 nghim trong

3
;
.
8 8

Bi 3.160 : Tm m phng trnh :

cos 3x cos 2x + m cos x = 1

c ng 7 nghim trong khong ; 2 .


2

TRN ANH TUN - 0974 396 391 - (04) 66 515 343

WWW.VNMATH.COM

Trang 59

www.VNMATH.comCHUYN LUYN THI I HC

www.luyenthi24h.com
www.luyenthi24h.com
www.VNMATH.com

3.4 a phng trnh v dng tch


Bi 3.161 : Gii phng trnh : cos x + cos 2x + cos 3x + cos 4x = 0.
Bi 3.162 : Gii phng trnh : sin x + sin 2x + sin 3x = 1 + cos x + cos 2x.
Bi 3.163 : Gii phng trnh : 1 + cos x + cos 2x + cos 3x = 0.
Bi 3.164 : Gii phng trnh : cos x + cos 2x + cos 3x + cos 4x = 0.
Bi 3.165 : Gii phng trnh : sin x + sin 2x + sin 3x + sin 4x + sin 5x + sin 6x = 0.
Bi 3.166 : Gii phng trnh : sin 3x sin x + sin 2x = 0.

Bi 3.167 : Gii phng trnh : cos 10x cos 8x cos 6x + 1 = 0.

Bi 3.168 : Gii phng trnh : 1 + sin x + cos 3x = cos x + sin 2x + cos 2x.
Bi 3.169 : Gii phng trnh : tan x = sin 4x.
Bi 3.170 : Gii phng trnh : (2 sin x 1)(2 sin 2x + 1) = 3 4 cos2 x.

Bi 3.171 : Gii phng trnh : (2 sin x + 1)(3 cos 4x + 2 sin x 4) + 4 cos2 x = 3.

Bi 3.172 : Gii phng trnh : (cos x sin x) cos x sin x = cos x cos 2x.
Bi 3.173 : Gii phng trnh : sin2 x + sin2 3x = cos2 2x + cos2 4x.
Bi 3.174 : Cho phng trnh :

sin x cos 4x sin 2x = 4 sin

x
7

.
4 2
2

Tm cc nghim ca phng trnh tha mn |x 1| < 3.

Bi 3.175 : Gii phng trnh : sin 2x cos 3x = sin 5x cos 6x.


Bi 3.176 : Gii phng trnh : sin x + sin 2x + sin 3x = cos x + cos 2x + cos 3x.
Bi 3.177 : Gii phng trnh : 4 sin3 x + 3 cos3 x 3 sin x sin2 x cos x = 0.
Bi 3.178 : Gii phng trnh : cos3 x sin3 x = sin 2x + sin x + cos x.
Bi 3.179 : Gii phng trnh : cos2 x + sin3 x + cos x = 0.
Bi 3.180 : Gii phng trnh : cos3 x + cos2 x + 2 sin x 2 = 0.

Bi 3.181 : Gii phng trnh : sin x + sin2 x + cos3 x = 0.

Bi 3.182 : Gii phng trnh : 2 sin3 x sin x = 2 cos3 x cos x + cos 2x.

Bi 3.183 : Gii phng trnh : 4 cos3 x + 3 2 sin 2x = 8 cos x.


Bi 3.184 : Gii phng trnh : sin x + sin2 x + sin3 x + sin4 x = cos x + cos2 x + cos3 x + cos4 x.
x
x
Bi 3.185 : Gii phng trnh : cos4 sin4 = sin 2x.
2
2
x
4 x
Bi 3.186 : Gii phng trnh : (sin x + 3) sin
(sin x + 3) sin2 + 1 = 0.
2
2

1
1
Bi 3.187 : Gii phng trnh : 2 2 sin x +
=
+
.
4
sin x cos x
1
1
2
Bi 3.188 : Gii phng trnh :
+
=
.
cos x sin 2x sin 4x
Bi 3.189 : Gii phng trnh : (sin6 x + cos6 x) = 2(sin8 x + cos8 x).
Bi 3.190 : Gii phng trnh : sin 2x(cot x + tan 2x) = 4 cos2 x.

TRN ANH TUN - 0974 396 391 - (04) 66 515 343

WWW.VNMATH.COM

Trang 60

www.luyenthi24h.com
www.luyenthi24h.com
CHUYN LUYN THI I HCwww.VNMATH.com

www.VNMATH.com

Bi 3.191 : Gii phng trnh : 2 tan x + cot 2x = 2 sin 2x +


Bi 3.192 : Gii phng trnh :

1
.
sin 2x

(1 cos x)2 + (1 + cos x)2


1
tan2 x sin x = (1 + sin x) + tan2 x.
4(1 sin x)
2

Bi 3.193 : Gii phng trnh : tan2 x cot2 2x cot 3x = tan2 x cot2 2x + cot 3x.
sin 3x sin 5x
=
.
Bi 3.194 : Gii phng trnh :
3
5
sin 5x
Bi 3.195 : Gii phng trnh :
= 1.
5 sin x
1
.
Bi 3.196 : Gii phng trnh : 2 cos 2x 8 cos x + 7 =
cos x
Bi 3.197 : Gii cc phng trnh sau :
1. sin 6x + sin 2x =

1
tan 2x ;
2

cos2 (1 + cot x) 3
= 3 cos x ;
sin x cos x

3. cos x sin
+ 6x + cos
x sin 6x = cos 6x + cos 4x ;
2
2

(sin x + cos x)2 2 sin2 x


4.
sin
x sin
3x =
;
2
4
4
1 + cot2 x
2.

5. 1 + sin

6.
7.

x
x
x
sin x cos sin2 x = 2 cos2

;
2
2
4 2

2x
1
2
=
1;
1 + cos 2x
cos2 2x

cos2

5 sin x 5 tan x
+ 4(1 cos x) = 0.
sin x + tan x

Bi 3.198 : Gii phng trnh : cos x + cos 3x + 2 cos 5x = 0.

Bi 3.199 : Tm cc nghim thuc khong 0;


ca phng trnh :
2
sin2 4x cos2 6x = sin(10, 5 + 10x).
Bi 3.200 : Gii phng trnh : cos6 x sin6 x = cos 2x.
Bi 3.201 : Gii phng trnh : sin x + cos x = cos 2x.

Bi 3.202 : Gii phng trnh : 1 + sin x + cos x + sin 2x + cos 2x = 0.


Bi 3.203 : Gii phng trnh : sin 2x cos 2x = 3 sin x + cos x 2.
1
1
Bi 3.204 : Gii phng trnh : 2 sin 3x
= 2 cos 3x +
.
sin x
cos x
x
3x
x
3x
Bi 3.205 : Gii phng trnh : cos x cos cos
sin x sin sin
= 0.
2
2
2
2
2
Bi 3.206 : Gii phng trnh : 3 tan 3x + cot 2x = 2 tan x +
.
sin 4x
Bi 3.207 : Gii cc phng trnh sau :
a) cos2 x + cos2 2x + cos2 3x + cos2 4x =

3
;
2

b) 4 sin 2x 3 cos 2x = 3(4 sin x 1) ;

TRN ANH TUN - 0974 396 391 - (04) 66 515 343

c) 2 cos3 x + cos 2x + sin x = 0 ;


d) sin

x+
= 2 sin x ;
4

WWW.VNMATH.COM

Trang 61

www.VNMATH.comCHUYN LUYN THI I HC

www.luyenthi24h.com
www.luyenthi24h.com
www.VNMATH.com

e) 3 sin x + 2 cos x = 2 + 3 tan x ;

g) 3(cot x cos x) 5(tan x sin x) = 2 ;

f) sin x + 2 cos x + cos 2x 2 sin x. cos x = 0 ;

h) 9 sin x + 6 cos x 3 sin 2x + cos 2x = 8 ;

Bi 3.208 : Cho phng trnh : sin 3x = m sin x + (4 2m) sin2 x.


a) Gii phng trnh khi m = 3 ;
b) Tm m phng trnh c ng 5 nghim thuc on [0; 2] ;
Bi 3.209 : Chp phng trnh :

m cos x 2
m sin x 2
=
.
m 2 cos x
m 2 sin x

a) Gii phng trnh khi m = 1.

b) Khi m , 0; 2, phng trnh c bao nhiu nghim thuc [20; 30].

3
Bi 3.210 : Tm m phng trnh sau c ng hai nghim thuc 0;
:
4
sin 2x + m = sin x + 2m cos x.
Bi 3.211 : Cho phng trnh :
(2 sin x 1)(2 cos 2x + 2 sin x + m) = 3 4 cos2 x.
Tm m phng trnh
a) c nhiu hn 2 nghim trong khong (0; ) ;
b) c ng 8 nghim thuc on [0; 7] ;

3.5 Phng php nh gi v phng php hm s


Bi 3.212 : Gii phng trnh : (cos 2x cos 4x)2 = 6 + 2 sin 3x.

Bi 3.213 : Gii phng trnh : cos 3x + 2 cos2 3x = 2(1 + sin2 2x).

Bi 3.214 : Gii phng trnh : cos5 x + sin5 x + cos 2x + sin 2x = 1 + 2.

Bi 3.215 : Gii phng trnh : 4 cos2 x + 3 tan2 x 4 3 cos x + 2 3 tan x + 4 = 0.


x2
= cos x.
2
Bi 3.217 : Gii cc phng trnh sau :
Bi 3.216 : Gii phng trnh : 1

a) sin x + cos x =

b) tan x + cot x =

2(2 sin 3x) ;

e) sin3 x + cos3 x = 2 sin4 x ;

2(sin x + cos x) ;

f) sin8 x + cos8 x = 2(sin10 x + cos10 x) +

c) cos13 x + sin14 x = 1 ;
d) | sin

x|

= | cos x| ;

TRN ANH TUN - 0974 396 391 - (04) 66 515 343

g) sin2 x +

5
cos 2x ;
4

1 2
sin 3x = sin x. sin2 3x ;
4

h) cos 2x cos 6x + 4(3 sin x 4 sin3 x + 1) = 0 ;


WWW.VNMATH.COM

Trang 62

www.VNMATH.com

www.luyenthi24h.com
www.luyenthi24h.com
CHUYN LUYN THI I HCwww.VNMATH.com

3.6 Gi tr ln nht v nh nht ca biu thc lng gic


Bi 3.218 : Tm GTLN, GTNN ca hm s : y = cos x +
Bi 3.219 : Tm GTLN ca hm s : y = sin3 x sin6 x.
Bi 3.220 : Tm GTNN ca :

y= 1+

1
cos 2x.
2

1
sin2 x

+ 1+

1
cos2 x

Bi 3.221 : Tm GTNN ca hm s :

y = sin2 x +

1
sin2 x

+ cos2 x +

1
cos2 x

Bi 3.222 : Tm GTLN v GTNN ca cc hm s :


a) y =

sin x + 2 cos x + 1
;
sin x + cos x + 2

b) y =

sin x + 2 cos x + 3
;
2 sin x + cos x + 3

Bi 3.223 : Cho x, y > 0 v x2 + y2 = 1. Tm GTLN ca P = x3 + y3 .


Bi 3.224 (C-2008) : Cho hai s thc x, y thay i v tho mn x2 + y2 = 2. Tm gi tr ln nht v gi tr nh nht ca
biu thc: P = 2(x3 + y3 ) 3xy.

Bi 3.225 (H-KB2008) : Cho hai s thc x, y thay i v tho mn x2 + y2 = 1. Tm gi tr ln nht v gi tr nh nht


2(x2 + 6xy)
ca biu thc: P =
.
1 + 2xy + 2y2

3.7 Lng gic trong cc k thi tuyn sinh H


Bi 3.226 (C08) : Gii phng trnh : sin 3x

3 cos 3x = 2 sin 2x.

Bi 3.227 (C09) : Gii phng trnh : (1 + 2 sin x)2 cos x = 1 + sin x + cos x.
5x
3x
Bi 3.228 (C10) : Gii phng trnh 4 cos
cos
+ 2(8 sin x 1) cos x = 5.
2
2
Bi 3.229 (A02) : Tm nghim thuc khong (0; 2) ca phng trnh :

5 sin x +

cos 3x + sin 3x
= cos 2x + 3.
1 + 2 sin 2x

cos 2x
1
+ sin2 x sin 2x.
1 + tan x
2

Bi 3.231 (A04) : Cho tam gic ABC khng t, tho mn iu kin : cos 2A + 2 2 cos B + 2 2 cos C = 3. Tnh ba gc
Bi 3.230 (A03) : Gii phng trnh : cot x 1 =
ca tam gic ABC.
Bi 3.232 (A05) : Gii phng trnh : cos2 3x cos 2x cos2 x = 0.

2 cos6 x + sin6 x sin x cos x

Bi 3.233 (A06) : Gii phng trnh :


= 0.
2 2 sin x

Bi 3.234 (A07) : Gii phng trnh : 1 + sin2 x cos x + 1 + cos2 x sin x = 1 + sin 2x.
Bi 3.235 (A08) : Gii phng trnh :

1
1

+
sin x sin x

3
2

= 4 sin 7 x .
4

(1 2 sin x) cos x
= 3.
(1 + 2 sin x)(1 sin x)

(1 + sin x + cos 2x) sin x +


4 = 1 cos x.
Bi 3.237 (A10) : Gii phng trnh
1 + tan x
2
Bi 3.236 (A09) : Gii phng trnh :

TRN ANH TUN - 0974 396 391 - (04) 66 515 343

WWW.VNMATH.COM

Trang 63

www.VNMATH.comCHUYN LUYN THI I HC

www.luyenthi24h.com
www.luyenthi24h.com
www.VNMATH.com

Bi 3.238 (B02) : Gii phng trnh : sin2 3x cos2 4x = sin2 5x cos2 6x.
2
Bi 3.239 (B03) : Gii phng trnh : cot x tan x + 4 sin 2x =
.
sin 2x
Bi 3.240 (B04) : Gii phng trnh : 5 sin x 2 = 3(1 sin x) tan2 x.

Bi 3.241 (B05) : Gii phng trnh : 1 + sin x + cos x + sin 2x + cos 2x = 0.

x
= 4.
Bi 3.242 (B06) : Gii phng trnh : cot x + sin x 1 + tan x tan
2
Bi 3.243 (B07) : Gii phng trnh : 2 sin2 2x + sin 7x 1 = sin x.

Bi 3.244 (B08) : Gii phng trnh : sin3 x 3 cos3 x = sin x cos2 x 3 sin2 x cos x.

Bi 3.245 (B09) : Gii phng trnh : sin x + cos x sin 2x + 3 cos 3x = 2(cos 4x + sin3 x).
Bi 3.246 (B10) : Gii phng trnh (sin 2x + cos 2x) cos x + 2 cos 2x sin x = 0.
Bi 3.247 (D02) : Tm x thuc on [0; 14] nghim ng phng trnh :

cos 3x 4 cos 2x + 3 cos x 4 = 0.

Bi 3.249 (D04) :
Bi 3.250 (D05) :
Bi 3.251 (D06) :
Bi 3.252 (D07) :
Bi 3.253 (D08) :
Bi 3.254 (D09) :

x
x

tan2 x cos2 = 0.
2 4
2
Gii phng trnh : (2 cos x 1)(2 sin x + cos x) = sin 2x sin x.

3
Gii phng trnh : cos4 x + sin4 x + cos x
sin 3x
= 0.
4
4
2
Gii phng trnh : cos 3x + cos 2x cos x 1 = 0.

x
x 2
Gii phng trnh : sin + cos
+ 3 cos x = 2.
2
2
Gii phng trnh : 2 sin x(1 + cos 2x) + sin 2x = 1 + 2 cos x.

Gii phng trnh : 3 cos 5x 2 sin 3x cos 2x sin x = 0.

Bi 3.248 (D03) : Gii phng trnh : sin2

Bi 3.255 (D10) : Gii phng trnh sin 2x cos 2x + 3 sin x cos x 1 = 0.

3.8 Bi tp tng hp
Bi 3.256 : Xc nh m phng trnh :

2 sin4 x + cos4 x + cos 4x + 2 sin 2x + m = 0

.
2
sin4 x + cos4 x 1
1
Bi 3.257 : Gii phng trnh :
= cot 2x
.
5 sin 2x
2
8 sin 2x
(2 sin2 2x) sin 3x
Bi 3.258 : Gii phng trnh : tan4 x + 1 =
.
cos4 x
c t nht mt nghim thuc on 0;

x
.
2
Bi 3.260 : Cho A, B, C l ba gc ca tam gic ABC. Chng minh rng tam gic ABC u th iu kin cn v l :
Bi 3.259 : Gii phng trnh : tan x + cos x cos2 x = sin x 1 + tan x tan

cos2
Bi 3.261 : Cho phng trnh :

A
B
C
1
AB
BC
CA
+ cos2 + cos2 2 = cos
cos
cos
.
2
2
2
4
2
2
2
2 sin x + cos x + 1
= a (1) (a l tham s).
sin x 2 cos x + 3

1
1. Gii phng trnh (1) khi a = .
3

TRN ANH TUN - 0974 396 391 - (04) 66 515 343

WWW.VNMATH.COM

Trang 64

www.VNMATH.com

www.luyenthi24h.com
www.luyenthi24h.com
CHUYN LUYN THI I HCwww.VNMATH.com

2. Tm a phng trnh (1) c nghim ?


r

1
= sin x.
8 cos2 x
3
Bi 3.263 : Cho tam gic ABC din tch bng . Gi a, b, c ln lt l di cc cnh BC, CA, AB v ha , hb , hc ln lt l
2
di cc ng cao k t nh A, B, C ca tam gic. Chng minh rng :

Bi 3.262 : Gii phng trnh :

1 1 1
+ +
a b c

1
1
1
+
+
ha hb hc

3.

Bi 3.264 : Gii phng trnh : 3 tan x(tan x + 2 sin x) + 6 cos x = 0.


Bi 3.265 : Gii phng trnh : cos 2x + cos x(2 tan2 x 1) = 2.
Bi 3.266 : Tnh cc gc ca tam gic ABC, bit rng :

8
>
<

4p(p a) bc

A
B
C 2
>
:sin sin sin
=

2
2
a+b+c
c, p =
.
2
Bi 3.267 : Gii phng trnh : 3 cos 4x 8 cos6 x + 2 cos2 x + 3 = 0.

2 x

(2 3) cos x 2 sin
2 4 = 1.
Bi 3.268 : Gii phng trnh :
2 cos x 1
cos2 x(cos x 1)
Bi 3.269 : Gii phng trnh :
= 2(1 + sin x).
sin x + cos x
Bi 3.270 : Cho cc gc A, B, C ca tam gic ABC biu thc :

33
8

trong BC = a, CA = b, AB =

Q = sin2 A + sin2 B + sin2 C


t gi tr nh nht.
2 cos 4x
.
sin 2x
Bi 3.272 : Xc nh dng ca tam gic ABC, bit rng :
Bi 3.271 : Gii phng trnh : cot x = tan x +

(p a) sin2 A + (p b) sin2 B = c. sin A sin B,


a+b+c
.
2
Bi 3.273 : Tm nghim trn khong (0; ) ca phng trnh :

trong BC = a, CA = b, AB = a, p =

x
3
2
4 sin
.
3 cos 2x = 1 + 2 cos x
2
4
2

Bi 3.274 : Gii phng trnh : 2 2 cos3 x


3 cos x sin x = 0.
4

cos 2x 1
Bi 3.275 : Gii phng trnh : tan
+ x 3 tan2 x =
.
2
cos2 x

3
sin x
Bi 3.276 : Gii phng trnh : tan
x +
= 2.
2
1 + cos x
Bi 3.277 : Gii phng trnh : sin 2x + cos 2x + 3 sin x cos x 2 = 0.

2+3 2
3
3
Bi 3.278 : Gii phng trnh : cos 3x cos x sin 3x sin x =
.
8

Bi 3.279 : Gii phng trnh : 2 sin 2x


+ 4 sin x + 1 = 0.
6
Bi 3.280 : Gii phng trnh : cos 2x + (1 + 2 cos x)(sin x cos x) = 0.

Bi 3.281 : Gii phng trnh : (2 sin2 x 1) tan2 2x + 3(2 cos2 x 1) = 0.

TRN ANH TUN - 0974 396 391 - (04) 66 515 343

WWW.VNMATH.COM

Trang 65

www.luyenthi24h.com
www.luyenthi24h.com
www.VNMATH.com

www.VNMATH.comCHUYN LUYN THI I HC


Bi 3.282 : Gii phng trnh : cos3 x + sin3 x + 2 sin2 x = 1.
Bi 3.283 : Gii phng trnh : 4 sin3 x + 4 sin2 x + 3 sin 2x + 6 cos x = 0.
1
1

= 2 cot 2x.
Bi 3.284 : Gii phng trnh : sin 2x + sin x
2 sin x sin 2x

Bi 3.285 : Gii phng trnh : 2 cos2 x + 2 3 sin x cos x = 3(sin x + 3 cos x).

3x
5x
x
Bi 3.286 : Gii phng trnh : sin
cos
= 2 cos .

2
4
2 4
2
sin 2x cos 2x
+
= tan x cot x.
Bi 3.287 : Gii phng trnh :
cos x
sin x

cos x = 1.
Bi 3.288 : Gii phng trnh : 2 2 sin x
12
Bi 3.289 : Gii phng trnh : (1 tan x)(1 + sin 2x) = 1 + tan x.
Bi 3.290 : Gii cc phng trnh sau :
1
1. tan x + cot x =
9
2. cot x 1 =
3.

1
1 1;
cos2 x

15. 2(1 + sin x)(tan2 x + 1) =

cos 2x
1
+ sin2 x sin 2x;
1 + tan x
2

2 cos
+x
4
(1 + sin 2x) = 1 + cot x;
sin x

4. 3 cos x 3 sin x tan x sin x + sin x tan2 x = 0;


1 cos 2x 1 cos3 x
;
5.
=
1 + cos 2x
1 sin3 x
sin 5x
6.
= 1;
5 sin x

7. (sin 3x 2 sin x) 2 cos x

1
= 3 tan x;
cos x

10. sin x + sin 2x + sin 3x + sin 4x + sin 5x = 0;


1
sin3 x sin 3x + cos3 x cos 3x

= ;
11.

8
tan x +
tan
x
3
6
12. sin 4x + 2 cos 2x + 4(sin x + cos x) = 1 + cos 4x;
13. cos 5x + sin 5x + 2 cos 3x 2 sin 3x cos x sin x = 0;

1
+ cos x +
= cos 2x 1;
4
4
3

18. 2(sin8 x cos8 x) = cos2 2x cos 2x;


19. tan x + tan 2x = sin 3x cos 2x;

22.

9. sin3 x(1 cot x) + cos2 x(cos x sin x) = cos x + sin x;

17. cos x

3 sin 4x = 2;

21. 4 cos3 x + 2 sin3 x = 3 sin x;

1
sin 2x

= 2 sin x +
;
8. cot x +
sin x + cos x
2
2

14. cos2 x +

16. 3 sin x + 1 = sin4 x cos4 x;

20. 2(sin4 x + cos4 x) +

cos x 1
;
sin x + cos x

1
+ sin2 x +
= 2 sin x ;
3
6
2

sin 3x(2 sin2 2x)


= tan4 x + 1;
cos4 x

23. sin

5x
x
3x

cos

= 2 cos ;
2
4
4 2
2

24. sin2 x +

(1 + cos 2x)2
= 2 cos 2x;
2 sin 2x

25. sin3 x(1 + cot x) + cos3 x(1 + tan x) = 2 sin x cos x;


26. 8 cos x + 6 sin x cos 2x 7 = 0;
27.

x
x
cos3
2
2 = 1 cos x;
2 + sin x
3

sin3

28. 2 sin2 x

= 2 sin2 x tan x.
4

Bi 3.291 : Gii cc phng trnh sau :


1. 1 + sin x cos x sin 2x + cos 2x = 0;

4. 8 cos4 x + 1 = cos 4x + 12 sin x;

2. sin 4x + 2 = cos 3x + 4 sin x + cos x;

1
2(cos x sin x)
3.
=
;
tan x + cot 2x
cot x 1

5.

TRN ANH TUN - 0974 396 391 - (04) 66 515 343

4 cos 3x cos x 2 cos 4x 4 cos x + tan 2x tan x + 2

= 0;
2 sin x 3
WWW.VNMATH.COM

Trang 66

www.luyenthi24h.com
www.luyenthi24h.com
CHUYN LUYN THI I HCwww.VNMATH.com

www.VNMATH.com
6. 2 tan x + cot 2x = 2 sin 2x +

1
;
sin 2x

11.

sin x + cos x
+ 2 tan 2x + cos 2x = 0;
sin x cos x

12. 2 sin x
= 2 sin2 x tan x;
4

13. cos x + cos 3x = 1 + 2 sin 2x +


;
4

(sin x + cos x)2 2 sin2 x


2

8.
= 2 sin 4 x sin 4 3x ;
1 + cot 2x cot x
1 + cot2 x
14.
+ 2(sin4 x + cos4 x) = 3;

cos2 x
1
2(cos x sin x)

9.
=
;
3

tan x + cot 2x
cot x 1
15. cos2 2x 2 cos x +
sin 3x
= 2;
4
4

5
= 4 sin
x 9;
10. 5 cos 2x +
16. 2 sin2 x sin 2x + sin x + cos x 1 = 0.
3
6

sin 3x 4 cos x
3
6
7.
= 0;
sin 3x 1

WWW.VNMATH.COM

TRN ANH TUN - 0974 396 391 - (04) 66 515 343

WWW.VNMATH.COM

Trang 67

www.luyenthi24h.com
www.luyenthi24h.com

Chng 4

T hp
4.1 Cc quy tc m. T hp, chnh hp, hon v
Bi 4.1 : Mt bn di c hai dy gh i din nhau, mi dy gm c 6 gh. Ngi ta mun xp ch ngi cho 6 hc sinh
trng A v 6 hc sinh trng B vo bn ni trn. Hi c bao nhiu cch xp ch ngi trong mi trng hp sau :
a) Bt k hai hc sinh no ngi cnh nhau hoc i din nhau th khc trng nhau ;
b) Bt k hai hc sinh no ngi i din nhau th khc trng nhau ;
Bi 4.2 : C 10000 v c nh s t 00000 n 99999. Hi c bao nhiu v gm 5 ch s khc nhau.
Bi 4.3 : Vi 10 ch s 0, 1, 2, . . . , 8, 9 c th lp c bao nhiu s c 5 ch s khc nhau.
Bi 4.4 : T cc ch s 0, 1, 3, 5, 7 c th lp c bao nhiu s gm 4 ch s khc nhau v khng chia ht cho 5.
Bi 4.5 : Xt mt dy s gm 7 ch s (mi s c chn t 0, 1, . . . , 8, 9) m ch s v tr s 3 l s chn, ch s v
tr cui khng chia ht cho 5, cc ch s v tr s 4, 5, 6 i mt khc nhau. Hi c bao nhiu cch chn nh vy.
Bi 4.6 : Cho 10 ch s 0, 1, 2,. . . , 8, 9. C bao nhiu s l c 6 ch s khc nhau nh hn 600000 xy dng t cc s trn.
Bi 4.7 : Mt ngi vit ngu nhin cc ch s 0, 1, 2, 3, 4, 5 ln cc tm phiu, sau xp ngu nhin thnh mt hng.
a) C bao nhiu s l gm 6 ch s c to thnh.
b) C bao nhiu s chn gm 6 ch s c to thnh.
Bi 4.8 : C th lp c bao nhiu s chn gm 5 ch s khc nhau c ly t cc ch s 0, 2, 3, 6, 9.
Bi 4.9 : Cho X = {0, 1, 2, 3, 4, 5}.
a) C bao nhiu s chn c 4 ch s khc nhau i mt ;
b) C bao nhiu s c 3 ch s khc nhau i mt chia ht cho 5 ;
c) C bao nhiu s c 3 ch s khc nhau i mt chia ht cho 9 ;
Bi 4.10 : Cho X = {0, 1, 2, 3, 4, 5}. Hi c th lp c bao nhiu s c 3 ch s khc nhau m s khng chia ht cho
3.

Bi 4.11 : C bao nhiu cch xp 5 hc sinh A, B, C, D, E vo mt gh di sao cho :

69

WWW.VNMATH.COM

www.VNMATH.com

CHUYN LUYN THI I HC

a) C ngi chnh gia ;

www.luyenthi24h.com
www.luyenthi24h.com
www.VNMATH.com

b) A, E ngi hai u gh ;

Bi 4.12 : Trong mt phng c hai bn di, mi bn c 5 gh. Ngi ta mun xp ch ngi cho 10 hc sinh gm 5 nam v
5 n. Hi c bao nhiu cch xp ch ngi, nu :
a) cc hc sinh ngi ty ;
b) cc hc sinh nam ngi mt bn v cc hc sinh n ngi mt bn ;
Bi 4.13 : Mt hc sinh c 12 cun sch i mt khc nhau, trong c 4 sch Vn, 2 sch Ton, 6 sch Anh vn. Hi c
bao nhiu cch sp xp cc cun sch ln mt k di nu cc cun cng mn xp k nhau.
Bi 4.14 : T X = {1, 2, 3, 4, 5, 6} thit lp cc s c 6 ch s khc nhau. Hi trong cc s lp c c bao nhiu s m hai

ch s 1 v 6 khng ng cnh nhau.

Bi 4.15 : Xt cc s gm 9 ch s, trong c 5 ch s 1 v 4 ch s cn li l 2, 3, 4, 5. Hi c bao nhiu s m :


a) 5 ch s 1 sp k nhau ;

b) cc ch s c sp xp ty ;

Bi 4.16 : C bao nhiu s gm 7 ch s i mt khc nhau c lp t 7 ch s 1, 2, 3, 4, 5, 7, 9 sao cho hai ch s chn


khng nm lin nhau.
Bi 4.17 : Mt i vn ngh c 15 ngi, gm 10 nam v 5 n. Hi c bao nhiu cch lp mt nhm ng ca gm 8 ngi,
bit rng trong nhm phi c t nht 3 n.
Bi 4.18 : T cc ch s 1, 2, 3, 4 , 5, 6, 7 c th lp c bao nhiu s t nhin, mi s gm 5 ch s khc nhau v nht
thit phi c hai ch s 1 v 5.
Bi 4.19 : Cho tam gic ABC. Xt tp hp 4 ng thng song song vi AB, 5 ng thng song song vi BC v 6 ng
thng song song vi CA.
a) Hi cc ng thng ny to c bao nhiu tam gic ;
b) Hi cc ng thng ny to c bao nhiu hnh thang (khng k cc hnh bnh hnh).
Cho bit khng c 3 ng thng no ca h l ng quy.
Bi 4.20 : C bao nhiu s t nhin gm 5 ch s u ln hn 4 v i mt khc nhau. Tnh tng cc s trn.
Bi 4.21 : Trong cc ch s 0, 1, 2, 3, 4 c th lp c bao nhiu s c 7 ch s trong ch s 4 c mt ng 3 ln, cn
cc ch s khc c mt ng 1 ln.
Bi 4.22 : T X = {0, 1, 2, 3, 4, 5, 6} lp c bao nhiu s t nhin c 5 ch s khc nhau, trong nht thit phi c mt
ch s 5.

Bi 4.23 : T 7 ch s 0, 1, 2, 3, 4, 5, 6 c th lp c bao nhiu s chn mi s gm 5 ch s khc nhau.


Bi 4.24 : Cho X = {0, 1, 2, 3, 4, 5, 6, 7} c th lp c bao nhiu s gm 5 ch s m l :
a) s chn ;
b) mt trong ba ch s u tin phi c mt ch s 1.
Bi 4.25 : T 7 ch s 1, 2, 3, 4 , 5, 6 c th lp c bao nhiu s c 4 ch s khc nhau v c th lp c bao nhiu s
c 4 ch s khc nhau trong c hai ch s 1 v 2.

TRN ANH TUN - 0974 396 391 - (04) 66 515 343

WWW.VNMATH.COM

Trang 70

www.VNMATH.com

CHUYN LUYN THI I HC

www.luyenthi24h.com
www.luyenthi24h.com
www.VNMATH.com

Bi 4.26 : T 10 ch s 0, 1, 2, . . . , 8, 9 c th lp c bao nhiu s c 6 ch s khc nhau sao cho cc s u phi c


mt 0 v 1.
Bi 4.27 : T cc s 1, 2, 3, 4, 5, 6, 7, 8, 9 c th lp c bao nhiu s t nhin, mi s gm 6 ch s khc nhau v tng
cc ch s hng chc, hng trm, hng ngn bng 8.
Bi 4.28 : C bao nhiu s t nhin gm 6 ch s i mt khc nhau trong c mt ch s 0 nhng khng c mt ch s
1.
Bi 4.29 : Tnh tng cc s t nhin gm 5 ch s khc nhau c lp t cc ch s 1, 3, 4, 5, 7, 8.
Bi 4.30 : C 12 hc sinh u t ca mt trng trung hc. Mun chn mt on i biu gm 5 ngi (mt trng on,
mt th k v ba thnh vin) i d tri h. Hi c bao nhiu cch chn nh vy.
Bi 4.31 : C 12 hc sinh u t ca mt trng trung hc. Mun chn mt on i biu gm 4 ngi i d tri h. Hi
c bao nhiu cch chn :
a) ty ;
b) hai hc sinh A v B khng i cng nhau ;
c) hai hc sinh A v B cng i hoc cng khng i.
Bi 4.32 : Mt on tu c ba toa tr khch : toa I, toa II, toa III. Trn sn ga c 4 hnh khch chun b i tu, bit rng
mi toa c t nht 4 ch trng. Hi :
a) C bao nhiu cch sp xp 4 hnh khch ln ba toa ;
b) C bao nhiu cch sp xp 4 hnh khch ln tu c mt toa trong c 3 trong 4 v khch.
Bi 4.33 (B04) : C 30 cu hi khc nhau gm 5 cu kh, 10 cu trung bnh v 15 cu d. T 30 cu , c th lp c
bao nhiu kim tra, mi gm 5 cu khc nhau, sao cho mi phi c 3 loi (kh, d, trung bnh) v s cu d khng
t hn 2.
Bi 4.34 : Mt chi on c 20 on vin, trong c 10 n. Mun chn mt t cng tc c 5 ngi. C bao nhiu cch
chn nu t cn t nht mt n.
Bi 4.35 : Mt i xy dng gm 10 cng nhn, 3 k s. lp mt t cng tc, cn chn 1 k s lm t trng, 1 cng
nhn lm t ph v 3 cng nhn lm t vin. Hi c bao nhiu cch lp t cng tc.
Bi 4.36 : Mt i vn ngh gm 10 hc sinh nam v 10 hc sinh n. C gio mun chn ra mt tp ca gm 5 em trong
c t nht l 2 em nam v 2 em n. Hi c bao nhiu cch chn.
Bi 4.37 : Mt i cnh st gm c 9 ngi. Trong ngy cn 3 ngi lm nhim v ti a im A, 2 ngi lm ti B v 4
ngi cn li trc ti n. Hi c bao nhiu cch phn cng.
Bi 4.38 : C 5 nh Ton hc nam, 3 nh Ton hc n v 4 nh Vt l nam. Mun lp mt on cng tc c 3 ngi gm
c nam v n, cn c c nh Ton hc ln nh Vt l. Hi c bao nhiu cch lp t cng tc.
Bi 4.39 : Mt i vn ngh c 10 ngi, trong c 6 n v 4 nam. C bao nhiu cch :
a) chia i vn ngh thnh hai nhm c s ngi bng nhau v mi nhm c s n bng nhau ;
b) chn 5 ngi, trong c khng qu 1 nam ;
Bi 4.40 : C 5 tem th khc nhau v 6 b th cng khc nhau. Ngi ta mun chn t ra 3 tem th, 3 b th v dn 3
tem th ln 3 b th chn, mt b th ch dn 1 tem th. Hi c bao nhiu cc lm nh vy.

TRN ANH TUN - 0974 396 391 - (04) 66 515 343

WWW.VNMATH.COM

Trang 71

www.VNMATH.com

CHUYN LUYN THI I HC

www.luyenthi24h.com
www.luyenthi24h.com
www.VNMATH.com

Bi 4.41 : C hai ng thng song song d1 v d2 . Trn d1 ly 15 im phn bit, trn d2 ly 9 im phn bit. Hi c bao
nhiu tam gic m cc nh l 3 trong s cc im cho.
Bi 4.42 : Mt lp c 20 hc sinh, trong c 2 cn b lp. Hi c bao nhiu cch chn 3 ngi i d hi ngh ca trng
sao cho trong c t nht mt cn b lp.
Bi 4.43 : C 16 hc sinh, gm 3 hc sinh gii, 5 kh, 8 trung bnh. C bao nhiu cch chia s hc sinh thnh hai t, mi
t 8 ngi, u c hc sinh gii v t nht 2 hc sinh kh.
Bi 4.44 : Mt ngi c 12 cy ging, trong c 6 cy xoi, 4 cy mt v 2 cy i. Ngi mun chn 6 cy ging
trng. Hi c bao nhiu cch chn sao cho :
a) mi loi c ng 2 cy ;

b) mi loi c t nht 1 cy ;

Bi 4.45 : Mt lp hc c 30 hc sinh nam v 15 hc sinh n. C 6 hc sinh c chn lp mt tp ca. Hi c bao


nhiu cch chn khc nhau v phi c t nht 2 n.
Bi 4.46 : Cho tp con gm 10 phn t khc nhau. Tm s tp con khc rng cha mt s chn cc phn t.
Bi 4.47 : Mt t c 20 sinh vin, trong c 8 sinh vin bit ni ting Anh, 7 SV bit ni ting Php, 5 SV bit ni ting
c (khng SV no bit ni c 2 trong 3 ngoi ng trn). Cn chn mt nhm i thc t gm 3 SV bit ting Anh, 4 SV
bit ting Php, 2 SV bit ting c. Hi c bao nhiu cch lp nhm.
Bi 4.48 : Trong mt hp c 7 qu cu xanh, 5 qu cu v 4 qu cu vng, cc qu cu u khc nhau. Chn ngu nhin
4 qu cu trong hp. Hi c bao nhiu cch chn sao cho trong 4 qu cu chn ra c 3 mu.
Bi 4.49 : Mt hp c 6 qu cu xanh c nh s t 1 n 6, 5 qu cu c nh s t 1 n 5, 4 qu cu vng c
nh s t 1 n 4.
a) C bao nhiu cch ly 3 qu cu cng mu ; 3 qu cu cng s ;
b) C bao nhiu cch ly 3 qu cu khc mu ; 3 qu cu khc mu v khc s ;
Bi 4.50 : C 9 vin bi xanh, 5 vin bi , 4 vin bi vng c kch thc i mt khc nhau. C bao nhiu cch chn ra :
a) 6 vin bi, trong c ng 2 vin bi ;
b) 6 vin bi, trong s bi xanh bng s bi ;
Bi 4.51 : T 5 bng hng vng, 3 bng hng trng v 4 bng hng (cc bng hoa xem nh i mt khc nhau). Ngi
ta mun chn ra mt b hoa gm 7 bng. C bao nhiu cch chn b hoa, trong :
a) c ng mt bng hng ;
b) c t nht mt bng hng vng v t nht 3 bng hng ;
Bi 4.52 : Xp 3 bi c bn knh khc nhau v 3 bi xanh ging nhau vo mt hc c 7 trng.
a) Hi c bao nhiu cch xp nh vy ;
b) C bao nhiu cch xp, sao cho 3 bi xp cnh nhau v 3 bi xanh xp cnh nhau ;
Bi 4.53 : Mt hp ng 4 vin bi , 5 vin bi trng v 6 vin bi vng. Ngi ta chn 4 vin bi t hp. Hi c bao nhiu
cch chn s bi ly ra khng 3 mu.
Bi 4.54 : Cho a gic u H c 20 cnh. Xt cc tam gic c ba nh ly t ba nh ca H.

TRN ANH TUN - 0974 396 391 - (04) 66 515 343

WWW.VNMATH.COM

Trang 72

www.VNMATH.com

CHUYN LUYN THI I HC

www.luyenthi24h.com
www.luyenthi24h.com
www.VNMATH.com

a) C bao nhiu tam gic nh vy ? C bao nhiu tam gic c ng hai cnh l hai cnh ca H ?
b) C my tam gic c ng mt cnh l mt cnh ca H ? C my tam gic khng c cnh no l cnh ca H ?
Bi 4.55 : Trn mt phng cho mt thp gic li. Xt cc tam gic m 3 nh ca n l ba nh ca thp gic. Hi trong s
cc tam gic , c bao nhiu tam gic m 3 cnh ca n u khng phi l 3 cnh ca thp gic.
Bi 4.56 (B02) : Cho a gic A1 A2 . . . A2n (n N v n 2) ni tip trong ng trn (O). Bit rng s tam gic c nh l
3 trong 2n nh A1 , A2 , . . . , A2n nhiu gp 20 ln s hnh ch nht c cc nh l 4 trong 2n nh A1 , A2 , . . . , A2n . Tm n.

Bi 4.57 : Trong mt trng tiu hc c 50 hc sinh t danh hiu "chu ngoan Bc H" trong c 4 cp anh em sinh
i. Cn chn mt nhm gm 3 trong s 50 hc sinh trn i d i hi "chu ngoan Bc H", sao cho trong nhm khng
c cp anh em sinh i no. Hi c bao nhiu cch chn nh vy.
Bi 4.58 : Mt tp th c 14 ngi, gm 6 nam v 8 n, trong c An v Bnh. Ngi ta mun chn mt t cng tc gm
6 ngi. Tm s cch chn trong mi trng hp sau :
a) Trong t phi c mt c nam ln n ;
b) Trong t phi c 1 t trng, 5 t vin, hn na An v Bnh khng ng thi c mt trong t ;
Bi 4.59 : Mt Thy gio c 12 cun sch i mt khc nhau, trong c 5 sch Vn, 4 sch Anh vn v 3 sch Ha. ng
ly ra 6 cun v tng 6 hc sinh A, B, C, D, E, F mi em mt cun.
a) Gi s Thy gio ch mun tng cc hc sinh trn nhng cun sch thuc loi Anh vn v Vn. Hi c bao nhiu cch
tng ;
b) Gi s Thy gio mun rng, sau khi tng xong, mi loi Vn, Anh vn, Ha cn t nht mt cun. Hi c bao nhiu
cch tng.
Bi 4.60 : T cc s 0, 1, 2, 3, 4 c th lp c bao nhiu s t nhin c 5 ch s khc nhau. Tnh tng tt c cc s .
Bi 4.61 : Cho hai ng thng song song d1 , d2 . Trn ng thng d1 c 10 im phn bit, trn ng thng d2 c n
im phn bit (n 2). Bit rng c 2800 tam gic c nh l cc im cho. Tm n.

Bi 4.62 : T cc ch s 0, 1, 2, 3, 4, 5, 6 c th lp c bao nhiu s chn, mi s c 5 ch s khc nhau, v mi s lp


c u nh hn 25000.
Bi 4.63 : T cc ch s 0, 1, 2, 3, 4, 5, 6 c th lp c bao nhiu s chn, mi s c 5 ch s khc nhau, trong c
ng 2 ch s l v hai s l ng cnh nhau.
Bi 4.64 : Mt lp hc c 33 hc sinh, trong c 7 n. Cn chia lp hc thnh 3 t, t 1 c 10 hc sinh, t 2 c 11 hc
sinh, t 3 c 12 hc sinh sao cho trong mi t c t nht 2 hc sinh n. Hi c bao nhiu cch chia nh vy.
Bi 4.65 : C bao nhiu s t nhin chn ln hn 2007, m mi s gm 4 ch s khc nhau.
Bi 4.66 : Cho A = {1, 2, 3, 4, 5, 6, 7, 8}.
a) C bao nhiu tp con ca A cha 1 m khng cha 2 ;
b) C bao nhiu s t nhin chn gm 5 ch s khc nhau m khng bt u bi 123 ;
Bi 4.67 : C th lp c bao nhiu s c 8 ch s 1, 2, 3, 4, 5,6 trong 1 v 6 u c mt ng 2 ln, cn cc ch s
khc xut hin ng 1 ln.
Bi 4.68 : a) C bao nhiu s chn gm 6 ch s khc nhau i mt, trong ch s u tin l ch s l ;

TRN ANH TUN - 0974 396 391 - (04) 66 515 343

WWW.VNMATH.COM

Trang 73

www.VNMATH.com

CHUYN LUYN THI I HC

www.luyenthi24h.com
www.luyenthi24h.com
www.VNMATH.com

b) C bao nhiu s chn gm 6 ch s khc nhau i mt, trong c ng 3 ch s l v 3 ch s chn ;


Bi 4.69 : C bao nhiu s t nhin gm 7 ch s, bit rng ch s 2 c mt ng 2 ln, ch s 3 c mt ng 3 ln, cn
cc ch s khc c mt khng qu 1 ln.
Bi 4.70 (B05) : Mt i thanh nin tnh nguyn c 15 ngi, gm 12 nam v 3 n. Hi c bao nhiu cch phn cng i
thanh nin tnh nguyn v gip 3 tnh min ni, sao cho mi tnh c 4 nam v 1 n.
Bi 4.71 (B06) : Cho tp A gm n phn t (n 4). Bit rng s tp con gm 4 phn t ca A bng 20 ln s tp con gm
2 phn t ca A. Tm k {1, 2, . . . , n} sao cho s tp con gm k phn t l ln nht.

Bi 4.72 (D06) : i thanh nin xung kch ca mt trng ph thng c 12 hc sinh, gm 5 hc sinh lp A, 4 hc sinh lp
B v 3 hc sinh lp C. Cn chn 4 hc sinh i lm nhim v, sao cho 4 hc sinh ny thuc khng qu 2 trong 3 lp trn.
Hi c bao nhiu cch chn nh vy.
Bi 4.73 : Trn cc cnh AB, BC, CD, DA ca hnh vung ABCD ln lt ly 1, 2, 3 v n im phn bit khc A, B, C, D.
Tm n, bit s tam gic c 3 nh t n + 6 im cho l 439.

4.2 Gii phng trnh, bt phng trnh, h


Bi 4.74 : Chng minh rng :
a) Pn Pn1 = (n 1)Pn1 ;
b) 1 + P1 + 2P2 + 3P3 + + (n 1)Pn1 = Pn .

n+1 n
.
2
Bi 4.76 : Chng minh rng vi mi n, k N v 2 k < n th :
Bi 4.75 : Chng minh rng vi mi n N c : n!

a) Akn = Akn1 + kAk1


n1 ;

n+1
2 n
b) An+2
n+k + An+k = k An+k ;

Bi 4.77 : Chng minh rng vi mi n N v n 2 th :


1
1
1
n1
+ 2 + + 2 =
.
2
An
n
A2 A3
Bi 4.78 : Cho n, k N v 2 k n. Chng minh rng :
k2
k(k 1)Cnk = n(n 1)Cn2
.

Bi 4.79 : Cho 4 k n. Chng minh rng :


k
Cnk + 4Cnk1 + 6Cnk2 + 4Cnk3 + Cnk4 = Cn+4
.

Bi 4.80 : Chng minh rng, nu k N v 0 k 2008 th :


k
k+1
1004
1005
C2009
+ C2009
C2009
+ C2009
.

Bi 4.81 : Cho mi n, k N v 0 k n. Chng minh rng :


n
n
n
C2n+k
.C2nk
C2n

2

Bi 4.82 : Cho n nguyn dng c nh v k {0; 1; 2; . . . ; n}. Chng minh rng, nu Cnk t gi tr ln nht ti k0 th k0
n1
n+1
tha mn
k0
.
2
2

TRN ANH TUN - 0974 396 391 - (04) 66 515 343

WWW.VNMATH.COM

Trang 74

CHUYN LUYN THI I HC

www.VNMATH.com

www.luyenthi24h.com
www.luyenthi24h.com
www.VNMATH.com

Bi 4.83 : Cho m, n N v 0 < m < n. Chng minh rng :


m1 ;
a) mCnm = nCn1
m1 + C m1 + + C m1 + C m1
b) Cnm = Cn1
m
n2
m1

n+1
Bi 4.84 (B08) : Chng minh rng
n+2

1
k
Cn+1

k+1
Cn+1

1
(n, k l cc s nguyn dng, k n).
Cnk

Bi 4.85 : Chng minh rng :


0
2007
1
2006
k
2007k
2007 0
C2008
.C2008
+ C2008
.C2007
+ + C2008
.C2008k
+ + C2008
.C1 = 1004.22008 .

Bi 4.86 : Cho n l s nguyn dng, chng minh rng :


Cn1 + 2.

C3
Cnn
n(n + 1)
Cn2
+ 3. n2 + + n. n1
=
.
1
Cn
Cn
Cn
2

Bi 4.87 : Cho n l s nguyn dng, chng minh rng :


Cn0
Cn1
Cn2
Cnn
1
+
+
+

+
= .
1
2
3
n+1
Cn+2 Cn+3 Cn+4
C2n+2 2
Bi 4.88 : Chng minh rng :
k , vi 5 k n.
1. C50Cnk + C51Cnk1 + + C55Cnk5 = Cn+5

n = C0
2. C2n
n

+ Cn1

+ + Cnn 2 .

k + C 1C k1 + C 1C k2 + + C k C 0 = C k
3. Cn0Cm
n m
n m
n m
n+m

Bi 4.89 : Tnh S =

0 2
C
n

Bi 4.90 : Chng minh rng :


Bi 4.91 : Chng minh rng :

1 2
C
n

2
1

1
C2009

2 2
C

1
2
C2009

n (1)k
P
k=1

1 + k2

+ +

+ +

1
2009
C2009

Cnn
n+1

2

1005
=
2009

1
1
C2008

1
2
C2008

+ +

1
2008
C2008

n+k < 0.
C2n

Bi 4.92 : Gii cc phng trnh :


1. Cn3 = 5Cn1 ;

2 + nP = 4A2 ;
3. 3Cn+1
2
n

n + C n+2 = 2C n+1 ;
2. C14
14
14

2 A2 4n3 = A1
4. Cn+1
n
2n .

x! (x 1)! 1
= .
(x + 1)!
6
Pn+4
15
Bi 4.94 : Gii bt phng trnh :
<
.
Pn Pn+2 Pn1
Bi 4.93 : Gii phng trnh :

Bi 4.95 : Gii phng trnh : P x A2x + 72 = 6(A2x + 2P x ).


Bi 4.96 : Tm x, y N tha mn h :

8
<A2 + C 3 = 22
x
y
: 3
Ay + C 2x = 66

Bi 4.97 : Gii bt phng trnh : A3x + 5A2x 21x.


Bi 4.98 : Gii bt phng trnh :

n3
Cn1
1
<
.
4
An+1 14P3

TRN ANH TUN - 0974 396 391 - (04) 66 515 343

WWW.VNMATH.COM

Trang 75

www.luyenthi24h.com
www.luyenthi24h.com
www.VNMATH.com

CHUYN LUYN THI I HC

www.VNMATH.com
Bi 4.99 : Gii phng trnh :

1
1
1
x = x.
x
C4 C5 C6

Bi 4.100 : Tm s nguyn dng x tha mn phng trnh :


C 1x + 6C 2x + 6C 3x = 9x2 14.
6
1
Bi 4.101 : Gii bt phng trnh : A22x A2x C 3x + 10.
2
x
Bi 4.102 : Tm s nguyn dng n tha mn iu kin sau :
8
>
<C 4

5
3
Cn1
< A2n2
4
7 3
>
:C n4
A
n+1
15 n+1
n1

Bi 4.103 : Gii h phng trnh :

8
<2Ayx + 5C yx = 90
:

5A x 2C x = 80

Bi 4.104 : Gii h phng trnh :

8
<5C y2
= 3C y1
x
x
: y

C x = C y1
x

Bi 4.105 (D05) : Tnh gi tr ca M =

A4n+1 + 3A3n
2 + 2C 2 + 2C 2 + C 2
, bit rng Cn+1
n+2
n+3
n+4 = 149.
(n + 1)!

Bi 4.106 : Tm s nguyn n > 1 tha mn ng thc : 2Pn + 6A2n Pn A2n = 12.


k
Bi 4.107 : Tm k {1, 2, . . . , 2005} sao cho C2005
t gi tr ln nht.

4.3 H s ca xk trong khai trin


4.4 H s ca xk trong khai trin nh thc (a + b)n
Bi 4.108 (D07) : Tm h s ca x5 trong khai trin thnh a thc ca : x(1 2x)5 + x2 (1 + 3x)10 .

x 4 18
.
+
2 x
Bi 4.110 : Tm h s ca x5 trong khai trin : P = (2x + 1)4 + (2x + 1)5 + (2x + 1)6 + (2x + 1)7 .

n
1
8
n+1 C n
Bi 4.111 (A03) : Tm s hng cha x trong khai trin 3 + x5 , bit : Cn+4
n+3 = 7(n + 3).
x

7
3
Bi 4.112 : Tm s hng hu t trong khai trin nh thc
16 + 3 .

4 124
.
Bi 4.113 : Trong khai trin sau y c bao nhiu s hng hu t :
3 5
Bi 4.109 : Tm s hng c lp vi x trong khai trin

Bi 4.114 (D04) : Tm s hng khng cha x (vi x > 0) trong khai trin

Bi 4.115 : Trong khai trin

28
3
x x + x 15

1
x + 4
x

7

hy tm s hng khng ph thuc vo x, bit rng Cnn + Cnn1 + Cnn2 = 79.

Bi 4.116 : Bit rng tng cc h s ca khai trin (x2 + 1)n bng 1024. Hy tm h s a ca s hng ax12 trong khai trin
.

Bi 4.117 : Tm n > 5, bit trong khai trin x +

1
2

thnh a thc i vi bin x, h s ca x6 bng 4 ln h s ca x4 .

TRN ANH TUN - 0974 396 391 - (04) 66 515 343

WWW.VNMATH.COM

Trang 76

CHUYN LUYN THI I HC

www.VNMATH.com

www.luyenthi24h.com
www.luyenthi24h.com
www.VNMATH.com

Bi 4.118 (A02) : Cho




x1
2

+ 2 2

n

= Cn0 2

x1
2

n

+ Cn1 2

x1
2

n1

2 3 + + Cnn1 2

x1
2

x n1

2 3

+ Cnn 2 3

Bit rng Cn3 = 5Cn1 v s hng th t bng 20n. Tm n v x.

1 2 10
= a0 + a1 x + + a9 x9 + a10 x10 .
Bi 4.119 : Cho
+ x
3 3
Tm s hng ak ln nht.
Bi 4.120 (A08) : Cho khai trin (1 + 2x)n = a0 + a1 x + + an xn , trong n N v cc h s a0 , a1 , . . . , an tha mn
a1
an
a0 +
+ + n = 4096. Tm s ln nht trong cc s a0 , a1 , . . . , an .
2
2


1 18
Bi 4.121 (C08) : Tm s hng khng cha x trong khai trin nh thc Niutn ca 2x + 5
, (x > 0).
x
Bi 4.122 (B07) : Tm s hng cha x10 trong khai trin nh thc Niutn ca (2 + x)n , bit :
3n Cn0 3n1 Cn1 + 3n2 Cn2 3n3 Cn3 + + (1)n Cnn = 2048.
1
3
Bi 4.123 : Tm h s ca x7 trong khai trin a thc (2 3x)2n , trong n l s nguyn dng tha mn : C2n+1
+ C2n+1
+
2n+1 = 1024.
+ C2n+1

Bi 4.124 : Tm h s ca x8 trong khai trin (x2 + 2)n , bit : A3n 8Cn2 + Cn1 = 49.
Bi 4.125 (A06) : Tm h s ca s hng cha

x26

trong khai trin nh thc Niutn ca

2
n
C2n+1
+ + C2n+1
= 220 1.

1
+ x7
x4

1
, bit rng C2n+1
+

4.5 H s ca xk trong khai trin (a + b)n(c + d)m


Bi 4.126 : a) Tm h s ca x2 trong khai trin (2 3x)5 (1 + x)4 .

b) Tm h s ca x3 trong khai trin ( x + 3)6 (1 + x)12 .

Bi 4.127 (D03) : Gi a3n3 l h s ca x3n3 trong khai trin thnh a thc ca (x2 + 1)n (x + 2)n . Tm n a3n3 = 26n.
Bi 4.128 : Tm hng t cha x20 trong khai trin : (1 + x + x3 + x4 )10 .

4.6 H s ca xk trong khai trin (a + b + c)n


Bi 4.129 : a) Tm h s ca x2 trong khai trin thnh a thc ca biu thc : P = (x2 + x 1)6 .
b) Tm h s ca x3 trong khai trin thnh a thc ca biu thc : P = (x2 + x 1)5 .
Bi 4.130 : Tm h s ca x4 trong khai trin (1 + x + 3x2 )10 .

Bi 4.131 (A04) : Tm h s ca x8 trong khai trin 1 + x2 (1 x) .

Bi 4.132 : Tm hng t khng cha x trong khai trin : 1 +

4.7 Tnh tng cc h s t hp :

n
P
k=0

10

6
+x
x

akCnk

4.8 Phng php c bn vi ak ch l hm s m theo bin k


1 + C 3 + + C 2n1 = 2048.
Bi 4.133 (D08) : Tm s nguyn dng n tha mn h thc C2n
2n
2n

TRN ANH TUN - 0974 396 391 - (04) 66 515 343

WWW.VNMATH.COM

Trang 77

CHUYN LUYN THI I HC

www.VNMATH.com

www.luyenthi24h.com
www.luyenthi24h.com
www.VNMATH.com

1 + C 2 + + C 10 .
Bi 4.134 : Tnh tng C20
20
20

Bi 4.135 : a) Khai trin nh thc (3x 1)16 ;


0 315 C 1 + 314 C 2 + C 16 = 216 ;
b) Chng minh rng : 316 C16
16
16
16

Bi 4.136 : Chng minh rng :

a) 2n Cn0 + 2n1 Cn1 + 2n2 Cn2 + + Cnn = 3n ;


b) 3n Cn0 3n1 Cn1 + 3n2 Cn2 + + (1)n Cnn = 2n ;
Bi 4.137 : Chng minh rng :
n1
X
k=1

Bi 4.138 : Chng minh :

0
C2n

2 .32
+ C2n

Cnk = 2(2n1 1);

4 .34
+ C2n

Bi 4.139 : Tnh cc biu thc sau :

n
X

Cnk (1)k = 0.

k=0

2n .32n
+ C2n

= 22n1 (22n + 1).

0 C 2 + + C 16 C 18 .
1. A = C19
19
19
19
1 C 3 + + C 17 C 19 .
2. B = C19
19
19
19
0 3C 2 + 9C 4 27C 6 + + (3)n C 2n .
3. C = C2n
2n
2n
2n
2n

Bi 4.140 (D02) : Tm s nguyn dng n sao cho : Cn0 + 2Cn1 + 4Cn2 + + 2nCnn = 243.

4.9 Phng php o hm vi ak l tch hm s m v a thc theo k


Bi 4.141 : Chng minh rng :
a) Cn1 + 2Cn2 + 3Cn3 + + nCnn = n.2n1 ;
b) Cn1 2Cn2 + 3Cn3 + (1)n1 Cnn = 0 ;
c) 2n1 Cn1 2n1 Cn2 + 3.2n3 Cn3 + (1)n1 nCnn = n ;
Bi 4.142 : Cho (x 2)100 = a0 + a1 x + a2 x2 + + a100 x100 . Tnh :
a) a97 ;
b) S = a0 + a1 + + a100 ;
c) M = a1 + 2a2 + 3a3 + + 100a100 ;
Bi 4.143 : Cho f (x) = (1 + x)n vi n 2.
a) Tnh f (1) ;
b) Chng minh : 2.1.Cn2 + 3.2.Cn3 + 4.3.Cn4 + + n(n 1)Cnn = n(n 1)2n2 .
Bi 4.144 : Chng minh : 2n1 Cn1 + 2n1 Cn2 + 3.2n3 Cn3 + 4.2n4 Cn4 + + nCnn = n.3n1 .

Bi 4.145 : Chng minh : Cn1 .3n1 + 2Cn2 .3n2 + 3Cn3 .3n3 + + nCnn = n.4n1 .

Bi 4.146 : Tnh A = Cn1 2Cn2 + 3Cn3 4Cn4 + + (1)n1 nCnn .

TRN ANH TUN - 0974 396 391 - (04) 66 515 343

WWW.VNMATH.COM

Trang 78

CHUYN LUYN THI I HC

www.VNMATH.com

www.luyenthi24h.com
www.luyenthi24h.com
www.VNMATH.com

Bi 4.147 : Chng minh vi n N v n > 2 th :

1 1
Cn + 2Cn2 + 3Cn3 + + nCnn < n!
n

Bi 4.148 : Chng minh rng :


a) 1.2Cn2 + 2.3Cn3 + + (n 1)nCnn = n(n 1)2n2 ;
b) 1.2Cn2 2.3Cn3 + + (1)n2 (n 1)nCnn = 0 ;
c) 2n1 Cn2 + 3.2n2 Cn3 + 3.4.2n4 Cn4 + + (n 1)nCnn = n(n 1)3n2 ;
d) 2n1 Cn2 3.2n2 Cn3 + 3.4.2n4 Cn4 + (1)n2 (n 1)nCnn = n(n 1) ;
Bi 4.149 : Chng minh rng :
a) 3Cn0 + 4Cn1 + + (n + 3)Cnn = 2n1 (6 + n) ;
b) 3Cn0 4Cn1 + + (1)n (n + 3)Cnn = 0 ;
Bi 4.150 (A05) : Tm s nguyn dng n sao cho :
1
2
3
4
2n+1
C2n+1
2.2.C2n+1
+ 3.22 .C2n+1
4.23 .C2n+1
+ + (2n + 1)22n C2n+1
= 2005.

Bi 4.151 : p dng khai trin nh thc Niutn ca (x2 + x)100 , chng minh rng :
99
0
100C100

1
2

100
1
101C100

1
2

199
100
+ + 200C100

1
2

= 0.

4.10 Phng php tch phn vi ak l tch hm s m v phn thc theo k


Bi 4.152 : Cho n N v n 2.
a) Tnh I =

R1 2
x (1 + x3 )n dx ;
0

1
1
1
1
2n+1 1
b) Chng minh : Cn0 + Cn1 + Cn2 + +
Cnn =
.
3
6
9
3(n + 1)
3(n + 1)
Bi 4.153 : Chng minh :

Cnk
2n+1 1
=
.
n+1
k=0 k + 1
n
P

22 1 1 23 1 2
2n+1 1 n
Cn +
Cn + +
C .
2
3
n+1 n
1
1
(1)n n+1 n 1 + (1)n
Bi 4.155 : Chng minh rng : 2.Cn0 .22 .Cn1 + .23 .Cn2 + +
.2 .Cn =
.
2
3
n+1
n+1
Bi 4.156 : Chng minh rng :
Bi 4.154 (B03) : Tnh Cn0 +

1
1
(1)n
a) (1)n Cn0 + (1)n1 Cn1 + +
Cnn =
;
2
n+1
n+1
1
1
1
b) Cn0 Cn1 + + (1)n
Cnn =
2
n+1
n+1
Bi 4.157 : a) Tnh

R1
0

x(1 x)19 dx ;

1 0
1 1
1 2
1 18 1 19
b) Rt gn S = C19
C19
+ C19
+ + C19
C19 .
2
3
4
20
21

TRN ANH TUN - 0974 396 391 - (04) 66 515 343

WWW.VNMATH.COM

Trang 79

CHUYN LUYN THI I HC

www.VNMATH.com
Bi 4.158 : a) Tnh

R1
0

www.luyenthi24h.com
www.luyenthi24h.com
www.VNMATH.com

x(1 x2 )n dx ;

1
1
1
(1)n n
1
1
b) Chng minh : Cn0 Cn1 + Cn2 Cn3 + +
Cn =
;
2
4
6
8
2n + 2
2(n + 1)
1
1
1
2n+1 (n2 + n + 2) 2
Bi 4.159 : Chng minh : Cn0 + Cn1 + +
Cnn =
3
4
n+3
(n + 1)(n + 2)(n + 3)
1 1
1 3
1 5
1 2n1 22n 1
Bi 4.160 (A07) : Chng minh rng : C2n
+ C2n
+ C2n
+ + C2n
=
.
2
4
6
2n
2n + 1

4.11 Bi tp tng hp
Bi 4.161 : Trong khai trin a thc sau :
(2x + 1)n (x + 2)n = a2n x2n + a2n1 x2n1 + + a1 x + a0 .
Tm n, bit a2n1 = 160.
2
2n
C2n
C4
C6
C 2n2
C2n
4096
+ 2n + C 2n + + 2n +
. Tm n, bit S =
.
3
5
7
2n 1 2n + 1
13
Bi 4.163 : Khai trin v rt gn biu thc 1 x + 2(1 x)2 + 3(1 x)3 + + n(1 x)n thu c a thc P(x) =
1
7
1
a0 + a1 x + a2 x2 + + an xn . Tnh h s a8 , bit rng n l s nguyn dng tha mn 2 + 3 = .
Cn Cn n
0 +
Bi 4.162 : Cho s nguyn dng n > 4 v S = C2n

Bi 4.164 : Mt t gm 10 hc sinh trong c 4 hc sinh nam v 6 hc sinh n. Chn ngu nhin 5 hc sinh lp nn i
c . Gi X l s hc sinh nam ca i c . Hy lp bng phn phi xc sut ca X.
Bi 4.165 : Trong k thi tuyn sinh nm 2009, trng THPT A c 5 hc sinh gm 3 nam, 2 n cng u vo khoa X ca
mt trng H. S sinh vin u vo khoa X c chia ngu nhin thnh 4 lp. Tnh xc sut mt lp c ng 2 nam
v 1 n ca trng THPT A.
Bi 4.166 : Tm s nguyn dng n tha mn
Cn1 .3 2Cn2 .32 + 3Cn3 .33 + + (1)n nCnn .3n = 33792.
Bi 4.167 : T cc ch s 0, 1, 2, 3, 4, 5 c th lp c bao nhiu s t nhin c 9 ch s sao cho ch s 2 xut hin ng
hai ln, ch s 3 xut hin ng ba ln, cc s khc xut hin ng mt ln.
Bi 4.168 : Tm s cc s t nhin gm 8 ch s phn bit c lp thnh t cc ch s 0, 1, 2, 3, 4, 5, 6, 7, 8, 9 sao cho
trong mi s khng c bt k hai ch s chn no ng cnh nhau.
Bi 4.169 : Tnh tng sau theo n
0
2
4
6
2n
S = C2n
3C2n
+ 9C2n
27C2n
+ + (3)n C2n
.
3
2n1
C2n
C5
(1)n1 C2n
+ 2n + +
.
3
9
3n1
Bi 4.171 : C hai t hc sinh. T th nht gm 8 hc sinh nam, trong c 2 hc sinh Hi Dng, 2 hc sinh Bc Ninh

1
Bi 4.170 : Tnh C2n

v 2 hc sinh Hng Yn. T th hai gm 6 hc sinh n, trong c 2 hc sinh Hi Dng, 2 hc sinh Bc Ninh v 2 hc
sinh Hng Yn. Chn mi t ra 3 hc sinh. Tnh xc sut trong 6 hc sinh c chn ra mi tnh c 1 hc sinh nam v 1
hc sinh n.
Bi 4.172 : Trn cc cnh AB, BC, CD, DA ca hnh vung ABCD ta ly ln lt 1, 2, 3 v 2010 im phn bit khc
A, B, C, D. Tnh s tam gic c to thnh m c cc nh c ly trong tp 2016 im nm trn cc cnh ca hnh vung
(khc cc nh ca hnh vung).

TRN ANH TUN - 0974 396 391 - (04) 66 515 343

WWW.VNMATH.COM

Trang 80

www.VNMATH.com

CHUYN LUYN THI I HC

www.luyenthi24h.com
www.luyenthi24h.com
www.VNMATH.com

Bi 4.173 : C 10 vin bi c bn knh khc nhau, 5 vin bi xanh c bn knh khc nhau v 3 vin bi vng c bn knh
khc nhau. Hi c bao nhiu cch chn ra 9 vin bi c ba mu?
Bi 4.174 : Tnh gi tr ca biu thc sau :
0
2
4
2008
2010
S = C2010
3C2010
+ 32 C2010
+ + 31004 C2010
31005 C2010
.

Bi 4.175 : t (1 x + x2 x3 )4 = a0 + a1 x + a2 x2 + + a12 x12 . Tnh h s a7 .

Bi 4.176 : T cc ch s 1, 2, 3, 4 c th lp c bao nhiu s t nhin c 5 ch s, trong ch s 3 c mt ng 3 ln,


cc ch s cn li c mt khng qu 1 ln. Trong cc s t nhin ni trn, chn ngu nhin mt s, tm xc sut s c
chn chia ht cho 3.
Bi 4.177 : Tm s nguyn dng n, bit
1
nC n
Cn1 2Cn2 3Cn3
2 + 3 + (1)n1 nn = .
2
2
2
2
32

10
1
+ x3
vi x , 0.
x
Bi 4.179 : T su ch s 1, 2, 3, 4, 5, 6 lp c bao nhiu s c nm ch s sao cho trong s c nm ch s c hai

Bi 4.178 : Tm h s ca x10 trong khai trin 1 +

ch s 1 cn cc ch s khc xut hin khng qu mt ln.


Bi 4.180 : C 3 hc sinh lp A, 4 hc sinh lp B, 5 hc sinh lp C. C bao nhiu cch chn ra 4 hc sinh t cc hc sinh
trn mi lp A, B, C u c t nht mt hc sinh c chn.
Bi 4.181 : T cc ch s 0, 1, 2, 3, 6, 7, 8, 9 c th lp c bao nhiu s t nhin c 6 ch s i mt khc nhau trong
phi c mt ch s 7.



nk  1 k
n

1 n P
k
x1
x1
3
Bi 4.182 : Cho
=
, bit n tha mn Cn1 + Cn3 = 2Cn2 v s hng th t trong
2 + 3
Cn
2
2x
2x
k=0
khai trin trn bng 2010n. Xc nh n v x.
Bi 4.183 : Tnh
1. A =

1
2
3
2010
0
21C2010
22 C2010
23 C2010
22010 C2010
20 C2010

+ +
.
1.2
2.3
3.4
4.5
2011.2012

2. B =

0
1
2
3
2010
20 C2010
21C2010
22 C2010
23 C2010
22010 C2010

+ +
.
1
2
3
4
2011

Bi 4.184 : Xp 3 bi c bn knh khc nhau v 3 bi trng c cng bn knh vo mt dy gm 7 trng. Hi :


1. C bao nhiu cch sp xp khc nhau.
2. C bao nhiu cch sp xp sao cho 3 bi xp cnh nhau v ba bi trng xp cnh nhau.
Bi 4.185 : Tnh tng sau

S =

Cn0
1

Cn1
2

+ +

Cnn
n+1

2

Bi 4.186 : Tnh tng S = Cn0 + 2Cn1 + 3Cn2 + + (n + 1)Cnn .

Bi 4.187 : Tm h s ca x8 trong khai trin 1 x4

Bi 4.188 : Cho khai trin

2x + 2 2 x

n

1
x

12

n
X

Cnk 2x

nk 

2 2 x

k

k=0

Tm x, bit tng ca s hng th 3 v th 5 bng 135, cn tng 3 h s ca 3 s hng cui bng 22.

TRN ANH TUN - 0974 396 391 - (04) 66 515 343

WWW.VNMATH.COM

Trang 81

CHUYN LUYN THI I HC

www.VNMATH.com

www.luyenthi24h.com
www.luyenthi24h.com
www.VNMATH.com

Bi 4.189 : Tm s hng ln nht trong khai trin (1 + 0, 2)1000 .


Bi 4.190 : Tm h s ca x8 trong khai trin (x2 2)n , bit A3n + Cn1 = 8Cn2 + 49.

1
2
n
Bi 4.191 : Tm h s ca x6 trong khai trin (x2 x 1)n , bit C2n+1
+ C2n+1
+ + C2n+1
= 220 1.

Bi 4.192 : Tm h s ca x11 trong khai trin (x2 + 2)n (3x2 + 1)n , bit

2n
2n1
2nk
0
C2n
3C2n
+ + (1)k 3k C2n
+ + 32n C2n
= 1024.

1 n
= a0 x3n + a1 x3n5 + a2 x3n10 + . Bit rng ba h s u a0 , a1 , a2 theo
2x2
th t lp lp thnh mt cp s cng. Tnh s hng cha x4 .

Bi 4.193 : Cho khai trin P(x) = x3 +

Bi 4.194 : Tnh S = Cn1

+ 2 Cn2

Bi 4.195 : Chng minh rng

+ 3 Cn3

+ + n Cnn n , vi n l s t nhin l.

12Cn1 + 22Cn2 + + n2 Cnn = n(n + 1)2n2 .


2 + C 4 + + C 2x 22003 1, vi x N .
Bi 4.196 : Gii bt phng trnh C2x
2x
2x

Bi 4.197 : Cho tp A gm n phn t (n > 4). Tm n bit rng trong s cc tp con ca A c ng 16n tp con c s phn
t l l.
Bi 4.198 : Cho tp A c n phn t (n > 7). Tm n bit rng s tp con gm 7 phn t ca A bng hai ln s tp con gm 3
phn t.

WWW.VNMATH.COM

TRN ANH TUN - 0974 396 391 - (04) 66 515 343

WWW.VNMATH.COM

Trang 82

www.luyenthi24h.com
www.luyenthi24h.com

Chng 5

Hm s
5.1 Tnh n iu
Vn 1 : Xt chiu bin thin ca hm s

Xt chiu bin thin ca hm s y = f (x) ta tin hnh cc bc nh sau :


1. Tm tp xc nh D ca hm s;
2. Tnh o hm y = f (x);
3. Tm cc gi tr ca x D f (x) = 0 hoc f (x) khng xc nh (gi l cc im ti hn ca hm s);
4. Lp bng bin thin hoc bng xt du y = f (x) trn tng khong x D;
5. Da vo bng xt du v iu kin ta suy ra cc khong n iu ca hm s.

Bi 5.1 : Xt s bin thin ca cc hm s sau :


1. y = x3 3x2 ;
2. y = x3 2x2 + 18x 1 ;
3. y = x3 3x2 + 24x + 26 ;
4. y = x3 + 3x2 + 3x + 2 ;
5. y = x4 2x2 + 7 ;

10. y =

x+2
;
x1

11. y =

x2 + 2x 1
;
x+2

12. y =

x2 + 4x + 3
;
x+2

4
;
x

14. y = x + 1 x2 ;
13. y = x +

1
6. y = x4 + 2x2 1 ;
4
7. y = x4 + 2x2 3 ;

8. y = x4 6x2 + 8x + 1 ;

15. y =

9. y =

16. y = sin x vi x (0; 2).

2x 1
;
x+1

83

3x2 x3 ;

WWW.VNMATH.COM

www.VNMATH.com

CHUYN LUYN THI I HC

www.luyenthi24h.com
www.luyenthi24h.com
www.VNMATH.com

Bi 5.2 : Chng minh rng hm s :


1. y =

x+1
nghch bin trn mi khong xc nh;
2x 1

x3
2. y =
x2 + x + 5 ng bin trn R;
3
2
3. y = x3 + 6x2 20x + 5 nghch bin trn R;
3

4. y =

4 x2 nghch bin trn [0; 2];

5. y = sin x + x ng bin trn R;


6. y = x3 + x cos x 4 ng bin trn R;
7. y = cos 2x 2x + 3 nghch bin trn R.

Vn 2 : Tm iu kin tham s hm s n iu trn mt min

S dng iu kin cn : Gi s hm s y = f (x) c o hm trn D = (a; b) v f (x) = 0 ti khng qu hu hn gi tr.


1. Hm s y = f (x) n iu tng trn D khi v ch khi f (x) 0 vi mi x D.
2. Hm s y = f (x) n iu gim trn D khi v ch khi f (x) 0 vi mi x D.
Chng ta cc bi ton sau :
Bi ton 1 : Tm cc gi tr ca tham s hm s lun ng bin hoc lun nghch bin trn R hoc trn (; a) v
(a; +).
C s bi ton l nh l sau :
nh l 1 : Cho tam thc bc hai f (x) = ax2 + bx + c (a , 0).

f (x) 0 vi mi x R khi v ch khi

f (x) 0 vi mi x R khi v ch khi

8
<a > 0
:

0.

8
<a < 0
:

0.

Ch : nh l 1 cn ng khi iu kin ca ta khng phi l mi x R m thay bng iu kin vi mi x khc x1 , x2 , . . .

Bi ton 2 : Tm cc gi tr ca tham s hm s lun ng bin hoc lun nghch bin trn (a; b) trong t nht a hoc
b l hu hn.
C s bi ton l nh l sau :
nh l 2 : Cho hm s y = f (x) lin tc trn min D v t GTLN, GTNN tng ng l max f (x), min f (x).

f (x) m vi mi x D khi v ch khi min f (x) m;


f (x) m vi mi x D khi v ch khi max f (x) m.
Mt cch tng qut vi bi ton phng trnh, bt phng trnh c tham s chng ta lm nh sau :

Chuyn v phng trnh, bt phng trnh v dng mt v ch cha n (v tri) v mt v ch cha tham s;
Lp bng bin thin ca hm s v tri (hn ch bng bin thin vi iu kin ca n ang xt);
Tnh u v cui tt c cc mi tn (tnh trc tip hoc qua cc gii hn c bn);
S dng nh l 2.

TRN ANH TUN - 0974 396 391 - (04) 66 515 343

WWW.VNMATH.COM

Trang 84

www.VNMATH.com

CHUYN LUYN THI I HC

www.luyenthi24h.com
www.luyenthi24h.com
www.VNMATH.com

Bi ton 3 : Tm cc gi tr ca tham s bit di ca khong ng bin hoc nghch bin.


Vi bi ton ny ta phi lp bng bin thin v tnh trc tip cc nghim ca y = 0 hoc s dng nh l Vit.

1
Bi 5.3 : Tm m hm s : y = x3 + 2x2 + (2m + 1)x 3m + 2 nghch bin trn R.
3
3
Bi 5.4 : Tm m hm s : y = x + (m 1)x2 + (m2 4)x + 9 ng bin trn R.

x3
+ (m + 1)x2 + 3x + 5. Tm m hm s ng bin trn R.
3
Bi 5.6 : Tm m hm s : y = x3 3x2 + 3mx + 3m + 4 ng bin trn R.
Bi 5.5 : Cho hm s y = (m2 1)

Bi 5.7 : Vi nhng gi tr no ca tham s m th hm s :


y = x3 + (m 1)x2 + (m2 4)x + 9
ng bin trn R.
Bi 5.8 : Cho hm s y = x3 3x2 + 3mx + 3m + 4. Tm m hm s ng bin trn R.

(m + 1)x2 2mx (m3 m2 + 2)


nghch bin trn cc khong xc nh.
xm
Bi 5.10 : Tm m hm s y = x + m sin x ng bin trn R.

Bi 5.9 : Tm m hm s : y =

Bi 5.11 : Tm m hm s y = 3 sin x 4 cos x mx + 1 ng bin trn R.


Bi 5.12 : Tm m hm s y = x + m(sin x + cos x) ng bin trn R.
Bi 5.13 : Tm m h y = (2m + 3) sin x + (2 m)x ng bin trn R.
Bi 5.14 : Tm m hm s y = (m 3)x (2m + 1) cos x nghch bin trn R.

Bi 5.15 : Xc nh k hm s y =

(k2

x2
2k) + kx + 3 x ng bin trn tp xc nh.
3

x2 + mx 1
c hai khong ng bin trn ton min xc nh ca n.
x1
(m + 1)x2 2mx 3m3 + m2 2
Bi 5.17 : Cho hm s y =
. Tm m sao cho hm s nghch bin trn tng khong xc
xm
nh.
Bi 5.16 : Tm m hm s y =

Bi 5.18 : Chng minh rng hm s :


y = x3 (m + 1)x2 (2m2 3m + 2)x + 2m(2m + 1)
khng th lun ng bin
Bi 5.19 : Cho hm s y = x3 3x2 + mx + 4. Tm tt c cc gi tr ca tham s m hm s nghch bin trn khong
(0; +).

Bi 5.20 : Tm a sao cho hm s :


1. y = x2 (a x) a tng trong khong (1; 2).

2. y = x3 + (a 1)x2 + (a + 3)x tng trong khong (0; 3).

Bi 5.21 : Cho hm s : y = x3 + 3x2 + (m + 1)x + 4m. Vi nhng gi tr no ca m th hm s nghch bin trn khong
(1; 1).
Bi 5.22 : Tm m hm s : y = (m + 1)x3 2mx2 + x ng bin trn khong (0; 1).

TRN ANH TUN - 0974 396 391 - (04) 66 515 343

WWW.VNMATH.COM

Trang 85

CHUYN LUYN THI I HC

www.VNMATH.com
Bi 5.23 : Cho hm s : y =

(0; 3).

www.luyenthi24h.com
www.luyenthi24h.com
www.VNMATH.com

x3
+ (m 1)x2 + (m + 3)x 4. Tm tt c cc gi tr ca m hm s ng bin trn khong
3

Bi 5.24 : Tm m hm s : y = x2 (m x) m ng bin trn khong (1; 2).


1
Bi 5.25 : Cho hm s : y = x3 mx2 + (2m 1)x m + 2. Vi gi tr no ca m th hm s nghc bin trn khong
3
(2; 0).
Bi 5.26 : Cho hm s : y = 2x3 + 3mx2 2m + 1. Vi gi tr no ca m th hm s nghch bin trn khong (1; 2).

Bi 5.27 : Xc nh m hm s : y = x3 3(2m + 1)x2 + (12m + 5)x + 2 ng bin trn c hai khong (; 1) v (2; +).
1
Bi 5.28 : Cho hm s y = x3 mx2 + (2m 1)x m + 2.
3
Vi nhng gi tr no ca m th hm s cho nghch bin trn (2; +).
m
1
Bi 5.29 : Tm m : y = x3 (m 1)x2 + 3(m 2)x + ng bin trn khong (2; +).
3
3
1 3
Bi 5.30 : Tm m : y = x (m + 1)x2 + m(m + 2)x + 7 ng bin trn [4; 9].
3
x+3
Bi 5.31 : Cho hm s y =
. Tm m sao cho hm s :
xm
1. tng trn (1; +) ;
Bi 5.32 : Cho hm s : y =

2. gim trn (; 2).


x2 2mx + 3m2
.
x 2m

1. Tm m hm s ng bin trn cc khong xc nh.

2. Tm m hm s ng bin trong khong (1; +).

2x2 + (1 m)x + 1 + m
. Xc nh m hm s nghch bin trn (2; +).
x + m
2x2 + kx + 2 k
Bi 5.34 : Tm k hm s y =
ng bin trn khong (1; +).
x+k1
x2 (m + 1)x + 4m2 4m 2
Bi 5.35 : Cho hm s y =
. Xc nh m sao cho hm s ng bin trn khong (0; +).
x (m 1)

Bi 5.33 : Cho hm s : y =

2x2 3x + m
. Vi nhng gi tr no ca m th hm s cho ng bin trn khong (3; +).
x1
x2 2mx + 2 + m
Cho hm s : y =
. Vi nhng gi tr no ca m th hm s cho ng bin trn (1; +).
xm
2x2 + (1 m)x + 1 + m
Tm m : y =
ng bin trn (1; +).
xm
mx2 + x + m
Cho hm s : y =
. Tm tt c cc gi tr ca m hm s ng bin trn khong (0; +).
mx + 1
mx2 + 6x 2
Tm m : y =
nghch bin trn (1; +).
x+2
Tm m hm s : y = x3 + 3x2 + (m + 1)x + 4m nghch bin trn (1; 1).

Bi 5.36 : Cho hm s y =
Bi 5.37 :
Bi 5.38 :
Bi 5.39 :
Bi 5.40 :
Bi 5.41 :

Bi 5.42 : Tm m hm s : y = x3 3(2m + 1)x2 + (12m + 5)x + 2 ng bin trn cc khong (; 1] v [2; +).
m
Bi 5.43 : Tm m hm s : y = x3 + 2(m 1)x2 + (m 1)x + m ng bin trn cc khong (; 0] v [2; +).
3
Bi 5.44 : Tm m hm s : y = x3 6mx2 + 2(12m 5)x + 1 ng bin trn cc khong (; 0) v (3; +).
m1 3
Bi 5.45 : Tm m hm s : y =
x + mx2 + (3m 2)x ng bin trn R.
3
Bi 5.46 : Tm m hm s : y = x3 mx2 (2m2 7m + 7)x + 2(m 1)(2m 3) ng bin trn [2; +).
Bi 5.47 : Tm m hm s :

TRN ANH TUN - 0974 396 391 - (04) 66 515 343

WWW.VNMATH.COM

Trang 86

www.VNMATH.com

CHUYN LUYN THI I HC

www.luyenthi24h.com
www.luyenthi24h.com
www.VNMATH.com

2 3
x + (m + 1)x2 + (m2 + 4m + 3)x m2 ng bin trn [1; +).
3
Bi 5.49 : Tm m hm s : y = x3 (m + 1)x2 (2m2 3m + 2)x + 1 ng bin trn [2; +).

Bi 5.48 : Tm m hm s : y =

Bi 5.50 : Tm m hm s : y = x3 3(m 1)x2 + 3m(m 2)x + 1 ng bin trn cc on [2; 1] v [1; 2].

1
Bi 5.51 : Tm m hm s : y = x3 2x2 + mx 1 ng bin trn 0; .
3
2
2x 3x + m
Bi 5.52 : Tm m hm s : y =
ng bin trn (3; +).
x1

2x2 3x + m
1
Bi 5.53 : Tm m hm s : y =
nghch bin trn ; + .
2x + 1
2
2
mx (m + 1)x 3
Bi 5.54 : Tm m hm s : y =
ng bin trn [4; +).
x
(2m 1)x2 3mx + 5
ng bin trn [2; 5].
Bi 5.55 : Tm m hm s : y =
x1
x2 2mx + 3m2
Bi 5.56 : Tm m hm s : y =
ng bin trn (1; +).
x 2m
x2 2mx + m + 2
Bi 5.57 : Tm m hm s : y =
ng bin trn (1; +).
xm
2x2 + mx + 2 m
ng bin trn (1; +).
Bi 5.58 : Tm m hm s : y =
x+m1
x2 8x
Bi 5.59 : Tm m hm s : y =
ng bin trn (1; +).
8(x + m)
Bi 5.60 : Cho hm s y = x3 + 3x2 + mx + m. Tm m hm s nghch bin trn khong c di bng 3.
Bi 5.61 : Cho hm s y = x3 + 3x2 + mx + m. Tm m hm s nghch bin trn khong c di bng 1.
Bi 5.62 : Tm cc gi tr ca m hm s y = x3 + 6x2 + mx + 5 ng bin trn khong c di bng 1.

Vn 3 : Gi tr ln nht, gi tr nh nht ca hm mt bin s

Bi ton 1 : Tm GTLN, GTNN ca hm s y = f (x) trn on [a; b].


1. Tnh y = f (x), gii phng trnh f (x) = 0 c cc nghim xi [a; b].
2. Tnh y(a) = f (a), y(b) = f (b), y(xi ) = f (xi ).
3. GTLN trong cc gi tr trn l max f (x), GTNN trong cc gi tr trn l min f (x).
x[a;b]

x[a;b]

Bi ton 2 : GTLN, GTNN ca hm s y = f (x) trn min D tng qut.


1. Lp bng bin thin ca hm s y = f (x) ch xt vi x D.
2. Tnh cc gi tr du v cui mi tn (tnh trc tip hoc qua cc gii hn c bn).
3. Cn c bng bin thin ta c c GTLN, GTNN (nu c) ca hm s.

3x 1
trn [0; 2].
x3
Bi 5.64 : Tm gi tr ln nht, gi tr nh nht ca cc hm s sau:

Bi 5.63 : Tm gi tr ln nht, b nht ca hm s y =

TRN ANH TUN - 0974 396 391 - (04) 66 515 343

WWW.VNMATH.COM

Trang 87

www.VNMATH.com

www.luyenthi24h.com
www.luyenthi24h.com
www.VNMATH.com

CHUYN LUYN THI I HC

9. y = x +

2. y = 1 + 4x x2 ;

10. y = x +

3. y = x4 2x2 + 5 (x [2; 3]);

x+1
11. y =
trn [1; 2];
x2 + 1

x2 + x + 1
1. y =
x

4. y =

x2+

(x>0);

4 x;

12. y =

2x2 + 4x + 5
;
x2 + 1

6. y = 2x 1 x2 ;
5. y =

4 x2 ;

x

+ sin2 x trn ; ;
2
2 2

13. y = 1 + x + sin x +

1
1
sin 2x + sin 3x trn [0; ];
4
9

14. y = sin x + cos x;

x+3
7. y =
;
x2 + 1
8. y = x +

2 cos x trn 0; ;
2

15. y = 2 sin x + cos 2x;

9
trn [2; 4];
x

16. y = sin5 x +

3 cos x.

Bi 5.65 : Tm gi tr ln nht, gi tr nh nht (nu c) ca cc hm s sau :


1
1. y = sin x cos x + ;
2
2. y = 2 sin x
Bi 5.66 :

4 3
sin x trn [0; ];
3

3. y =

2 cos2 +| cos x| + 1
;
| cos x| + 1

4. y =

3 cos4 x + 4 sin2 x
.
3 sin4 x + 2 cos2 x

1. Tm gi tr ln nht, gi tr nh nht ca hm s y = |x2 + 2x 3| +

3
1
trn ; 4 .
2
2

2. Tm gi tr ln nht ca hm s y = |x3 + 3x2 72x + 90| trn [5; 5].


3. Tm gi tr ln nht, gi tr nh nht ca hm s y = x3 8x2 + 16x 9 trn (1; 3].
Bi 5.67 : Tm GTLN, GTNN ca hm s y = x6 + 4(1 x2 )3 vi x [1; 1]
1
Bi 5.68 : Tm x hm s sau t gi tr nh nht y = f (x) = lg2 x + 2
lg x + 2
ln2 x
, x [1; e3 ].
x
Tm gi tr ln nht, b nht ca hm s y = x4 2x2 + 3 trn [3; 2].

Tm gi tr ln nht, b nht ca hm s y = x + cos2 x trn 0; .


4

Tm gi tr ln nht, b nht ca hm s y = x 1 + 3 x.
2x
4x
Tm gi tr ln nht, b nht ca hm s y = sin
+ cos
+ 1.
2
1+x
1 + x2
Cho x, y l cc s khng m c tng bng 1. Tm gi tr ln nht v gi tr nh nht ca S = x3 + y3 + xy.

Bi 5.69 : Tm GTLN, GTNN ca y =


Bi 5.70 :
Bi 5.71 :
Bi 5.72 :
Bi 5.73 :
Bi 5.74 :

Bi 5.75 : Cho x, y l hai s khng m, tha mn xy + x + y = 3. Tm gi tr ln nht, gi tr nh nht ca S = x3 + y3 +


x2 y + xy2 5xy.

5
4
1
Bi 5.76 : Gi s x, y l hai s dng tho mn iu kin x + y = . Tm gi tr nh nht ca biu thc S = +
4
x 4y
Bi 5.77 : Cho x, y tha mn x 0, y 0, x + y = 1. Tm GTLN, GTNN ca
P=

TRN ANH TUN - 0974 396 391 - (04) 66 515 343

x
y
+
.
y+1 x+1
WWW.VNMATH.COM

Trang 88

www.VNMATH.com

CHUYN LUYN THI I HC

www.luyenthi24h.com
www.luyenthi24h.com
www.VNMATH.com

Bi 5.78 : Cho x, y, z l cc s dng tho mn


1 1 1
+ + =4
x y z
Tm GTLN ca
1
1
1
+
+
2x + y + z x + 2y + z x + y + 2z
Bi 5.79 : Cho x, y, z > 0 tho mn xyz = 1 Tm GTNN ca

1 + x3 + y3
1 + y3 + z3
1 + z3 + x3
+
+
xy
yz
zx
Bi 5.80 : Cho x, y, z > 0 v x + y + z = 1. Tm GTLN
y
z
x
+
+
x+1 y+1 z+1
Bi 5.81 : Cho a, b, c > 0 v a + b + c = 1. Tm GTNN

1
a+
a

1
b+
b

1
c+
.
c

Vn 4 : S dng tnh n iu chng minh bt ng thc

Bi ton 1 : Bt ng thc mt bin.

a bt ng thc v dng f (x) c vi mi x D.


Xt hm s y = f (x) vi x D.
Lp bng bin thin ca hm s y = f (x) vi x D.
T bng bin thin ta c kt lun bi ton.
Bi ton 2 : Bt ng thc "phn" i xng hai bin a v b vi a b (tng t a b).

a bt ng thc v dng f (a) f (b).


S dng nh ngha v tnh n iu : Gi s y = f (x) xc nh trn D = (a; b) v x1 < x2 thuc khong
(i) y = f (x) ng bin trn D th f (x1 ) < f (x2 );
(ii) y = f (x) nghch bin trn D th f (x1 ) > f (x2 ).
Bi ton 3 : Bt ng thc i xng hai bin a v b.

Bin i bt ng thc v dng f (a, b) c hoc f (a, b) c vi c l hng s (thng a v trng hp c = 0). Quay
v bi ton tm max f (a, b) hoc min f (a, b).

t S = a + b v P = ab vi (S 2 4P), t cc iu kin rng buc ta a f (a, b) theo S (hoc P) v tm min rng


buc cho S v P tng ng.

T ta quay v bi ton tm max, min ca hm mt bin s

Bi 5.82 : Cho hm s y = f (x) = tan x + sin x 2x. Chng minh rng

TRN ANH TUN - 0974 396 391 - (04) 66 515 343

WWW.VNMATH.COM

Trang 89

CHUYN LUYN THI I HC

www.VNMATH.com

1. hm s ng bin trn 0;
.
2

www.luyenthi24h.com
www.luyenthi24h.com
www.VNMATH.com

2. sin x + tan x > 2x vi mi x 0;


.
2

Bi 5.83 : Chng minh rng

4
1

1
2.
.
< 2 + 1 2 vi x 0;
2

2
sin x x

2x

vi x 0;
1. sin x >
;

Bi 5.84 : Cho hm s y = f (x) = tan x + 2 sin x 3x.Chng minh rng

1. hm s ng bin trn 0;
Bi 5.85 :

.
2

2. 2 sin x + tan x > 3x vi mi x 0;

.
2

1. Chng minh rng tan x > x vi mi x 0;


.
2

x3

vi mi x 0;
.
3
2
4x
tan x.
Bi 5.86 : Cho hm s y = f (x) =

2. Chng minh rng tan x > x +

.
4

4x

tan x vi mi 0; .
2. Chng minh rng

n
n
n
n
n
n
Bi 5.87 : Cho n l s nguyn dng, chng minh rng 1 +
+ 1
< 2.
n
n
Bi 5.88 : Chng minh rng
1. Xt chiu bin thin ca hm s y = f (x) trn 0;

1. sin x x vi mi x 0;

;
2

3. cos x < 1

x3

2. sin x > x
vi mi x 0;
;
3!
2

4.

sin x
x

x2 x4

+
vi mi x 0;
;
2
24
2

> cos x vi mi x 0;
.
2

Bi 5.89 : Chng minh rng


1. ex 1 + x vi mi x R;
Bi 5.90 :

2. ex 1 + x +

x2
vi mi x 0.
2

1. Cho a < b, chng minh rng sin a sin b < b a;

2. Chng minh rng sin 2010 sin 2009 + 1 < 0.

tan x

Bi 5.91 : Chng minh rng hm s y = f (x) =


ng bin trn 0;
. T suy ra 4. tan . tan
< 3. tan . tan .
x
4
36
20
30
18
Bi 5.92 : Chng minh rng vi 0 < < <

sin
6 ta c
>
sin

3
6
3
6

x2
vi mi x 0.
2
Bi 5.94 : Tm s thc a nh nht bt ng thc ln(1 + x) x ax2 ng vi mi x 0.
Bi 5.93 : Chng minh rng ln(1 + x) x

Bi 5.95 : Tm tt c cc s thc dng a ax 1 + x vi mi 0.

1 b
1 a
2b + b .
a
2
2
x
x
y
y
y
x
Bi 5.97 : Chng minh rng (2 + 3 ) < (2 + 3 ) vi mi x > y > 0.

b
x + a x+b
a
>
.
Bi 5.98 : Cho x, a, b > 0 v a , b. Chng minh rng
x+b
b
Bi 5.99 : Chng minh rng x > ln(1 + x) vi mi x > 0.

Bi 5.96 : Cho a b > 0. Chng minh rng 2a +

TRN ANH TUN - 0974 396 391 - (04) 66 515 343

WWW.VNMATH.COM

Trang 90

www.VNMATH.com

CHUYN LUYN THI I HC

www.luyenthi24h.com
www.luyenthi24h.com
www.VNMATH.com

Bi 5.100 : Chng minh rng vi x (4; +) ta lun c 2x > x2 .

Vn 5 : ng dng s bin thin vo vic gii phng trnh, bt phng trnh, h


1. Nu f (x) c v g(x) c th phng trnh f (x) = g(x) tng ng vi

8
< f (x) = c
:

g(x) = c.

2. Nu hm s y = f (x) lun ng bin (hoc nghch bin) trn (a; b) th phng trnh f [u(x)] = f [v(x)] tng ng
vi u(x) = v(x).
3. Nu hm s y = f (x) ng bin trn (a; b) v hm s y = g(x) l hm s nghch bin trn (a; b) th phng trnh
f (x) = g(x) nu c nghim th nghim l duy nht.
4. Nu hm s y = f (x) ng bin (hoc nghch bin) trn (a; b) th phng trnh f (x) = c (vi c l hng s) nu c
nghim th nghim l duy nht.
5. Nu hm s y = f (x) ng bin trn (a; b) th bt phng trnh f (u) f (v) tng ng vi u v.
6. Nu hm s y = f (x) nghch bin trn (a; b) th bt phng trnh f (u) f (v) tng ng vi u v.

Bi 5.101 : Gii cc phng trnh


1.

2.

3x + 1 +

x+

5x3 1 +

7x + 2 = 4;

3
2x 1 + x = 4;

3.

x+2+

x+1=

4.

x+1+

x+2+

2x2 + 1 +

2x2 ;

x + 3 = 0.

Bi 5.102 : Gii bt phng trnh


1.

5x 1 +

x + 3 4;

5
2. 3 3 2x +
2x 6;
2x 1

3. (x + 2)(2x 1) 3 x + 6 4 (x + 6)(2x 1) + 3 x + 2;
4.

5.

2x3 + 3x2 + 6x + 16 < 2 3 + 4 x;


x+9+

2x + 4 > 5.

Bi 5.103 : Gii cc h phng trnh


8
>
<

1
1
=y
x
y
1.

>

: x + y y = 2.
x

2.

8
< x3 3x = y3 3y
: 6
x + y6 = 1.

3.

< 2x + 1 2y + 1 = x y
: 2
x 12xy + 9y2 + 4 = 0.

Bi 5.104 : Gii phng trnh : x5 + x3 1 3x + 4 = 0.

Bi 5.105 : Gii phng trnh : x2 + 15 = 3x 2 + x2 + 8.

TRN ANH TUN - 0974 396 391 - (04) 66 515 343

WWW.VNMATH.COM

Trang 91

CHUYN LUYN THI I HC

www.VNMATH.com

www.luyenthi24h.com
www.luyenthi24h.com
www.VNMATH.com

3
4
5
x + 1 + 5x 7 + 7x 5 + 13x 7 < 8.

Bi 5.107 : Gii bt phng trnh : 2x + x + x + 7 + 2 x2 + 7x < 49.


1
1
1
Bi 5.108 : Gii phng trnh : 5x + 4x + 3x + 2x = x + x + x 2x3 + 5x2 7x + 17.
2
3
6
Bi 5.106 : Gii bt phng trnh :

Bi 5.109 : Tm x, y (0; ) tha mn h :

8
<cot x cot y = x y
:

5x + 8y = 2.

Vn 6 : ng dng s bin thin vo bi ton s nghim phng trnh c tham s


Bc 1 : t n ph (nu cn) t = u(x), t iu kin cht cho t.
Bc 2 : T gi thit bi ton bin i v mt trong cc dng sau:
f (t) = g(m); f (t) g(m); f (t) g(m); f (t) > g(m); f (t) < g(m).
Tc l bin i c lp m v mt v, cn v kia c lp vi m.
Bc 3 : Lp bng bin thin ca hm s f (t) trn min gi tr ca t tm c sau bc 1.
Bc 4 : T bng bin thin suy ra min gi tr ca f (t). S dng cc kt qu nu mc 2, tm ra kt lun ca bi
ton.
Ch : iu kin cht cho t c ngha l tm cc gi tr ca t khi x bin thin phng trnh t = u(x) c nghim.
1
Chng hn, nu t t = 3x th iu kin t > 0, nhng vn t t = 3x , x [1; 1] th iu kin
t 3 v nu t
3

2
t = u(x) = 3 x +2x , x [0; 2] iu kin cht ca t phi l 1 t 3.

Bi 5.110 : Cho hm s : y = mx2 + 2mx 3.


1. Tm m phng trnh f (x) = 0 c nghim trong on [1; 2].
2. Tm m bt phng trnh f (x) 0 nghim ng vi mi x trong on [1; 3].
3. Tm m bt phng trnh f (x) 0 nghim ng vi mi x trong on (1; 4).
Bi 5.111 : Tm m phng trnh :
2(sin4 x + cos4 x) + cos 4x + 2 sin 2x + m = 0

.
2
Bi 5.112 : Tm m phng trnh sau c nghim :

c t nht mt nghim thuc on 0;

2x2 2(m + 4)x + 5m + 10 + 3 x = 0.

Bi 5.113 : Tm m phng trnh :

2 + 2 sin 2x = m(1 + cos x)2


c nghim trn on ; .
2 2

TRN ANH TUN - 0974 396 391 - (04) 66 515 343

WWW.VNMATH.COM

Trang 92

www.VNMATH.com

CHUYN LUYN THI I HC

Bi 5.114 : Tm m phng trnh

www.luyenthi24h.com
www.luyenthi24h.com
www.VNMATH.com

x2 + mx + 2 = 2x + 1 c hai nghim thc phn bit.


4

Bi 5.115 : Tm m phng trnh 3 x 1 + m x + 1 = 2 x2 1 c nghim.


Bi 5.116 : Vi gi tr no ca m bt phng trnh sau ng vi mi x [5; 1]
4

54xx2

+ 21+

54xx2

m.

Bi 5.117 : Cho phng trnh 9x m3x + 2m = 0.


1. Gii phng trnh vi m = 1;

2. Tm m phng trnh trn c nghim.

Bi 5.118 : Tm GTLN, GTNN ca hm s


y=

1 + sin x +

cos6 x + sin6 x
= m tan 2x.
cos2 x sin2 x
13
1. Gii phng trnh khi m = ;
8

1 + cos x.

Bi 5.119 : Cho phng trnh

2. Tm m phng trnh v nghim.

Bi 5.120 : Tm cc gi tr ca m phng trnh c nghim thuc (0; 1)


4(log2

x)2 log 1 x + m = 0.
2

Bi 5.121 : Tm m bt phng trnh sau c nghim :


4x m.2x m + 3 0

Bi 5.122 : Tm m phng trnh sau c t nht mt nghim thuc on 0;

2(sin4 x + cos4 x) + cos 4x + 2 sin 2x + m = 0.


Bi 5.123 : Tm a phng trnh sau y c 4 nghim phn bit :

Bi 5.124 : Tm m :

2|x2 5x + 4| = x2 5x + a.

18 + 3x x2 m2 m + 1 nghim ng vi mi x [3; 6].

Bi 5.125 : Tm a bt phng trnh : x3 + 3x2 1 a( x x 1)3 c nghim.

Bi 5.126 : Tm a : x x + x + 12 = m( 5 x + 4 x) c nghim.
3+x+

6x

Bi 5.127 : Tm m phng trnh sau c nghim

m( 1 + x2 1 x2 + 2) = 2 1 x4 + 1 + x2 1 x2
Bi 5.128 : Cho phng trnh :
log23 x +

log23 x + 1 2m 1 = 0.

1. Gii phng trnh khi m = 2;

2. Tm m phng trnh c t nht mt nghim thuc on [1; 3 3 ].

Bi 5.129 : Tm m phng trnh 3 m x + x + 2 = 1 c ng 3 nghim thc phn bit.

5.2 Cc tr ca hm s
TRN ANH TUN - 0974 396 391 - (04) 66 515 343

WWW.VNMATH.COM

Trang 93

www.VNMATH.com

CHUYN LUYN THI I HC

www.luyenthi24h.com
www.luyenthi24h.com
www.VNMATH.com

Vn 1 : S dng du hiu 1 v du hiu 2 xc nh cc im cc tr ca hm s


Bi 5.130 : Xc nh cc im cc tr ca cc hm s sau :

1. y = 2x + 3 x2 + 1;

4. y = x3 (1 x2 );

7. y = |x|(x 2);
8. y = cos x +

2. y =

3x + 14
;
(x 2)(x + 3)

5. y = 1 2 4x x2 ;

3. y =

2x2 + 3x + 1
;
x2 4x + 3

6. y =

x2
1

x2

9. y =

1
cos 2x;
2

3 sin x + cos x +

2x + 3
.
2

Bi 5.131 : Tm cc tr ca cc hm s :

1. y = x 3 x ;
2. y =

4. y = x2 2|x| + 2 ;

x
;
x2 + 4

5. y = xe3x ;
6. y =

3. y = 3x4 4x3 24x2 + 48x 3 ;


Bi 5.132 : Tm cc tr ca hm s y = sin2 x

x
;
ln x

3 cos x, x [0; ].

Bi 5.133 : Cho m l s nguyn dng, hy tm cc tr ca hm s : y = xm (4 x)2 .

Vn 2 : iu kin ca tham s hm s t cc tr (cc i hoc cc tiu) ti x = x0 hoc th hm s


t cc tr ti im (x0 ; y0 )

Chng ta lm theo phng php iu kin cn v :

Bc 1 : Gi s hm s t cc tr ti x = x0 suy ra f (x0 ) = 0, tm c tham s m.


Bc 2 : Vi tng gi tr ca m va tm c, th li xem x0 c ng l im cc tr (cc i hoc cc tiu).
Bc 3 : Kt lun.
Ch :

C th dng du hiu 2 kim t ti bc 2;


Mt s li gii sai lm l dng
trc tip du hiu 2 gii bi ton trn, chng hn hm s t cc i ti
8

< f (x0 ) = 0
x = x0 khi v ch khi
l li gi sai lm, v d hm s y = x4 t cc tiu ti x = 0, nhng
:
f (x0 ) < 0,

f (0) = 0 ch khng phi l f (0) < 0.

Bi 5.134 : Tm m hm s : y = x3 3mx2 + (m2 1)x + 2 t cc tiu ti x = 2.

Bi 5.135 : Xc nh cc s a, b, c hm s : y = x3 + ax2 + bx + c c gi tr 0 khi x = 1 v t cc tr 0 khi x = 1.

TRN ANH TUN - 0974 396 391 - (04) 66 515 343

WWW.VNMATH.COM

Trang 94

www.VNMATH.com

CHUYN LUYN THI I HC

www.luyenthi24h.com
www.luyenthi24h.com
www.VNMATH.com

Bi 5.136 : Xc nh hm s bc ba y = f (x), bit rng n c cc tiu 2 khi x = 1 v nu em chia f (x) cho x2 + 3x + 2


th cn d x + 3.

Bi 5.137 : Cho hm s : y = x3 (m + 3)x2 + mx + m + 5. Tm m hm s t cc tiu ti x = 2.

Bi 5.138 : Cho hm s y = (x m)3 3x. Tm m hm s cho t cc tiu ti x = 0.


5
5
Bi 5.139 : Tm a v b cc cc tr ca hm s y = a2 x3 + 2ax2 9x + b u l nhng s dng v x0 = l im
3
9
cc i.
Bi 5.140 : Cho hm s y = x3 3mx2 + 3(m2 1)x (m2 1). Tm m hm s t cc i ti x = 1.
1
Bi 5.141 : Tm m hm s : y = x3 + (m2 m + 2)x2 + (3m2 + 1)x + m 5 t cc tiu ti x = 2.
3
1

Bi 5.142 : Cho hm s y = a sin x + sin 3x. Tm a hm s t cc tr ti x = .


3
3
2
x + mx + 1
Bi 5.143 : Cho hm s y =
. Tm m hm s t cc i ti x = 2.
x+m
ax2 + bx + ab
Bi 5.144 : Tm a, b hm s y =
t cc tiu ti x = 0 v cc i ti x = 4.
bx + a
1
Bi 5.145 : Tm a, b hm s y = x4 ax2 + b c gi tr cc tr bng 2 khi x = 1.
4

Vn 3 : Tm iu kin hm s c cc tr v tha mn mt vi iu kin

1. Cc tr hm bc 3 : y = ax3 + bc2 + cx + d (a , 0)
Hm s c cc tr (hoc hai im cc tr) khi v ch khi phng trnh y = 0 c hai nghim phn bit. Khi honh hai
im cc tr l nghim ca phng trnh y = 0.
2. Cc tr hm bc 4 : y = ax4 + bx3 + cx2 + dx + e (a , 0)
- Hm s c ba im cc tr khi v ch khi phng trnh y = 0 c 3 nghim phn bit;
- Khi vit c y = (x x0 )P(x) vi P(x) l a thc bc 2, khi hm s c 1 im cc tr khi v ch khi y = 0 i du 1
ln, tng ng vi P(x0 ) = 0 hoc P(x) 0.

Ch : Hm bc 4 c s im cc tiu nhiu hn cc i th a > 0 v s im cc i nhiu hn cc tiu th a < 0.


ax2 + bc + c
3. Cc tr ca hm phn thc : y =
(a v d khc 0)
dx + e
Hm s c cc tr (hoc hai im cc tr) khi v ch khi phng trnh y = 0 c hai nghim phn bit. Khi honh hai
im cc tr l nghim ca phng trnh y = 0.
Ch :
- Vi bi ton yu cu c th im no l im cc i, im no l im cc tiu th ta cn lp bng bin thin xc nh
im cc tr.
- Vi bi ton c vai tr ca im cc i v im cc tiu l nh nhau th ta thng dng nh l Vit.

Bi 5.146 : Chng minh rng hm s y = x3 3mx2 + (m 1)x + 2 c cc tr vi mi gi tr ca m.


1
Bi 5.147 : Tm m hm s : y = x3 + mx2 + (m + 6)x (2m + 1) c cc i, cc tiu.
3

TRN ANH TUN - 0974 396 391 - (04) 66 515 343

WWW.VNMATH.COM

Trang 95

www.VNMATH.com

CHUYN LUYN THI I HC

www.luyenthi24h.com
www.luyenthi24h.com
www.VNMATH.com

Bi 5.148 : Tm m hm s : y = (m + 2)x3 + 3x2 + mx 5 c cc i, cc tiu.

Bi 5.149 : Cho hm s : y = x3 3mx2 + 2. Vi gi tr no ca m th hm s c cc i v cc tiu.

Bi 5.150 : Cho hm s y = mx3 + 3mx2 (m 1)x 1. Vi nhng gi tr no ca m th hm s khng c cc tr.

Bi 5.151 : Chng minh rng : vi mi m, hm s y = 2x3 3(2m + 1)x2 + 6m(m + 1)x + 1 lun t cc tr ti x1 , x2 vi
x2 x1 khng ph thuc vo m.

1 3
x + (m 2)x2 + (5m + 4)x + m2 + 1 t cc tr ti x1 < 1 < x2 .
3
1
Bi 5.153 : Tm m hm s : y = x3 + (m + 3)x2 + 4(m + 3)x + m2 m t cc tr ti x1 , x2 tha mn 1 < x1 < x2 .
3
3
Bi 5.154 : Cho hm s : y = 2x 3(m + 2)x2 + 6(5m + 1)x (4m3 + 2). Tm m hm s c :
Bi 5.152 : Tm m hm s : y =

1. ng mt im cc tr ln hn 1.
2. hai im cc tr nh hn 2.
3. t nht mt im cc tr (1; 1).
4. t nht mt im cc tr ln hn 9.
5. t nht mt im cc tr c gi tr tuyt i ln hn 4.
Bi 5.155 : Cho hm s y = x3 (m 3)x2 + (4m 1)x m. Tm m hm s t cc tr ti cc im x1 , x2 tho mn iu

kin x1 < 2 < x2 .

1 3
1
x (m 1)x2 + 3(m 2)x + .
3
3
Vi gi tr no ca m th hm s c cc i, cc tiu ln lt l x1 , x2 tha mn x1 + 2x2 = 1.

Bi 5.156 : Cho hm s y =

Bi 5.157 : Chng minh rng vi mi a, hm s :


y = 2x3 3(2a + 1)x2 + 6a(a + 1)x + 1
lun t cc tr ti x1 , x2 . Tm a sao cho cc gi tr cc tr tng ng y1 , y2 tha mn y1 + y2 = 1.
Bi 5.158 : Cho hm s : y = mx3 3mx2 + (2m + 1)x + 3 m. Tm tt c cc gi tr ca m hm s c cc i v cc

tiu. Chng minh rng khi ng thng ni hai im cc i v cc tiu lun i qua mt im c nh.
2
Bi 5.159 : Gi s hm s : y = x3 + (cos a 3 sin a)x2 8(cos 2a + 1)x + 1 t cc tr ti x1 , x2 . Chng minh rng :
3
x21 + x22 18 vi mi a.
1
1
3x
Bi 5.160 : Cho hm s : y = x3 (sin a + cos a)x2 +
sin 2a. Tm a hm s t cc tr ti cc im x1 , x2 tha
3
2
4
mn : x1 + x2 = x21 + x22 .
2
Bi 5.161 : Cho hm s : y = x3 + (m + 1)x2 + (m2 + 4m + 3)x.
3
1. Tm m hm s c cc i, cc tiu.
2. Tm m hm s t cc tr ti t nht mt im ln hn 1.
3. Gi cc im cc tr l x1 , x2 . Tm max ca A = |x1 x2 2(x1 + x2 )|.
Bi 5.162 : Tm m th hm s y =
Bi 5.163 : Tm m hm s y =

1 3
x mx2 x + m + 1 c khong cch gia cc im cc tr l nh nht.
3

1 3
x mx2 + mx 1 t cc tr ti x1 , x2 tha mn |x1 x2 | 8.
3

TRN ANH TUN - 0974 396 391 - (04) 66 515 343

WWW.VNMATH.COM

Trang 96

www.VNMATH.com

CHUYN LUYN THI I HC

www.luyenthi24h.com
www.luyenthi24h.com
www.VNMATH.com

Bi 5.164 : Tm a cc im cc tr ca th hm s : y = x3 3ax2 + 4a3 i xng qua ng thng y = x.

Bi 5.165 : Vit phng trnh ng thng i qua im cc i v cc tiu ca th hm s :


y = x3 3(m 1)x2 + 2(m2 3m + 2)x m(m 1).

Bi 5.166 : Cho hm s y = 4x3 mx2 3x + m. Chng minh rng vi mi m hm s lun c cc i, cc tiu, ng thi

honh im cc i, cc tiu ca hm s l tri du.

Bi 5.167 : Cho hm s y = mx3 3mx2 + (2m + 1)x + 3 m. Tm tt c cc gi tr ca m hm s c cc i, cc tiu.


Chng minh rng khi ng thng ni hai im cc i, cc tiu ca th hm s lun i qua mt im c nh.

Bi 5.168 : Cho hm s y = x3 + 2(m 1)x2 + (m2 4m + 1)x 2(m2 + 1). Tm m hm s t cc tr ti hai im x1 , x2


1
1
1
sao cho :
+
= (x1 + x2 ).
x1 x2 2
Bi 5.169 : Cho hm s y = x3 3x2 mx + 2. Tm m hm s c cc i, cc tiu v ng thi hai im cc i, cc
tiu ca th hm s cch u ng thng y = x 1.

Bi 5.170 : Tm m th hm s : y = 2x3 + 3(m 1)x2 + 6m(1 2m)x c cc im cc i, cc tiu nm trn ng


thng y = 4x.

Bi 5.171 : Tm m th hm s : y = x3 + mx2 + 7x + 3 c ng thng i qua hai im cc tr vung gc vi ng


thng y = 3x 7.

Bi 5.172 : Tm m th hm s y = x3 + 3(m 1)x2 + (2m2 3m + 2) m(m 1) c ng thng i qua hai im cc


1
tr to vi ng thng y = x + 5 mt gc 45 .
4
1
5
Bi 5.173 : Tm m th hm s y = x3 3x2 + m2 x + m c cc im cc tr i xng nhau qua ng thng y = x .
2
2
Bi 5.174 : Tm m th hm s y = 2x3 3(3m + 1)x2 + 12(m2 + m)x + 1 c cc i, cc tiu. Vit phng trnh ng
thng i qua hai im cc tr .

Bi 5.175 : Tm m th hm s y = x3 3(m + 1)x2 + 2(m2 + 7m + 2)x 2m(m + 2) c cc i, cc tiu. Vit phng


trnh ng thng i qua hai im cc tr .

Bi 5.176 : Tm m th hm s y = 2x3 3(2m + 1)x2 + 6m(m + 1)x + 1 c cc i, cc tiu i xng nhau qua ng
thng y = x + 2.

Bi 5.177 : Tm m th hm s y = mx3 3mx2 + (2m + 1)x + 3 m c cc i, cc tiu. Chng minh rng, khi

ng thng i qua hai im cc tr lun i qua mt im c nh.

Bi 5.178 : Tm m th hm s y = x3 + mx2 + 2 c cc i, cc tiu. Chng minh rng, khi ng thng i qua


hai im cc tr lun i qua mt im c nh.
Bi 5.179 : Tm m th hm s y = mx3 x2 + 2x 8m c cc i, cc tiu. Chng minh rng, khi ng thng i
qua hai im cc tr lun i qua mt im c nh.

Bi 5.180 : Tm m th hm s y = x3 (2m + 1)x2 + (3m + 1)x m 1 c ng thng i qua hai ng thng cc


19
tr to vi ng thng y =
x + 1 mt gc 45 .
13
Bi 5.181 : Chng minh rng th hm s y = x3 + 3mx2 3x 3m + 2 lun c cc i, cc tiu. Tm m cc i, cc
x 1
tiu i xng nhau qua ng thng y = .
4 4
4 3
Bi 5.182 : Tm a hm s y = x 2(1 sin a)x2 + (1 + cos 2a)x + 1 t cc tr ti x1 , x2 tha mn : x21 + x22 = 1.
3
1 3 1
3 sin 2a
Bi 5.183 : Cho hm s y = x (sin a + cos a)x2 +
x.
3
2
3

TRN ANH TUN - 0974 396 391 - (04) 66 515 343

WWW.VNMATH.COM

Trang 97

www.VNMATH.com

CHUYN LUYN THI I HC

www.luyenthi24h.com
www.luyenthi24h.com
www.VNMATH.com

1. Tm a hm s ng bin trn R.
2. Tm a hm s t cc tr ti x1 , x2 tha mn x1 + x2 = x21 + x22 .
Bi 5.184 : Tm m th hm s y = x3

3m 2
x + m c cc im cc i, cc tiu nm v hai pha ca ng thng
2

y = x.

mx2 + (2 m2 )x (2m + 1)
c cc tr.
xm
x2 2x + m + 2
. Vit phng trnh ng thng qua cc i, cc tiu ca th hm s trn.
Bi 5.186 : Cho hm s y =
x+m1
Bi 5.187 : Tm m cc hm s sau c cc tr :
Bi 5.185 : Tm m hm s y =

1. y =

x2 + 2m2 x + m2
;
x+1

3. y =

x2 + 2mx m
;
x+m

5. y =

x2 + (m + 2)x + 3m + 2
;
x+1

x2 + (m + 1)x m
x2 + (m 1)x m
mx2 + (m + 1)x + 1
;
;
.
4. y =
6. y =
x+1
x+1
mx + 2
x2 + mx m2
Bi 5.188 : Cho hm s y =
. Tm m th hm s c cc tr, vit phng trnh ng thng qua cc
xm
im cc tr trn.
2. y =

Bi 5.189 : Tm cc hm s sau c cc i, cc tiu.


x2 + 2x cos + 1
;
x + 2 sin

x2 cos + x + sin2 cos + sin


.
x + cos

(x a cos )(x a sin2 )

Bi 5.190 : Tm ta cc im cc tr ca th hm s sau : y =
vi a > 0 v 0;
.
x
2
x2 + mx 2m 4
c cc i v cc tiu.
Bi 5.191 : Tm m hm s y =
x+2
2m2 x2 + (2 m2 )(mx + 1)
Bi 5.192 : Tm m hm s : y =
c cc tr.
mx + 1
x2 + mx 8
.
Bi 5.193 : Vit phng trnh ng thng i qua cc im cc tr ca th hm s y =
xm
(m + 1)x2 2mx (m3 m2 2)
Bi 5.194 : Tm m , 1 hm s y =
c cc tr thuc khong (0; 2).
xm
ax2 + bx + c
Bi 5.195 : Tm a, b, c y =
c cc tr bng 1 khi x = 1 v ng tim cn xin ca th hm s vung
x2
1x
gc vi ng thng y =
.
2
x2 + m2 x + 2m2 5m + 3
Bi 5.196 : Tm m > 0 hm s y =
t cc tr ti x0 (0; 2m).
x
2x2 + (m 2)x
Bi 5.197 : Tm m th hm s y =
c cc tr v tm qu tch hai im cc tr .
x1
x2 + mx m 1
. Tm m th hm s trn c cc tr. Tm qu tch cc im cc tr .
Bi 5.198 : Cho hm s y =
x+1
x2 + 3x + m
Bi 5.199 : Tm m hm s y =
c |yc yct | = 4.
x4
2x2 + 3x + m 2
Bi 5.200 : Tm m hm s y =
c |yc yct | < 12.
x+2
x2 (m + 1)x m2 + 4m 2
Bi 5.201 : Tm m hm s y =
c cc tr v yc .yct nh nht.
x1
x2 mx + m
Bi 5.202 : Cho hm s y =
. Chng minh rng vi mi m th hm s c cc tr v khong cch gia hai
x1
im cc tr khng i.
1. y =

TRN ANH TUN - 0974 396 391 - (04) 66 515 343

2. y =

WWW.VNMATH.COM

Trang 98

www.VNMATH.com

CHUYN LUYN THI I HC

www.luyenthi24h.com
www.luyenthi24h.com
www.VNMATH.com

mx2 + 3mx + (2m + 1)


c cc tr nm hai pha Ox.
x1
x2 + (m + 1)x m + 1
c cc i, cc tiu nm v cng mt pha Ox.
Bi 5.204 : Tm m th hm s y =
xm
x2 + 2mx 5
Bi 5.205 : Tm m th hm s y =
c cc i, cc tiu nm v hai pha ng thng y = 2x.
x1
x2 + 3x + a
Bi 5.206 : Cho hm s : y =
.
x+1
Bi 5.203 : Tm m th hm s y =

1. Vi nhng gi tr no ca tham s a th th hm s y c tip tuyn vung gc vi ng phn gic th nht ca


h trc ta .
2. Chng minh rng khi , th ca hm s c im cc i v im cc tiu.
x2 + 2m2 x + m2
c cc i v cc tiu.
x+1
2m2 x2 + (2 m2 )(mx + 1)
Bi 5.208 : Cho hm s : y =
. Chng minh rng vi mi m , 0 hm s lun c cc i v cc
mx + 1
tiu.
x2 2kx + k2 + 1
Bi 5.209 : Cho hm s y =
. Chng minh rng vi mi k, hm s lun c gi tr cc i, cc tiu tri
xk
du.
x2 (3m + 2)x + m + 4
Bi 5.210 : Cho hm s : y =
. Tm m hm s c cc i, cc tiu v ng thi khong cch
x1
gia hai im cc i, cc tiu ca th hm s nh hn 3.
Bi 5.207 : Vi gi tr no ca m th hm s : y =

x2 + mx + 3
. Tm m hm s c cc i, cc tiu v ng thi hai im cc i, cc tiu
x+1
ca th hm s nm v hai pha ca ng thng 2x + y 1 = 0.
Bi 5.211 : Cho hm s y =

Bi 5.212 : Cho hm s y =
ca hm s cng m.

x2 (m + 3)x + 3m + 1
. Tm m hm s c cc i, cc tiu v cc gi tr cc i, cc tiu
x1

Bi 5.213 : Cho hm s : y = x4 + 4mx3 + 3(m + 1)x2 + 1. Vi gi tr no ca m th hm s ch c cc tiu v khng c cc


i.
Bi 5.214 : Cho hm s : y = x4 + (m + 3)x3 + 2(m + 1)x2 . Chng minh rng : vi mi m , 1 hm s lun c cc i,
ng thi xc 0.

Bi 5.215 : Cho hm s : y = x4 + 8ax3 + 3(2a + 1)x2 4. Vi gi tr no ca a th hm s ch c cc tiu v khng c cc


i.

Bi 5.216 : Cho hm s y =

1 4
1
x mx2 + .
2
2

1. Xc nh m hm s c cc tiu m khng c cc i.
2. Xc nh m th hm s c ba im cc tr lp thnh mt tam gic :
(a) u ;

(b) vung ;

1
(c) c din tch bng .
2

Bi 5.217 : Cho hm s y = x4 2mx2 + m 1. Tm m th hm s c ba im cc tr to thnh mt tam gic u.

Bi 5.218 : Tm m th hm s : y = x4 2mx2 + 2m + m4 c ba im cc tr lp thnh tam gic u.

Bi 5.219 : Chng minh rng : hm s y = x4 + mx3 + mx2 + mx + 1 khng th c ng thi cc i v cc tiu vi mi


m R.

TRN ANH TUN - 0974 396 391 - (04) 66 515 343

WWW.VNMATH.COM

Trang 99

www.VNMATH.com

CHUYN LUYN THI I HC

www.luyenthi24h.com
www.luyenthi24h.com
www.VNMATH.com

Bi 5.220 : Chng minh rng : x4 + px3 + q 0x R 256q 27p4 .

Bi 5.221 : Tm m hm s : y = x4 4x3 + x2 + mx 1 c cc i v cc tiu.

Bi 5.222 : Cho hm s : y = x4 + 2x3 + mx2 . Tm m hm s c cc tiu m khng c cc i.


Bi 5.223 : Tm m y = x4 8mx3 3(2m + 1)x2 + 4 ch c cc i m khng c cc tiu.
3
1
Bi 5.224 : Tm m hm s y = x4 mx2 + ch c cc tiu m khng c cc i.
4
2
Bi 5.225 : Tm m y = mx4 + (m 1)x2 + (1 2m) ch c ng mt cc tr.
Bi 5.226 : Cho hm s : y = 3x4 + 4mx3 + 6mx2 + 24mx + 1.
1. Bin lun theo m s cc tr ca hm s.
2. Tm m hm s t cc tiu ti x0 [2; 2].
3
1 4
x 2x3 + (m + 2)x2 (m + 6)x + 1. Tm m hm s c ba cc tr.
4
2
1. Chng minh rng : x4 + px + q 0x R 256q3 27p4 .

Bi 5.227 : Cho hm s : y =
Bi 5.228 :

2. Cho 256q3 27p4 . Chng minh rng : qx4 + px3 + 1 0 x R.

Bi 5.229 : Chng minh rng : y = 2x4 6mx2 + (m2 + 1)x + 3m2 lun c ba cc tr, ng thi gc ta l trng tm ca

tam gic to bi ba nh l ba cc tr .

5.3 Tim cn
Vn 1 : Tm tim cn ca th hm s

1. Nu y =

P(x)
, vi P(x), Q(x) l cc a thc.
Q(x)

(a) Nu Q(x) = 0 c nghim x = x0 th ng thng x = x0 l ng tim cn ng.


h s cao nht ca P(x)
(b) Nu P(x) v Q(x) c bc bng nhau th ng thng y =
.
h s cao nht ca Q(x)
(c) Nu bc ca P(x) bng bc ca Q(x) cng 1 th th hm s c mt ng tim cn xin (l thng khi chia
t cho mu).
2. xc nh h s a, b trong ng tim cn xin y = ax + b vi a , 0 ca th hm s y = f (x) ta lm bc sau :
f (x)
, v b = lim ( f (x) ax) ;
x+ x
x+
f (x)
(b) hoc a = lim
, v b = lim ( f (x) ax).
x x
x
(a) a = lim

Ch rng nu trn ta tnh c a = 0 th th hm s khng c tim cn xin m ch c tim cn ngang y = b.

Bi 5.230 : Tm tim cn ca cc hm s sau:

TRN ANH TUN - 0974 396 391 - (04) 66 515 343

WWW.VNMATH.COM

Trang 100

www.VNMATH.com

CHUYN LUYN THI I HC

x+1
;
1. y =
2x + 1
2. y =

3
;
4. y = 2x 1
x+2

2x2 1
;
x2 3x + 2

3. y = 4 +

x+3
;
x+1

8. y = x2 x + 1;
7. y =

2x2 + x + 1
;
x+1

6. y = x2 1;
5. y =

1
;
x2

www.luyenthi24h.com
www.luyenthi24h.com
www.VNMATH.com

9. y = x +

x2 + 2x.

Bi 5.231 : Cho th cc hm s :
a) y =

3x2 + x + 1
;
x1

b) y =

x2 4x + 5
;
2x + 1

c) y =

x2 + x 1
.
x+3

1. Tnh din tch hnh phng gii hn bi tim cn xin th cc hm s trn chn trn hai trc ta .
2. Tnh din tch hnh phng gii hn bi tip tuyn ti im c honh bng 2 vi cc tim cn ca th cc hm s
trn.
2x + 1
c th (C)1 . M l mt im ty trn (C). Tip tuyn vi (C) ti M ct tim cn
x2
ngang v tim cn ng ti A v B.
Bi 5.232 : Cho hm s y =

1. Chng minh rng M l trung im ca AB.


2. Chng minh rng khi M di ng trn (C) th tip tuyn to vi hai tim cn mt tam gic c din tch khng i.
3. Chng minh khng c tip tuyn no ca (C) i qua giao im ca hai tim cn.
Bi 5.233 : Tm cc im thuc th hm s y =
din tch nh nht.

x+1
sao cho tip tuyn ti to vi hai tim cn mt tam gic c
x1

Vn 2 : Cc bi ton v tim cn c tham s


ax + b
a
r
= +
(vi a v c khc 0) c tim cn ng (ngang) khi v ch khi r , 0. Khi
1. th hm s y =
cx + d
c cx + d
a
d
y = l ng tim cn ngang v x = l ng tim cn ng.
c
c
ax2 + bx + c
r
= px + q +
(vi a v d khc 0) c tim cn ng (xin) khi v ch khi r , 0.
dx + e
dx + e
e
Khi y = px + q l ng tim cn xin v x = l ng tim cn ng.
d

2. th hm s y =

x2
c tim cn.
xm
2x2 3x + m
Bi 5.235 : Tm m th hm s y =
khng c tim cn ng.
xm
mx2 + 6x 2
Bi 5.236 : Tm m hm s y =
khng c tim cn ng.
x+2
Bi 5.234 : Tm m hm s y =

Cc khng nh ca bi ny ng cho mi hm s phn thc y =

TRN ANH TUN - 0974 396 391 - (04) 66 515 343

ax + b
ax2 + bx + c
v y =
cx + d
dx + e

WWW.VNMATH.COM

Trang 101

www.VNMATH.com

CHUYN LUYN THI I HC

www.luyenthi24h.com
www.luyenthi24h.com
www.VNMATH.com

x2 + x + a
c tim cn xin i qua A(2; 0).
x+a
x2 + mx 1
. Tm m th hm s c tim cn xin v tim cn xin to vi hai trc ta
Bi 5.238 : Cho hm s y =
x1
mt tam gic c din tch bng 8.
Bi 5.237 : Tm a y =

Bi 5.239 : Cho hm s y =

x2 + 2x cos + 1
, vi [0; ].
x + 2 sin

1. Tm tim cn xin ca th hm s.
2. Tm khong cch t gc ta n tim cn xin t gi tr ln nht.
Bi 5.240 : Cho hm s y =

mx2 + (3m2 2)x 2


c th l (C).
x + 3m

1. Tm m gc gia hai ng tim cn ca (C) bng 45 .


2. Tm m (C) c tim cn xin ct hai trc ta ti A v B sao cho tam gic OAB c din tch bng 4.

5.4 Tm i xng v trc i xng. im thuc th


Vn 1 : Tm i xng, trc i xng

1. im M(x0 ; y0 ) thuc th hm s y = f (x) nn y0 = f (x0 ).

8
>
<

x M + xN
2
2. Hai im M, N i xng qua im I khi v ch khi I l trung im MN, tc l
y + yN
>
:y I = M
.
2
3. Hai im M, N i xng nhau qua ng thng khi v ch khi

xI =

8
u
< MN

Trung im I ca MN thuc .

x2 + 2m2 x + m2
c hai im i xng vi nhau qua gc ta .
x+1
Tm m trn th hm s y = x3 + mx2 + 9x + 4 c hai im i xng vi nhau qua gc ta .
3x + 4
Tm trn th hm s y =
cc cp im i xng nhau qua im I(1; 1).
2x 1

x2 + x + 2
5
Tm trn th hm s y =
cc cp im i xng nhau qua im I 0; .
x1
2
2
x
Tm trn th hm s y =
cc cp im i xng nhau qua ng thng d : y = x 1.
x1

Bi 5.241 : Tm m trn th hm s y =
Bi 5.242 :
Bi 5.243 :
Bi 5.244 :
Bi 5.245 :

Vn 2 : Khong cch


1. MN =

(xM xN )2 + (yM yN )2 ;

2. M(x0 ; y0 ) v : Ax + By + C = 0 th d(M, ) =

|Ax0 + By0 + C|

.
A2 + B2

TRN ANH TUN - 0974 396 391 - (04) 66 515 343

WWW.VNMATH.COM

Trang 102

www.VNMATH.com

CHUYN LUYN THI I HC

www.luyenthi24h.com
www.luyenthi24h.com
www.VNMATH.com

Bi 5.246 : Cho hm s : y = x3 3x2 + 1 c th (C).


1. Tm hai im trn (C) i xng nhau qua im A(0; 2).
2. Tm im M trn (C) sao cho khong cch t M n ng thng y = x 1 l bng

2, bit im M c tung

dng.

Bi 5.247 : Tm im M thuc parabol (P) : y = x2 1 OM t gi tr nh nht. Chng minh rng khi ng thng
OM vung gc vi tip tuyn ca (P) ti M.

Bi 5.248 : Tm im M thuc th hm s y =
Bi 5.249 : Tm im M thuc th hm s y =
nht.

3x 5
tng khong cch t M n hai ng tim cn l nh nht.
x2
x2 + 2x 1
tng khong cch t M n hai ng tim cn l nh
x2

Bi 5.250 : Tm im M thuc th hm s y =

x1
tng khong cch t M n hai trc ta t gi tr nh nht.
x+1

Bi 5.251 : Tm im M thuc th hm s y =

x2 + x 6
tng khong cch t M n hai trc ta t gi tr nh
x3

nht.

Bi 5.252 : Tm trn mi nhnh ca th hm s y =

4x 9
cc im M, N di MN nh nht.
x3

Bi 5.253 : Tm trn mi nhnh ca th hm s y =

x2 + 2x 5
cc im M, N di MN nh nht.
x1

Bi 5.254 : Cho th hm s y =

x2 + 5x + 15
(C).
x+3

1. Tm trn (C) cc im c ta nguyn.

2. Tm trn (C) cc im M khong cch t M n Ox bng 2 ln khong cch t M n Oy.

5.5 Bin lun s nghim ca phng trnh, bt phng trnh bng phng php th
+ Hai im M0 (x0 ; y0 ) v M1 (x0 ; y0 ) i xng nhau qua gc to .
+ Hai im M0 (x0 ; y0 ) v M2 (x0 ; y0 ) i xng nhau qua trc honh.

+ Hai im M0 (x0 ; y0 ) v M3 (x0 ; y0 ) i xng nhau qua trc trc tung.


V th hm s
y = f (x) =

TRN ANH TUN - 0974 396 391 - (04) 66 515 343

(x + 1)(x 3)
.
x2
WWW.VNMATH.COM

(C0 )
Trang 103

8
7
6
5
4
3
2
1
4

www.luyenthi24h.com
www.luyenthi24h.com
www.VNMATH.com

CHUYN LUYN THI I HC

www.VNMATH.com

1 1
2
3
4
5
6
7

y
y=

(x+1)(x3)
x2

T hy v cc th cc hm s sau:
y = f (x) =

(x + 1)(x 3)
.
x2

(C1 )

T (C0 ) chuyn sang (C1 ) bng cch ly i xng (C0 ) qua trc honh.

6
5
4
3
2
1
4

1 1
2
3
4
5
6
7
8
9

y = (x+1)(x3)
x2

8


(x + 1)(x 3) < f (x)
=
y = | f (x)| =
:
x2

f (x)

nu f (x) 0 ( pha trn trc honh)

nu f (x) < 0.

(C2 )

T (C0 ) chuyn sang (C2 ) bng cch gi nguyn phn nm trn trc honh, ly i xng phn di trc honh ca (C0 ) qua
trc honh.

TRN ANH TUN - 0974 396 391 - (04) 66 515 343

WWW.VNMATH.COM

Trang 104

CHUYN LUYN THI I HC

www.VNMATH.com

8
7
6
5
4
3
2
1
4

y = f (|x|) =

1 1
2
3
4
5
6
7

www.luyenthi24h.com
www.luyenthi24h.com
www.VNMATH.com



(x + 1)(x 3)


y=

x2

8
< f (x)

(|x| + 1)(|x| 3)
=
:
|x| 2
f (x)

nu x 0 ( bn phi trc tung)

(C3 )

nu x < 0.

T (C0 ) chuyn sang (C3 ) bng cch gi nguyn phn nm bn phi trc tung, ly i xng phn bn phi trc tung ca
(C0 ) qua trc tung.

8
7
6
5
4
3
2
1
7

y=

1 1
2
3
4
5
6
7

8
< (x+1)(x3) = f (x)
x2

y
y=

|(x + 1)|(x 3)
=
: (x+1)(x3)
x2
x2 = f (x)

(|x|+1)(|x|3)
|x|2

nu x 1 ( bn phi x = 1)

nu x < 1.

(C4 )

T (C0 ) chuyn sang (C4 ) bng cch gi nguyn phn nm bn phi ng thng x = 1, ly i xng phn bn tri ng
thng x = 1 ca (C0 ) qua trc honh.

TRN ANH TUN - 0974 396 391 - (04) 66 515 343

WWW.VNMATH.COM

Trang 105

8
7
6
5
4
3
2
1
4

www.luyenthi24h.com
www.luyenthi24h.com
www.VNMATH.com

CHUYN LUYN THI I HC

www.VNMATH.com

1 1
2
3
4
5
6
7

y
y=

(x+1)(x3)
(x + 1)|x 3| < x2 = f (x)
=
y=
: (x+1)(x3)
x2
x2 = f (x)

|x+1|(x3)
x2

nu x 3 ( bn phi x = 3)

nu x < 3.

(C5 )

T (C0 ) chuyn sang (C5 ) bng cch gi nguyn phn nm bn phi ng thng x = 2, ly i xng phn bn tri ng

thng x = 2 ca (C0 ) qua trc honh.

8
7
6
5
4
3
2
1
4

1 1
2
3
4
5
6
7

y
y=

8
(x + 1)(x 3)
>
<

y=

= f (x)
(x + 1)(x 3)
x2
=
(x + 1)(x 3)
>
|x 2|
:
= f (x)
x2

(x+1)|x3|
x2

nu x 2 ( bn phi x = 2)
nu x < 2.

(C6 )

T (C0 ) chuyn sang (C6 ) bng cch gi nguyn phn nm bn phi ng thng x = 1, ly i xng phn bn tri ng

thng x = 1 ca (C0 ) qua trc honh.

TRN ANH TUN - 0974 396 391 - (04) 66 515 343

WWW.VNMATH.COM

Trang 106

CHUYN LUYN THI I HC

www.VNMATH.com

8
7
6
5
4
3
2
1

www.luyenthi24h.com
www.luyenthi24h.com
www.VNMATH.com

y
y=

(x+1)(x3)
|x2|

x
O
1 1
1
2
3
4
5
6
2
3
4
5
6
7
u(x)
u(x)
mun v th cc hm s y = |u(x)|v(x) (C1 ) hoc y =
. Ta gi
Ch : Vi hm s y = u(x).v(x) (C) hoc y =
v(x)
|v(x)|
nguyn th (C) trong min lm cho u(x) > 0 hoc v(x) > 0, (tng ng). Ly xng phn cn li qua trc honh.
4

Bi 5.255 : Bin lun theo m s nghim phng trnh : x3 + 3x2 + m = 0.


(x 1)2
Bi 5.256 : Cho hm s y =
c th (C).
x+2
1. Kho st v v th (C) ;
2. Bin lun theo m s nghim phng trnh

(x 1)2
= m.
|x + 2|

Bi 5.257 : Cho hm s y = x3 3x2 + 2 c th (C).


1. Kho st s bin thin v v th (C) ;

2. Bin lun theo k c nghim phng trnh x2 2x 2 =


Bi 5.258 : Cho hm s y =

k
.
|x 1|

x2 + x 3
.
x+2

1. Kho st s bin thin v v th ca hm s ;


2. Bin lun theo m s nghim phng trnh :
t4 + (1 m)t2 3 2m = 0.
Bi 5.259 :

1. Kho st s bin thin v v th (C) ca hm s y =

x2 + x + 1
;
x+1

2. Bin lun theo m s nghim ca phng trnh : x2 + (1 m)|x| + 1 m = 0.

Bi 5.260 : Tm m phng trnh : |x4 2x2 1| = log2 m c 6 nghim phn bit.


2x 1
;
Bi 5.261 :
1. Kho st v v th hm s y =
x+2
2 sin x 1
= m c ng 2 nghim trn [0; ].
sin x + 2
1. Kho st s bin thin v v th hm s y = x4 + 2x2 + 3 ;

2. Tm cc gi tr ca m phng trnh
Bi 5.262 :

2. Bin lun theo m s nghim ca phng trnh x4 2x2 = m4 2m2 ;

TRN ANH TUN - 0974 396 391 - (04) 66 515 343

WWW.VNMATH.COM

Trang 107

www.VNMATH.com

CHUYN LUYN THI I HC

www.luyenthi24h.com
www.luyenthi24h.com
www.VNMATH.com

Bi 5.263 : Cho hm s y = (x + 1)2 (2 m).


1. Kho st v v th hm s trn ;
2. Bin lun theo m s nghim ca phng trnh :
(x + 1)2 (2 x) = (m + 1)2 (2 m).

5.6 Bi ton v s tng giao


Bi 5.264 :

1. Kho st s bin thin v v th hm s y =

2x 1
.
x+1

2. Vi gi tr no ca m, ng thng dm i qua im A(2; 2) v c h s gc m ct th hm s cho


(a) Ti hai im phn bit ?
Bi 5.265 :

(b) Ti hai im thuc hai nhnh ca th ?

1. Kho st s bin thin v v th (C) ca hm s y =

x+2
.
2x + 1

2. Chng minh rng ng thng y = mx + m 1 lun i qua mt im c nh ca ng cong (C) khi m bin thin.
3. Tm cc gi tr ca m sao cho ng thng cho ct (C) ti hai im thuc cng mt nhnh ca (C).
Bi 5.266 :

1. Kho st s bin thin v v th hm s y =

2x2 x + 1
.
x1

2. Vi gi tr no ca m ng thng y = m x ct th hm s cho ti hai im phn bit ?


3. Gi A v B l hai giao im . Tm tp hp trung im M ca on thng AB khi m bin thin.
Bi 5.267 : Tm m th hm s y = x3 3(m + 1)x2 + 2(m2 + 4m + 1)x 4m(m + 1) ct trc honh ti ba im phn
bit c honh ln hn 1.

Bi 5.268 : Chng minh rng th hm s y =


xng vi nhau qua ng thng y = x.

x2 + 2x
v ng thng y = x 3 ct nhau ti hai im phn bit i
x+1

x2 + 3
2
c th (C). Vit phng trnh ng thng d i qua im 2;
sao cho d ct (C)
x+1
5
ti hai im phn bit A v B sao cho M l trung im on AB.

Bi 5.269 : Cho hm s y =

x2 + mx 8
c th (C). Tm m (C) ct trc honh ti hai im phn bit A v B sao cho
xm
tip tuyn vi (C) ti A v B vung gc vi nhau.
Bi 5.270 : Cho hm s y =

Bi 5.271 : Cho ng cong y = x3 x2 + 18mx 2m. Tm m ng cong ct trc honh ti ba im phn bit c honh
x1 , x2 , x3 sao cho x1 < 0 < x2 < x3 .

x2 + x 1
c th (C). Tm m (C) ct ng thng y = x + m ti hai im phn bit A
x1
v B. Chng minh rng khi y A, B thuc cng mt nhnh ca th (C).
Bi 5.272 : Cho hm s y =

x2 + x + 1
c th (C). Chng minh rng vi mi m, ng thng y = m lun ct (C) ti hai im
x1
phn bit A, B. Tm m A, B ngn nht.

Bi 5.273 : Cho y =

Bi 5.274 : Cho hm s y =
ca (C).
Bi 5.275 : Cho hm s y =
cng mt nhnh ca (C).

2x2 3x
c th (C). Tm k ng thng y = 2kx k ct (C) ti hai im thuc hai nhnh
x2
x2 + 4x + 1
c th (C). Tm m ng thng y = mx + 2 m ct (C) ti hai im thuc
x+2

TRN ANH TUN - 0974 396 391 - (04) 66 515 343

WWW.VNMATH.COM

Trang 108

www.VNMATH.com
Bi 5.276 : Cho hm s y =
OAOB.

CHUYN LUYN THI I HC

www.luyenthi24h.com
www.luyenthi24h.com
www.VNMATH.com

x2 + mx 1
. Tm m ng thng y = m ct th hm s trn ti hai im A, B tha mn
x1

Bi 5.277 : Tm m (Cm ) : y = x3 + (1 + m)x2 + 2mx + m2 ct trc honh ti ba im phn bit c honh m.


x3
x m c th (C). Tm m (C) ct trc honh ti 3 im phn bit.
3
Bi 5.279 : Cho (Cm ) : y = x3 3mx2 + 3(m2 1)x m2 + 1. Tm m (Cm ) ct trc honh ti ba im phn bit c honh
Bi 5.278 : Cho hm s y =

dng.

Bi 5.280 : Cho (Cm ) : y = x3 2mx2 + (2m2 1)x + m(1 m2 ). Tm m (Cm ) ct trc honh ti ba im phn bit c
honh dng.

Bi 5.281 : Tm m ng thng y = mx m + 2 ct th hm s y =
tam gic OAB u.

x2 + x 1
ti hai im A, B phn bit sao cho
x1

2x + 1
(C) v im A(2; 5). Xc nh ng thng d song song vi ng thng
x1
y = x + 5 v ct (C) ti hai im B, C sao cho tam gic ABC u.
3
Bi 5.283 : Chng minh rng ng thng y = 2x + m lun ct th hm s y = x + 3 +
ti hai im A v B phn
x1
bit. Tm m khong cch AB l ngn nht.
Bi 5.282 : Cho th hm s y =

Bi 5.284 : Vit phng trnh ng thng d i qua im M 2;


bit v M l trung im AB.

2
x2 + 3
sao cho d ct th hm s y =
ti A, B phn
5
x+1

Bi 5.285 : Tm m ng thng y = m(x 5) + 10 ct th hm s y =


M(5; 10) l trung im AB.

x2 2x + 9
ti hai im A, B phn bit sao cho
x2

5.7 S tip xc ca hai ng cong v tip tuyn


Vn 1 : Vit phng trnh tip tuyn bit tip im

1. Tnh x0 v y0 ;
2. Thay vo phng trnh tip tuyn y = y (x0 )(x x0 ) + y0 .

2
(C). Vit phng trnh tip tuyn vi (C) ti im A(0; 3).
2x 1
Bi 5.287 (TN07) : Cho hm s y = x3 + 3x2 2 (C). Vit phng trnh tip tuyn vi (C) ti im un.

Bi 5.286 (TN07) : Cho hm s y = x + 1

Bi 5.288 (TN07) : Cho hm s y = x4 2x2 + 1 (C). Vit phng trnh tip tuyn vi (C) ti im cc i ca (C).
x1
Bi 5.289 (TN07) : Cho hm s y =
(C). Vit phng trnh tip tuyn vi (C) ti giao im ca (C) vi trc tung.
x+2
3x + 4
Bi 5.290 (TN07) : Cho hm s y =
(C). Vit phng trnh tip tuyn vi (C) ti im M(1; 7).
2x 3
Bi 5.291 (TN07) : Cho hm s y = x3 3x + 2 (C). Vit phng trnh tip tuyn vi (C) ti im A(2; 4).

Bi 5.292 (TN08) : Cho hm s y = x3 3x2 + 1 (C). Vit phng trnh tip tuyn vi (C) ti im c honh bng 3.

Bi 5.293 (TN08) : Cho hm s y = x4 2x2 (C). Vit phng trnh tip tuyn vi (C) ti im c honh bng x = 2.

TRN ANH TUN - 0974 396 391 - (04) 66 515 343

WWW.VNMATH.COM

Trang 109

CHUYN LUYN THI I HC

www.VNMATH.com

www.luyenthi24h.com
www.luyenthi24h.com
www.VNMATH.com

Bi 5.294 : Trong cc bi t 5.286 n 5.293 hy tnh din tch tao gic to bi tip tuyn vi hai trc ta .
2x 3
. Tm cc im c ta nguyn ca th hm s v vit phng trnh tip tuyn ti cc
Bi 5.295 : Cho hm s
x2
im .
1
Bi 5.296 : Cho hm s y = x3 2x2 + 3x. Vit phng trnh tip tuyn ti im un ca th hm s ti im un v
3
chng minh rng tip tuyn l tip tuyn c h s gc b nht.
Bi 5.297 : Cho hm s y = 2x3 3x2 + 9x 4 (C). Vit phng trnh tip tuyn ca (C) ti cc giao im ca (C) vi

th cc hm s sau :

1. d : y = 7x + 4;

2. (P) : y = x2 + 8x 3;

3. (C ) : y = x3 4x2 + 6x 7.

Bi 5.298 : Cho hm s y = x4 + 2mx2 2m + 1 c th l (Cm ).


1. Chng minh rng (Cm ) lun i qua hai im c nh A v B.
2. Tm m hai tip tuyn ti A v B vung gc vi nhau.
Bi 5.299 : Cho hm s y =

x2 + 2x + 2
c th l (C).
x+1

1. Gi s A l im thuc (C) c honh l a. Vit phng trnh tip tuyn d ca (C) ti A.


2. Xc nh a d i qua im M(1; 0).
Bi 5.300 : Cho hm s y = x
nhau.

1
(C). Tm cc cp im phn bit trn (C) m cc tip tuyn ti song song vi
x+1

Bi 5.301 : Cho im A(x0 ; y0 ) thuc th (C) hm s y = x3 3x + 1. Tip tuyn ca (C) ti A ct (C) ti im B khng

trng vi A. Tm honh ca B theo x0 .


x1
. Vit phng trnh tip tuyn vi th hm s ti im c honh l x0 . Tm ta
Bi 5.302 : Cho hm s y =
x+1
cc giao im ca tip tuyn vi cc trc ta v vi cc ng tim cn ca th hm s.
Bi 5.303 : Cho hm s y = x3 + 1 m(x + 1) (Cm ). Vit phng trnh tip tuyn ti giao im ca (Cm ) vi trc tung. Tm
m tip tuyn ni trn chn hai trc ta mt tam gic c din tch bng 8.
Bi 5.304 : Cho hm s y =
A, B v tam gic OAB cn.

x2 + x 2
(C). Tm im M trn (C) sao cho tip tuyn ti M ct hai trc ta ti hai im
x2

1
, (C). Tm cc im trn (C) c honh ln hn 1 sao cho tip tuyn ti im
x1
to vi hai ng tim cn mt tam gic c chu vi nh nht.

Bi 5.305 : Cho hm s y = x + 1 +

2x2 + (6 m)x
(C) (vi m , 0). Chng minh rng ti mi im ca (C) tip tuyn lun ct
mx + 2
hai ng tim cn mt tam gic c din tch khng i.
Bi 5.306 : Cho hm s y =

x2 + mx 8
(C). Xc nh m (C) ct trc Ox ti hai im phn bit v cc tip tuyn ti
xm
hai im vung gc vi nhau.

Bi 5.307 : Cho hm s y =

2x2 + mx + m
(C). Xc nh m (C) ct trc Ox ti hai im phn bit v cc tip tuyn ti
x+1
hai im vung gc vi nhau.
Bi 5.308 : Cho hm s y =

Bi 5.309 : Cho hm s y = x3 + mx2 m 1. Vit phng trnh tip tuyn ti im c nh ca th hm s.

Bi 5.310 : Cho hm s y = x3 3mx + 3m 2. Chng minh rng tip tuyn ca th hm s ti im un lun i qua
mt im c nh.

TRN ANH TUN - 0974 396 391 - (04) 66 515 343

WWW.VNMATH.COM

Trang 110

www.VNMATH.com

CHUYN LUYN THI I HC

www.luyenthi24h.com
www.luyenthi24h.com
www.VNMATH.com

Bi 5.311 : Cho hm s y = x3 + 3x2 + mx. Tm m tip tuyn ca th hm s ti im un i qua im M(1; 0).


Bi 5.312 : Cho hm s y = x3 + 3x2 + 3x + 5. Chng minh rng trn th hm s khng tn ti hai im sao cho tip
tuyn ca th hm s ti hai im y vung gc vi nhau.
x3 + 1
Bi 5.313 : Tm phng trnh ca tt c cc tip tuyn ca th hm s y =
, bit mi tip tuyn to vi cc trc
x
1
ta mt tam gic c din tch .
2
Bi 5.314 : Cho hm s y = x3 +3x2 +mx+1 (Cm ). Tm m (Cm ) ct ng thng y = 1 ti 3 im phn bit C(0; 1), D, E.
Tm m cc tip tuyn ti D v E vung gc.
2x 1
(C) v im M bt k thuc (C). Gi I l giao im ca hai ng tim cn. Tip tuyn
Bi 5.315 : Cho hm s y =
x1
ti M ct hai ng tim cn ti A v B.
1. Chng minh rng M l trung im AB.
2. Chng minh rng S IAB khng i.
3. Tm M tam gic IAB c chu vi nh nht.
x2 3x + 4
(C) v im M bt k thuc (C). Gi I l giao im hai ng tim cn. Tip
2x 2
tuyn ti M ct hai ng tim cn ti A v B.
Bi 5.316 : Cho hm s y =

1. Chng minh rng M l trung im AB.


2. Chng minh rng tch khong cch t M n hai ng tim cn l khng i.
3. Chng minh rng S IAB khng i.
4. Tm M tam gic IAB nh nht.

Vn 2 : Hai ng cong tip xc

Hai ng cong y = f (x) v y = g(x) tip xc vi nhau khi v chit khi h phng trnh
8
< f (x) = g(x)
:
f (x) = g (x)

c nghim. Khi nghim ca h l honh tip im tip xc.

Bi 5.317 : Tm tt c cc gi tr ca a th hm s y = x4 5x2 + 4 tip xc vi th hm s y = x2 + a.


Bi 5.318 : Cho hm s y =
parabol (P) : y = x2 + 5.

2x2 + (m + 1)x 3
c th (Cm ). Tm m tim cn xin ca (Cm ) c th tip xc vi
x+m

Bi 5.319 : Tm a (C) : y = (x + 1)2 (x 1)2 tip xc vi (P) : y = ax2 3. Khi vit phng trnh tip tuyn chung

ca (P) v (C).

Bi 5.320 : Cho y =

mx2 + 3mx + 2m + 1
c th (Cm ). Tm m (Cm ) tip xc vi d : y = m.
x+2

TRN ANH TUN - 0974 396 391 - (04) 66 515 343

WWW.VNMATH.COM

Trang 111

www.VNMATH.com

CHUYN LUYN THI I HC

www.luyenthi24h.com
www.luyenthi24h.com
www.VNMATH.com

Bi 5.321 : Cho y = 2x3 3(m + 3)x2 + 18mx 8. Tm m (Cm ) tip xc vi trc honh.

Bi 5.322 : Cho hm s y = x3 + k(x + 1) + 1. Tm tt c gi tr k ng thng y = x + 1 tip xc vi th hm s.

Vn 3 : Tip tuyn i qua mt im

Yu cu bi ton l vit phng trnh tip ca th hm s y = f (x) bit tip tuyn i qua A(xA ; yA ).
Cch 1 :

(a) Gi s tip tuyn c h s gc l k, khi phng trnh tip tuyn l y = k(x xA ) + yA .

8
< f (x) = k(x xA ) + yA
(b) Theo iu kin tip xc ta c h
:

f (x) = k.

(c) Thay k vo phng trnh u ta tm c x, th tr li tm c k, khi ta c phng trnh tip tuyn.


Cch 2 :

(a) Gi s honh tip im l x0 , khi phng trnh tip tuyn l y = f (x0 )(x x0 ) + y0 .
(b) Do tip tuyn i qua A, nn ta c yA = f (x0 )(xA x0 ) + y0 . T tm c x0 , v vy vit c phng trnh
tip tuyn.

Bi 5.323 (TN05) : Cho hm s y =


A(1; 3).

2x + 1
(C). Vit phng trnh tip tuyn ca (C), bit tip tuyn i qua im
x+1

Bi 5.324 : Vit phng trnh ng thng i qua im A(0; 1) v tip xc vi th hm s y = x3 3x2 + 2.

Bi 5.325 : Vit phng trnh tip tuyn ca y = 3x 4x3 qua A(1; 3).
1
Bi 5.326 : Cho y = x +
c th (C). Chng minh rng qua A(1; 1) ta lun v c n (C) hai tip tuyn vung
x+1
gc.
Bi 5.327 : Cho hm s y = x3 3x2 + 2 (C).

23
1. Vit phng trnh tip tuyn vi (C), bit tip tuyn xut pht t A
; 2 .
9
2. Tm trn ng thng y = 2 cc im k n (C) hai tip tuyn vung gc vi nhau.
Bi 5.328 : Cho hm s y = x3 + 3x + 2 (C). Tm trn trc honh cc im t k c 3 tip tuyn n (C).

Bi 5.329 : Cho hm s y = x3 12x + 12 (C). Tm trn ng thng y = 4 cc im k c 3 tip tuyn n (C).

Bi 5.330 : Cho hm s y = x3 12x + 12. Tm trn ng thng y = 4 nhng im t c th k c 3 tip tuyn

n th.

Bi 5.331 : Tm im A trn trc tung, sao cho qua A c th k c ba tip tuyn ti th hm s y = x4 + x2 1.


Bi 5.332 : Cho hm s y =
3
song vi d : y = x.
4

x2 6x + 9
(C). Tm cc im M trn trc tung t k c tip tuyn n (C) song
x + 2

Bi 5.333 : Cho hm s y =

x+1
c th (C). Tm nhng im trn trc tung m t mi im ch k c ng mt
x1

tip tuyn n (C).

Bi 5.334 : Tm trn ng thng y = 2x + 1 cc im m t k c ng 1 tip tuyn ti th hm s y =

TRN ANH TUN - 0974 396 391 - (04) 66 515 343

WWW.VNMATH.COM

x+3
.
x1

Trang 112

CHUYN LUYN THI I HC

www.VNMATH.com

Bi 5.335 : Tm trn trc tung cc im c th k c n th hm s y =


nhau.

www.luyenthi24h.com
www.luyenthi24h.com
www.VNMATH.com

2x2 + x + 1
hai tip tuyn vung gc vi
x+1

Bi 5.336 : Tm trn trc honh cc im k c ng mt tip tuyn ti th hm s y =

x2 + x 3
.
x+2

Vn 4 : Tip tuyn c h s gc cho trc


1. Nu tip tuyn c h s gc k th phng trnh honh tip im l f (x) = k.
1
2. Nu tip tuyn vung gc vi ng thng y = ax + b th h s gc k = .
a
3. Nu tip tuyn song song vi ng thng y = ax + b th h s gc k = a.
Trong trng hp ny ta8phi kim tra iu kin song song : Hai ng thng y = ax + b v y = a x + b song song
vi nhau khi v ch khi

<a = a
:

b , b .



ka

.
4. Tip tuyn hp vi ng thng y = ax + b mt gc th tan =
1 + ka

Bi 5.337 : Cho hm s y = x3 + 3x2 9x + 5. Trong s cc tip tuyn ca th hm s, hy tm tip tuyn c h s gc


b nht.

Bi 5.338 : Cho hm s y = x3 + 3x2 3x + 1. Trong s cc tip tuyn ca th hm s, hy tm tip tuyn c h s gc

ln nht.

Bi 5.339 : Vit phng trnh tip tuyn ca th hm s y = x3 3x2 , bit tip tuyn vung gc vi ng thng
1
y = x + 5.
3
Bi 5.340 : Vit phng trnh tip tuyn vi th hm s y = x3 3x2 + 1, bit tip tuyn song song vi ng thng
y = 9x + 2010.

3x 2
, bit tip tuyn to vi trc honh mt gc 45 .
x1
Bi 5.342 : Cho hm s y = 2x3 3x2 12x 5. Vit phng trnh tip tuyn vi th hm s, bit
Bi 5.341 : Vit phng trnh tip tuyn ca th hm s y =

1. Tip tuyn song song vi ng thng y = 6x 4.


1
2. Tip tuyn vung gc vi ng thng y = x + 2.
3
1
3. Tip tuyn to vi ng thng y = x + 5 mt gc 45 .
2
Bi 5.343 : Cho hm s y =

1 3
x 2x2 + x 4. Vit phng trnh tip tuyn vi th hm s, bit
3

1. Tip tuyn c h s gc bng 2.


2. Tip tuyn to vi chiu dng Ox mt gc 60 .
3. Tip tuyn to vi chiu dng Ox mt gc 45 .

TRN ANH TUN - 0974 396 391 - (04) 66 515 343

WWW.VNMATH.COM

Trang 113

www.VNMATH.com

CHUYN LUYN THI I HC

www.luyenthi24h.com
www.luyenthi24h.com
www.VNMATH.com

4. Tip tuyn to vi trc Ox mt gc 75 .


5. Tip tuyn song song vi ng thng y = x + 2.
6. Tip tuyn vung gc vi ng thng y = 2x 3.
7. Tip tuyn to vi ng thng y = 3x + 7 gc 45 .
1
8. Tip tuyn to vi ng thng y = x + 3 gc 30 .
2
Bi 5.344 : Vit phng trnh tip tuyn ca th hm s y =
y = 2x 1.

1 4 1 3 1 2
x x + x + x 5 song song vi ng thng
4
3
2

1
Bi 5.345 : Vit phng trnh tip tuyn ca th hm s y = x4 2x2 + 4x 1 song song vi ng thng y = x + 3.
4
2
x x1
.
Bi 5.346 : Vit phng trnh tip tuyn song song vi ng thng y = x ca th hm s y =
x+1
x2 + 3x + 4m
Bi 5.347 : Cho hm s y =
. Vi nhng gi tr no ca m th th ca hm s c tip tuyn vung gc vi
x+1
ng phn gic th nht.
2x 3
Bi 5.348 : Vit phng trnh tip tuyn ca th hm s y =
, bit tip tuyn vung gc vi ng thng
5x 4
y = 2x.
4x 3
Bi 5.349 : Vit phng trnh tip tuyn ca th hm s y =
, bit tip tuyn to vi ng thng y = 3x mt
x1

gc 45 .

5.8 Hm s trong cc k thi tuyn sinh H


Bi 5.350 (C08) : Cho hm s y =

x
.
x1

1. Kho st s bin thin v v th (C) ca hm s cho.


2. Tm m ng thng d : y = x + m ct th (C) ti hai im phn bit.
Bi 5.351 (C09) : Cho hm s y = x3 (2m 1)x2 + (2 m)x + 2 (1), vi m l tham s thc.
1. Kho st s bin thin v v th hm s (1) khi m = 2.
2. Tm cc gi tr ca tham s m hm s (1) c cc i, cc tiu v cc im cc tr ca th hm s (1) c honh
dng.
Bi 5.352 (C10) :

1. Kho st s bin thin v v th (C) ca hm s y = x3 + 3x2 1.

2. Vit phng trnh tip tuyn ca th (C) ti im c honh bng 1.

Bi 5.353 (A02) : Cho hm s : y = x3 + 3mx2 + 3(1 m2 )x + m3 m2 (1) (m l tham s).


1. Kho st s bin thin v v th hm s (1) khi m = 1.
2. Tm k phng trnh : x3 + 3x2 + k3 3k2 = 0 c ba nghim phn bit.
3. Vit phng trnh ng thng i qua hai im cc tr ca th hm s (1).
Bi 5.354 (A03) : Cho hm s : y =

mx2 + x + m
(1) (m l tham s ).
x1

TRN ANH TUN - 0974 396 391 - (04) 66 515 343

WWW.VNMATH.COM

Trang 114

www.VNMATH.com

CHUYN LUYN THI I HC

www.luyenthi24h.com
www.luyenthi24h.com
www.VNMATH.com

1. Kho st s bin thin v v th hm s (1) khi m = 1.


2. Tm m th hm s (1) ct trc honh ti hai im phn bit v hai im c honh dng.
Bi 5.355 (A04) : Cho hm s : y =
1. Kho st hm s (1).

x2 + 3x 3
(1).
2(x 1)

2. Tm m ng thng y = m ct th hm s (1) ti hai im A, B sao cho AB = 1.


Bi 5.356 (A05) : Gi (Cm ) l th hm s : y = mx +

1
(*) (m l tham s).
x

1. Kho st s bin thin v v th hm s (*) khi m =

1
.
4

1
2. Tm m hm s (*) c cc tr v khong cch t cc im cc tiu ca (Cm ) n tim cn xin ca (Cm ) bng .
2
Bi 5.357 (A06) :

1. Kho st s bin thin v v th ca hm s : y = 2x3 9x2 + 12x 4.

2. Tm m phng trnh sau c 6 nghim phn bit : 2|x|3 9x2 + 12|x| = m.


Bi 5.358 (A07) : Cho hm s : y =

x2 + 2(m + 1)x + m2 + 4m
(1), m l tham s.
x+2

1. Kho st s bin thin v v th hm s (1) khi m = 1.


2. Tm m hm s (1) c cc i cc tiu, ng thi cc im cc tr ca th cng vi gc to O to thnh mt
tam gic vung ti O.
Bi 5.359 (A08) : Cho hm s y =

mx2 + (3m2 2)x 2


(1), vi m l tham s thc.
x + 2m

1. Kho st s bin thin v v th hm s (1) khi m = 1.


2. Tm cc gi tr ca m gc gia hai ng tim cn ca th hm s (1) bng 45 ,
Bi 5.360 (A09) : Cho hm s y =

x+2
(1).
2x + 3

1. Kho st s bin thin v v th hm s (1).


2. Vit phng trnh tip tuyn ca th hm s (1), bit tip tuyn ct trc honh, trc tung ln lt ti hai im
phn bit A v B v tam gic OAB cn ti gc ta O.
Bi 5.361 (A10) : Cho hm s y = x3 2x2 + (1 m)x + m (1), m l tham s thc.
1. Kho st s bin thin v v th hm s khi m = 1.
2. Tm m th hm s (1) ct trc honh ti 3 im phn bit c honh x1 , x2 , x3 tha mn iu kin x21 + x22 + x33 <
4.
Bi 5.362 (B02) : Cho hm s : y = mx4 + (m2 9)x2 + 10 (1) (m l tham s).
1. Kho st s bin thin v v th hm s (1) khi m = 1.
2. Tm m hm s (1) c ba cc tr.
Bi 5.363 (B03) : Cho hm s : y = x3 3x2 + m (1) (m l tham s).

TRN ANH TUN - 0974 396 391 - (04) 66 515 343

WWW.VNMATH.COM

Trang 115

www.VNMATH.com

CHUYN LUYN THI I HC

www.luyenthi24h.com
www.luyenthi24h.com
www.VNMATH.com

1. Tm m th hm s (1) c hai im phn bit i xng vi nhau qua gc to .


2. Kho st s bin thin v v th hm s (1) khi m = 2.
Bi 5.364 (B03) : Tm gi tr ln nht v nh nht ca hm s : y = x +
1
Bi 5.365 (B04) : Cho hm s : y = x3 2x2 + 3x (1) c th (C).
3

4 x2 .

1. Kho st hm s (1).
2. Vit phng trnh tip tuyn ca (C) ti im un v chng minh rng l tip tuyn ca (C) c h s gc nh
nht.
Bi 5.366 (B04) : Tm gi tr ln nht v gi tr nh nht ca hm s y =
Bi 5.367 (B05) : Gi (Cm ) l th hm s : y =

ln2 x
trn on [1; e3 ].
x

x2 + (m + 1)x + m + 1
() (m l tham s).
x+1

1. Kho st s bin thin v v th hm s (*) khi m = 1.


2. Chng minh rng vi m bt k, th (Cm ) lun c im cc i, im cc tiu v khong cch gia hai im

bng 20.
Bi 5.368 (B06) : Cho hm s : y =

x2 + x 1
.
x+2

1. Kho st s bin thin v v th (C) ca hm s cho.


2. Vit phng trnh tip tuyn ca th (C), bit tip tuyn vung gc vi tim cn xin ca th (C).
Bi 5.369 (B07) : Cho hm s : y = x3 + 3x2 + 3(m2 1)x 3m2 1 (1), m l tham s.
1. Kho st s bin thin v v th hm s (1) khi m = 1.
2. Tm m hm s (1) c cc i, cc tiu v cc im cc tr ca th hm s (1) cch u gc to O.
Bi 5.370 (B08) : Cho hm s : y = 4x3 6x2 + 1 (1).
1. Kho st s bin thin v v th hm s (1).
2. Vit phng trnh tip tuyn ca th hm s (1), bit rng tip tuyn i qua im (1; 9).
Bi 5.371 (B09) : Cho hm s y = 2x4 4x2 (1).
1. Kho st s bin thin v v th hm s (1).
2. Vi cc gi tr no ca m, phng trnh x2 |x2 2| = m c ng 6 nghim thc phn bit.
Bi 5.372 (B09) : Tm cc gi tr ca tham s m ng thng y = x + m ct th hm s y =
phn bit A v B sao cho AB = 4.

Bi 5.373 (B10) : Cho hm s y =

x2 1
ti hai im
x

2x + 1
.
x+1

1. Kho st s bin thin v v th (C) ca hm s cho.


2. Tm m ng thng y = 2x + m ct th (C) ti hai im phn bit A, B sao cho tam gic OAB c din tch

bng 3 (O l gc ta ).

TRN ANH TUN - 0974 396 391 - (04) 66 515 343

WWW.VNMATH.COM

Trang 116

www.VNMATH.com

CHUYN LUYN THI I HC

Bi 5.374 (D02) : Cho hm s : y =

www.luyenthi24h.com
www.luyenthi24h.com
www.VNMATH.com

(2m 1)x m2
(1) (m l tham s).
x1

1. Kho st s bin thin v v th (C) ca hm s (1) ng vi m = 1.


2. Tnh din tch hnh phng gii hn bi ng cong (C) v hai trc to .
3. Tm m th hm s (1) tip xc vi ng thng y = x.
Bi 5.375 (D03) :

1. Kho st s bin thin v v th hm s : y =

x2 2x + 4
(1).
x2

2. Tm m ng thng dm : y = mx + 2 2m ct th ca hm s (1) ti hai im phn bit.


x+1
Bi 5.376 (D03) : Tm gi tr ln nht, gi tr nh nht ca hm s : y =
trn on [1; 2].
x2 + 1
Bi 5.377 (D04) : Cho hm s : y = x3 3mx2 + 9x + 1 (1) vi m l tham s.
1. Kho st hm s (1) khi m = 2.
2. Tm m im un ca th hm s (1) thuc ng thng y = x + 1.
Bi 5.378 (D05) : Gi (Cm ) l th ca hm s : y =

1 3 m 2 1
x x + () (m l tham s).
3
2
3

1. Kho st s bin thin v v th hm s (*) khi m = 2.


2. Gi M l im thuc (Cm ) c honh bng 1. Tm m tip tuyn ca (Cm ) ti im M song song vi ng
thng 5x y = 0.

Bi 5.379 (D06) : Cho hm s : y = x3 3x + 2.


1. Kho st s bin thin v v th (C) ca hm s cho.
2. Gi d l ng thng i qua im A(3; 20) v c h s gc l m. Tm m ng thng d ct th (C) ti ba im
phn bit.
Bi 5.380 (D07) : Cho hm s : y =

2x
.
x+1

1. Kho st s bin thin v v th (C) ca hm s cho.


2. Tm to im M thuc (C), bit tip tuyn ca (C) ti M ct hai trc Ox, Oy ti A, B v tam gic OAB c din tch
1
bng .
4
Bi 5.381 (D08) : Cho hm s : y = x3 3x2 + 4 (1).
1. Kho st s bin thin v v th hm s (1).
2. Chng minh rng mi ng thng i qua im I(1; 2) vi h s gc k (k > 3) u ct th ca hm s (1) ti ba
im phn bit I, A, B ng thi I l trung im ca on AB.

Bi 5.382 (D09) : Cho hm s y = x4 (3m + 2)x2 c th l (Cm ), m l tham s.


1. Kho st s bin thin v v th hm s cho khi m = 0.
2. Tm m ng thng y = 1 ct th (Cm ) ti 4 im phn bit u c honh nh hn 2.
Bi 5.383 (D09) : Tm cc gi tr ca tham s m ng thng y = 2x + m ct th hm s y =
phn bit A, B sao cho trung im ca on thng AB thuc trc tung.

TRN ANH TUN - 0974 396 391 - (04) 66 515 343

WWW.VNMATH.COM

x2 + x 1
ti hai im
x
Trang 117

www.VNMATH.com

CHUYN LUYN THI I HC

www.luyenthi24h.com
www.luyenthi24h.com
www.VNMATH.com

Bi 5.384 (D10) : Cho hm s y = x4 x2 + 6.


1. Kho st s bin thin v v th (C) ca hm s cho.
2. Vit phng trnh tip tuyn ca th (C), bit tip tuyn vung gc vi ng thng y =

1
x 1.
6

Bi 5.385 : Cho hm s : y = x4 mx2 + m 1 (1) (m l tham s).


1. Kho st s bin thin v v th hm s (1) khi m = 8.
2. Xc nh m sao cho th hm s (1) ct trc honh ti bn im phn bit.
Bi 5.386 : Cho hm s : y =

x2 2x + m
(1) (m l tham s).
x2

1. Xc nh m hm s (1) nghch bin trn on [1; 0].


2. Kho st s bin thin v v th hm s (1) khi m = 1.
Bi 5.387 : Cho hm s : y =

1 3
1
x + mx2 2x 2m (1) (m l tham s).
3
3

1
1. Cho m = .
2
(a) Kho st s bin thin v v th (C) ca hm s (1).
(b) Vit phng trnh tip tuyn ca th (C), bit rng tip tuyn song song vi ng thng d : y = 4x + 2.

5
sao cho hnh phng gii hn bi th ca hm s (1) v cc ng thng : x = 0, x = 2, y = 0
6
c din tch bng 4.

2. Tm m thuc 0;

Bi 5.388 : Cho hm s : y = (x m)3 3x (m l tham s).


1. Xc nh m hm s t cc tiu ti im c honh x = 0.
2. Kho st s bin thin v v th hm s cho khi m = 1.
3. Tm k h bt phng trnh sau c nghim :
8
>
<

|x 1|3 3x k < 0
1
1
>
: log2 x2 + log2 (x 1)3 1.
2
3
Bi 5.389 : Cho hm s : y =

x2 + mx
(1) (m l tham s).
1x

1. Kho st s bin thin v v th hm s (1) khi m = 0.


2. Tm m hm s (1) c cc i, cc tiu. Vi gi tr no ca m th khong cch gia hai im cc tr ca th hm
s (1) bng 10 ?
Bi 5.390 :

1. Kho st s bin thin v v th hm s : y =

1 3
x 2x2 + 3x (1).
3

2. Tnh din tch hnh phng gii hn bi th hm s (1) v trc honh.


Bi 5.391 :

1. Kho st s bin thin v v th hm s : y =

2x2 4x 3
.
2(x 1)

2. Tm m phng trnh 2x2 4x 3 + 2m|x 1| = 0 c hai nghim phn bit.

TRN ANH TUN - 0974 396 391 - (04) 66 515 343

WWW.VNMATH.COM

Trang 118

CHUYN LUYN THI I HC

www.VNMATH.com

Bi 5.392 : Tm gi tr ln nht v gi tr nh nht ca hm s : y = sin5 x +


Bi 5.393 : Cho hm s : y =

x2

www.luyenthi24h.com
www.luyenthi24h.com
www.VNMATH.com

3 cos x.

m2

+ (2m + 1)x +
+m+4
(1) (m l tham s).
2(x + m)

1. Tm m hm s (1) c cc tr v tnh khong cch gia hai im cc tr ca th hm s (1).


2. Kho st s bin thin v v th hm s (1) khi m = 0.
Bi 5.394 : Cho hm s : y = (x 1)(x2 + mx + m) (1) (m l tham s).
1. Tm m th hm s (1) ct trc honh ti ba im phn bit.
2. Kho st s bin thin v v th hm s (1) khi m = 4.
Bi 5.395 : Tm gi tr ln nht v gi tr nh nht ca hm s : y = x6 + 4(1 x2 )3 , vi x [1; 1].
2x 1
Bi 5.396 : Cho hm s : y =
(1).
x1
1. Kho st s bin thin v v th (C) ca hm s (1).

2. Gi I l giao im hai ng tim cn ca (C). Tm im M thuc (C) sao cho tip tuyn ca (C) ti M vung gc
vi ng thng I M.
Bi 5.397 : Chng minh rng : ex + cos x 2 + x
Bi 5.398 : Cho hm s : y =

x2
, x R.
2

x2 + 5x + m2 + 6
(1) (m l tham s).
x+3

1. Kho st s bin thin v v th hm s (1) khi m = 1.


2. Tm m hm s (1) ng bin trn khong (1; +).
Bi 5.399 :

1. Kho st s bin thin v v th hm s : y = 2x3 3x2 1.

2. Gi dk l ng thng i qua im M(0; 1) v c h s gc bng k. Tm k ng thng dk ct (C) ti ba im


phn bit.

Bi 5.400 : Gi (Cm ) l th hm s : y =

x2 + 2mx + 1 3m2
() (m l tham s).
xm

1. Kho st s bin thin v v th hm s (*) ng vi m = 1.

2. Tm m th hm s (*) c hai im cc tr nm v hai pha trc tung.


Bi 5.401 :

1. Kho st s bin thin v v th (C) ca hm s : y =

x2 + x + 1
.
x+1

2. Vit phng trnh ng thng qua M(1; 0) v tip xc vi th (C).


Bi 5.402 :

1. Kho st s bin thin v v th (C) ca hm s : y = x4 6x2 + 5.

2. Tm m phng trnh sau c 4 nghim phn bit : x4 6x2 log2 m = 0.


Bi 5.403 : Cho hm s : y =

x2 + 2x + 2
().
x+1

1. Kho st s bin thin v v th (C) ca hm s (*).


2. Gi I l giao im ca hai tim cn ca (C). Chng minh rng khng c tip tuyn no ca (C) i qua im I.
Bi 5.404 : Gi Cm l th hm s y = x3 + (2m + 1)x2 m 1 (1) (m l tham s).

TRN ANH TUN - 0974 396 391 - (04) 66 515 343

WWW.VNMATH.COM

Trang 119

CHUYN LUYN THI I HC

www.VNMATH.com

www.luyenthi24h.com
www.luyenthi24h.com
www.VNMATH.com

1. Kho st s bin thin v v th hm s (1) khi m = 1.


2. Tm m th (Cm ) tip xc vi ng thng y = 2mx m 1.
Bi 5.405 :

1. Kho st s bin thin v v th ca hm s : y =

2. Tm m phng trnh
Bi 5.406 :

x2 + 3x + 3
.
x+1

x2 + 3x + 3
= m c bn nghim phn bit.
|x + 1|

1. Kho st s bin thin v v th (C) ca hm s : y =

x2 + 2x + 5
.
x+1

2. Da vo th (C), tm m phng trnh sau y c hai nghim dng phn bit :


x2 + 2x + 5 = (m2 + 2m + 5)(x + 1).
Bi 5.407 :

1. Kho st s bin thin v v th (C) ca hm s : y =

x4
2(x2 1).
2

2. Vit phng trnh ng thng i qua im A(0; 2) v tip xc vi (C).


Bi 5.408 : Cho hm s : y = x3 + (1 2m)x2 + (2 m)x + m + 2 (1).
1. Kho st s bin thin v v th hm s (1) khi m = 2.
2. Tm cc gi tr ca m th hm s (1) c im cc i, im cc tiu, ng thi honh ca im cc tiu nh
hn 1.
Bi 5.409 : Cho hm s : y =

x2 x 1
.
x+1

1. Kho st s bin thin v v th (C) ca hm s cho.


2. Vit phng trnh cc tip tuyn ca th (C), bit tip tuyn i qua A(0; 5).
Bi 5.410 : Cho hm s : y =

x3
11
+ x2 + 3x .
3
3

1. Kho st s bin thin v v th (C) ca hm s cho.


2. Tm trn th (C) hai im phn bit M, N i xng nhau qua trc tung.
Bi 5.411 : Cho hm s : y =

x+3
(C).
x1

1. Kho st s bin thin v v th (C) ca hm s cho.


2. Cho im M0 (x0 ; y0 ) (C). Tip tuyn ca (C) ti M0 ct cc tim cn ca (C) ti cc im A v B. Chng minh M0
l trung im ca on AB.

Bi 5.412 : Cho hm s : y =

x2 + 4x + 3
.
x2

1. Kho st s bin thin v v th hm s cho.


2. Chng minh rng tch cc khong cch t mt im bt k trn th hm s n cc ng tim cn ca n l hng
s.
Bi 5.413 : Cho hm s : y = x + m +

m
(Cm ).
x2

1. Kho st s bin thin v v th hm s vi m = 1.

TRN ANH TUN - 0974 396 391 - (04) 66 515 343

WWW.VNMATH.COM

Trang 120

www.VNMATH.com

CHUYN LUYN THI I HC

www.luyenthi24h.com
www.luyenthi24h.com
www.VNMATH.com

2. Tm m th (Cm ) c cc tr ti hai im A, B sao cho ng thng AB i qua gc to O.


Bi 5.414 : Cho hm s : y = 2x3 + 6x2 5.
1. Kho st s bin thin v v th (C) ca hm s cho.
2. Vit phng trnh tip tuyn ca (C), bit tip tuyn i qua im A(1; 13).
Bi 5.415 : Cho hm s : y = x + 1 +

m
(Cm ).
2x

1. Kho st s bin thin v v th hm s vi m = 1.


2. Tm m th (Cm ) c cc i ti im A sao cho tip tuyn vi (Cm ) ct trc Oy ti B m tam gic OBA vung
cn.
Bi 5.416 : Cho hm s : y =

x + 1
(C).
2x + 1

1. Kho st s bin thin v v th ca hm s cho.


2. Vit phng trnh tip tuyn vi (C), bit rng tip tuyn i qua giao im ca ng tim cn v trc Ox.
Bi 5.417 : Cho hm s : y =

x
(C).
x1

1. Kho st s bin thin v v th ca hm s cho.


2. Vit phng trnh tip tuyn d ca (C) sao cho d v hai tim cn ca (C) ct nhau to thnh mt tam gic cn.
Bi 5.418 (TN2008) : Cho hm s y =

2x 1
, gi th ca hm s l (C).
x1

1. Kho st s bin thin v v th ca hm s cho.

2. Vit phng trnh tip tuyn ca th (C) ti im A(2; 3).

5.9 Bi tp tng hp
Bi 5.419 : Cho hm s y = 2x3 9x2 + 12x 4 c th l (C ).
1. Kho st s bin thin v v th ca hm s cho.
2. Tm m
(a) Phng trnh 2x3 9x2 + 12x + log2 (m 1) = 0 c ng 3 nghim thc phn bit.
(b) Phng trnh 2|x|3 9x2 + 12|x| + m = 0 c ng 6 nghim thc phn bit.
(c) Phng trnh |2x3 9x2 + 12x 4| + 2m 1 = 0 c nhiu hn ba nghim thc phn bit.
(d) Phng trnh sin 3x 9 cos 2x 27 sin x + 4m 5 = 0 c nghim.
3. Vit phng trnh tip tuyn ca th hm s, bit
(a) Honh tip im bng -1.
(b) Tung tip im bng 1.
(c) Tip tuyn i qua A(0; 1) v honh tip im l s nguyn.
(d) H s gc ca tip tuyn bng 12.

TRN ANH TUN - 0974 396 391 - (04) 66 515 343

WWW.VNMATH.COM

Trang 121

www.VNMATH.com

CHUYN LUYN THI I HC

www.luyenthi24h.com
www.luyenthi24h.com
www.VNMATH.com

(e) H s gc ca tip tuyn t gi tr nh nht.


(f) Tip tuyn song song vi ng thng y 36x + 5 = 0.
4. Tm trn th hm s hai im phn bit i xng nhau qua im E(1; 2).
5. Gi k l h s gc ca ng thng d i qua
B(2; 0). Tm k ng thng d ct (C ) ti hai im phn bit khc B
2
sao cho khong cch gia hai im bng
.
2
6. Tnh din tch hnh phng gii hn bi (C ) v trc honh.
7. Tnh th tch ca khi trn xoay khi quay hnh phng gii hn bi (C ) v trc honh quanh trc Ox.
Bi 5.420 : Cho hm s y = x3 + 3x2 + 3(m2 1)x 3m2 1 c th l (Cm ).
1. Kho st s bin thin v v th ca hm s cho khi m = 1.
2. Tm m hm s c cc tr v
(a) Hai im cc tr ca th hm s cch u gc ta .

(b) Khong cch gia hai im cc tr bng 2 5.


11
, bit m > 0.
5
(d) Chng minh rng khi hai im cc tr cch u ng thng x = 1. Vit phng trnh ng thng qua hai
(c) Khong cch t im cc tiu n ng thng 3x + 4y + 5 = 0 bng
im cc tr .
3. Tm m hm s
(a) Nghch bin trn tp xc nh.
(b) Nghch bin trn (; 2).
(c) ng bin trn (3; 5).
4. Tm m trn (Cm ) c hai im phn bit i xng nhau qua im E(1; 2).
5. Tm m (Cm ) ct trc honh ti 3 im phn bit.
Bi 5.421 : Cho h ng cong bc ba (Cm ) v h ng thng dk ln lt c phng trnh l
y = x3 + mx2 m v y = kx + k + 1.
1. Vi m = 1. Kho st v v th (C) ca hm s.
(a) Gi A v B l 2 im cc i v cc tiu ca (C) v M l im bt k trn cung AB vi M khc A, B. Chng
minh rng trn (C) ta tm c hai im ti c tip tuyn vung gc vi tip tuyn ti M vi (C).
(b) Gi l ng thng c phng trnh y = 1. Bin lun s tip tuyn vi (C) v t E vi (C).
(c) Tm E qua E c ba tip tuyn vi (C) v c hai tip tuyn vung gc vi nhau.
(d) nh p trn (C) c 2 tip tuyn c h s gc bng p, trong trng hp ny chng t trung im ca hai tip
im l im c nh.
(e) Tm M (C) qua M ch c mt tip tuyn vi (C).
2. Vi m l tham s

TRN ANH TUN - 0974 396 391 - (04) 66 515 343

WWW.VNMATH.COM

Trang 122

www.VNMATH.com

CHUYN LUYN THI I HC

www.luyenthi24h.com
www.luyenthi24h.com
www.VNMATH.com

(a) Tm im c nh ca (Cm ). nh m hai tip tuyn ti hai im c nh ny vung gc nhau.


(b) nh m (Cm ) c 2 im cc tr. Vit phng trnh ng thng qua 2 im cc tr.
(c) nh m (Cm ) ct Ox ti 3 im phn bit.
(d) nh m hm s ng bin trong (1; 2).
(e) nh m hm s nghch bin trong (0; +).
(f) Tm m (Cm ) ct Ox ti 3 im c honh to thnh cp s cng.
(g) Tm iu kin gia k v m dk ct (Cm ) ti 3 im phn bit. Tm k dk ct (Cm ) thnh hai on bng nhau.
(h) Vit phng trnh tip tuyn vi (Cm ) v i qua im (1; 1).
(i) Chng minh rng trong cc tip tuyn vi (Cm ) th tip tuyn ti im un c h s gc ln nht.
Bi 5.422 : Cho hm s y = x4 + 8ax3 4(1 + 2a)x2 + 3 c th l (Ca ).
1. Khi a = 0 :
(a) Kho st s bin thin v v th (C0 ).
(b) Xc nh m tip tuyn vi (C0 ) ti M c honh m ct (C0 ) ti hai im P, Q khc im M. Tm qu tch
trung im I ca PQ khi M thay i. Xc nh m m l trung im PQ.
1
2. Khi a = :
2
(a) Kho st s bin thin v v th (C 1 ).
2

(b) Vit phng trnh tip tuyn vi (C 1 ) v c h s gc bng -8. Tm ta cc tip im.
2

3. Khi a cha bit :


(a) Bin lun theo a s im cc tr ca hm s. Tm a hm s ch c im cc tiu m khng c im cc i.
(b) Trong trng hp th hm s c 3 im cc tr, hy vit phng trnh parabol i qua ba im cc tr ny.
Bi 5.423 : Cho hm s y =
1. Khi m = 1 :

(m + 1)x2 2mx (m3 m2 2)


c th (Cm ).
xm

(a) Gi dm l ng thng c phng trnh y = 2x + m. Chng minh rng dm lun ct (C1 ) ti hai im A, B phn
bit. Xc nh m AB ngn nht.

(b) Tim hai im M, N thuc hai nhnh ca (C1 ) khong cch MN l ngn nht.
(c) Tm M thuc (C1 ) I M l ngn nht vi I l giao im hai ng tim cn. Trong trng hp ny hy chng
t tip tuyn vi (C1 ) ti M s vung gc vi I M.

2. Khi m = 1 :
(a) Bin lun theo k s tip tuyn k t K(0; k) n (C1 );
(b) Tm cc im trn Ox cc im t v c ng 1 tip tuyn vi (C1 );
(c) Gi l tip tuyn ti im J thuc (C1 ), ct hai ng tim cn ti E, F. Chng minh rng khi J l trung
im EF v tam gic IEF c din tch khng i.

TRN ANH TUN - 0974 396 391 - (04) 66 515 343

WWW.VNMATH.COM

Trang 123

www.VNMATH.com

CHUYN LUYN THI I HC

www.luyenthi24h.com
www.luyenthi24h.com
www.VNMATH.com

Bi 5.424 : Tm cc gi tr ca tham s m th hm s y = x3 + 3x2 4 ct ng thng y = mx + 2 ti ba im phn

bit c honh lp thnh mt cp s cng.

Bi 5.425 : Tm cc gi tr ca tham s m th hm s y =
A, B sao cho OAOB.

x2 + 3x 2
ct ng thng y = m ti hai im phn bit
x+1

Bi 5.426 : Tm ta im M thuc ng thng y = 3x + 2 sao cho t M c th k c hai tip tuyn n th hm


s y = x3 3x + 2 v hai tip tuyn vung gc vi nhau.

Bi 5.427 : Chng minh rng vi ba im A, B, C phn bit thuc th hm s y =


cng thuc th hm s ny.

x+1
th trc tm ca tam gic ABC
x2

Bi 5.428 : Chng minh rng vi mi m , 0 th th hm s y = x4 (m2 + 10)x2 + 9 lun ct trc honh ti bn im


phn bit trong c hai im nm trong khong (3; 3) v hai im nm ngoi khong (3; 3).

Bi 5.429 : Tm m hm s y = x3 3(m + 1)x2 + 9x m t cc tr ti hai im x1 , x2 sao cho |x1 x2 | 2.


x+1
Bi 5.430 : Tm cc tip tuyn ca th hm s y =
, bit cc tip tuyn song song vi ng thng 3x + y = 0.
x2
mx
Bi 5.431 : Tm m ng thng 2x + 2y 1 = 0 ct th hm s y =
ti hai im cng vi gc ta to thnh
x+2
3
mt tam gic c din tch bng .
8
x
, bit tip tuyn vi th hm s ny ti M ct hai trc Ox, Oy ti
Bi 5.432 : Tm im M thuc th hm s y =
x1
hai im A, B v tam gic OAB c s o cc gc lp thnh mt cp s cng.
2x2 + 3x 3
v cch u hai ng tim cn.
x1
mx2 + (m2 + 1)x + 4m2 + m
Bi 5.434 : Tm cc gi tr ca tham s m th hm s y =
c mt im cc tr thuc gc
x+m
phn t th II v mt im cc tr thuc gc phn t th IV ca mt phng ta .
2x + 1
x
Bi 5.435 : Tm cc im M trn th hm s y =
sao cho khong cch t M n ng thng y = + 2 c gi
x+1
4
tr nh nht.
Bi 5.433 : Tm cc im thuc th hm s y =

Bi 5.436 : Tm tt c cc gi tr ca m th hm s y = x3 3x2 + mx c cc im cc i v cc tiu i xng nhau


qua ng thng x 2y 5 = 0.

1
2
(m + 1)x3 mx2 + 2(m 1)x t cc i, cc tiu ti x1 , x2 tha mn 2x1 + x2 = 1.
3
3
4
Bi 5.438 : Tm tt c cc gi tr ca m th hm s y = x 2m(m 1)x2 + m + 1 c ba im cc tr to thnh ba nh
Bi 5.437 : Tm m hm s y =

ca mt tam gic vung.

Bi 5.439 : Cho hm s y = 8x4 9x2 + 1.


1. Kho st s bin thin v v th (C) ca hm s.
2. Hy bin lun theo m s nghim ca phng trnh
8 cos4 x 9 cos2 x + m = 0 vi x [0; ].
Bi 5.440 : Tm m ng thng x + y m + 1 = 0 ct th hm s y =
IAIB, vi I(1; 4).

2x + 3
ti hai im phn bit A v B sao cho
x+1

Bi 5.441 : Tm cc im trn th hm s y = 2x3 3x2 + 1 nhng im M sao cho tip tuyn ti M ct trc tung ti
im c tung bng 8.

TRN ANH TUN - 0974 396 391 - (04) 66 515 343

WWW.VNMATH.COM

Trang 124

www.VNMATH.com

CHUYN LUYN THI I HC

Bi 5.442 : Chng minh rng mi tip tuyn ca th hm s y =


khng i.

www.luyenthi24h.com
www.luyenthi24h.com
www.VNMATH.com

x+1
to vi hai tim cn mt tam gic c din tch
x1

Bi 5.443 : Tm a, b ng thng y = ax + b ct th hm s y =
ng thng x 2y + 3 = 0.

x1
ti hai im phn bit i xng nhau qua
x+1

Bi 5.444 : Vit phng trnh tip tuyn vi th hm s y = x4 2x2 + 2, bit tip tuyn ny i qua im A(0; 2).
1
Bi 5.445 : Tm cc gi tr ca m th hm s y = x3 mx2 + (2m 1)x m + 2 c hai im cc tr vi honh
3
dng.
Bi 5.446 : Tm m th hm s y = x4 2(mx)2 + 1 c ba im cc tr l ba nh ca mt tam gic vung.
2x + 1
sao cho tng khong cch t im n hai tim cn l nh
Bi 5.447 : Tm nhng im trn th hm s y =
x+1
nht.
Bi 5.448 : Tm m th hm s y = (m + 1)x3 3(m + 1)x2 + 2mx m c cc im cc i, cc tiu. Chng minh rng
khi hai im cc tr lun cch u ng thng x = 1.

Bi 5.449 : Tm m th hm s y = mx3 3mx2 + 2(m 1)x 1 m c cc im cc i, cc tiu. Chng minh rng

khi hai im cc tr lun cch u ng thng x = 1.


Bi 5.450 : Cho hm s y = x3 3x + 2 (1).
1. Kho st s bin thin v v th hm s (1).

2. Tm a > 0 phng trnh |x3 3x + 2| = log2 a c ng bn nghim thc phn bit.


Bi 5.451 : Tm tt c cc gi tr ca m hm s y = 2x3 + 9mx2 + 12m2 x + 1 t cc i ti x1 , t cc tiu ti x2 tha
mn x21 = x2 .
Bi 5.452 : Lp phng trnh tip tuyn ca th hm s y =
A v B tha mn OA = 4OB.

2x 1
, bit tip tuyn ny ct cc trc Ox, Oy ln lt ti
x1

Bi 5.453 : Chng minh rng ng thng y = x + 1 lun ct th hm s y = x4 + 2m2 x2 + 1 ti hai im phn bit vi
mi gi tr ca m.
Bi 5.454 : Chng minh rng vi mi gi tr ca m, th hm s y = 2x3 3(2m + 1)x2 + 6m(m + 1)x + 1 lun c cc
i, cc tiu v khong cch gia hai im cc i, cc tiu l khng i.

Bi 5.455 : Tm cc gi tr ca m ng thng y = mx m + 2 ct th hm s y =

v on AB c di nh nht.

2x
ti hai im phn bit A, B
x1

Bi 5.456 : Tm cc gi tr ca tham s m th hm s y = x4 2mx2 + 3m + 1 c im cc i, im cc tiu, ng

thi cc im cc i, cc tiu lp thnh tam gic c din tch bng 1.


2x 1
Bi 5.457 : Tm im M thuc d th hm s y =
sao cho khong cch t im I(1; 2) ti tip tuyn ti M l ln
x+1
nht.
x2 (m + 5)x + m
Bi 5.458 : Tm cc gi tr ca tham s m th hm s y =
ct trc honh ti hai im phn bit c
x1
honh ln lt l x1 , x2 sao cho T = |x1 x2 | t gi tr nh nht.
Bi 5.459 : Tm cc gi tr ca tham s m th hm s y =
th ti im vung gc vi ng thng y = x.

x2 2mx + m2
tn ti t nht mt im m tip tuyn ca
x1

WWW.VNMATH.COM
TRN ANH TUN - 0974 396 391 - (04) 66 515 343

WWW.VNMATH.COM

Trang 125

www.luyenthi24h.com
www.luyenthi24h.com

Chng 6

M v lgart
6.1 Hm s m, hm s ly tha
Bi 6.1 : Rt gn biu thc sau trong min xc nh ca n :

x . 3 x y
x + y
:
1. P =
.
(x2 xy) x x y y
2
4

2. Q = a3 4

a+


3. R = x + y : x
:

4 2 4
4 2 3

b +
a b
5 . 3 a. a.

a + ab

x y
y

+
.

x
x y

2 3x
3
4. T =
3
x2 + 8x + 16.
x 4x x x 4 x

Bi 6.2 : Cho x < 0, chng minh rng :

1 x
(2 2x )2 1 2x
4
r
=
.
1 + 2x
1 x
2
x
1 + 1 + (2 2 )
4

1 +

1+

Bi 6.3 : Tm cc khong ng bin v nghch bin ca hm s y =


Bi 6.4 : Xt hm s f (x) =

2x + 2x
.
2

2x + 2x
2x 2x
v g(x) =
. Chng minh rng vi mi x1 , x2 ta c cc h thc sau :
2
2

1. f (x1 + x2 ) + f (x1 x2 ) = 2 f (x1 ) f (x2 ).


2. g(2x1 ) = 2g(x1 ) f (x1 ).
3. f (2x1 ) = 2 f 2 (x1 ) 1.
Bi 6.5 : Cho hm s f (x) =

4x
. Tnh tng : S = f
4x + 2

1
+f
1993

2
+ + f
1993

1992
.
1993

6.2 Hm s logarit
Bi 6.6 : Tnh cc i lng sau :
1. A = 92 log3 4+4 log81 2 .

2. B = loga

5 !
a2 . 3 a. a4
4
, vi a > 0, a , 1.
a

Bi 6.7 : Cho log12 27 = a. Tnh theo a gi tr ca log6 16.


127

WWW.VNMATH.COM

www.luyenthi24h.com
www.luyenthi24h.com
www.VNMATH.com

www.VNMATH.comCHUYN LUYN THI I HC


Bi 6.8 : Cho log14 28 = a. Tnh theo a gi tr ca log49 16.
2a 2
Bi 6.9 : log49 16 =
2a
Bi 6.10 : Cho lg 392 = a; lg 112 = b. Tnh log5 7 theo a v b.
Bi 6.11 : Bit log2 3 = a; log3 5 = b; log7 2 = c. Tnh theo a, b, c gi tr ca log140 63.
Bi 6.12 : Cho log4 75 = a; log8 45 = b. Tnh log 3 25 135 theo a v b.
Bi 6.13 : Cho a, b > 0 v a2 + b2 = 7ab. Chng minh rng vi mi > 0, , 1, ta c :

a+b 1
=
log a + log b
log
3
2

Bi 6.14 : Chng minh rng : 2008 = log5

q
5

log5

5
... 5

{z

du cn
Bi 6.15 : Cho a, b, c l di ba cnh ca mt tam gic vung, vi di cnh huyn l c. Gi s c b , 1. Chng minh
2008

rng :

logc+b a + logcb a = 2 logc+b a. logcb a.


Bi 6.16 : Cho log12 18 = , log24 54 = . Chng minh rng : . + 5( ) = 1.
x(y + z x) y(z + x y) z(x + y z)
=
=
. Chng minh rng :
Bi 6.17 : Gi s :
lg x
lg y
lg z
xy yx = zy yz = zx xz .
Bi 6.18 : Cho N > 0 v N , 1. Chng minh rng :
1
1
1
1
+
+ +
=
.
log2 N log3 N
log2008 N log2008! N
1
1
1
1

lg
x
1

lg
y
1

lg z .
Bi 6.19 : Cho y = 10
; z = 10
. Chng minh rng : x = 10
Bi 6.20 : Tm cc gii hn sau :
e5x+3 e3
.
x0
2x

1. A = lim

2. B = lim
x0

ex 1
.
x+11

ln(1 + x3 )
.
x0
2x

3. C = lim

ln(1 + 2x)
.
x0
tan x

4. D = lim

1
. Chng minh rng : xy + 1 = ey .
1+x
1
Bi 6.22 : Cho hm s y =
. Chng minh rng : xy = y(y ln x 1).
1 + x + ln x
Bi 6.23 : Cho hm s y = ex sin x. Chng minh rng : y + 2y + 2y = 0.

Bi 6.21 : Cho hm s y = ln

Bi 6.24 : Cho y = sin(ln x) + cos(ln x). Chng minh rng : y + xy + x2 y = 0.


Bi 6.25 : Tm cc khong ng bin v nghch bin ca cc hm s y =

2x
1

v y = 3x

3x+1 .

Bi 6.26 : Cho 0 < x < 1; 0 < y < 1; y > x. Chng minh rng :


1
y
x
ln
ln
> 4.
yx
1y
1x
x+y
xy
Bi 6.27 : Cho x > y > 0. Chng minh rng :
>
.
2
ln x ln y

Bi 6.28 : Chng minh rng, nu x > 0 th ln x < x.


Bi 6.29 : Tm gi tr ln nht v gi tr nh nht ca hm s y =

TRN ANH TUN - 0974 396 391 - (04) 66 515 343

ln2 x
, trn 1; e3 .
x

WWW.VNMATH.COM

Trang 128

www.luyenthi24h.com
www.luyenthi24h.com
CHUYN LUYN THI I HCwww.VNMATH.com

www.VNMATH.com

6.3 Phng trnh m v logarit


Vn 1 : Phng trnh c bn

Khi gii phng trnh cha m hoc logarit ta cn t iu kin cho n, c th

ax xc nh khi 0 < a , 1;
loga x xc nh khi 0 < a , 1 v x > 0.
Ta c mt s phng trnh c bn sau (gi s 0 < a , 1) :
1. a f (x) = ag(x) f (x) = g(x).
2. a f (x) = b f (x) = loga f (x).
3. loga f (x) = loga (g(x))

8
< f (x) > 0 (hoc g(x) > 0)
:

f (x) = g(x).

4. loga f (x) = b f (x) = ab .

Bi 6.30 : Gii cc phng trnh sau :


1. 2x = 8;

4. 42x+1 = 1;

1
7. log2 x = ;
2

2. 9x = 27;

5. ex = 2;

8. ln x = 0;

3. 3x = 5;

6. log3 x = log3 5;

9. log x = 4.

Bi 6.31 : Gii cc phng trnh sau :


1. (2 +
2. 2x

3)2x = 2

2 3x+2

3;

5. log2

1
= log 1 (x2 x 1);
2
x

6. log4 (x + 12). log x 2 = 1;

= 4;

3. 2.3x+1 6.3x1 3x = 9;

7. log3 x + log9 x + log27 x = 11;

4. 9x+1 = 272x+1 ;

8. log3 (3x + 8) = 2 + x.

Bi 6.32 : Gii cc phng trnh sau :

1. log2 [x(x 1)] = 1;

3. log2 x + log4 x = log 1

2. log2 x + log2 (x 1) = 1;

4. log2 (3 x) + log2 (1 x) = 3;

TRN ANH TUN - 0974 396 391 - (04) 66 515 343

3;

1
1
log(2x 1) = log(x 9);
2
2

1
1
6. log2 (x2) = log 1 3x 5.
8
6
3
5. 1

WWW.VNMATH.COM

Trang 129

www.luyenthi24h.com
www.luyenthi24h.com
www.VNMATH.com

www.VNMATH.comCHUYN LUYN THI I HC


Vn 2 : Phng php logarit hai v

Khi phng trnh mi v l tch ca cc hm s m hoc cc hng s.


Phng php l ly logarit hai v theo mt c s thch hp.

Bi 6.33 : Gii cc phng trnh sau :


4. 2x+1 .5x = 200;

1. 3x1 .2x = 8.4x2 ;


2

7. 34 = 43 ;
x

2. 2x .5x = 0, 2. log 10x1 ;

5. 3x .8 x+1 = 36;

8. 5x1 = 10x .2x .5x+1 ;

x
3. 0, 125.42x3 = 4 2 ;

6. 32log3 x = 81x;

9. 32 x7 = 0, 25.128 x3 .

x+5

x+17

Vn 3 : Phng php t n ph


1. Nu t t = ax , iu kin t > 0;
2. Nu t t = loga x, v c bn khng cn t iu kin cho t;
3. Nu phng trnh cha tham s ta cn t iu kin cht cho n t.
4. Mt s cch t thng thng :
1
;
t
1
(b) Nu t t = loga b th logb a = ;
t

(c) Nu t t = u(x) th u(x) = t2 ;

1
(d) Vi phng trnh cha (a b) m (a + b)(a b) = 1, nu t t = (a + b) x th (a b) x = .
t
(e) Vi phng trnh dng .ax + .bx + .cx = 0, ta thng chia hai v cho ax (hoc bx hoc cx ) ri t n ph.
(a) Nu t = ax th a2x = t2 , ax =

Bi 6.34 : Gii cc phng trnh sau :


1. 32x+5 = 3x+2 + 2;
2.

6
4
+
= 3;
log2 2x log2 x2

3. log22 x 3 log2 x + 2 = 0;
4.

1
2
+
= 1;
5 log x 1 + log x

5. log 1 x + log22 x = 2;
2

6. 3.4x 2.6x = 9x ;

11.

7. 3x+1 + 18.3x = 29;

12;

13. log x1 4 = 1 + log2 (x 1);

x + 1 = 0;

10. log9x 27 log3x 3 + log9 243 = 0;

TRN ANH TUN - 0974 396 391 - (04) 66 515 343

12. log3 (3x 1). log3 3x+1 3

8. 27x + 12x = 2.8x ;


9. log2 x3 20 log

log2 x
log8 4x
=
;
log4 2x log16 8x

14. 5

log2 (x) = log2

WWW.VNMATH.COM

x2 ;

Trang 130

www.VNMATH.com
log4 x+

15. 3

www.luyenthi24h.com
www.luyenthi24h.com
CHUYN LUYN THI I HCwww.VNMATH.com

1
1
log4 x

2 +3
2 = x;

17. 4ln x+1 6ln x 2.3ln x

18. 3

1
1
1

16. 4 x + 6 x = 9 x ;

+2

20. 2sin

= 0;

+ 4.2cos

= 6;

log2 x log2 8x + 1 = 0;

19. log21 (4x) + log2


2

x2
= 8.
8

21.

43+2 cos 2x

7.41+cos 2x

1
= 42 .

Vn 4 : Phng php phn tch thnh nhn t

Bng cch s dng cc hng ng thc, tm cch t nhn t chung a v dng AB = 0, tng ng vi 4

A=0
B = 0.

Bi 6.35 : Gii cc phng trnh sau :


1. 8.3x + 3.2x = 24 + 6x ;

4. 2. log29 x = log3 x. log3

2. 12.3x + 3.15x 5x+1 = 20;

5. 4x

3. log2 x + 2 log7 x = 2 + log2 x. log7 x;

6. 4x

2 3x+2

2 +x

2 +6x+5

+ 4x
2

2x + 1 1 ;

2 +3x+7

= 42x

+ 1;

+ 21x = 2(x+1) + 1.

Vn 5 : Phng php nh gi

C s ca phng php ny l chng ta s dng bt ng thc hoc phng php hm s nh gi.


Cch 1 : C s nhn dng :
(a) Nu hm s y = f (x) ng bin trn (a; b) v hm s y = g(x) nghch bin trn (a; b) th phng trnh
f (x) = g(x) nu c nghim th nghim l duy nht.
(b) Nu hm s y = f (x) ng bin (hoc nghch bin) trn (a; b) th phng trnh f (x) = c (vi c l hng s)
nu c nghim th nghim l duy nht.
Phng php gii l :
(a) Nhn thy x = x0 l mt nghim ca phng trnh cho.
(b) Nu x > x0 , ta suy ra v tri ln hn v phi hoc ngc li.
(c) Nu x < x0 , ta suy ra v tri ln hn v phi hoc ngc li.
(d) Kt lun phng trnh cho c nghim duy nht x = x0 .
Cch 2 : Nu hm s y = f (x) ng bin (hoc nghch bin) trn (a; b) th phng trnh f (u) = f (v) tng ng vi
u = v.
Cch 3 : Nu hm s y = f (x) tha mn f (x) = 0 c nhiu hn 1 nghim th chng ta lp bng bin thin suy ra
phng trnh c ti ta bao nhiu nghim, ri nhm s nghim , dn n l tt c cc nghim ca phng
trnh.

TRN ANH TUN - 0974 396 391 - (04) 66 515 343

WWW.VNMATH.COM

Trang 131

www.luyenthi24h.com
www.luyenthi24h.com
www.VNMATH.com

www.VNMATH.comCHUYN LUYN THI I HC


Cch 4 : Nu f (x) c v g(x) c th phng trnh f (x) = g(x) tng ng vi

8
< f (x) = c
:

g(x) = c.

Bi 6.36 : Gii cc phng trnh sau :


1. 2x = 3 x;

5. 4x 3x = 1;

2. 2x = 2 log3 x;

6.

3. log2 x = 3 x;
4.

3x

4x

7.

5x ;

1
3

sin
5

= x + 4;
x

+ cos
5

= 1.

6.4 Bt phng trnh m v logarit


Vn 1 : Bt phng trnh c bn

Gii bt phng trnh cha m v logarit chng ta cn ch n c s :

Nu c s a > 1 th bt phng trnh t c cng chiu;


Nu c s 0 < a < 1 th bt phng trnh t c ngc chiu.
Khi bin i bt phng trnh phi bo m biu thc trong logarit l dng.
Di y l mt s dng bt phng trnh c bn :
1. a f (x) > ag(x) , ta c cc kh nng sau :
(a) Nu a > 1 th bt phng trnh tng ng vi f (x) > g(x);
(b) Nu 0 < a < 1 th bt phng trnh tng ng vi f (x) < g(x).
2. a f (x) < b. Khi b 0 th bt phng trnh v nghim. Khi b > 0, ta c cc kh nng sau :
(a) Nu a > 1 th bt phng trnh tng ng vi f (x) < loga b;
(b) Nu 0 < a < 1 th bt phng trnh tng ng vi f (x) > loga b.
3. a f (x) > b. Khi b 0 th bt phng trnh nghim ng vi mi x thuc tp xc nh. Khi b > 0, ta c cc kh nng
sau :

(a) Nu a > 1 th bt phng trnh tng ng vi f (x) > loga b;


(b) Nu 0 < a < 1 th bt phng trnh tng ng vi f (x) < loga b.
4. loga f (x) = loga g(x), ta c cc kh nng sau :
(a) Nu a > 1 th bt phng trnh tng ng vi f (x) > g(x) > 0;
(b) Nu 0 < a < 1 th bt phng trnh tng ng vi 0 < f (x) < g(x).

TRN ANH TUN - 0974 396 391 - (04) 66 515 343

WWW.VNMATH.COM

Trang 132

www.luyenthi24h.com
www.luyenthi24h.com
CHUYN LUYN THI I HCwww.VNMATH.com

www.VNMATH.com

5. loga f (x) > b, ta c cc kh nng sau :


(a) Nu a > 1 th bt phng trnh tng ng vi f (x) > ab ;
(b) Nu 0 < a < 1 th bt phng trnh tng ng vi

8
< f (x) > 0
:

f (x) < ab .

6. loga f (x) < b, ta c cc kh nng sau :


(a) Nu a > 1 th bt phng trnh tng ng vi

8
< f (x) > 0
:

f (x) < ab ;

(b) Nu 0 < a < 1 th bt phng trnh tng ng vi f (x) > ab .

Bi 6.37 : Gii cc bt phng trnh sau :


9. log0,5 (4x + 11) < log0,5 (x2 + 6x + 8);

1. 236x > 1;
2. 16x > 0, 125;

10. log 1 (x + 1) > log3 (2 x);


3

3. log5 (3x 1) < 1;

11. log0,1 (x2 + x 2) > log0,1 (x + 3);

4. log 1 (5x 1) > 0;

12. log 1 (x2 6x + 5) + 2 log3 (2 x) 0;

5. log0,5

(x2

5x + 6) 1;

13. log 1 (x2 6x + 18) + 2 log5 (x 4) < 0;

6. log3 log 1 (x2 1) < 1;

14. log2 log0,5 2x

1 2x
7. log3
0;
x

15.

8. 2x+2 2x+3 2x+4 > 5x+1 5x+2 ;

31
16

2;

i
log 3 log 1 x22 +2log2 x1 +3
1
2

1.

Vn 2 : Phng php t n ph

Chng ta thc hin ging nh phng php gii phng trnh.

Bi 6.38 : Gii cc bt phng trnh sau :


1. 9x < 2.3x + 3;

5. 4x 2.52x < 10x ;

2. 52x+1 > 5x + 4;

6. 4x 3.2x + 2 > 0;

3. log20,5 x + log0,5 x 2 0;

7. log23 x 5 log3 x + 6 0;

4. 2x + 2x+1 3 < 0;

8. log20,2 x 5 log0,2 x < 6;

TRN ANH TUN - 0974 396 391 - (04) 66 515 343

WWW.VNMATH.COM

Trang 133

www.luyenthi24h.com
www.luyenthi24h.com
www.VNMATH.com

www.VNMATH.comCHUYN LUYN THI I HC


Vn 3 : Phng php phn tch thnh nhn t

Bng cch s dng cc hng ng thc, tm cch t nhn t chung a v dng


1. AB 0, tng ng vi

2. AB 0, tng ng vi

8
<A 0
:

8
<A 0
:

hoc

B0
hoc

B0

8
<A 0
:

B 0.

8
<A 0
:

B 0.

Ch rng nu bit chc chn mt trong hai nhn t A v B l dng hoc m th ta c th chia hai v cho s . Tuy nhin,

nu ch bit A 0 hoc A 0 th khng c chia. Chng hn, bt phng trnh AB 0 khng th tng ng vi
B 0, chng ta x l bt phng trnh ny nh sau :

Nu

Nu

A = 0, bt phng trnh lun ng vi iu kin tha mn tp xc nh.


A > 0, bt phng trnh tng ng vi B 0.

Bi 6.39 : Gii cc bt phng trnh sau :


1. 3 + x2 (2x1 + 22x ) > 3x2 + 22x + 2x1 ;

2. 2x+1 + (5x2 + 11)21x x2 < 24 x 1 (x2 9)2x ;


3.

3x2 5x + 2 + 2x 3x .2x 3x2 5x + 2 + 4x2 .3x ;

6.5 H phng trnh


Khi gii h phng trnh m v lgarit, ta cng dng cc phng php gii h phng trnh hc nh phng php th,
phng php cng i s, phng php t n ph, phng php bin i a v h phng trnh i s thng thng,
phng php nh gi, phng php a v cng c s,. . .
Bi 6.40 : Gii cc h phng trnh sau :

1.

2.

3.

8
<2 x+y + 3y = 5

4.

: x+y y1
2 .3 = 2;

5.

log2 x. log4 y 1 = 4;

8
< xy = 1
:

7.

log4 x + log4 y = 1 + log4 9;

8
<22xy + 2 x = 21+y
:

8
< x + y = 20

log2 x + log2 y = 2;

6.

8
<x + y = 1
: 2x
4 + 42y = 0, 5;
8
<3x .2y = 1152
:

log 5 (x + y) = 2;

TRN ANH TUN - 0974 396 391 - (04) 66 515 343

8.

9.

8
< x2 y2 = 2
:

log2 (x + y) log3 (x y) = 1;

8
<3.2 x + 2.3y = 2, 75
: x
2 3y = 0, 75;
8
<log x + log 7. log y = 1 + log 2
7
5
5
5
:

3 + log2 y = log2 5(1 + 3 log5 x);

WWW.VNMATH.COM

Trang 134

www.VNMATH.com

www.luyenthi24h.com
www.luyenthi24h.com
CHUYN LUYN THI I HCwww.VNMATH.com

8
>
<

log2 (x y) = 5 log2 (x + y)
10. log x log 4
>
= 1;
:
log y log 3

8
<2 log x 3y = 15
2

11.

12.

: y
3 . log2 x = 2 log2 x + 3y+1 ;

8
< x2 + y = y2 + x
: x+y
2 2x1 = x y.

6.6 Phng trnh m v lgarit trong cc k thi tuyn sinh H


Bi 6.41 (C08) : Gii phng trnh : log22 (x + 1) 6 log2

Bi 6.42 (A02) : Cho phng trnh : log23 x +

x + 1 + 2 = 0.

log23 x + 1 2m 1 = 0 (m l tham s).

a) Gii phng trnh khi m = 2 ;


b) Tm m phng trnh c t nht mt nghim thuc on [1; 3
Bi 6.43 (A04) : Gii h phng trnh :

8
>
<

log 1 (y x) log4
4

>
: x2 + y2 = 25.

3 ].

1
=1
y

Bi 6.44 (A06) : Gii phng trnh : 3.8x + 4.12x 18x 2.27x = 0.


Bi 6.45 (A07) : Gii bt phng trnh : 2 log3 (4x 3) + log 1 (2x + 3) 2.
3

Bi 6.46 (A08) : Gii phng trnh : log2x1


Bi 6.47 (A09) : Gii h phng trnh

(2x2

+ x 1) + log x+1 (2x 1)2 = 4.

8
<log (x2 + y2 ) = 1 + log (xy)
2
2

(x, y R).

: x2 xy+y2
3
= 81


Bi 6.48 (B02) : Gii bt phng trnh : log x log3 (9x 72) 1.


8

< x1+ 2y =1
Bi 6.49 (B05) : Gii h phng trnh :
:
3 log9 (9x2 ) log3 y3 = 3.

Bi 6.50 (B06) : Gii bt phng trnh : log5 (4x + 144) 4 log5 2 < 1 + log5 (2x2 + 1).

Bi 6.51 (B07) : Gii phng trnh : ( 2 1) x + ( 2 + 1) x 2 2 = 0.

Bi 6.52 (B08) : Gii bt phng trnh : log0,7


Bi 6.53 (B10) : Gii h phng trnh

x2 + x
log6
x+4

8
<log (3y 1) = x
2
: x
4 + 2x = 3y2
8
>
<23x = 5y2 4y

Bi 6.54 (D02) : Gii h phng trnh :

< 0.

(x, y R).

4x + 2x+1
>
:
= y.
x
2 +2

Bi 6.55 (D03) : Gii phng trnh :

2
2x x

22+xx = 3.

Bi 6.56 (D06) : Chng minh rng vi mi a, h phng trnh sau c nghim duy nht :
8
<e x ey = ln(1 + x) ln(1 + y)
:

y x = a.

Bi 6.57 (D06) : Gii phng trnh : 2x

2 +x

2 x

4.2x

22x + 4 = 0.

Bi 6.58 (D07) : Gii phng trnh : log2 (4x + 15.2x + 27) + 2 log2

TRN ANH TUN - 0974 396 391 - (04) 66 515 343

1
4.2x

= 0.

WWW.VNMATH.COM

Trang 135

www.luyenthi24h.com
www.luyenthi24h.com
www.VNMATH.com

www.VNMATH.comCHUYN LUYN THI I HC


x2 3x + 2
0.
x

Bi 6.59 (D08) : Gii bt phng trnh : log 1


2

Bi 6.60 (D10) : Gii phng trnh

42x+ x+2

Bi 6.61 (D10) : Gii h phng trnh

+ 2x = 42+

x+2

8
< x2 4x + y + 2 = 0

+ 2x

+4x4 .

2 log2 (x 2) log 2 y = 0.

6.7 Bi tp tng hp
Bi 6.62 : Gii cc phng trnh sau :
1. 5x+1 + 6.5x 3.5x1 = 51.

4. log3 x(x + 2) = 1.

2. 3x+1 + 3x+2 + 3x+3 = 9.5x + 5x+1 + 5x+2 .

5. log2 (x2 3) log2 (6x 10) + 1 = 0.

3. 3x .2x+1 = 72.

6. log2 (2x+1 5) = x.

Bi 6.63 : Gii cc phng trnh sau :


1. 52x+1 + 7x+1 175x 35 = 0.

3. x2 .2x+1 + 2|x3|+2 = x2 .2|x3|+4 + 2x1 .

1
1
2. 3.4x + .9x+2 = 6.4x+1 .9x+1 .
3
2

4. 4x

2 +x

+ 21x = 2(x+1) + 1.

Bi 6.64 : Gii cc phng trnh sau :


1. log x 2. log x 2 = log x 2.
16

2. log5x

64

3. log2 x + log3 x + log4 x = log20 x.

5
+ log25 x = 1.
x

Bi 6.65 : Gii cc phng trnh sau :


1. 4x+

x2 2

5.2x1+

x2 2

6=0;

2. 43+2 cos x 7.41+cos x 2 = 0 ;

3. 8x + 18x = 2.27x ;

x
x
x
4. 26 + 15 3 + 2 7 + 4 3 2 2 3 = 0.

Bi 6.66 : Gii cc phng trnh sau :

1. log2 4x+1 + 4 . log2 (4x + 1) = 3 ;




2. log4 log2 x + log2 log4 x = 2 ;

3. log x (125x). log225 x = 1 ;


4. log x 3 + log3 x = log x 3 + log3

1
x+ .
2

Bi 6.67 : Gii cc phng trnh sau :


1. xlog4 x2 = 23(log4 x1) ;

2. xlg

x+lg x3 +3

= x.

Bi 6.68 : Gii cc phng trnh sau :


1.

4x+1
2

3x2
1

2. xlg x = 1000x2 .

Bi 6.69 : Gii cc phng trnh sau :

TRN ANH TUN - 0974 396 391 - (04) 66 515 343

WWW.VNMATH.COM

Trang 136

www.luyenthi24h.com
www.luyenthi24h.com
CHUYN LUYN THI I HCwww.VNMATH.com

www.VNMATH.com

1. log (x 1) + log (x + 1) log 1 (7 x) = 1 ;


2

3. 1 + log2 (9x 6) = log2 (4.3x 6) ;

2. 3x .2x = 1 ;

4. log 2

x + 1 log (3 x) log8 (x 1)3 = 0.

Bi 6.70 : Gii phng trnh : 3.25x2 + (3x 10).5x2 + 3 x = 0.

Bi 6.71 : Gii phng trnh : x2 .3x + 3x (12 7x) = x3 + 8x2 19x + 12.

Bi 6.72 : Gii phng trnh : log2 (1 + x) = log3 x.

Bi 6.73 : Gii phng trnh : log2 x + 3log6 x = log6 x.


Bi 6.74 : Gii phng trnh : xlog2 9 = x2 .3log2 x xlog2 3 .
1
1
1
Bi 6.75 : Gii phng trnh : 5x + 4x + 3x + 2x = x + x + x 2x3 + 5x2 7x + 17.
2
3
6


Bi 6.76 : Gii phng trnh : 4(x 2) log2 (x 3) + log3 (x 2) = 15(x + 1).
Bi 6.77 : Gii phng trnh : 4sin x 21+sin x cos(xy) + 2|y| = 0.
8
Bi 6.78 : Gii phng trnh : 22x+1 + 232x =
.
log3 (4x2 4x + 4)
Bi 6.79 : Gii h phng trnh :

Bi 6.80 : Gii h phng trnh :

Bi 6.81 : Gii h phng trnh :

8
>
>
log2 x + log4 y + log4 z = 2
>
<

log3 x + log9 y + log9 z = 2


>
>
>
:log x + log 6y + log 6z = 2.
4
1
1
8
<4log3 (xy) = 2 + (xy)log3 2
: 2
x + y2 3x 3y = 12.
8
< xlog3 y + 2ylog3 x = 27
:

log3 y log3 x = 1.

Bi 6.82 : Gii cc h phng trnh sau :

1.

8
>
<

log (y x) log4

>
: x2 + y2 = 25;

1
=1
y

2.

8
<log (x2 + y2 ) = 5
2
:

2 log4 x + log2 y = 4.

Bi 6.83 : Gii cc phng trnh sau :

1. x2 .3x1 + x(3x 2x ) = 2(2x 3x1 ) ;

3. 3 log3 (1 +

2. 4sin x 21+sin x cos(xy) + 2|y| = 0 ;

4. (x + 2) log23 (x + 1) + 4(x + 1) log3 (x + 1) 16 = 0.

x+

x) = 2 log2

x;

Bi 6.84 : Gii cc bt phng trnh sau :


1.

9x 3x+1 + 2 > 3x 9 ;

2. 252xx

2 +1

3. 52x103

2 +1

+ 92xx
x2

4. log2 (2x 1). log (2x+1 2) > 2 ;

34.152xx ;

4.5x5 < 51+3

x2

5.

log22 x + log x2 3 >

6. log x 2x

5(log4 x2 3) ;

log x (2x)3 .

Bi 6.85 : Gii cc bt phng trnh sau :

TRN ANH TUN - 0974 396 391 - (04) 66 515 343

WWW.VNMATH.COM

Trang 137

www.luyenthi24h.com
www.luyenthi24h.com
www.VNMATH.com

www.VNMATH.comCHUYN LUYN THI I HC


1. xlog2 x+4 < 32 ;

2. xlg
h

i x4

Bi 6.86 : Gii bt phng trnh : log2 x + log (x + 3)


Bi 6.87 : Gii cc bt phng trnh sau :


3 lg x+1

> 1000.

1.


1. log x log9 (3x 9) < 1 ;

2. log x log2 (4x 6) 1.

Bi 6.88 : Gii bt phng trnh :5x +

6x2 + x3 x4 . log2 x > (x2 x) log2 x + 5 + 5 6 + x x2 .

Bi 6.89 : Gii cc bt phng trnh sau :


1. 25x + 15x 2.9x ;

4. 3

2. log x + 2 log (x 1) + log2 6 0 ;


5.

3. log2 (2x 1) log2 (2x+1 2) > 2 ;


Bi 6.90 : Gii bt phng trnh : log7 x < log3 (2 +

log x + log4 x2 2 > 0 ;

log20,5 x + 4 log2

2(4 log16 x4 ).

x).

Bi 6.91 : Gii phng trnh, bt phng trnh sau :




1. log x2

4x 2
|x 2|

1
13. log (x 3) = 1 + log4 ;
x

1
;
2

2. 4x+1 + 2x+4 = 2x+2 + 16 ;


3.

log42

x log

3
x

14.

+ 9 log2

32
< 4 log2 x ;

x2

1
1
<
;
2
log4 (x + 3x) log2 (3x 1)

15. (4x + 2x 2) log2 (2x 1) 0 ;

4. 4.4x 9.2x+1 + 8 = 0 ;

16. 4

5. 9x + 6x = 22x+1 ;

17. 3x

6. ( 1 x + 1 + x 2). log x (x2 x) = 0 ;

18. 5 + 5+log5 sin x = 15+log15 cos x ;

2 4

7. log2 x + 2 log7 x = 2 + log2 x. log7 x ;


8. 3x + 5x = 6x + 2 ;
9. 32x 8.3x+

x+4

x+5+1

9.9

x+4

+ 2.2

x+5+x

= 2.4x ;

+ (x2 4).3x2 1 0 ;

1
89x 25
19. 3 +
= log x

;
log32 x
2
2x

20. ( 2 + 1) x+1 (3 + 2 2) x = x 1 ;
21. 2 ln x + ln(2x 3)2 = 0 ;
1

10. log x(x2 3x + 2) 1 ;

22. log2 (3x 1) +

11. log5 x = log7 (x + 2) ;

23. log4 (x x2 1). log5 (x+ x2 1) = log20 (x x2 1) ;

12. log2 (x2 3) log2 (6x 10) + 1 = 0 ;

24. 4x

log x+3 2

= 2 + log2 (x + 1) ;

2 3x+2

2 +6x+5

+ 4x

2 +3x+7

= 42x

+1;

Bi 6.92 : Gii cc h sau :


8
>
<

2xy

2xy

2
2 2
+ 7.
3.
6=0
3
3
1.
>
:
lg(3x y) + lg(x + y) 4 lg 2 = 0;

TRN ANH TUN - 0974 396 391 - (04) 66 515 343

2.

8
<2 x + log y + 2 x log y = 5
2
2
: x
4 + log22 y = 5;

WWW.VNMATH.COM

Trang 138

www.luyenthi24h.com
www.luyenthi24h.com
CHUYN LUYN THI I HCwww.VNMATH.com

www.VNMATH.com
3.

8
<lg2 y = lg3 x 4 lg2 x + 7 lg x

7.

: 2
lg x = lg3 y 4 lg2 y + 7 lg y;

8.

x + y = 1;

8
>
<

log4 (x2 + y2 ) log4 (2x) + 1 = log4 (x + 3y)

x
>
:log4 (xy + 1) log4 (4y2 + 2y 2x + 4) = log4 1;
y

8
<23x+1 + 2y2 = 3.2y+3x
5.

:
2

6.

log4 x log4 y = 1;

8
<e x ey = (log y log x)(xy + 1)
2
2
4.
: 2
2

3x + 1 + xy =

8
< xlog8 y + ylog8 x = 4

9.

x + 1;

8
<9log2 (xy) = 3 + 2(xy)log2 3

10.

: 2
x + y2 = 3x + 3y + 6;

8
<3lg x = 4lg y
:

(4x)lg 4 = (3y)lg 3 ;

8
< x4 + y = 3 x4 y
:

4 y

8(x4 + y) = 6x

Bi 6.93 : Tm tt c cc gi tr ca a bt phng trnh sau nghim ng vi mi x R :


a.9x + (a 1).3x+2 + a 1 > 0.
Bi 6.94 : Cho phng trnh : 2 log4 (2x2 x + 2m 4m2) + log (x2 + mx 2m2 ) = 0. Xc nh tham s m phng trnh
trn c hai nghim x1 , x2 tha mn x21 + x22 > 1.

Bi 6.95 : Cho bt phng trnh : m.92x x (2m + 1)62x x + m.42x x 0. Tm m bt phng trnh nghim ng vi
1
mi x tha mn iu kin |x| .
2
Bi 6.96 : Gii bt phng trnh : log 1 (4x + 4) log 1 (22x+1 3.2x ).
2

1
1
Bi 6.97 : Gii phng trnh : . log 2 (x + 3) + log4 (x 1)8 = log2 (4x).
2
4
Bi 6.98 : Tm a phng trnh sau c nghim :
91+

1x2

(a + 2).31+

1x2

+ 2a + 1 = 0

8
< x 4|y| + 3 = 0
Bi 6.99 : Gii h phng trnh :

log4 x

log2 y = 0.

Bi 6.100 : Tm k h bt phng trnh sau c nghim :


8
>
<

|x 1|3 3x k < 0
1
1
>
: log2 x2 + log2 (x 1)3 1.
2
3
Bi 6.101 : Gii phng trnh : 16 log27x3 x 3 log3x x2 = 0.
Bi 6.102 : Gii h phng trnh :

8
<log (x3 + 2x2 3x 5y) = 3
x
:

logy (y3 + 2y2 3y 5x) = 3

8
<log xy = log y
y
x
Bi 6.103 : Gii h phng trnh :
: x
y

2 + 2 = 3.

Bi 6.104 : Gii bt phng trnh : 15.2x+1 + 1 |2x 1| + 2x+1 .

Bi 6.105 : Tm m phng trnh :

4 log2

2
x log 1 x + m = 0
2

c nghim thuc khong (0; 1).

TRN ANH TUN - 0974 396 391 - (04) 66 515 343

WWW.VNMATH.COM

Trang 139

www.VNMATH.comCHUYN LUYN THI I HC

www.luyenthi24h.com
www.luyenthi24h.com
www.VNMATH.com

Bi 6.106 : Gii phng trnh : log 1 x + 2 log 1 (x 1) + log2 6 0.


2

Bi 6.107 : Cho hm s f (x) = x log x 2, vi x > 0, x , 1.


Tnh f (x) v gii bt phng trnh f (x) 0.

Bi 6.108 : Gii phng trnh : log5 (5x 4) = 1 x.

Bi 6.109 : Gii cc phng trnh, bt phng trnh, h ...


4
=1;
a) (2 log3 x) log9x 3
1 log3 x
b) log4 (x 1) +

1
log2x+1 4

f)

1
+ log2 x + 2 ;
2

8
<ln(1 + x) ln(1 + y) = x y
: 2
x 12xy + 20y2 = 0;

g) 2(log2 x + 1) log4 x + log2

1
=0;
4

c) log3 (x 1)2 + log 3 (2x 1) = 2 ;

h) 9x

< x + x2 2x + 2 = 3y1 + 1
d)

:
x1
2

i) 4x 2x+1 + 2(2x 1) sin(2x + y 1) + 2 = 0 ;

y+

y 2y + 2 = 3

e) (log x 8 + log4 x2 ) log2

2 +x1

+ 1;

2 +x2

10.3x

+1=0;

j) log3 (3x 1) log3 (3x+1 3) = 6 ;


k) log 2

2x 0 ;

Bi 6.110 : Chng minh rng h :

x + 1 log 1 (3 x) log8 (x 1)3 = 0 ;


2

8
y
>
x
>
<e = 2009 2
y 1
>
x
y
>
:e = 2009

x2 1

c ng hai nghim tho mn iu kin x > 0, y > 0.

Bi 6.111 : a) Gii bt phng trnh : (2, 5) x 2.(0, 4) x+1 + 1, 6 = 0.


b) Gii phng trnh : log4 (log2 x) + log2 (log4 x) = 2.
Bi 6.112 : Trong cc nghim (x, y) ca bt phng trnh : log x2 +2y2 (2x + y) 1 hy ch ra nghim c tng 2x + y ln nht.
Bi 6.113 : Gii phng trnh : log2 (3x 1) +
Bi 6.114 : Tm tp xc nh ca hm s : y =

log x+3 2

= 2 + log2 (x + 1).

log2 (x2 + 2). log2x 2 2

Bi 6.115 : Gii cc bt phng trnh :


a) 3

x2 2x

x|x1|
1

b)

Bi 6.116 : Gii bt phng trnh : log3

x2 5x + 6 + log 1
3

log2 (x + 1)2 log3 (x + 1)3


> 0.
x2 3x 4

x2 >

1
log 1 (x + 3).
3
2

Bi 6.117 : Gii phng trnh : 4lg(10x) 6lg x = 2.3lg(100x ) .

Bi 6.118 : Gii phng trnh : log4 (x + 1)2 + 2 = log 2 4 x + log8 (4 + x)3 .


x
x

7+3 5
73 5
Bi 6.119 : Cho phng trnh :
+ a.
= 8.
2
2
2

a) Gii phng trnh khi a = 7.


b) Bin lun theo a s nghim ca phng trnh.

TRN ANH TUN - 0974 396 391 - (04) 66 515 343

WWW.VNMATH.COM

Trang 140

www.VNMATH.com

www.luyenthi24h.com
www.luyenthi24h.com
CHUYN LUYN THI I HCwww.VNMATH.com

Bi 6.120 : Tm min xc nh ca hm s : y =

x2 + x 2 log3 (9 x2 ).

Bi 6.121 : Gii phng trnh : log x x2 14. log16x x3 + 40. log4x x = 0.


2

Bi 6.122 : Cho h phng trnh :

8
1
>
y
x
9
>
<
= 9 2y

x + my 2x
>
>
=
4.
:
x

a) Gii h phng trnh vi m = 3.


b) Tm m h phng trnh trn c nghim duy nht.
Bi 6.123 : Cho phng trnh : 4x m.2x+1 + 2m = 0.
a) Gii phng trnh khi m = 2.
b) Tm m h phng trnh cho c hai nghim phn bit x1 , x2 sao cho x1 + x2 = 3.
Bi 6.124 : Cho hm s y = 2x + m + log2 (mx2 2(m 2)x + 2m 1). Tm tt c cc gi tr ca m hm s xc nh vi

mi x.

Bi 6.125 : Gii cc phng trnh sau :


a) log2 (log3 (log2 x)) = 1.
b)

sin x
sin x
5+2 6
+
52 6
= 2.

Bi 6.126 : Gii bt phng trnh : 3x+1 22x+1 12 2 < 0.


x

Bi 6.127 : Gii phng trnh : log x 2 + log2 (4x) = 3.


2

1
x1
log 3
+ log3 |x 3|.
2
2
Bi 6.129 : Tm tt c cc gi tr ca m sao cho bt phng trnh sau c nghim ng vi mi x 0 :

Bi 6.128 : Gii phng trnh : log9 (x2 5x + 6)2 =

m.2x+1 + (2m + 1)(3


Bi 6.130 : Gii bt phng trnh : 4x2 + x.2x

2 +1

5) x + (3 +

5) x < 0.

+ 3.2x > x2 .2x + 8x + 12.


lg(mx)
Bi 6.131 : Xc nh gi tr ca tham s m phng trnh
= 2 c nghim duy nht.
lg(x + 1)
21x 2x + 1
0.
2x 1
Bi 6.133 : Gii bt phng trnh : (x + 1) log21 x + (2x + 5). log 1 x + 6 0.
2
2

x
x
x
Bi 6.134 : Cho phng trnh : ( 5 + 1) + a.( 5 1) = 2 .

Bi 6.132 : Gii bt phng trnh :

1
a) Gii phng trnh khi a = .
4
b) Tm mi gi tr ca a phng trnh c ng mt nghim.
Bi 6.135 : Gii phng trnh : 9cot x + 3cot x 2 = 0.
8
>
<

log22 x log2 x2 < 0


Bi 6.136 : Gii h bt phng trnh : x3
>
:
3x2 + 5x + 9 > 0.
3
Bi 6.137 : Gii cc phng trnh sau :

TRN ANH TUN - 0974 396 391 - (04) 66 515 343

WWW.VNMATH.COM

Trang 141

www.luyenthi24h.com
www.luyenthi24h.com
www.VNMATH.com

www.VNMATH.comCHUYN LUYN THI I HC


a) 9x + 2(x 2)3x + 2x 5 = 0.

b) log2 (3.2x 1) = 2x + 1.

Bi 6.138 : Tm m bt phng trnh sau y nghim ng vi mi x :


logm (x2 2x + m + 1) > 0.

Bi 6.139 : Gii h phng trnh :

8
<log (6x + 4y) = 2
x
:

logy (6y + 4x) = 2.

Bi 6.140 : Cho bt phng trnh : (m 1).4x + 2x+1 + m + 1 > 0.


a) Gii bt phng trnh khi m = 1.
b) Tm cc gi tr ca m bt phng trnh nghim ng vi mi x.
Bi 6.141 : Gii bt phng trnh :

x+1
x3

10 + 3 x 1 <
10 3 x + 3 .

Bi 6.142 : Tm cc gi tr ca a bt phng trnh sau :

x2 2 log2

a
a
a
+ 2x 1 + log2
2 1 + log2
>0
a+1
a+1
a+1

nghim ng vi mi x.
Bi 6.143 : Tm tt c gi tr ca x, tho mn x > 1, nghim ng bt phng trnh sau :
log 2(x2 + x) (x + m 1) < 0
m
vi mi gi tr ca m : 0 < m 4.

2.3x 2x+2
1.
3x 2x

1
Bi 6.145 : Gii bt phng trnh : log x x
2.
4
Bi 6.146 : Gii phng trnh : log2 (x2 1) = log 1 (x 1).

Bi 6.144 : Gii bt phng trnh sau :

Bi 6.147 : Cho h phng trnh :

8
<log (x + y) + log (x y) = 1
2
a
: 2
x y2 = 1

vi a l s dng khc 1. Xc nh a h phng

trnh c nghim duy nht v gii h phng trnh trong trng hp .


Bi 6.148 : Gii phng trnh : 12.3x + 3.15x 5x+1 = 20.
x1
x
Bi 6.149 : Gii phng trnh : 5 .8 x = 500.
Bi 6.150 : Tm tt c cc cp s dng tho mn h phng trnh :
8
x
< xy+4x = y5(y 3 )
: 3
x = y1 .

Bi 6.151 : Gii bt phng trnh : (4x2 16x + 7). log3 (x 3) > 0.


Bi 6.152 : Gii bt phng trnh :

lg(x2 3x + 2)
> 2.
lg x + lg 2

Bi 6.153 : Cho phng trnh : (x 2)log2 4(x2) = 2 (x 2)3 .

TRN ANH TUN - 0974 396 391 - (04) 66 515 343

WWW.VNMATH.COM

Trang 142

www.luyenthi24h.com
www.luyenthi24h.com
CHUYN LUYN THI I HCwww.VNMATH.com

www.VNMATH.com
a) Gii phng trnh vi = 2.

b) Xc nh phng trnh c hai nghim phn bit x1 , x2 tho mn :




5
x1 , x2 4.
2

1
. log 1 (x 1)
5
2x 1 1
Bi 6.155 : Gii bt phng trnh : log x+2 x 2 log x+1 2.
Bi 6.154 : Gii bt phng trnh : 2 log225 (x 1) log5
Bi 6.156 : Tm m phng trnh :

log22 x + log 1 x2 3 = m(log4 x2 3)


2

c nghim thuc khong (32; +).


Bi 6.157 : Gii phng trnh :

cos x 
cos x
7+4 3
+
74 3
= 4.

Bi 6.158 : Gii bt phng trnh :

log22 x + log 1 x2 3 =
2

5(log4 x2 3).

Bi 6.159 : Gii phng trnh : 4log2 2x xlog2 6 = 2.3log2 4x .


2

Bi 6.160 : Cho bt phng trnh :

9x 2(m + 1).3x 2m 3 > 0


trong m l tham s thc. Tm tt c cc gi tr ca m bt phng trnh trn lun ng vi mi s thc x.
Bi 6.161 : Gii h phng trnh :

8
>
<

log4 (x2 + y2 ) log4 (2x) + 1 = log4 (x + 3y)

x
>
:log4 (xy + 1) log4 (4y2 + 2y 2x + 4) = log4 1.
y

Bi 6.162 : Gii v bin lun theo tham s thc a h phng trnh :


8
<x + y + a = 1
: a2 x+yxy
2 .4
=2

trong (x, y) l n.
5
Bi 6.163 : Tnh tch cc nghim ca phng trnh sau : xlog6 (3x) 36. x7 = 0.
Bi 6.164 : Gii h phng trnh :

8
<(x4 + y).3xyx4 = 1
:

4 y

8(x4 + y) 6x

= 0.

Bi 6.165 : Gii phng trnh : 25x + 10x = 22x+1 .


Bi 6.166 : Gii h phng trnh :

8
<x + y = 1

: x
2 2y = 2.


log3

Bi 6.167 : Gii bt phng trnh : 5

x2
x

< 1.

Bi 6.168 : Gii phng trnh : (2 + 3) x + (7 + 4 3)(2 3) x = 4(2 + 3).


7
Bi 6.169 : Gii phng trnh : log x 2 log4 x + .
6
Bi 6.170 : Cho phng trnh :
(m + 3).16x + (2m 1).4x + m + 1.
Tm m phng trnh c hai nghim tri du.

Bi 6.171 : Gii phng trnh : (2 3) x + (2 + 3) x = 14.

TRN ANH TUN - 0974 396 391 - (04) 66 515 343

WWW.VNMATH.COM

Trang 143

www.VNMATH.comCHUYN LUYN THI I HC

www.luyenthi24h.com
www.luyenthi24h.com
www.VNMATH.com

Bi 6.172 : Vi gi tr no ca m th phng trnh :


|x2 4x+3|
1

= m4 m2 + 1

c bn nghim phn bit.


Bi 6.173 : Gii phng trnh : log3 (x2 + x + 1) log3 x = 2x x2 .

Bi 6.174 : Gii v bin lun phng trnh : 5x

2 +2mx+2

Bi 6.175 : Gii phng trnh : log3

x2 + x + 3
2x2 + 4x + 5

2 +4mx+m+2

52x

= x2 + 2mx + m.

= x2 + 3x + 2.

Bi 6.176 : Gii bt phng trnh : 2x + log2 (x2 4x + 4) > 2 (x + 1) log 1 (2 x).


2

Bi 6.177 : Gii bt phng trnh : log0,5

(9x1 )

2 > log0,5

(3x1

+ 7).

Bi 6.178 : Gii v bin lun bt phng trnh :

1
loga 2.
2

Bi 6.179 : Cho phng trnh : log2 (mx3 5mx2 + 6 x) = log2+m (3 x 1), m l tham s.
loga (loga2 x) + loga2 (loga x)

a) Gii phng trnh vi m = 0.


b) Tm cc gi tr ca x nghim ng bt phng trnh cho vi mi m 0.
Bi 6.180 : Gii phng trnh : log2 (x2 + 3x + 2) + log2 (x2 + 7x + 12) = 3 + log2 3.
Bi 6.181 : Gii phng trnh : 125x + 50x = 23x+1 .

Bi 6.182 : Gii phng trnh : ( 1 x + 1 + x 2). log2 (x2 x) = 0.


1
1

Bi 6.183 : Gii bt phng trnh :


>
.
2
log 1 2x 3x + 1 log 1 (x + 1)
3

Bi 6.184 : Gii phng trnh : log5

(5x

1). log25

(5x+1

5) = 1.

8
<23x+1 + 2y2 = 3.2y+3x
Bi 6.185 : Gii h phng trnh :

:
2
r

Bi 6.186 : Gii bt phng trnh :

3x + 1 + xy =

log3

x + 1.

2x 3
< 1.
1x

1
Bi 6.187 : Gii phng trnh : loga (ax). log x (ax) = loga2 , vi 0 < a , 1.
a

2
Bi 6.188 : Gii phng trnh : 2(log9 x) = log3 x. log3 ( 2x + 1 1).
Bi 6.189 : Gii phng trnh : 4x 2.6x = 3.9x .

logx1 (2x1)

5 3
3
4
8

Bi 6.191 : Gii phng trnh : 2 log6 ( x + x) = log4 x.

Bi 6.190 : Gii bt phng trnh :

(0, 12)log x1 x

Bi 6.192 : Gii bt phng trnh : (4x 12.2x + 32). log2 (2x 1) 0.


Bi 6.193 : Gii phng trnh : 2 log5 x log x 125 < 1.
Bi 6.194 : Gii phng trnh : 4x

x2 5

12.2x1

x2 5

2

Bi 6.195 : Gii bt phng trnh : 2 log121 (x 2)

+ 8 = 0.



log 1 ( 2x 3 1) log 1 (x 2) .


11

Bi 6.196 : Gii bt phng trnh : log 1 (x 1) + log 1 (2x + 2) +


3

TRN ANH TUN - 0974 396 391 - (04) 66 515 343

11

log

3 (4

x) < 0.
WWW.VNMATH.COM

Trang 144

www.VNMATH.com

www.luyenthi24h.com
www.luyenthi24h.com
CHUYN LUYN THI I HCwww.VNMATH.com

2
2
Bi 6.197 : Cho phng trnh : ( 2 + 1) x + ( 2 1) x 1 + m = 0. Tm m phng trnh trn c nghim?

Bi 6.198 : Gii bt phng trnh : (2, 5) x 2.(0, 4)x+1 + 1, 6 < 0.

Bi 6.199 : Cho phng trnh :

(3 + 2 2)tan x + (3 2 2)tan x = m.

a) Gii phng trnh khi m = 6.


b) Xc nh m phng trnh c ng hai nghim phn bit nm trong khong ;
.
2 2
Bi 6.200 : Gii bt phng trnh : 2log2 x + xlog2 x 4.
2

Bi 6.201 : Cho bt phng trnh : log5 (x2 + 4x + m) log5 (x2 + 1) < 1. Tm m bt phng trnh nghim ng vi mi
x thuc khong (2; 3).

Bi 6.202 : Gii phng trnh : (x + 1) log23 x + 4x log3 x 16 = 0.


Bi 6.203 : Gii h phng trnh :

8
<log (3x + 2y) = 2
x
:

logy (3y + 2x) = 2

Bi 6.204 : Gii bt phng trnh : lg(x2 3) >

1
lg(x2 2x + 1).
2

Bi 6.205 : Gii phng trnh : 32x

+2x+1

28.9x

+x

+ 9 = 0.

Bi 6.206 : Gii bt phng trnh : log4 x2 + log8 (x 1)3 1.


Bi 6.207 : Gii bt phng trnh :

log9 (3x2 + 4x + 2) + 1 > log3 (3x2 + 4x + 2).

Bi 6.208 : Cho phng trnh : 342x 2.32x + 2m 3 = 0.


2

a) Gii phng trnh khi m = 0.


b) Xc nh m phng trnh c nghim?
Bi 6.209 : Gii v bin lun phng trnh sau theo tham s m : 2 log3 x log3 (x 1) log3 m = 0.
Bi 6.210 : Gii h phng trnh :

8
<3x .2y = 1152
:

log x5 (x + y) = 2.

Bi 6.211 : Gii bt phng trnh : log x1 (x + 1) > log x2 1 (x + 1).

Bi 6.212 : Gii bt phng trnh : 2x log3 8 + x2 log3 (2x) log3 x3 x2 3 + x log3 (4x2 ).

x
x
Bi 6.213 : Gii phng trnh : log3 sin sin x + log 1 sin + cos 2x = 0.
3
2
2
Bi 6.214 : Tm a bt phng trnh sau c nghim ng vi mi x :
a.4x + (a 1).2x+2 + a 1 > 0.
Bi 6.215 : Gii v bin lun theo k h phng trnh :

8
<log (3x + ky) = 2
x
:

logy (3y + kx) = 2.

Bi 6.216 : Gii bt phng trnh : log x+1 (2x) > 2.


Bi 6.217 : Gii phng trnh : 4x 2x+1 + 2(2x 1) sin(2x + y 1) + 2 = 0.
2x 1
Bi 6.218 : Gii phng trnh : log2
= 1 + x 2x .
|x|

TRN ANH TUN - 0974 396 391 - (04) 66 515 343

WWW.VNMATH.COM

Trang 145

www.VNMATH.comCHUYN LUYN THI I HC


8
>
<

3
= 2y
2
Bi 6.219 : Gii h phng trnh :
>
:(x2 y + 2x)2 2x2 y 4x + 1 = 0.
1x2
x2

Bi 6.220 : Gii phng trnh :

+ xy +

log22

www.luyenthi24h.com
www.luyenthi24h.com
www.VNMATH.com

x
x + x log7 (x + 3) =
+ 2 log7 (x + 3) log2 x.
2

3
Bi 6.221 : Gii phng trnh : 2 + (1 log3 x) log 2 (4x2 ) = (1 + log2 x) log 2 (4x2 ) + 2 log3 . log2x 2.
x
x
x

Bi 6.222 : Gii phng trnh : ln(2 + sin 2x) = 2 cos2 x


.
4
4x2 + 2
= x3 1.
Bi 6.223 : Gii phng trnh : log2 3
x + 4x2 + 1
Bi 6.224 : Gii phng trnh : 42x 5.4x
2

2 +x

+ 42x+1 = 0.

1
Bi 6.225 : Gii bt phng trnh : log3 (9x 3) log3 x
.
3
Bi 6.226 : Gii bt phng trnh : 2 log3 (x + 1) + 2 log9 (4x + 1) 3 log27 (10x + 7) > 1.

Bi 6.227 : Gii bt phng trnh : 15.2x+1 + 1 |2x 1| + 2x+1 .

Bi 6.228 : Vi gi tr no ca m, phng trnh sau c nghim duy nht :

2 log 1 (mx + 28) = log5 (12 4x x2 ).


25

Bi 6.229 : Gii bt phng trnh : log7 (x2 + x + 1) log2 x.


8
>
<

|x| + y = 4 + y2 + 2

Bi 6.230 : Gii h phng trnh : 1


y
>
: lg x2 2 lg 2 = lg 1 +
.
2
2

x2 + 5x + 8
Bi 6.231 : Gii phng trnh : ln
= 4x + 4.
x2 x + 2

Bi 6.232 : Gii bt phng trnh : log2 (1 + x) > log3 x.

Bi 6.233 : Gii bt phng trnh : log x

x2 1

Bi 6.234 : Gii bt phng trnh : 32x 8.3x+

x3 + 1
2x2 + 1

x+4

9.9

> log x+ x2 1
x+4

2x + 1
.
x2 + 1

0.


Bi 6.235 : Gii bt phng trnh : log2 log3 x log5 log7 x .


Bi 6.236 : Gii h phng trnh :

8
>
<

2 log2 (y + x) log2 x = 1 + log2 (3y x)





xy + 3
y
>

log
= 0.
:log2
4
2
x y + 3x 1
x

Bi 6.237 : Gii bt phng trnh :

log9 (3x2 + 4x + 2) + 1 > log3 (3x2 + 4x + 2).

Bi 6.238 : Tm m bt phng trnh : m log2 (3x 1) log2 (2.3x 2) < 1 + m c nghim trn (0; 2).

Bi 6.239 : Gii bt phng trnh : log|x|


9 x2 x 1 1.
Bi 6.240 : Gii bt phng trnh : log7x2

log 1 x

Bi 6.241 : Gii bt phng trnh : 3 + x

3 sin 2x 2 sin x
= log7x2 (sin 2x(cot x + tan x)).
sin 2x cos x
2

log 1 x

.2log2 x > 6.x 2 .


1
Bi 6.242 : Gii bt phng trnh : log 1 (x2 + 2x + 5) log 1 (2x2 + 4x + 3) 2.
2
2
2
2

8
<log x + ylog2 3 = 6
2
Bi 6.243 : Tm m h phng trnh sau c nghim :

log2 x + ylog2

TRN ANH TUN - 0974 396 391 - (04) 66 515 343

= 2m.
WWW.VNMATH.COM

Trang 146

www.luyenthi24h.com
www.luyenthi24h.com
CHUYN LUYN THI I HCwww.VNMATH.com

www.VNMATH.com

Bi 6.244 : Tm cc gi tr ca m phng trnh sau c nghim duy nht


25x + (m 1)5x + 2m + 3 = 0.
Bi 6.245 : Tm cc gi tr ca a 3x + (a 1)2x + (a 1) > 0 vi mi x R.
Bi 6.246 : Tm m h phng trnh sau c nghim

8


<(2x + 1) [ln(x + 1) + ln x] = (2y + 1) ln(y + 1) + ln y
:

y 1 2 4 (y + 1)(x 1) + m x + 1 = 0.

Bi 6.247 : Gii phng trnh


log2

x2 + x + 1 + log16 (x2 x + 1)2 =

3
3
log2 x4 + x2 + 1 + log4 (x4 x2 + 1).
2

1
Bi 6.248 : Tm m phng trnh 4 log22 log 1 x + m = 0 c nghim thuc khong (0; 1).
2
x

Bi 6.249 : Tm a bt phng trnh sau c nghim log 1 x2 + 1 > log 1 (ax + a).
3

Bi 6.250 : Tm tt c cc gi tr ca tham s a phng trnh log5

(25x

log5 a) = x c nghim duy nht.

Bi 6.251 : Tm cc gi tr ca m phng trnh sau c nghim duy nht

log0,5 (m + 6x) + log2 (3 2x x2 ) = 0.


Bi 6.252 : Cho phng trnh log(x2 + 10x + m) = 2 log(2x + 1) (vi m l tham s).
Tm cc gi tr ca m phng trnh trn c ng hai nghim thc phn bit.
Bi 6.253 : Gii cc phng trnh, bt phng trnh :
1. log32x (2x2 9x + 9) + log3x (4x2 12x + 9) 4 = 0;

2. log 2 x2 6x 3 + log x2 6x3 2 = 3;


3. 2 log3 (3x 1) + 1 = log 3 (2x + 1);
4. log3 (x3 + 1) =

1
log3 (2x 1)2 + log 3 (x + 1) ;
2

5. 6x1 = 5 log7 (6x 5) + 1;

6. log 1 log3
x2 + 1 + x log2 log 1
x2 + 1 x ;
2

9. 2log2 x x2 ;
10. 42x 5.4x
2

11.

+ 42x+1 = 0;

log2 (4x 2) log2 x

1
;
2

3
12. ( 5 1) x + ( 5 + 1) x 2x+ 2 = 0;
13. log3

2 +x

x2 5x + 6 + log 1

x 2 > log 1

x + 3;

14. 3log2 x = x2 1;

7. 64log4 x = 3.2log2 x + 3.xlog4 x + 4;


3
log8 x
log2 1 + 2x
8.

.
log2 (1 + 2x)
log2 x

15.

4x + (x 11)2x 8(x 3)
0.
log2 x 2

Bi 6.254 : Gii cc phng trnh, bt phng trnh :


1. log20,5 x + 4 log2
2. log3

x 4 log16 x4 ;

x2 5x + 6 + log 1
3

x2>

5.
1
log 1 (x + 3);
3
2

x2 3x + 2 log2 x2

6. 2x + 3.2x

x2 3x + 2(5 log x 2);

2 log xlog (x+6)


2
2

> 1;

3. 9x + (x 12)3x + 11 x = 0;

7. ( 3 + 1)log2 x + x.( 3 1)log2 x = 1 + x2 ;

4. log4 (x + 1)2 =

8. (4x 2.2x 3) log2 x 3 > 4

1
log2 (x + 2)3 + 2 log2 4 x + 1;
3

TRN ANH TUN - 0974 396 391 - (04) 66 515 343

WWW.VNMATH.COM

x+1
2

4x ;
Trang 147

www.luyenthi24h.com
www.luyenthi24h.com
www.VNMATH.com

www.VNMATH.comCHUYN LUYN THI I HC


9. 2 log3 (x2 4) + 3

log3 (x + 2)2 log3 (x 2)2 = 4;

10.

2x + 1
2x+3 2 + < 4x + 9.2x+1 3.
2

Bi 6.255 : Gii cc h phng trnh

1.

8
>
<4log3 (xy) 2 = 2log3 (xy)

1
>
:log4 (4x2 + 4y2 ) = + log4 x + log4 (x + 3y);

2.

8
>
<2 x 21y + log2

x
=0
1y

>
: x(1 y) + 5y + 1 = 0.

WWW.VNMATH.COM

TRN ANH TUN - 0974 396 391 - (04) 66 515 343

WWW.VNMATH.COM

Trang 148

www.luyenthi24h.com
www.luyenthi24h.com

Chng 7

Tch phn
7.1 Cc dng ton c bn v nguyn hm
Vn 1 : Chng minh mt hm s F(x) l mt nguyn hm ca hm s f (x)

Cho hm s f xc nh trn K. Hm s F c gi l mt nguyn hm ca f trn K nu F (x) = f (x) vi mi x K

Bi 7.1 :

1. Chng minh rng


F(x) = 4 sin x + (4x + 5)ex + 1

l mt nguyn hm ca hm s f (x) = 4 cos x + (4x + 9)ex .


2. Chng minh rng hm s F(x) = |x| ln(1 + |x|) l mt nguyn hm ca hm s f (x) =
3. Chng minh rng

8
x2
>
<

F(x) =

l mt nguyn hm ca hm s f (x) =

8
< x ln x
:

>
:1

ln x

x2
+1
4

x
.
1 + |x|

khix > 0
khix = 0

khix > 0
khix = 0

trn [0; +).

Bi 7.2 : Xc nh cc h s a, b, c hm s F(x) = (ax2 + bx + c) 3 2x l mt nguyn hm ca hm s f (x) =

x 3 2x.
2x 3
Bi 7.3 :
1. Tm m hm s F(x) = ln(x2 + 2mx + 4) l mt nguyn hm ca hm s f (x) = 2
.
x 3x + 4
2. Cho hm s f (x) = xex v F(x) = (ax + b)ex . Vi gi tr no ca a v b th F(x) l mt nguyn hm ca f (x).

Vn 2 : S dng bng nguyn hm c bn

Ta c bng nguyn hm cc hm s c bn sau

149

WWW.VNMATH.COM

www.luyenthi24h.com
www.luyenthi24h.com
www.VNMATH.com

www.VNMATH.comCHUYN LUYN THI I HC


1.
2.

3.

0 dx = C;

1 dx = x + C;

(b)

R
x+1
1 (ax + b)+1
+ C; (ax + b) dx = .
+C
+1
a
+1
(vi , 1, a , 0);

(c)

dx =

x dx =

R 1

dx = ln |x| +C;

5.

4. Vi a l hng s khc 0
(a)

(d)

1
1
dx = ln |ax + b| +C (a , 0);
ax + b
a

sin(ax + b) dx =

cos(ax + b)
+ C;
a

R
R
R

cos(ax + b) dx =
e(ax+b) dx =
x dx =

sin(ax + b)
+ C;
a

e(ax+b)
+ C;
a

x
+ C (vi 0 < , 1);
ln

1
dx = tan x + C;
cos2 x
R 1
(b)
dx = cot x + C.
sin2 x
(a)

Bi 7.4 : Tm nguyn hm ca cc hm s sau :


1.

x+

x+1
3
;
x

2.

3.

x+1

7.

8. e32x ;

x+1 ;

1
;
sin x cos2 x

11.

6.

12.

1
;
(1 + x)(1 2x)

1 x2
x

1
;
x(1 + x)2

14.

x4 2
;
x3 x


(1 + sin 2x);
4
16. sin x sin 2x cos 5x;
15. sin x

1
1
10. 3 ;
x
x

x3 + 1
5.
;
1 x2

13.

9. x(x + 1)(x + 2);

cos 2x
4.
;
sin x + cos x

2x 1
;
ex

2

17. sin6 x + cos6 x;


1
;
18.
2 + sin x cos x

3x2 + 3x + 3
;
x3 3x + 2

19. sin x cos2 x.

Vn 3 : Tm hng s C
Bi 7.5 :

1. Tm nguyn hm F(x) ca hm s f (x) =

x3 + 3x2 + 3x 1
1
, bit rng F(1) = .
2
x + 2x + 1
3

1 + sin x
, bit rng F(0) = 2.
1 + cos x
Bi 7.6 : Tm cc hm s tha mn cc iu kin sau :
2. Tm nguyn hm F(x) ca hm s f (x) =

1. f (x) = 2x + 1, th ca n i qua im (1; 5);

2. f (x) = 2 x2 v f (2) =

Bi 7.7 : Tm hm s y = f (x) c th i qua im (1; 2) v tha mn f (x) = ax +

7
.
3

b
, y f (1) = 4 v f (1) = 0.
x2

Vn 4 : Phng php nguyn hm tng phn

Cng thc

u dv = uv

v du.

V vic chn u, v nh th no chng ta xem phn phng php tch phn tng phn.

Bi 7.8 : Tnh cc nguyn hm sau :

TRN ANH TUN - 0974 396 391 - (04) 66 515 343

WWW.VNMATH.COM

Trang 150

www.luyenthi24h.com
www.luyenthi24h.com
CHUYN LUYN THI I HCwww.VNMATH.com

www.VNMATH.com
1.
2.
3.
4.
5.

R
R
R
R
R

(1 2x)e3x dx;

8.

(x2 + 2x 1)e x dx;

9.

x sin(2x + 1) dx;

10.

(x 1) sin x dx;

11.

x ln(1 x) dx;
R
6.
x ln2 x dx;

7.

12.

R
R
R
R
R

e x sin x dx;

15.

e3x sin 5x dx;

16.
17.

e3x cos 7x dx;

18.

xe cos x dx;
x

xe sin(2x + 1) dx;
R
x
13. x sin dx;
2
R
14. x2 cos x dx;

e x cos x dx;

2x

19.

R
R
R
R

x ln x dx;

22.

x2 e x dx;

23.

3 x cos x dx;

24.

xe sin 2x dx;
x

R 1 + sin x

25.

e dx;
x

1 + cos x
20. sin(ln x) dx;

R
21. ln x + 1 + x2 dx;
R

26.
27.

R
R

x ln

1+x
dx;
1x

cos (ln(tan x)) dx;

R x cos x
R
R
R

sin2 x

dx;

x2 x dx;
xex dx;
25e3x cos 4x dx.

Vn 5 : Phng php i bin s

Cho hm s u = u(x) c o hm lin tc trn [a; b] v hm s f (u) lin tc sao cho f [u(x)] xc nh trn [a; b]. Khi nu F l mt
nguyn hm ca f , tc

f (u) du = F(u) + C th
Z

f [u(x)] u (x) dx = F [u(x)] + C.

Vic chn u = u(x) nh th no chng ta xem thm phn i bin tch phn.

Bi 7.9 : Tm cc nguyn hm sau :


1.

R
R

11.

7
dx;
4 3x
R
3

3.
dx;
2x + 1

R 4x 5
4.
e + 3x + 2 dx;
2.

5.
6.
7.
8.

R
R
R
R

cos

 

2
6x + 5

dx;

2x(x2 + 1)3 dx;

dx;

x3 4

R
9. x x 1 dx;

R
10. 2x x2 + 1 dx;

3x2
dx;
x3 + 1
R
x
14.
dx;
2
(3x + 9)4

R
15. 2x e x2 +4 dx;
13.

(2x + 1)4 dx;

3x2 x3 + 1 dx;

R
12. 2x3 4 x4 dx;

2(4x 1)6 dx;

2x + 4
16.
dx;
x2 + 4x 5

R 3
17. x 2 t2 dx;
18.
19.
20.

R
R
R

cos xesin x dx;


ex
dx;
x
e +1

cos x sin4 x dx;


R
21. x x + 1 dx;

cos x
dx;
1 + sin x
R
x
23.
dx;
x2 + 4

R
24. (x + 1) x 1 dx;
22.

25.
26.
27.
28.
29.
30.

R tan x

sin2 x
R
R

dx;

4x
dx;
(1 2x2 )
4x
dx;
(1 2x2 )2

R ln x

dx;

ex
dx;
1 + ex

1
dx.
x ln x

dx;
2x + 3

x
dx;
(1 + x2 )2

dx
;
e x ex

Bi 7.10 : Tnh cc nguyn hm sau :


1.

(2x + 1)20 dx;

x
dx;
x2 + 1

R
3. x2 x3 + 5 dx;
2.

4.
5.

e3 cos x sin x dx;

R ln4 x

dx;

e2x

dx;
ex + 1

R
7. 3x 7 3x2 dx;
6.

8.
9.

R
R

9x2

dx;
1 x3

dx;
x(1 + x)3

TRN ANH TUN - 0974 396 391 - (04) 66 515 343

10.
11.
12.
13.

R ln2 x

WWW.VNMATH.COM

dx;

Trang 151

www.luyenthi24h.com
www.luyenthi24h.com
www.VNMATH.com

www.VNMATH.comCHUYN LUYN THI I HC


14.
15.
16.

R
R
R

sin x cos x
, (a2 , b2 );

2
a sin2 x + b2 cos2 x
R
dx
18.
;
cos x sin2 x
R
19. x 1 + x2 dx;

1 + ln x
dx;
x

17.

cos x sin3 x dx;


cos x + sin x

dx;
sin x cos x

20.
21.
22.

R
R
R

sin2 x cos3 x dx;


e3 sin x cos x dx;
(3x + 2)10 dx.

Bi 7.11 : Tnh cc nguyn hm sau :


1.

x3 ex dx;
R

2. sin x dx;

7.

R ln(ln x)

8.

3.

dx;

x
R
2
4. cos (ln x) dx;

5.

6.

dx;

R
R
R

9.

10.

sin(ln x) dx;

cos2 x dx;

1
1

dx;
ln2 x ln x
x cos x
dx;
sin2 x


sin
x + 1 dx;

11.

R ln (tan x)

cos2

dx;

x
x
x
12. sin cos dx;
3
3
R 1
1
1
sin cos dx;
13.
x2
x
x
R

14.

15.

dx
;
sin x + cos x

dx
;
8 4 sin x + 7 cos x

dx
;
3 + 5 cos x

16.

17.

R 4 sin x + 6 cos x + 5

sin x + 2 cos x + 2

dx.

7.2 Cc dng ton tch phn


Vn 1 : S dng tch phn c bn

Nu F l mt nguyn hm l mt nguyn hm ca f trn [a; b] th


Z

f (x) dx = F(x) = F(b) F(a).


a

Bi 7.12 : Tnh cc tch phn sau :


1.

R2

x(x + 1)2 dx;

R2

5.

2.

R
0

3.

R2
1
2

4.

9.

(2 cos x sin 2x) dx;


1
dx;
x(x + 1)

ex

+1

dx;

6.

R8

7.

1
4x 3
3 x2

R1

8.

10.

(sin 6x sin 2x 6) dx;

3x e

x
4

R4
1

ln
R 2 e2x+1
0

2x2 + cos x dx;

11.

dx;

dx
;
2
x (x + 1)
3

sin x
dx;
1 cos x

R2
0

12.

dx;

R3

13.

R4
0

dx
;
(1 + tan2 x) cos4 x

14.

R2

cos2 2x dx;

x3 2x2 + x dx;

dx
;
2
sin x cos2 x

15.

R2

sin 2x sin 6x dx;

16.

R6

tan x dx.

Vn 2 : Tch phn hm cha du tr tuyt i


1. Cng thc tch cn tch phn

f (x) dx =

2. Tch phn cha du tr tuyt i

Rb
a

c
a

f (x) dx +

f (x) dx.

| f (x)| dx (gi s a > b).

(a) Gii phng trnh f (x) = 0, c cc nghim xi [a; b], gi s a x1 < x2 < < xn b.

TRN ANH TUN - 0974 396 391 - (04) 66 515 343

WWW.VNMATH.COM

Trang 152

www.luyenthi24h.com
www.luyenthi24h.com
CHUYN LUYN THI I HCwww.VNMATH.com

www.VNMATH.com
(b) Dng cng thc tch cn

Zb
a

Zx1

| f (x)| dx =

Zx2

| f (x)| dx +

Zb

| f (x)| dx + +

x1

xn

| f (x)| dx






Zx1
Zx2

Zb






= f (x) dx + f (x) dx + + f (x) dx .





a

x1

xn

Ch : Sau khi tch cn chng ta c th ph du gi tr tuyt i ch khng nht thit phi a gi tr tuyt i ra ngoi tch phn.

Bi 7.13 :

1. Cho

R5
0

f (t) dt = 3 v

R7

f (u) du = 4, tnh

R7

f (x) dx.

2. Xc nh hm s f (x) = A sin x + B, bit rng f (1) = 2 v

R2

f (x) dx = 4.

Bi 7.14 :

1. Cho hm s f (x) = a.3 x + b, bit rng f (0) = 2 v

R2

f (x) dx = 12. Tm cc gi tr ca a v b.

2. Cho hm s f (x) = a sin 2x + b, bit rng f (0) = 4 v

R2

f (x) dx = 3. Tm cc gi tr ca a v b.

Bi 7.15 :

1. Cho

2. Cho a

R4

f (x) dx = 1 v

R6

f (t) dt = 5. Tnh tch phn I =

R6

f (x) dx.

R1
3
v tho mn cos(x + a2 ) dx = sin a. Tnh gi tr ca a.
;
2 2
0

Bi 7.16 : Tnh cc tch phn sau :


1.

R2
0

2.

R2
0

6.

|x2 x| dx;

7.

4.

R2
0

1 + x2

8.

R4

9.

dx;

R5

|x 2| dx;

10.

R2
0

R3
0

12.

x2 6x + 9 dx;

|2 x 4| dx;

13.

16.

4 |x| dx;

17.

|x2 + 2x 3| dx;

15.

tan2 x + cot2 x 2 dx;

19.

R
0

1 sin 2x dx;

cos x cos x cos3 x dx;

18.

R3

R2

R2

14.

x3 4x2 + 4x dx;

1 + cos x dx;

1 sin x dx;

R2

R1
1

(|x + 2| |x 2|) dx;

R3
0

11.

|x2 1| dx;

1 cos 2x dx;

R 3 |1 x2 |

R3
3

R2

3.

5.

|1 x| dx;

| sin x| dx;

1 + cos 2x dx;

20.

R2

1 + cos x dx.

Vn 3 : Phng php tch phn tng phn


Zb

b Zb

u dv = uv v du.
a

Dng phng php tch phn tng phn khi tch phn ca chng ta va cha ln ln cc hm : hm a thc, hm m, hm lga (hoc
ch cha hm lga), hm lng gic, hoc cha hm v t.
Nu cha lga chng ta thng t u l lga v dv l phn cn li hoc t u l a thc v dv l phn cn li.
Ch :

Tch phn I =

e x sin x dx t u = e x v dv = sin x dx . . .;

Trc khi dng tch phn tng phn chng ta phi kim tra xem c lm c bng phng php i bin s khng ;

TRN ANH TUN - 0974 396 391 - (04) 66 515 343

WWW.VNMATH.COM

Trang 153

www.luyenthi24h.com
www.luyenthi24h.com
www.VNMATH.com

www.VNMATH.comCHUYN LUYN THI I HC

Mt cch tng qut, chng ta t u l biu thc d xc nh o hm, dv l phn cn li d xc nh nguyn hm.

Bi 7.17 : Tnh cc tch phn sau :


ln
R2

1.

xe2x dx;

9.

R1

(2x2 + x + 1)e x dx;

10.

3.

R
0

(1 x) sin x cos x dx;

4.

R4

11.

5.

13.

2x ln x dx;

6.

x + 1e

x+1

dx;

18.

19.

14.

x3 ln2 x dx;

dx;

20.

7.

15.

e2x sin 3x dx;

(x2 + 2x + 3) cos x dx;

21.

(x 1) sin x dx;

22.

8.

16.

e cos 2x dx;

R3
R

(ln(x 1) ln(x + 1)) dx;


e x cos2 x dx;

R1

e x sin2 (x) dx;

x cos x sin2 x dx;

23.

R2

x2 cos x dx;

R2 x sin x

1 + x2 dx;

x ln x +

R2

R1

R2

x ln2 x dx;

R2

Re

2x ln(x 1) dx;

Re

(2x 1)e2x dx;

R3
R1

R5
2

R3

17.

12.

x sin x dx;

R1
0

(x2 + 1)e2x dx;

2.

R1

1 + cos x

dx;

24.

R3
0

(2 x) sin x dx.

Vn 4 : Phng php i bin s


1. Phng php i bin s n gin
1R
f (ax + b) d(ax + b);
a
R
1R
1 (2x 3)3
VD : (2x 3)2 dx =
(2x 3)2 d(2x 3) =
+ C.
2
2
3
1
Ch : d(ax + b) = a dx dx = d(ax + b).
a
R
1
f (xn+1 )xn dx =
f (xn+1 ) d(xn+1 ), t t = xn+1 ;
(b)
n+1
R
R
VD : I = (4x3 + 1)2 x5 dx = (4x3 + 1)2 x3 .x2 dx.
1t
t t = 4x3 + 1 dt = 12x2 dx v x3 =
.
4

3
R
1t
dt
Vy I = t2
=
4
12
(c) V c bn khi c cn chng ta thng t t l ton b cn, ri ly tha hai v cho mt cn; nu biu thc trong cc hm
(a)

f (ax + b) dx =

sin, cos, tan, cot, ln hoc ly tha l phc tp th ta thng t t l biu thc phc tp .
VD :

R t dt
t dt
x2 2x3 + 1 dx, t t = 2x3 + 1 t2 = 2x3 + 1 2t dt = 6x2 dx x2 dx =
, nn I = t.
=
3
3
R
R
R
dt
2
2
ii. I = x3 .e x +1 dx, t t = x2 + 1 dt = 2x dx v x2 = t 1, nn I = x2 .e x +1 x dx = (t 1)et
ri dng phng
2
php nguyn hm tng phn.
R 1
R
1R
1
1
1
dx
sin 2t dt.
iii. I =
sin cos dx, t t = dt = 2 , nn I = sin t cos t dt =
x2
x
x
x
x
2
i. I =

2. Phng php i bin s tch phn lng gic


Tch phn ch cha cc hm s lng gic sin, cos, tan, cot chng ta bin i v mt trong cc dng sau :

TRN ANH TUN - 0974 396 391 - (04) 66 515 343

WWW.VNMATH.COM

Trang 154

www.VNMATH.com
(a)

www.luyenthi24h.com
www.luyenthi24h.com
CHUYN LUYN THI I HCwww.VNMATH.com

f (sin x) cos x dx, hm f (sin x) l tnh theo sin x (ch rng cos2 x = 1 sin2 x v ni chung ly tha bc chn ca cos x

u a c v sin x), t t = sin x dt = cos x dx (tc l tch phn cha sin x m l).
(b)

f (cos x) sin x dx, hm f (cos x) l tnh theo cos x (ch rng sin2 x = 1 cos2 x v ni chung ly tha bc chn ca sin x

u a c v cos x), t t = cos x dt = sin x dx (tc l tch phn cha cos x m l).
R
dx
dx
(c)
f (tan x) 2 , t t = tan x dt =
(tc l tch phn c ly tha ca sin x v cos x cng tnh chn l). Trng
cos x
cos2 x
2t
1 t2
hp c bit, tch phn cha tan x, sin 2x, cos 2x cng t t = tan x, khi sin 2x =
, cos 2x =
.
2
1+t
1 + t2
R
dx
dx
(d)
f (cot x) 2 , t t = cot x dt = 2 .
sin x
sin x


(e) Tch phn cha (sin x + cos x) dx hoc sin x +
dx t t = sin x cos x.
4



(f) Tch phn cha (sin x cos x) dx hoc sin x


dx t t = sin x + cos x.
4
R cos x dx R
R
dx
1
1
=
=
cos x dx =
cos x dx, t t = sin x.
2
2
cos x
cos x
cos x
1 sin2 x

3. Phng php i bin vi tch phn cha ax2 + bx + c

(a) Nu cha a2 x2 t x = a sin t, t .


2
2

2
2
(b) Nu cha x a t x =
, t v t , 0.
sin t 2
2

(c) Nu cha x2 + a2 t x = a tan t, < t < .


2
2

VD : I =

VD :

dx

, t x = 2 sin t ( t ) dx = 2 cos t dt. Ta c :


2
2
2
2x

R 2 cos t dt R
2
2
2

= dt = t + C.
2 x = 2 2 sin t = 2 cos t = 2 cos t, v I =
2 cos t

dt
1
(b) I =
x2 + 1 dx, t x = tan t, < t < , nn dx =
v
x2 + 1 =
. Ta c :
2
2
cos2 t
cos t
2
R dt
R
R
d(sin t)
1
(sin t + 1) (sin t 1)
I=
d(sin t) = . . .
=
=
cos3 t
(sin t + 1)(sin t 1)
(1 sin2 t)2 2

R
dx
(c) I =
, t x = tan t v ta c I = ln |x + x2 + a2 | + C.
x2 + a 2
x
(d) Mt cch tng qut tch phn cha sin x, cos x, tan x, cot x chng ta t t = tan .
2
(a) I =

4. Phng php i bin vi tch phn ch cha hm m


Ta t t l c hm m , chng hn :
R t dt
ex
dt
dx, t t = e x dt = e x dx = t dx dx =
, vy th I =
= . . ..
x
e +1
t
t+1 t
R dx
R t lndt2
dt
x
x
(b) J =
,
t
t
=
2

dt
=
2
ln
2
dx
=
t
ln
2
dx

dx
=
,
vy
th
J
=
= ...
2x + 1
t ln 2
t+1

(a) I =

5. Phng php i bin tch phn cha lga v phn thc


R
dx
dx
I = f (ln x). , t t = ln x, ta c dt =
.
x
x
R ln x + 1
R
dx
VD : Tnh I =
dx, t t = ln x + 1 dt =
, vy I = t dt.
x
x

Bi 7.18 : Tnh cc tch phn sau :

TRN ANH TUN - 0974 396 391 - (04) 66 515 343

WWW.VNMATH.COM

Trang 155

www.luyenthi24h.com
www.luyenthi24h.com
www.VNMATH.com

www.VNMATH.comCHUYN LUYN THI I HC

22

1.

R3
3

12.

3x + 5 dx;

2.

R1

13.

x3 (1 + x4 )3 dx;

3.

2 3x3

xe

14.

dx;

R2
0

5.

sin x
dx;
1 + cos x

a
2

15.

dx

, (a > 0);
a 2 x2
Ra
dx
6.
, (a > 0);
2
2
0 a +x
R1
0

8.

R2
1

9.

R1
0

10.
11.

dx
;
2
x +x+1

16.
17.

R8

R1

19.

24.

Re5 ln(ln x)

dx;

25.

R1

x+1
3
dx;
3x + 1

20.

R
0

21.

R
0

26.

22.

R2

dx;

Re

27.

R2

35.

cos(ln x) dx;

29.

1 + 4 sin x cos x dx;

dx;
cos2 x + 2 sin2 x

30.

dx
;
(sin x + 2 cos x)2

31.

sin 2x

36.

R3

37.

R1

38.

ln
R2

39.

xe x

dx;
ex + 1

32.

40.

e x ln(e x + 1) dx;

R4 x sin x dx

cos3 x

x dx
;
+ x2 + 1

e2x sin2 (e x ) dx;

xe x cos x dx;

x4

esin x + cos x cos x dx;

sin x ln(tan x) dx;

x dx
;
1 + sin 2x

esin x + cos x cos x dx;

dx;

cos x ln(sin x) dx ;

ln
R3

R1

(x + sin2 x) cos x dx;

R2

R4

(1 + sin x)1+cos x
dx;
1 + cos x

Re 1 + x ln x

R2

28.

ln

x3 e x dx;

1+x
dx;
x

R2

34.

1 + ln x
dx;
x

x5 1 + x2 dx;

cos2 x

cos x dx;

e2

x2 2 x2 dx;

R1

33.

e x 1 dx;

x + 2x + 10x + 1
dx;
x2 + 2x + 9

R3

Re
1

18.

x(1 x)5 dx;


3

ln
R2
0

7.

Re ln2 x

R4

23.

2x

dx;
1 + x2

4.

R2

R3 x dx

4 cos2 x

R1

R2 sin 2x dx

Re ln(ln x)
2

41.

dx.

Bi 7.19 : Tch phn cc hm s lng gic


1.

sin4 x cos4 x dx;

12.

2.

cos 3x tan x dx;

13.

3.

sin x sin 2x cos 5x dx;

R3

5.

14.
15.

16.

R2

sin x + cos x dx;


6

17.

8.

1 + cos x

9.

18.
19.

10.

R6 tan4 x
0

27.

tan2 x + tan4 x dx;

R4
0

R2 sin x + 7 cos x + 5

29.

dx;

dx;

cos 2x
dx
;
cos4 x

R2

sin3 x cos2 x dx;

4 sin x + 3 cos x + 5

R2 9 sin x 2 cos x

cos x + 2 sin x + 1

20.

R
0

dx

;
sin 2x + 2 3 cos2 x + 2 3


R3 2 sin x 4
11.
dx;
cos x
0

12

dx;
1 + cos2 x

28.

sin 2x sin 5x dx;

2 sin x + cos x + 1

26.

cos x

dx;

R12

dx
;
(sin x + 2 cos x)2

dx;

R2 3 cos x + sin x + 2

dx;

R2

dx;

R2 4 sin3 x

25.

3 cos 2x 4 sin 2x + 5

R4
0

tan2 x + cot2 x 2 dx;

R3

24.

R4 3 sin 2x + 4 cos 2x + 5

R4

dx
;
1 + sin x + cos x

dx
;
1 + cos 2x
1 + cos x

R2

R2
0

R2 4 sin3 x

cos4 x dx;

7.

6.

cos 3x cos 5x dx;

cos10 x + sin10 x sin4 x cos4 x dx;

4.

23.

R3

R3

21.

dx;

30.

R6
0

sin x
dx;
3 + cos2 x

31.

sin2 x cos4 x dx;

32.

R4
0

R2

cos x cos x cos3 x dx;

TRN ANH TUN - 0974 396 391 - (04) 66 515 343

33.

cos7 x

dx;
dx

cos x cos x +

R2

 ;
4

dx
;

2 + sin x cos x

R4 sin x dx
0

22.

R4 sin5 x

1 + sin 2x

dx
.
sin 2x 2 sin x

WWW.VNMATH.COM

Trang 156

www.luyenthi24h.com
www.luyenthi24h.com
CHUYN LUYN THI I HCwww.VNMATH.com

www.VNMATH.com
Bi 7.20 : Tch phn hm v t
1.

R3

2.

R2

5.
6.

R1

dx

R1
0

8.

9.

R1

20.
21.

;
1+ 3x

22.

1x
dx;
1+x

23.
24.

R
0

11.

R2
1

12.

R16
1

25.

x dx

;
1x

26.

15.

16.

x dx

;
1+ x1

27.

dx

;
x+9 x

28.

R1
R2
1

30.
31.

R1

32.

dx;

R5

R1

39.

2x 3

x2 + x + 1
x2 + 1

x2 4x + 3

2R 3

dx;
dx;

x
dx;
1+ 3x

42.

R3

dx

x x2 + 4

R1

x+1
x2 2x + 2

R3
1

43.

1+x
dx;
x3

x3 x2 + 1 dx;

x3 1 x2 dx;

3 2
R5

x5

p
3

1
3
5

dx;
44.

dx
dx;
(x2 + 8)3

45.

R1

(2 5x3 )2 dx;
dx
n
, n N;
1 + xn

xn )

(1 +

R1

7
x7 8x4 + 1 dx;

dx

;
4 x2

R1

1x
dx;
1+x

x2

dx;
3 + 2x x2

R1

38.

41.

dx

;
1 + x + x2 + 1

1
3

2x x2 dx;

1+x
dx;
1x

R1

40.

x(x + 2) dx;

37.

x 4 x2 ;

x 1 x dx;

dx
;
2 2x + 2
2
x
x 2x + 2
2x + 2

R2

R1 x2 2x + 5
0

2x2

dx;
1+x

x
dx;
2x

R1
R2

34.

36.

R0

29.

x2

dx;

1 x2

12

;
(x 1) x2 4x + 3

2+ 2

R0

R2 x3 x5

dx

R9 3
1

x+
R4

R1

x dx
;
13.

1+x
0
R3
14.
x3 2x2 + x dx;
R1

(1 x)

R1

33.

10.

R1

x2

35.

dx

(x2 + x) x + 1 dx;

3
4

x 3 + x2 dx;

R2

dx

;
x+1+ x1

R2

R1
0

dx
;

x+ 3x

R64

7.

19.

x dx

;
1+ 2+x

R7

R1

x dx
3
;
x+1

R7
0

4.

dx
4 ;
x
0 1+
a

R
18. x2 a2 x2 dx, vi a > 0;
17.

x+3

dx;
x 2x + 3

3.

dx
p
;
x + 1 + (x + 1)2

46.

R1

x15 1 + 3x8 dx.

Vn 5 : Tch phn cc hm hu t

P(x)
dx, vi P(x) l mt a thc no .
+ bx + c
3
2
R 2x + 3x x
VD : Tnh I =
dx.
x2 + 2x + 2

Xt tch phn dng

ax2

Chia t cho mu (bc t ln hn bc mu), c


Z

I=
vn l cn tnh I1 =

(2x 1) dx +

3x + 2
dx
+ 2x + 2

x2

3x + 2
dx.
+ 2x + 2

x2

TRN ANH TUN - 0974 396 391 - (04) 66 515 343

WWW.VNMATH.COM

Trang 157

www.luyenthi24h.com
www.luyenthi24h.com
www.VNMATH.com

www.VNMATH.comCHUYN LUYN THI I HC


Tch t theo o hm ca mu : o hm ca mu l 2x + 2, v t l 3x + 2 =
I1 =
Vi

R (2x + 2) dx

3
2

(2x + 2) dx
+5
x2 + 2x + 2

3
(2x + 2) + 5, vy :
2

dx
.
x2 + 2x + 2

R d(x2 + 2x + 2)

= ln |x2 + 2x + 2| + C.
x2 + 2x + 2
x2 + 2x + 2
R
dx
Vi
, ta nhn thy mu x2 + 2x + 2 v nghim, nn x2 + 2x + 2 = (x + 1)2 + 1 (tng qut : ax2 + bx + c =
2 + 2x + 2
x

b 2
a x+
+ ) v ta c
2a
4a
Z
Z
dx
dx
=
x2 + 2x + 2
(x + 1)2 + 1
=

t x + 1 = tan t dx =

1
dt
v (x + 1)2 + 1 = tan2 t + 1 =
, thay vo ta c
2
cos t
cos2 t
Z

Dng tng qut :

x2

dx
=
2
x + 2x + 2

dt
cos2 t
1
cos2 t

(x + 1) dx +

dt = t + C.

dx
, t x = a tan t.
+ a2

Mt v d khc vi mu l a thc hai nghim phn bit.


R 2x3 x2 + x 4
VD : Tnh I =
dx v bin i nh trn ta c :
2x2 3x + 1
I=

3x 5
dx =
2x2 3x + 1

(x + 1) dx +

3
4

4x 3
11
dx
2x2 3x + 1
4

dx
2x2 3x + 1

R d(2x2 3x + 1)
4x 3
dx
=
= ln |2x2 3x + 1| + C.
2x2 3x + 1
2x2 3x + 1

R
dx
1
1
2
2
Vi
, nhn thy mu 2x 3x + 1 c hai nghim phn bit 1 v , nn 2x 3x + 1 = 2(x 1) x
.
2x2 3x + 1
2

2
1

(x 1) x
1
1
1
1
1
1
2

= .(2).

=
= .

.
Ta bin i 2
1
1
2x 3x + 1 2
2
x 12 x 1
(x 1) x
(x 1) x
2
2
Ta c :

Vi

dx
=
2x2 3x + 1

dx
dx

x 12 x 1

d x
x

1
2

1
2

d(x 1)

x1


1
= ln x ln |x 1| + C.
2

V cui cng ta xt v d vi mu l a thc c nghim kp.


R
dx
1R
dx
1 R d(x + 1)
1 1
VD : Tnh
=
=
= .
+ C.
2
2
2x 4x + 2 2 (x + 1)
2 (x + 1)2
2 x+1
Ch rng :

Nu ax2 + bx + c c hai nghim x1 , x2 th ax2 + bx + c = a(x x1 )(x x2 ).


Nu ax2 + bx + c c nghim kp x = x0 th ax2 + bx + c = a(x x0 )2 .
d(x + a) = dx vi mi s thc a.

Bi 7.21 : Tnh cc nguyn hm sau :


1.

2.

dx
;
3x + 1

R x2 + 3x 1

2x + 3

3.

dx;

4.

2x2
R

dx
;
x+1

dx
;
2
x 4x + 4

TRN ANH TUN - 0974 396 391 - (04) 66 515 343

5.

R x3 + 5x2 + 3x 7

6.

R x2 6x + 10

x2

x2

+ 6x + 9

6x + 8

dx;

dx;

7.

8.

dx
;
+ 1)

x2 (x
R

WWW.VNMATH.COM

x2

2x 7
dx;
3x + 2

Trang 158

www.VNMATH.com
R

x1
dx;
+ 3x + 2)2
R
2x + 5
dx;
10.
9x2 6x + 1
R
2x + 1
11.
dx;
2
(x 4x + 4)3
9.

(x2

www.luyenthi24h.com
www.luyenthi24h.com
CHUYN LUYN THI I HCwww.VNMATH.com
12.

13.

R 3x + 1

(x +

1)3

dx;

14.

x3
dx;
(x2 + 1)2

15.

x dx
;
(x2 + 1)2

R x2 + 2x + 1

x2 + 1

16.

dx;

17.

R x4 1

x3 x

dx;

(x2 + 1) dx
;
(x 1)3 (x + 3)

Bi 7.22 : Tnh cc tch phn sau :


1

1.

R2
0

2.

R2
0

3.

R1
0

4.

R1
0
1

6.

R2
0

7.

R1
0

8.

R2
1

9.

R1
0

10.

x dx
;
4
x + x2 + 1

13.

1+

x4

dx;

R1
0

R1
0

14.

R4
0

dx
;
x4 + 4x2 + 3
dx
;
3x2 2x 1
dx
;
x2 4x + 5
dx
;
x2 2x + 2

15.

R1
0

16.
17.

R2
1

18.

20.

x dx
;
+x+1

R1
0

R1
0

21.

(x 4) dx
;
2x3 4x2 + 6x 12

22.

3x + 8
dx;
x2 9x + 14

23.

x dx
;
x4 + 4x2 + 3

24.

x2
dx;
(1 + x)2

25.

x2 + x

x1
x+2

dx;
2

(2x + 1)3

R2

(x2

R1 (1 3x)4
0

19.

x2

R4 2x + 1
2

x dx
;
x4 5x2 + 4
x2 + 1

R1
1

12.

R1 x2 dx
0

11.

3x dx
;
2
x + 2x + 1
3

R2 1 x2
1

5.

x3 dx
;
x2 3x + 2

26.
dx;

27.
28.

dx;

x3 dx
;
(x2 + 1)2

29.

x5 dx
;
6
4x + 4x3 + 1

30.

R1
(x2 + 1) dx
dx
; 31.
;
2
+ 3x 1)(x + 5x 1)
(11
+
5x)2
2

(x2 4) dx
(x2 + 1) dx
; 32. R2
2
2
2
2
1 (x 3x + 4)(x 2x + 4)
1 (x + 5x + 1)(x 3x + 1)
2
1
R
6x + x + 2
dx;
R2
(4x + 2) dx
(4x
+ 1)(x2 + 1)
0
33.
;
2
2
1 (x + x)(x + x + 2)
4
2
1
R x + 5x + 4
dx;
R2
x(x2 + 2)2
(x2 6) dx
1
2
34.
2
2
1 (x + 3x + 2)(x + 9x + 18
R1
4x 2
dx;
2
2
R0 
x
0 (x + 2)(x + 1)
35.
dx;

2
2+ 6
1 x 3x + 2
R2 x2 + 1
dx;
x4 + 1
R1 x dx
1
36.
;
2
2
R1 x dx
0 (x + 1)
;
3
0 (x + 1)
R2
5x + 3
37.
dx;
2
1
R x 1
3
2
1 x 2x 3x
dx;
4
1 x + 1
2
3
R2
dx

R1 2x + 1 2
38.
;
3 4x
x
dx;
1
x+1
0
R2

R1
0

x2
dx;
2
(x + 1)2

39.

R2 3x2 8x + 13
0

(x + 3)(x 1)2

dx.

Vn 6 : Tch phn mt s hm c bit


1. i vi hm chn, l
(a) Nu hm s y = f (x) l hm s chn, lin tc trn [a; a] th
Za

f (x) dx = 2

Za

f (x) dx.

(b) Nu hm s y = f (x) l hm s l, lin tc trn [a; a] th


Za

f (x) dx = 0.

Nhn xt : Nh vy, trc khi tnh tch phn ta cn ch n hai cn, nu thy hai cn i nhau ta cn n tnh chn l
ca hm s di du tch phn ri p dng kt qu khng nh trn.
2. Tch phn kt hp gia hm chn v hm m
Cho f (x) l hm s chn, lin tc trn [a; a]. Khi :
Za
a

f (x)
dx =
mx + 1

TRN ANH TUN - 0974 396 391 - (04) 66 515 343

Za

f (x) dx.

WWW.VNMATH.COM

Trang 159

www.luyenthi24h.com
www.luyenthi24h.com
www.VNMATH.com

www.VNMATH.comCHUYN LUYN THI I HC


3. Tnh bt bin ca tch phn khi bin s thay i cn cho nhau
Ta c h thc :

Rb
a

f (a + b x) dx =

Rb

f (x) dx.

4. Tch phn hm tun hon


Nu hm s y = f (x) lin tc, tun hon vi chu k T th
a+T
Z

f (x) dx =

ZT

f (x) dx.

5. Hm s di du tch phn c trc i xng thng ng


Nu hm s y = f (x) lin tc trn [a; b] v f (a + b x) = f (x) th
Zb

a+b
x f (x) dx =
2

c bit :

x f (sin x) dx =

Zb

f (x) dx.

R
f (sin x) dx.
20

6. Tch phn ca cc hm s i xng nhau - tch phn lin kt lng gic

Nu f (x) lin tc trn [0; 1] th

R2

f (sin x) dx =

c bit

f (cos x) dx.

Z2

R2

Z2

sink x
=
(sin x + cos x)n

cosk x
;
(sin x + cos x)n

7. Bin i tch i hm s v co cn tch phn


Nu f (x) l hm lin tc trn [0; 2a], th

R2a

f (x) dx =

Bi 7.23 :

1. Chng minh rng

R1

sink x
=
sinn x + cosn x

Z2

cosk x
.
sin x + cosn x
n

( f (x) + f (2a x)) dx.

R1

ecos x dx = 2 ecos x dx.

2. Tnh cc tch phn sau:

Ra

Z2

Z2

I1 =

ln(x +

Z2

x2 + 1) dx; I2 =

cos x ln
12

1+x
1x

dx.

Bi 7.24 : Tnh cc tch phn sau :

1.

R3

cos7 x dx;

8.

2.

12

R1 x6 + tan x

x2 + 1

3.

Ra

x2

5.

6.

R1
1

7.

ln x +

R1

1 + x2

2007

(2 x

x2 |sin x|
;
2009 x + 1

10.

dx;

dx

;
(e x + 1) 1 x2

R2 sin x sin 2x cos 5x

ex + 1

3x
x
2+x
x + cos 6x + sin
sin
ln
2
2
2x

x4
sin4 x + cos4 x

R2

R1
1

9.

sin x + a2 x2 dx (a > 0);

R1

dx;

4.

R2

dx;

x5 x3 + x sin x
dx;
x4 + x2 + 1 + cos x

dx
;
+ 1)(x2 + 1)

TRN ANH TUN - 0974 396 391 - (04) 66 515 343

11.

R1 x ln 1 + 1 + x2

12.

dx;
(3 x + 1) 1 + x2

R1 x2 ln(1 + x2 )
1
1

13.

dx;

2x + 1

R2 x ln
12

1+x
1x

ex + 1

dx;

dx;

WWW.VNMATH.COM

Trang 160

www.luyenthi24h.com
www.luyenthi24h.com
CHUYN LUYN THI I HCwww.VNMATH.com

www.VNMATH.com

14.

R2 x2 cos x

ex + 1

dx;

27.

ln(1 + tan x) dx;

28.

R1 ln(1 + x)

1+

x2

29.

1 + cos10 x

dx;

0
5

20.

R4

21.

22.

1 cos 2x dx;

sin 2x
dx;
x + sin4 x

cos4

R x sin x dx
R

9 + 4 cos2 x

23.

33.
34.
35.

x sin x dx;

25.

R
0

26.

R
0

R3

sin x sin 2x sin 3x dx;

sin 5x
cos 7x dx;
sin 3x

R2

1 + sin x dx;

R2

1 + sin2 x

ln(sin x) dx;

x sin x dx;

24. I =

R2

dx
dx;
2009 x
1
+
tan
0

R4
ln(9 x)
32.

dx;

ln(9 x) + ln(x + 3)
2

R x sin x d x

ln(tan x) dx;

31.

R2

tan2007 2x + sin2009 6x dx;

2007
R

sinn x
;
x + cosn1 x

30.

19.

dx;

sin

n1

dx;

R4 sin7 3x cos8 5x
0

18.

R2
0

17.

1
tan2 (cos x)
cos2 (sin x)

R4
0

16.

15.

R2

36.

x sin x cos2 x dx;

cos4 x
;
sin4 x + cos4 x

37.

sin x
;
(sin x + cos x)3

38.

R2
0

cos3 x
dx;
sin x + cos x

R1
1

x4
dx;
1 + 2x

Bi 7.25 : Chng minh cc h thc sau :


1.

R1
0

2.

Ra

xm (1 x)n d x =
x3 f (x2 ) d x =

1
2

R1
0

Ra2

xn (1 x)m dx;
x f (x) dx (a > 0; x > 0);

3. Chng minh rng nu y = f (x) l hm chn, tun hon vi chu k T th

RT

f (x) dx = 2

R2

f (x) dx.

7.3 ng dng tch phn tnh din tch hnh phng


Bi 7.26 : Tnh din tch hnh phng gii hn bi :
1. elp :

x2 y2
+
= 1, (a, b > 0).
a2 b2

2. th hm s y = x3 1, ng thng x = 2, trc tung v trc honh.


3. th hm s y = 4 x2 , ng thng x = 3, trc tung v trc honh.
4. parabol y = 2 x2 v ng thng y = x.
5. ng thng y = x + 2 v parabol y = x2 + x 2.

6. th hm s y = x, trc honh v ng thng y = x 2.


Bi 7.27 : Tnh din tch hnh phng gii hn bi :

TRN ANH TUN - 0974 396 391 - (04) 66 515 343

WWW.VNMATH.COM

Trang 161

www.VNMATH.comCHUYN LUYN THI I HC


1. th cc hm s y =

www.luyenthi24h.com
www.luyenthi24h.com
www.VNMATH.com

27
x2
,y =
v y = x2 .
x
27

2. parabol y = 2x2 4x 6, trc honh, v hai ng thng x = 2, x = 4.


3. parabol (P) : y = x2 4x + 5 v hai tip tuyn ca (P) ti A(1; 2) v B(4; 5).
4. th cc hm s y = |x2 1| v y = |x| + 5.

5. th cc hm s y = 4 x2 v x2 + 3y = 0.

6. th cc hm s y = sin |x| v y = |x| .


7. th cc hm s x2 = 4y v y =

x2

8
+4

Bi 7.28 : Cho parabol (P) : y = x2 + 1 v cho ng thng dm : y = mx + 2.


1. Chng minh rng vi mi m th (P) v dm lun ct nhau ti hai im phn bit.
2. Tm m hnh phng gii hn bi (P) v dm c din tch nh nht.
Bi 7.29 : Tnh din tch hnh phng gii hn bi :
1. th cc hm s y = x2 , y =

2
8
x2
, y = v y = .
4
x
x

2. th cc hm s y2 = 2x, x 2y + 2 = 0 v trc honh.


3. th hm s y2 + x 5 = 0 v ng thng x + y 3 = 0.
4. th cc hm s x2 = 3y v y2 = 3x.
5. parabol y = x2 2x + 2 v cc tip tuyn ca n i qua im A(2; 2).

7.4 ng dng tch phn tnh th tch vt th trn xoay


Bi 7.30 : Cho hnh phng S gii hn bi parabol y = 2x x2 v trc honh.
1. Tnh th tch V x ca hnh trn xoay to bi khi quay S quanh trc Ox.
2. Tnh th tch Vy ca hnh trn xoay to bi khi quay S quanh trc Oy.
Bi 7.31 : Cho hnh phng S gii hn bi th cc hm s y = x2 , y =
khi quay S quanh trc Ox, Oy (tng ng).

27
x2
v y =
. Tnh th tch V x , Vy ca hnh trn xoay to bi
x
27

Bi 7.32 : Cho hnh phng S gii hn bi cc parabol y = 4 x2 v y = x2 + 2. Tm th tch V x , Vy ca hnh trn xoay to bi khi quay
S quanh trc Ox, Oy.

Bi 7.33 : Cho S l hnh trn tm I(2; 0) v bn knh R = 1. Tm th tch V x , Vy ca hnh trn xoay to bi khi quay S quanh trc
Ox, Oy.
Bi 7.34 : Cho hnh phng S gii hn bi th hm s y = xe x , trc honh v ng thng x = 1 . Tm th tch V x ca hnh trn xoay
to bi khi quay S quanh trc Ox.
Bi 7.35 : Cho hnh phng S gii hn bi th hm s y = sin x, trc honh, cc ng thng x = 0 v x = . Tnh th tch V x ca
hnh trn xoay to bi khi quay S quanh trc Ox.
Bi 7.36 : Cho hnh phng S gii hn bi th hm s y = x ln x, trc honh, cc ng thng x = 1 v x = e. Tnh th tch V x ca
hnh trn xoay to bi khi quay S quanh trc Ox.

TRN ANH TUN - 0974 396 391 - (04) 66 515 343

WWW.VNMATH.COM

Trang 162

www.luyenthi24h.com
www.luyenthi24h.com
CHUYN LUYN THI I HCwww.VNMATH.com

www.VNMATH.com

7.5 Tch phn trong cc k thi H


R1

Bi 7.37 (C09) : Tnh tch phn : I =

e2x + x e x dx.

R1 2x 1

Bi 7.38 (C10) : Tnh tch phn I =

dx.

x+1

Bi 7.39 (A02) : Tnh din tch hnh phng gii hn bi cc ng : y = |x2 4x + 3|, y = x + 3.

2R 3

Bi 7.40 (A03) : Tnh tch phn : I =

R2

Bi 7.41 (A04) : Tnh tch phn : I =

dx

.
x x2 + 4

dx.
1+ x1

R2 sin 2x + sin x

Bi 7.42 (A05) : Tnh tch phn : I =

dx.
1 + 3 cos x

R2

sin 2x

dx.
2
0
cos x + 4 sin2 x
Bi 7.44 (A07) : Tnh din tch hnh phng gii hn bi cc ng : y = (e + 1)x, y = (1 + e x )x.
Bi 7.43 (A06) : Tnh tch phn : I =

R6 tan4 x

Bi 7.45 (A08) : Tnh tch phn : I =

cos 2x

dx.

R2

Bi 7.46 (A09) : Tnh tch phn : I =

Bi 7.47 (A10) : Tnh tch phn I =

cos3 x 1 cos2 dx.

R2 x2 + e x + 2x2 e x

1 + 2e x

dx.
r

Bi 7.48 (B02) : Tnh din tch hnh phng gii hn bi cc ng : y =


Bi 7.49 (B04) : Tnh tch phn I =

1 + 3 ln x ln x
dx.
x

Re
1

x2
x2
v y = .
4
4 2

Bi 7.50 (B05) : Tnh tch phn I =

R2 sin 2x cos x

1 + cos x

Bi 7.51 (B06) : Tnh tch phn I =

ln
R5
ln 3

dx.

dx
.
e x + 2.ex 3

Bi 7.52 (B07) : Cho hnh phng H gii hn bi cc ng : y = x ln x, y = 0, x = e. Tnh th tch ca khi trn xoay to thnh khi
quay hnh H quanh trc Ox.

Bi 7.53 (B08) : Tnh tch phn : I =

R4

R3 3 + ln x

(x + 1)2

Bi 7.55 (B10) : Tnh tch phn I =

Re
1

Bi 7.56 (D03) : Tnh tch phn : I =

R2
R3
2

dx

dx.

|x2 x| dx.
ln(x2 x) dx.

Bi 7.58 (D05) : Tnh tch phn : I =

ln x
dx.
x(2 + ln x)2
0

Bi 7.57 (D04) : Tnh tch phn : I =

sin 2x + 2(1 + sin x + cos x)

Bi 7.54 (B09) : Tnh tch phn : I =

sin x

R2

esin x + cos x cos x dx.

R1

Bi 7.59 (D06) : Tnh tch phn : I = (x 2)e2x dx.


0

Bi 7.60 (D07) : Tnh tch phn : I =

Re

x3 ln2 x dx.

TRN ANH TUN - 0974 396 391 - (04) 66 515 343

WWW.VNMATH.COM

Trang 163

www.VNMATH.comCHUYN LUYN THI I HC


R2 ln x

Bi 7.61 (D08) : Tnh tch phn : I =

x3

R3

Bi 7.62 (D09) : Tnh tch phn : I =

Bi 7.63 (D10) : Tnh tch phn I =

Re

dx.

dx
.
ex 1

www.luyenthi24h.com
www.luyenthi24h.com
www.VNMATH.com

2x

3
x

dx.

7.6 Bi tp tng hp
Bi 7.64 : Tnh tch phn : I =

R2 x3 dx

x2 + 1

Bi 7.65 : Tnh tch phn : I =

ln
R3

e x dx
.
(e x + 1)3

Bi 7.66 : Tnh tch phn : I =

R0

x 22x +

x + 1 dx.

Bi 7.67 : Tnh tch phn : I =

R4

x
dx.
1 + cos 2x

Bi 7.68 : Tnh tch phn : I =

R1
0

Bi 7.69 : Tnh tch phn : I =

x3 1 x2 dx.

ln
R5
ln 2

Bi 7.70 : Tnh tch phn : I =

R1

e2x dx

.
ex 1

x3 e x dx.
2

Bi 7.71 : Tnh tch phn : I =

Re x2 + 1

ln x dx.

Bi 7.72 : Tnh tch phn : I =

R3

sin2 x tan x dx.

Bi 7.73 : Tnh tch phn : I =

R7 x + 2

Bi 7.74 : Tnh tch phn : I =

Re

x+1

dx.

x2 ln x dx.

R4

Bi 7.75 : Tnh tch phn : (tan x + esin x . cos x) dx.


0

Re

Bi 7.76 : Tnh tch phn : I =

ln2 x

dx.
x ln x + 1

R2

Bi 7.77 : Tnh tch phn : I = (2x 1) cos2 x dx.


0

Bi 7.78 : Tnh tch phn : I =

R6
2

dx

.
2x + 1 + 4x + 1

Bi 7.79 : Tnh din tch hnh phng gii hn bi parabol : y = x2 x + 3 v ng thng d : y = 2x 1.


Bi 7.80 : Tnh tch phn : I =

R e 3 2 ln x
2

Bi 7.81 : Tnh tch phn : I =

R10
5

R2

dx.
x 1 + 2 ln x

dx

.
x2 x1

Bi 7.82 : Tnh tch phn : I = (x + 1) sin 2x dx.


0

R2

Bi 7.83 : Tnh tch phn : I = (x 2) ln x dx.


1

TRN ANH TUN - 0974 396 391 - (04) 66 515 343

WWW.VNMATH.COM

Trang 164

www.luyenthi24h.com
www.luyenthi24h.com
CHUYN LUYN THI I HCwww.VNMATH.com

www.VNMATH.com
Bi 7.84 : Tnh tch phn : I =

R4
0

2x + 1

dx.
1 + 2x + 1

x(1 x)
.
x2 + 1

Bi 7.86 : Tnh din tch hnh phng gii hn bi cc ng y = x2 v y = 2 x2 .


Bi 7.85 : Tnh din tch hnh phng gii hn bi cc ng y = 0 v y =

Bi 7.87 : Tnh tch phn : I =

R1 x(x 1)

x2 4

Bi 7.88 : Tnh tch phn : I =

R2

dx.

x2 cos x dx.

3x
x2
v y =
.
4
x+1

Bi 7.90 : Tnh th tch ca khi trn xoay to thnh khi quay hnh phng gii hn bi cc ng y = 3x 2x + 1; y = 0; x = 1 xung
Bi 7.89 : Tnh din tch hnh phng gii hn bi cc ng y =
quanh trc Ox.
Bi 7.91 : Tnh th tch khi trn xoay nhn c do quay quanh trc Oy hnh phng c gii hn bi cc ng y2 = x v 3y x = 2.
Bi 7.92 : Tnh din tch hnh phng gii hn bi cc ng y = |x2 4x| v y = 2x.

Bi 7.93 : Cho D l hnh hnh gii hn bi (x 1)2 + (y 1)2 = 1. Tnh th tch vt th khi quay D quanh trc Ox.
Bi 7.94 : Tnh cc tch phn sau :
1.

Re
1
e

2.

R1
0

3.

(2x 1)2 e3x dx;

e+1
R
2

4.

x2 ln(x 1) dx;

11.

6.

7.

x2

13.

dx;

R1 x3 x2
0

x 3x 4

sin 2x
dx;
1 + cos4 x

R4 x sin2 x dx
15.
;
sin 2x cos2 x
6
16.

Re
1

x dx

;
2+x+ 2x

R4 x sin x
12.
dx;
3
0 cos x

x2 + 1

dx;
x 3x + 1

R3 ln(x2 + 3)

R2

R
0

cos3 x
dx;
9.
cos x sin x
2



cos
x

4
R
4 dx;
10.
0 4 3 cos x

x sin x cos x dx;

R5

14.

dx

;
x + 1 x2

R1
0

5.

R2 x + sin x
8.
dx;
0 1 + cos x

ln x
dx;
(1 + x)2

ln3 x
dx;
x(ln2 x + 1)

17.

R2 

3x(x 1) + e1+cos x sin 2x dx;

1 dx;

18.

R4

cos2

dx
.
x 1 + e3x

Bi 7.95 : Tnh cc tch phn sau :


1.

lnR 3
0

2.

dx

;
1 + 1 x2

R2

R1
0

3.

e2x dx

;
3e x + 1
0 1+

R2 x x 1
8.
dx;
x5
1

dx
;
2x
e +1

7.

cos 2x sin4 x + cos4 x dx;

9.

4.

5.

dx
;
4
x + 4x2 + 3

R2
0

10.

R2
0

x(2 x) + ln(4 + x2 ) dx;

R3 x + sin2 x
6.
dx;
0 1 + cos 2x

R1
1

R1

3Rln 2

11.

12.

R3
0

13.

x ln(x + 1) + x
x2 + 1

dx;
1+ x
sin x

dx;
cos x 3 + sin2 x

TRN ANH TUN - 0974 396 391 - (04) 66 515 343

sin3 x

14.

dx

;
1 + x + 1 + x2

R1 1 + x
0

R2 x cos x

dx;

ln
R5
ln 2

(10.ex

dx

;
1) e x 1

15.

R4 x sin x
0

cos3 x

dx;

16.

17.

R2 sin 2x 3 cos x
0

2 sin x + 1

R2

4 x2
dx;
x2

WWW.VNMATH.COM

dx;

Trang 165

www.luyenthi24h.com
www.luyenthi24h.com

Chng 8

S phc
Bi 8.1 :

1. Mi quan h z = z ng nu v ch nu z l s thc ;

2. Vi bt k s phc z quan h z = z l ng ;
3. Vi bt k s phc z, s phc z.z R l mt s thc khng m ;
4. z1 + z2 = z1 + z2 (lin hp ca mt tng bng tng cc lin hp) ;
5. z1 .z2 = z1 .z2 (lin hp ca mt tch bng tch cc lin hp) ;
6. Vi bt k s phc z , 0, c z1 = (z)1 ;
7.

z1
z2

z1
, z2 , 0 (lin hp ca mt thng bng thng cc lin hp) ;
z2

z+2
zz
v (z) =
.
2
2i
20
5 + 5i
+
;
1. Tnh z =
3 4i 4 + 3i

8. (z) =
Bi 8.2 :

2. Gi s z1 , z2 C. Chng minh rng s E = z1 .z2 + z1 .z2 l mt s thc.

Bi 8.3 : Chng minh cc khng nh sau :

1. |z| (z) |z| v |z| (z) |z| ;


2. |z| = | z| = |z| ;
3. z.z = |z|2 ;
4. |z1 .z2 | = |z1 |.|z2 | (mun ca mt tch bng tch cc mun) ;
5. |z1 | |z2 | |z1 + z2 | |z1 | + |z2 | ;
6. |z1 | = |z|1 , z , 0 ;


z1 |z1 |
7. =
, z2 , 0 (mun ca mt thng bng thng cc mun) ;
z2
|z2 |

8. |z1 | |z2 | |z1 z2 | |z1 | + |z2 |.


Bi 8.4 : Chng minh rng

|z1 + z2 |2 + |z1 z2 |2 = 2(|z1 |2 + |z2 |2 )


vi mi s phc z1 , z2 .
Bi 8.5 : Chng minh rng nu |z1 | = |z2 | = 1 v z1 .z2 , 1, th
Bi 8.6 : Gii s a l mt s thc dng v

Ma =

z1 + z2
l s thc.
1 + z1 z2


1

z C : z + = a .
z

Tm gi tr nh nht v gi tr ln nht ca |z| khi z Ma .

167

WWW.VNMATH.COM

www.VNMATH.com

CHUYN LUYN THI I HC

www.luyenthi24h.com
www.luyenthi24h.com
www.VNMATH.com

Bi 8.7 : Chng minh rng vi bt k s phc z, c


1
|z + 1| hoc |z2 + 1| 1.
2
Bi 8.8 : Chng minh rng :

7
|1 + z| + |1 z + z2 | 3
2

7
6

vi mi s phc m |z| = 1.
Bi 8.9 : Xt tp

H = {z C : z = x 1 + xi, x R}.
Chng minh rng c duy nht s z H sao cho |z| |w| vi mi w H.
Bi 8.10 : Gi s x, y, z l cc s phc phn bit sao cho

y = tx + (1 t)z, t (0; 1).


Chng minh rng

Bi 8.11 : Gii phng trnh trn tp s phc

|z| |y| |z| |x| |y| |x|

.
|z y|
|z x|
|y x|
z2 8(1 i)z + 63 16i = 0.

Bi 8.12 : Gi s p, q l cc s phc vi q , 0. Chng minh rng nu cc nghim ca phng trnh bc hai x2 + px + q2 = 0 c cng
p
mun, th l mt s thc.
q
Bi 8.13 : Gi s a, b, c l cc s phc khc khng vi |a| = |b| = |c|.
1. Chng minh rng nu cc nghim ca phng trnh az2 + bz + c = 0 c mun bng 1, th b2 = ac.
2. Nu mi phng trnh
az2 + bz + c = 0 v bz2 + cz + a = 0
c mt nghim c mun bng 1, th |a b| = |b c| = |c a|.
Bi 8.14 : Gii cc phng trnh sau trong C :
1. z2 + z + 1 = 0 ;

2. z3 + 1 = 0.

Bi 8.15 : Tm cc s thc x, y tha mn mi trng hp sau :


1. (1 2i)x + (1 + 2i)y = 1 + i ;
x3 y3
+
=i;
2.
3+i
3i

3. (4 3i)x2 + (3 + 2i)xy = 4y2

1 2
x + (3xy 2y2 )i.
2

Bi 8.16 : Tnh :
1. (2 i)(3 + 2i)(5 4i) ;

4.

2. (2 4i)(5 + 2i) + (3 + 4i)(6 i) ;


3.

1+i
1i

16

1i
1+i

5.

6
6
1 + i 3
1i 7
+
;
2
2

3 + 7i 5 8i
+
.
2 + 3i 2 3i

Bi 8.17 : Tnh :
1. i2000 + i1999 + i201 + i82 + i47 ;

3. i1 .i2 .i3 . . . i2000 ;

2. En = 1 + i + i2 + + in , vi n 1 ;

4. i5 + (i)7 + (i)13 + i100 + (i)94 .

TRN ANH TUN - 0974 396 391 - (04) 66 515 343

WWW.VNMATH.COM

Trang 168

www.VNMATH.com

CHUYN LUYN THI I HC

www.luyenthi24h.com
www.luyenthi24h.com
www.VNMATH.com

Bi 8.18 : Gii phng trnh trong C :


1. z2 = i ;

1
2
3. z = i
.
2
2

2. z2 = i ;

Bi 8.19 : Tm tt c cc s phc z , 0 sao cho z +

1
R.
z

Bi 8.20 : Chng minh rng :

1. E1 = (2 + i 5)7 + (2 i 5)7 R ;

2. E2 =

19 + 7i
9i

20 + 5i
7 + 6i

R.

Bi 8.21 : Chng minh cc ng thc sau :


1. |z1 + z2 |2 + |z2 + z3 |2 + |z3 + z1 |2 = |z1 |2 + |z2 |2 + |z3 |2 + |z1 + z2 + z3 |2 ;
2. |1 + z1 z2 |2 + |z1 z2 |2 = (1 + |z1 |2 )(1 + |z2 |)2 ;
3. |1 z1 z2 |2 |z1 z2 |2 = (1 |z1 |2 )(1 |z2 |)2 ;
4. |z1 + z2 + z3 |2 + | z1 + z2 + z3 |2 + |z1 z2 + z3 |2 + |z1 + z2 z3 |2 = 4(|z1 |2 + |z2 |2 + |z3 |2 ).







1
1
3


Bi 8.22 : Gi s z C sao cho z + 3 2. Chng minh rng z + 2.
z
z

Bi 8.23 : Tm tt c cc s phc z sao cho :


1. |z| = 1 v |z2 + z2 | = 1 ;

2. 4z2 + 8|z|2 = 8 ;

3. z3 = z.

Bi 8.24 : Xt s phc z C vi (z) > 1. Chng minh rng



1 1 1
< .
z 2 2

1
3
Bi 8.25 : Gi s a, b, c l cc s thc v = + i
. Tnh
2
2
(a + b + c2 )(a + b2 + c).
Bi 8.26 : Gii cc phng trnh :
1. |z| 2z = 3 4i ;

4. iz2 + (1 + 2i)z + 1 = 0 ;

2. |z| + z = 3 + 4i ;

5. z4 + 6(1 + i)z2 + 5 + 6i = 0 ;

3. z3 = 2 + 11i, y z = x + yi v x, y Z ;

6. (1 + i)z2 + 2 + 11i = 0.

Bi 8.27 : Tm tt c cc s thc m sao cho phng trnh


z3 + (3 + i)z2 3z (m + i) = 0
c t nht mt nghim thc.
Bi 8.28 : Tm tt c cc s phc z sao cho
z = (z 1)(z + i)
l mt s thc.


1
Bi 8.29 : Tm tt c cc s phc z sao cho |z| = .
z

Bi 8.30 : Gi s z1 , z2 C l cc s phc sao cho |z1 + z2 | =


Bi 8.31 : Tm tt c cc s nguyn dng n sao cho

3 v |z1 | = |z2 | = 1. Tnh |z1 z2 |.

n
n
1 + i 3
1 i 3
+
= 2.
2
2

TRN ANH TUN - 0974 396 391 - (04) 66 515 343

WWW.VNMATH.COM

Trang 169

www.VNMATH.com

CHUYN LUYN THI I HC

www.luyenthi24h.com
www.luyenthi24h.com
www.VNMATH.com

Bi 8.32 : Gi s n > 2 l mt s nguyn. Tm cc nghim ca phng trnh


zn1 = iz.
Bi 8.33 : Gi s z1 , z2 , z3 l cc s phc vi
|z1 | = |z2 | = |z3 | = R > 0.
Chng minh rng
|z1 z2 |.|z2 z3 | + |z3 z1 |.|z1 z2 | + |z2 z3 |.|z3 z1 | 9R2 .
v(u z)
Bi 8.34 : Gi s u, v, w, z l cc s phc sao cho |u| < 1, |v| = 1 v w =
. Chng minh rng |w| 1 nu v ch nu |z| 1.
u.z 1
Bi 8.35 : Gi s z1 , z2 , z3 l cc s phc sao cho
z1 + z2 + z3 = 0 v |z1 | = |z2 | = |z3 | = 1.
Chng minh rng
z21 + z22 + z23 = 0.
Bi 8.36 : Xt cc s phc z1 , z2 , . . . , zn vi
|z1 | = |z2 | = = |zn | = r > 0.
Chng minh rng s
(z1 + z2 )(z2 + z3 ) (zn1 + zn )(zn + z1 )
z1 .z2 zn

E=
l s thc.

Bi 8.37 : Gi s z1 , z2 , z3 l cc s phc khc nhau sao cho


|z1 | = |z2 | = |z3 > 0.|
Nu z1 + z2 z3 , z2 + z1 z3 v z3 + z1 z2 l cc s thc, chng minh rng z1 z2 z3 = 1.
Bi 8.38 : Gi s x1 v x2 l cc nghim ca phng trnh x2 x + 1 = 0. Tnh
1. x2000
+ x2000
;
1
2

2. x1999
+ x1999
;
1
2

3. xn1 + xn2 , vi n N.

Bi 8.39 : Phn tch thnh tch cc nh thc bc nht cc a thc sau :


1. x4 + 16 ;

2. x3 27 ;

3. x3 + 8 ;

4. x4 + x2 + 1.

Bi 8.40 : Tm tt c cc phng trnh bc hai vi h s thc c mt nghim l :


1. (2 + i)(3 i) ;

2.

5+i
;
2i

3. i51 + 2i80 + 3i45 + 4i38 .

Bi 8.41 (Bt ng thc Hlawka) : Chng minh bt ng thc sau


|z1 + z2 | + |z2 + z3 | + |z3 + z1 | |z1 | + |z2 | + |z3 | + |z1 + z2 + z3 |
ng vi mi s phc z1 , z2 , z3 .
Bi 8.42 : Biu din hnh hc ca cc s phc sau : z1 = 3 + i ; z2 = 4 + 2i ; z3 = 5 4i ; z4 = 5 i ; z5 = 1 ; z6 = 3i ; z7 = 2i ;

z8 = 4.

Bi 8.43 : Tm tp hp cc im trong mt phng phc biu din mi s phc z tha mn cc trng hp di y :


1. |z 2| = 3 ;

3. |z 1 + 2i| > 3 ;

2. |z + i| < 1 ;

4. |z 2| |z + 2| < 2 ;

TRN ANH TUN - 0974 396 391 - (04) 66 515 343

WWW.VNMATH.COM

Trang 170

www.VNMATH.com

CHUYN LUYN THI I HC

1+z
R;
z

9. | x2 + 4 + i y 4| = 10, vi z = x + yi ;

5. 0 < (iz) < 1 ;

8.

6. 1 < (z) < 1 ;

z2
7.
z1

www.luyenthi24h.com
www.luyenthi24h.com
www.VNMATH.com




1

10. z + = 2.
z

=0;

Bi 8.44 : Biu din lng gic ca cc s phc sau v xc nh argument ca chng :


1. z1 = 1 i ;

3. z3 = 1 + i 3 ;

2. z2 = 2 + 2i ;

4. z4 = 1 i 3.

Bi 8.45 : Biu din lng gic ca cc s phc sau v xc nh argument ca chng :


1. z1 = 2i ;

2. z2 = 1 ;

3. z3 = 2 ;

4. z4 = 3i.

Bi 8.46 : Tm biu din lng gic ca s phc


z = 1 + cos a + i sin a, a (0; 2).



z z

Bi 8.47 : Tm tt c cc s phc z sao cho |z| = 1 v + = 1.
z z

Bi 8.48 : Tnh (1 + i)1000 .


Bi 8.49 : Chng minh rng
sin 5t = 16 sin5 t 20 sin3 t + 5 sin t; cos 5t = 16 cos5 t 20 cos3 t + 5 cos t.

(1 i)10 ( 3 + i)5

Bi 8.50 : Tnh z =
.
(1 i 3)10
Bi 8.51 : Tnh :
1. (1 cos a + i sin a)n vi a [0; 2) v n N ;

2. zn +

1
1
, nu z + = 3.
n
z
z

Bi 8.52 : Gi s z1 , z2 , z3 l cc s phc sao cho


|z1 | = |z2 | = |z3 | = r > 0
v z1 + z2 + z3 , 0. Chng minh rng



z1 z2 + z2 z3 + z3 z1

= r.
z +z +z

1

Bi 8.53 : Gi s z1 , z2 l cc s phc sao cho


|z1 | = |z2 | = r > 0.
Chng minh rng

z1 + z2
r2 + z1 z2

Bi 8.54 : Gi s z1 , z2 , z3 l cc s phc sao cho

z1 z2
r2 z1 z2

1
.
r2

|z1 | = |z2 | = |z3 | = 1


v

z21
z2
z2
+ 2 + 3 + 1 = 0.
z2 z3 z3 z1 z1 z2

Chng minh rng


|z1 + z2 + z3 |{1; 2}.
Bi 8.55 : Gi s z1 , z2 l cc s phc sao cho |z1 | = |z2 | = 1. Chng minh rng
|z1 + 1| + |z2 + 1| + |z1 z2 + 1| 2.

TRN ANH TUN - 0974 396 391 - (04) 66 515 343

WWW.VNMATH.COM

Trang 171

www.VNMATH.com

CHUYN LUYN THI I HC

www.luyenthi24h.com
www.luyenthi24h.com
www.VNMATH.com

Bi 8.56 : Gi s n > 0 l mt s nguyn v z l s phc sao cho |z| = 1. Chng minh rng
n|1 + z| + |1 + z2 | + |1 + z3 | + + |1 + z2n | + |1 + z2n+1 | 2n.
Bi 8.57 : Dng cng thc khai trin Newton (1 + i)19 v cng thc Moa-vr tnh
0
2
4
16
18
C19
C19
+ C19
+ C19
C19
.

Bi 8.58 (C10) : Cho s phc z tha mn iu kin (2 3i)z + (4 + i)z = (1 + 3i)2 . Tm phn thc v phn o ca z.

Bi 8.59 (C10) : Gii phng trnh z2 (1 + i)z + 6 + 3i = 0 trn tp hp cc s phc.

Bi 8.60 (A09) : Gi z1 , z2 l hai nghim phc ca phng trnh z2 + 2z + 10 = 0. Tnh gi tr ca biu thc A = |z1 |2 + |z2 |2 .

Bi 8.61 (A10) : Tm phn o ca s phc z, bit z = ( 2 + i)2 (1 2i).

(1 3i)3
Bi 8.62 (A10) : Cho s phc z tha mn z =
. Tm mun ca s phc z + iz.
1i

Bi 8.63 (B09) : Tm s phc z tha mn |z (2 + i)| = 10 v z.z = 25.


Bi 8.64 (B10) : Trong mt phng ta Oxy, tm tp hp im biu din cc s phc z tha mn :
|z i| = |(1 + i)z| .
Bi 8.65 (D09) : Trong mt phng ta Oxy, tm tp hp im biu din cc s phc z tha mn iu kin |z (3 4i)| = 2.

Bi 8.66 (D10) : Tm s phc z tha mn : |z| = 2 v z2 l s thun o.

TRN ANH TUN - 0974 396 391 - (04) 66 515 343

WWW.VNMATH.COM

Trang 172

www.luyenthi24h.com
www.luyenthi24h.com

Chng 9

Phng php ta trong trong mt phng


9.1 Phng php ta trong mt phng


Bi 9.1 : Trong mt phng vi h ta Oxy, cho ba im A(1; 1), B(2; 5), C(4; 3). Tnh ta im D xc nh bi AD = 3AB2AC.
Bi 9.2 : Trong mt phng vi h ta Oxy, cho ba im A(2; 5), B(1; 1), C(3; 3). Tnh ta im D sao cho t gic ABCD l hnh
bnh hnh. Tm ta tm I ca hnh bnh hnh
Bi 9.3 : Trong mt phng vi h ta Oxy, cho tam gic ABC c trung im cc cnh AB, BC, CA ln lt l M(1; 4), N(3; 0), P(1; 1).
Tm ta cc nh ca tam gic ABC.
Bi 9.4 : Trong mt phng vi h ta Oxy, cho tam gic ABC c A(1; 1), B(5; 3); nh C trn trc Oy v trng tm G ca tam

gic nm trn trc Ox. Tm ta nh C.

Bi 9.5 : Trong mt phng vi h ta Oxy, cho A(1; 2). Tm trn trc honh im M sao cho ng trung trc ca on AM i

qua gc ta O.

Bi 9.6 : Trong mt phng vi h ta Oxy, cho tam gic ABC c : A(1; 2), B(2; 0), C(3; 1).
a) Xc nh tm ng trn ngoi tip tam gic ABC.
b) Tm im M trn ng thng BC sao cho din tch tam gic ABM bng

1
din tch tam gic ABC.
3

Bi 9.7 : Trong mt phng vi h ta Oxy, cho ba im A(3; 0), B(3; 0), C(2; 6).
a) Tm to trng tm G, trc tm H v tm ng trn ngoi tip I ca tam gic ABC.

b) Chng minh rng ba im I, H, G thng hng v IH = 3IG.


Bi 9.8 : Trong mt phng vi h ta Oxy, cho hai im A(3; 2), B(4; 3). Tm im M trn trc honh sao cho tam gic MAB
vung ti M.
Bi 9.9 : Trong mt phng vi h ta Oxy, cho tam gic ABC c A(1; 5), B(4; 5), C(4; 1).
a) Tm ta chn ng phn gic trong v chn ng phn gic ngoi ca gc A.
b) Tm ta tm ng trn ni tip tam gic ABC.

a (2t; t),
Bi 9.10 : Trong mt phng vi h ta Oxy, cho hai vect
b =


2 3 2
t;
t , vi t , 0. Chng minh rng gc gia hai
2
2

vect khng i khi t thay i.

Bi 9.11 : Trong mt phng vi h ta Oxy, cho tam gic ABC vi AB = (a1 ; a2 ) v AC = (b1 ; b2 ).
a) Chng minh rng din tch S ca tam gic ABC c tnh theo cng thc S =

1
|a1 b2 a2 b1 |.
2

b) p dng, tnh din tch tam gic ABC, bit A(2; 4), B(2; 8), C(10; 2).

175

WWW.VNMATH.COM

www.VNMATH.com

CHUYN LUYN THI I HC

www.luyenthi24h.com
www.luyenthi24h.com
www.VNMATH.com

9.2 Phng trnh ca ng thng


9.2.1 Cc bi ton thit lp phng trnh ng thng
Bi 9.12 : Cho tam gic ABC nh A(2; 2). Lp phng trnh cc cnh ca tam gic, bit rng 9x 3y 4 = 0 v x + y 2 = 0 ln lt
l phng trnh cc ng cao k t B v C ca tam gic.

Bi 9.13 : Vit phng trnh cc ng trung trc ca tam gic ABC, bit trung im ca cc cnh BC, CA, AB tng ng l
M(1; 1), N(1; 9), P(9; 1).

Bi 9.14 : Bit rng A(1; 3) l nh ca tam gic ABC v x 2y + 1 = 0, y = 0 l phng trnh ca hai ng trung tuyn ca tam gic
ny. Lp phng trnh cc cnh ca tam gic ABC.

Bi 9.15 : Trong mt phng ta cho P(2; 5) v Q(5; 1). Lp phng trnh ng thng qua P sao cho khong cch t Q ti ng
thng ny bng 3.
Bi 9.16 : Cho im A(8; 6). Lp phng trnh ng thng qua A v to vi hai trc ta mt tam gic c din tch bng 12.

9.2.2 Cc bi ton lin quan n vic s dng phng trnh ng thng


Bi 9.17 : Trong mt phng ta cho bn im A(1; 0), B(2; 4), C(1; 4), D(3; 5). Gi s l ng thng c phng trnh 3x

y 5 = 0. Tm im M trn sao cho hai tam gic MAB, MCD c din tch bng nhau.
3
Bi 9.18 : Cho tam gic ABC c din tch bng v hai im A, B c ta l A(2; 3) v B(3; 2). Trng tm G ca tam gic nm
2
trn ng thng 3x y 8 = 0. Tm ta nh C ca tam gic.

Bi 9.19 : Trong mt phng vi h ta Oxy, cho tam gic ABC c nh A(2; 1), ng cao qua B c phng trnh x 3y 7 = 0 v

ng trung tuyn qua nh C c phng trnh x + y + 1 = 0. Xc nh ta nh B v C ca tam gic ABC.

Bi 9.20 : Cho ng thng d : x 2y + 2 = 0 v hai im A(0; 6), B(2; 5). Tm im M trn d sao cho MA + MB nh nht.

Bi 9.21 : Vit phng trnh ng thng i qua M(4; 3) v to vi hai trc ta Ox, Oy thnh mt tam gic c din tch bng 3.
Bi 9.22 : Trong mt phng Oxy cho tam gic ABC. Bit cnh AC c phng trnh x + 3y 3 = 0, ng cao AH c phng trnh
x + y 1 = 0, nh C nm trn Ox, B nm trn Oy. Tm ta cc nh ca tam gic ABC.

Bi 9.23 : Trong mt phng Oxy cho hai ng thng d1 : x y + 2 = 0 v d2 : 2x + y 5 = 0 v im M(1; 4). Vit phng trnh
ng thng ct d1 , d2 ti A v B tng ng M l trung im ca AB.

Bi 9.24 : Trong mt phng Oxy cho A(1; 0), B(2; 3). Vit phng trnh ng thng d cch AB mt khong bng

10.

Bi 9.25 : Trong mt phng Oxy cho tam gic ABC c A(1; 2), ng trung tuyn BM, phn gic trong CD tng ng c phng trnh
2x + y + 1 = 0 v x + y 1 = 0. Vit phng trnh ng thng cha cnh BC.

Bi 9.26 : Mt hnh thoi c mt ng cho cho phng trnh x + 2y 7 = 0, mt cnh c phng trnh x + 3y 3 = 0, mt nh l
(0; 1). Tm phng trnh cc cnh hnh thoi.

Bi 9.27 : Cho tam gic ABC vi A(6; 3), B(4; 3), C(9; 2).
1. Vit phng trnh ba cnh ca tam gic.
2. Vit phng trnh ng phn gic trong ca gc A.
3. Tm im M trn AB, N thuc AC sao cho MN song song BC v AM = CN.
Bi 9.28 : Trong mt phng ta cho d : 2x + 3y + 1 = 0 v im M(1; 1). Vit phng trnh ca cc ng thng qua M v to vi
d gc 45 .
Bi 9.29 : Trong mt phng ta cho tam gic ABC cn, vi A(1; 1), C(3; 5), nh B nm trn ng thng d : 2x y = 0. Vit

phng trnh cnh AB, BC.

Bi 9.30 : Trong mt phng ta cho tam gic ABC c nh A nm trn ng thng d : x 4y 2 = 0. Cnh BC song song vi d,
phng trnh ng cao BH : x + y + 3 = 0 v trung im ca AB l M(1; 1). Tm ta cc nh.

TRN ANH TUN - 0974 396 391 - (04) 66 515 343

WWW.VNMATH.COM

Trang 176

www.VNMATH.com

CHUYN LUYN THI I HC

Bi 9.31 : Trong mt phng ta cho tam gic ABC cn nh A, c trng tm G


0, phng trnh ng thng BG l 7x 4y 8 = 0. Tm ta cc nh A, B, C.

www.luyenthi24h.com
www.luyenthi24h.com
www.VNMATH.com

4 1
;
. Phng trnh ng thng BC l x2y4 =
3 3

Bi 9.32 : Trong mt phng ta cho tam gic ABC c nh A(1; 0), hai ng cao xut pht t B v C c phng trnh x 2y + 1 = 0
v 3x + y 1 = 0. Tm din tch tam gic ABC.

Bi 9.33 : Trn mt phng ta cho hai ng thng d1 : 2x y + 5 = 0, d2 : 3x + 6y 1 = 0 v im P(2; 1). Lp phng trnh
ng thng d qua P sao cho d cng vi d1 , d2 to thnh mt tam gic cn nh A, vi A l giao im d1 v d2 .

Bi 9.34 : Tm trn trc honh cho im P sao cho tng khong cch t P ti cc im A(1; 2), B(3; 4) l nh nht.
Bi 9.35 : Tam gic ABC c cc cnh AB, AC, BC tng ng c phng trnh x y 2 = 0, 3x y + 5 = 0, x 4y 1 = 0. Vit phng
trnh cc ng cao ca tam gic.

Bi 9.36 : Vit phng trnh ng thng d i qua giao im ca hai ng thng d1 : 2x y + 1 = 0; d2 : x 2y 3 = 0 ng thi
chn trn hai trc ta nhng on bng nhau.

Bi 9.37 : Cho h ng thng ph thuc tham s l d : (x 1) cos + (y 1) sin 4 = 0. Chng minh rng vi mi , h ng
thng ni trn lun tip xc vi mt ng trn c nh.

9.2.3 Bi tp tng hp
Bi 9.38 : Vit phng trnh ng thng trong mi trng hp sau :
a) i qua hai im A(2; 1) v B(1; 3).
b) ct trc Ox ti im A(4; 0) v ct trc Oy ti im B(0; 3).
Bi 9.39 : Vit phng trnh ng thng trong mi trng hp sau :
3
.
4
b) i qua im M(8; 2) v song song vi ng thng d : 2x 3y + 5 = 0.
a) i qua im M(3; 5) v c h s gc k =

c) i qua im M(3; 2) v vung gc vi ng thng d : 3x + 4y + 7 = 0.


Bi 9.40 : Vit phng trnh ng thng trong mi trng hp sau :
1
v hp vi hai trc ta mt tam gic c din tch bng 1.
2
b) i qua im M(8; 6) v to vi hai trc ta mt tam gic c din tch bng 2.
a) c h s gc k =

Bi 9.41 : Trong mt phng vi h ta Oxy, bit ba trung im cc cnh ca mt tam gic l M(2; 1), N(5; 3), P(3; 4). Hy lp

phng trnh cc cnh ca tam gic .

Bi 9.42 : Trong mt phng vi h ta Oxy, vit phng trnh ng thng i qua im M(3; 1) v ct trc Ox, Oy ln lt ti B
v C sao cho tam gic ABC cn ti A vi A(2; 2).

Bi 9.43 : Trong mt phng vi h ta Oxy, cho tam gic ABC c nh A(1; 3).
a) Cho bit ng cao BH : 5x + 3y 25 = 0, CK : 3x + 8y 12 = 0. Vit phng trnh cnh BC.
b) Xc nh ta cc nh B v C nu bit ng trung trc ca AB l 3x + 2y 4 = 0 v ta trng tm G(4; 2) ca tam gic
ABC.

Bi 9.44 : Trong mt phng vi h ta Oxy, cho hai ng thng d1 : x + y + 5 = 0, d2 : x + 2y 7 = 0 v im A(2; 3). Tm im


B thuc d1 v im C thuc d2 sao cho tam gic ABC c trng tm G(2; 0).

Bi 9.45 : Trong mt phng vi h ta Oxy, cho hai ng thng 1 : x y + 1 = 0, 2 : 2x + y + 1 = 0 v im M(2; 1). Vit

phng trnh ng thng d i qua im M v ct hai ng thng 1 , 2 ln lt ti A v B sao cho M l trung im ca on thng
AB.
Bi 9.46 : Trong mt phng vi h ta Oxy, cho hai ng thng d1 : 2x y + 5 = 0, d2 : x + y 3 = 0 v im M(2; 0). Vit

phng trnh ng thng d i qua im M v ct hai ng thng d1 , d2 ln lt ti A v B sao cho MA = 2 MB.

TRN ANH TUN - 0974 396 391 - (04) 66 515 343

WWW.VNMATH.COM

Trang 177

CHUYN LUYN THI I HC

www.VNMATH.com

www.luyenthi24h.com
www.luyenthi24h.com
www.VNMATH.com

Bi 9.47 : Trong mt phng vi h ta Oxy, cho tam gic ABC c nh A(2; 7), phng trnh mt ng cao v mt trung tuyn

v t hai nh khc nhau ln lt l : 3x + y + 11 = 0 v x + 2y + 7 = 0. Vit phng trnh cc cnh ca tam gic ABC.

Bi 9.48 : Trong mt phng vi h ta Oxy, cho tam gic ABC c nh A(1; 2), ng trung tuyn BM v ng phn gic trong
CD c phng trnh ln lt l : 2x + y + 1 = 0 v x + y 1 = 0. Hy vit phng trnh ng thng BC.

Bi 9.49 : Vit phng trnh cc cnh ca tam gic ABC, bit A(1; 3) v hai trung tuyn c cc phng trnh l : x 2y + 1 = 0 v
y 1 = 0.

Bi 9.50 : Phng trnh hai cnh ca tam gic ABC l : 5x 2y + 6 = 0, 4x + 7y 21 = 0. Vit phng trnh cnh th ba ca tam gic
ABC, bit trc tm ca tam gic trng vi gc ta .

Bi 9.51 : Cho A(2; 1) v hai phn gic trong ca gc B, C ca tam gic ABC ln lt c phng trnh : x2y+1 = 0 v x+y+3 = 0.
Vit phng trnh cnh BC.

Bi 9.52 : Trong mt phng vi h ta Oxy, vit phng trnh ng thng qua M(4; 1) v ct cc tia Ox, Oy ln lt ti A v B sao
cho OA + OB t gi tr nh nht.
Bi 9.53 : Trong mt phng vi h ta Oxy, vit phng trnh ng thng i qua im A(27; 1) v ct cc tia Ox, Oy ln lt ti
M v N sao cho MN c di nh nht.
Bi 9.54 : Bin lun theo m v tr tng i ca hai ng thng 1 : 4x my + 4 m = 0 v 2 : (2m + 6)x + y 2m 1 = 0.
Bi 9.55 : Cho hai ng thng d1 : (m + 1)x + 6y + m = 0 v d2 : x + (m + 2)y + 1 = 0. Tm m hai ng thng d1 v d2
a) ct nhau.

b) song song vi nhau.

c) trng nhau.

Bi 9.56 : Cho hai ng thng d1 : (a + 1)x 2y a 1 = 0 v d2 : x + (a 1)y a2 = 0.


a) Tm giao im I ca d1 v d2 .
b) Tm a ng thng qua M(0; a), N(a; 0), vi (a , 0) i qua giao im I.
Bi 9.57 : Cho tam gic ABC c phng trnh cc cnh l
AB : 2x + 3y 5 = 0; BC : 3x 4y + 1 = 0; CA : x 2y + 1 = 0.
Vit phng trnh ng cao ca tam gic ABC xut pht t nh A.
Bi 9.58 : Trong mt phng vi h ta Oxy, cho hai ng thng d1 : mx + (m 1)y + m 3 = 0 v d2

8
< x = (m 1)t

:y = m 1 2t.

a) Tm m hai ng thng d1 v d2 trng nhau.


b) Tm m d1 , d2 v : 2x + y 1 = 0 ng quy.
Bi 9.59 : Tnh gc gia hai ng thng d1 : 2x y + 3 = 0 v d2 : x 3y + 9 = 0.
Bi 9.60 : Trong mt phng vi h ta Oxy, cho ng thng d1 :
bi d1 v d2 bng 45 .

8
< x = 2 + at
:y = 1 2t

v d2 : 3x + 4y + 12 = 0. Xc nh a gc hp

Bi 9.61 : Trong mt phng vi h ta Oxy, lp phng trnh ng thng i qua im M(2; 1) v to vi ng thng d :
2x + 3y + 4 = 0 mt gc 45 .
Bi 9.62 : Trong mt phng vi h ta Oxy, cho mt tam gic cn c mt cnh y v mt cnh bn l c phng trnh ln lt l :
3x y + 5 = 0 ; x + 2y 1 = 0. Lp phng trnh cnh bn cn li bit rng n i qua im M(1; 3).

Bi 9.63 : Trong mt phng vi h ta Oxy, cho hai ng thng d1 : 2x y + 1 = 0 ; d2 : x + 2y 7 = 0.

Lp phng trnh ng thng i qua gc ta O v to vi d1 , d2 mt tam gic cn c nh l giao im A ca d1 v d2 .

Bi 9.64 : Cho hnh ch nht ABCD, bit AB : 2x y + 5 = 0, ng thng AD i qua gc ta O v tm hnh ch nht l I(4; 5).
Vit phng trnh cc cnh cn li.

TRN ANH TUN - 0974 396 391 - (04) 66 515 343

WWW.VNMATH.COM

Trang 178

www.VNMATH.com

CHUYN LUYN THI I HC

www.luyenthi24h.com
www.luyenthi24h.com
www.VNMATH.com

Bi 9.65 : Trong mt phng vi h ta Oxy, cho hnh vung ABCD c nh A(4; 5) v mt ng cho nm trn ng thng
7x y + 8 = 0. Lp phng trnh cc cnh ca hnh vung.

Bi 9.66 : Trong mt phng vi h ta Oxy, cho cc im A(0; 1), B(2; 1) v cc ng thng :


d1 : (m 1)x + (m 2)y + 2 m = 0 v d2 : (2 m)x + (m 1)y + 3m 5 = 0.

Chng minh d1 v d2 lun ct nhau. Gi P l giao im ca d1 v d2 , tm m PA + PB t gi tr ln nht.


Bi 9.67 : Trong mt phng vi h ta Oxy, cho hai im A(1; 1), B(4; 3). Tm im M thuc ng thng d : x 2y 1 = 0 sao

cho khong cch t M n ng thng AB bng 6.

Bi 9.68 : Trong mt phng vi h ta Oxy, cho ng thng d : 2x y + 3 = 0. Vit phng trnh ng thng song song vi d

v cch d mt khong bng 5.


Bi 9.69 : Trong mt phng vi h ta Oxy, vit phng trnh ng thng i qua im M(4; 5) v cch im A(3; 2) mt khong
bng 1.
Bi 9.70 : Trong mt phng vi h ta Oxy,vit phng trnh ng thng cch im A(2; 5) mt khong bng 2 v cch im
B(5; 4) mt khong bng 3.
Bi 9.71 : Trong mt phng vi h ta Oxy, cho hnh bnh hnh ABCD c din tch bng 4. Bit nh A(1; 0), B(0; 2) v giao im
I ca hai ng cho nm trn ng thng y = x. Tm ta cc nh C v D.
Bi 9.72 : Cho A(1; 1), hy tm im B trn ng thng y = 3 v im C trn trc honh sao cho tam gic ABC u.
Bi 9.73 : Trong mt phng vi h ta Oxy, cho ng thng m : (m 2)x + (m 1)y + 2m 1 = 0.
a) Chng minh rng m lun i qua mt im c nh M khi m thay i.
b) Tm m m ct on thng AB, vi A(2; 3), B(1; 0).
c) Tm m khong ch t A n m l ln nht.
Bi 9.74 : Trong mt phng vi h ta Oxy, vit phng trnh cc ng phn gic ca cc gc to bi hai ng thng 1 :
3x 4y + 1 = 0, 2 : 8x + 6y 5 = 0.

Bi 9.75 : Trong mt phng vi h ta Oxy, vit phng trnh phn gic ca gc nhn to bi hai ng thng d1 : 7x + y 6 = 0
v d2 : x y + 2 = 0.

Bi 9.76 : Trong mt phng vi h ta Oxy, cho tam gic ABC bit A(6; 4), B(3; 1), C(4; 2). Vit phng trnh ng phn gic

trong ca gc A.

Bi 9.77 : Trong mt phng vi h ta Oxy, cho tam gic ABC c A(2; 0), B(4; 1), C(1; 2). Vit phng trnh ng phn gic trong
ca gc A trong tam gic ABC.
Bi 9.78 : Trong mt phng vi h ta Oxy, cho ng thng d : 2x + y 2 = 0 v im M(6; 5).
a) Xc nh ta hnh chiu vung gc ca M trn ng thng d.
b) Xc nh ta im M i xng vi im M qua ng thng d.
Bi 9.79 : Trong mt phng vi h ta Oxy, cho ng thng d : x 2y + 1 = 0 v im A(0; 3). V AH vung gc vi d ti H v
ko di AH v pha H mt on HB = 2AH. Tm ta im B.

Bi 9.80 : Trong mt phng vi h ta Oxy, cho ng thng d : x 2y + 2 = 0 v hai im A(0; 6), B(2; 5). Trn ng thng d
tm ta im M sao cho :

a) MA + MB t gi tr nh nht.
b) |MA MB| t gi tr ln nht.
Bi 9.81 : Trong mt phng vi h ta Oxy, cho ng thng d : 3x 2y + 8 = 0 v im M(1; 5). Vit phng trnh ng thng
i xng vi ng thng d qua im M.

TRN ANH TUN - 0974 396 391 - (04) 66 515 343

WWW.VNMATH.COM

Trang 179

www.VNMATH.com

CHUYN LUYN THI I HC

www.luyenthi24h.com
www.luyenthi24h.com
www.VNMATH.com

Bi 9.82 : Trong mt phng vi h ta Oxy, cho hai ng thng song song


1 : 3x 2y + 1 = 0 v 2 : 6x 4y 3 = 0.
Vit phng trnh ng thng 3 i xng vi 1 qua 2 .
Bi 9.83 : Trong mt phng vi h ta Oxy, cho hai ng thng : 2x y + 5 = 0 v d : x + 3y 8 = 0. Vit phng trnh ng
thng i xng vi qua d.

Bi 9.84 : Trong mt phng vi h ta Oxy, cho ng thng : 2x + 3y 6 = 0.


a) Vit phng trnh ng thng 1 i xng vi qua trc Ox.
b) Vit phng trnh ng thng 2 i xng vi qua trc Oy.

9.3 ng trn
Bi 9.85 : Xc nh tm v tnh bn knh ng trn (C) trong cc trng hp sau :
a) (C) : x2 + y2 2x 2y 2 = 0.
b) (C) : 16x2 + 16y2 + 16x 8y 11 = 0.
Bi 9.86 : Cho h ng trn (Cm ) c phng trnh :
x2 + y2 + 4mx 2my + 2m + 3 = 0.
a) Xc nh m (Cm ) l ng trn.
b) Tm tp hp tm I ca h ng trn.
Bi 9.87 : Cho h ng trn (Cm ) c phng trnh :
x2 + y2 2mx + 2(m + 1)y 12 = 0.
a) Tm qu tch tm ca h ng trn (Cm ).
b) Tm m sao cho bn knh ng trn (Cm ) nh nht.
c) Khi m, cho ng thng d : 3x 4y + 12 = 0. Tm im M trn (C2 ) sao cho khong cch t M n d l ngn nht.
Bi 9.88 : Cho h ng trn (Cm ) c phng trnh :
x2 + y2 2mx + 2(m + 2)y + 2m2 + 4m

1
= 0.
2

a) Chng minh rng (Cm ) lun l mt ng trn c bn knh khng i.


b) Tm tp hp tm cc ng trn (Cm ), t suy ra (Cm ) lun tip xc vi hai ng thng.
Bi 9.89 : Vit phng trnh ng trn (C) c tm I(4; 2) v tip xc vi ng thng : 3x + 4y 16 = 0.
Bi 9.90 : Vit phng trnh ng trn (C) c ng knh AB, vi A(1; 2), B(3; 4).

Bi 9.91 : Vit phng trnh ng trn (C) i qua ba im A(3; 3), B(1; 1), C(5; 1).
Bi 9.92 : Vit phng trnh ng trn (C) c tm I(3; 1) v chn trn ng thng : x 2y + 4 = 0 mt dy cung c di bng 4.
Bi 9.93 : Vit phng trnh ng trn (C) i qua hai im A(2; 3), B(1; 1) v c tm nm trn ng thng : x 3y 11 = 0.

Bi 9.94 : Trong mt phng vi h ta Oxy, cho hai im A(0; 5), B(2; 3). Vit phng trnh ng trn (C) i qua hai im A, B

v c bn knh R = 10.

Bi 9.95 : Trong mt phng vi h ta Oxy, vit phng trnh ng trn (C) c tm nm trn ng thng : x + y 5 = 0, c

bn knh R = 10 v tip xc vi ng thng d; 3x + y 3 = 0.

TRN ANH TUN - 0974 396 391 - (04) 66 515 343

WWW.VNMATH.COM

Trang 180

CHUYN LUYN THI I HC

www.VNMATH.com

www.luyenthi24h.com
www.luyenthi24h.com
www.VNMATH.com

Bi 9.96 : Trong mt phng vi h ta Oxy, vit phng trnh ng trn (C) tip xc vi ng thng : 3x 4y 31 = 0 ti

im A(1; 7) v c bn knh R = 5.

Bi 9.97 : Trong mt phng vi h ta Oxy, vit phng trnh ng trn (C) c tm thuc ng thng : 2x + y = 0 v tip xc
vi ng thng d : x 7y + 10 = 0 ti im A(4; 2).

Bi 9.98 : Vit phng trnh ng trn (C) i qua im A(6; 4) v tip xc vi ng thng : x + 2y 5 = 0 ti im B(3; 1).

Bi 9.99 : Trong mt phng vi h ta Oxy, vit phng trnh ng trn (C) c tm nm trn ng thng : 4x + 3y 2 = 0 v

tip xc vi hai ng thng d1 : x + y + 4 = 0 v d2 : 7x y + 4 = 0.

Bi 9.100 : Trong mt phng vi h ta Oxy, vit phng trnh ng trn (C) tip xc vi trc honh ti im A(2; 0) v khong

cch t tm ca (C) n im B(6; 4) bng 5.


Bi 9.101 : Trong mt phng vi h ta Oxy, cho ng thng d : x y + 1

2 = 0 v im A(1; 1). Vit phng trnh ng

trn (C) i qua im A, qua gc to O v tip xc vi ng thng d.

Bi 9.102 : Trong mt phng vi h ta Oxy, vit phng trnh ng trn i qua im A(2; 1) v tip xc vi hai trc ta Ox

v Oy.

Bi 9.103 : Trong mt phng vi h ta Oxy, cho ng trn (C) : (x 1)2 + (y 2)2 = 4 v ng thng d : x y 1 = 0. Vit

phng trnh ng trn (C ) i xng vi ng trn (C) qua ng thng d. Tm ta cc giao im (C) v (C ).

Bi 9.104 : Trong mt phng vi h ta Oxy, cho ng thng d : x 7y + 10 = 0 v ng trn (C ) : x2 + y2 2x + 4y 20 = 0.

Vit phng trnh ng trn (C) i qua im A(1; 2) v cc giao im ca ng thng d v (C ).

Bi 9.105 : Trong mt phng vi h ta Oxy, cho ng trn (C ) : x2 + y2 = 100. Vit phng trnh ng trn (C) tip xc vi
ng trn (C ) ti im M(6; 8) v c bn knh R = 6.
Bi 9.106 : Trong mt phng vi h ta Oxy, cho ng trn (C) : x2 + y2 12x 4y + 36 = 0. Vit phng trnh ng trn (C1 )
tip xc vi hai trc ta Ox, Oy ng thi tip xc ngoi vi ng trn (C).

Bi 9.107 : Trong mt phng vi h ta Oxy, cho ba im A(1; 7), B(4; 3), C(4; 1). Hy vit phng trnh ng trn (C) ni

tip tam gic ABC.

Bi 9.108 : Trong mt phng vi h ta Oxy, cho im A(2; 1) v ng trn (C) : (x 1)2 + (y 2)2 = 9. Vit phng trnh ng
thng i qua im A v ct ng trn (C) ti hai im phn bit E, F sao cho A l trung im EF.

Bi 9.109 : Lp phng trnh ng thng i qua gc ta O v ct ng trn (C) : (x 1)2 + (y + 3)2 = 25 theo mt dy cung

c di bng 8.

Bi 9.110 : Cho ng trn (C) : x2 + y2 + 2x 4y 20 = 0 v im A(3; 0). Vit phng trnh ng thng i qua im A v ct

ng trn (C) theo mt dy cung MN c di :


a) ln nht ;

b) nh nht.

Bi 9.111 : Cho ng trn (C) : x2 +y2 2x+4y+4 = 0. Vit phng trnh ng thng song song vi ng thng d : 3x+4y7 = 0

v chia ng trn (C) thnh hai cung m t l di bng 2.

Bi 9.112 : Cho ng trn (C) : x2 + y2 2x + 4y 4 = 0 c tm I v im M(1; 3). Vit phng trnh ng thng d i qua im
M v ct (C) ti hai im phn bit A v B sao cho tam gic IAB c din tch ln nht.

Bi 9.113 : Cho ng thng d : x y + 3 = 0 v ng trn (C) : x2 + y2 2x 2y + 1 = 0. Tm ta im M nm trn d sao cho


ng trn tm M c bn knh gp i ng trn (C) v tip xc ngoi vi ng trn (C).

Bi 9.114 : Cho cc ng trn


(C1 ) : x2 + y2 x 6y + 8 = 0 v (C2 ) : x2 + y2 2mx 1 = 0.
Tm m (C1 ) v (C2 ) tip xc vi nhau.
Bi 9.115 : Cho ng trn (C) : x2 + y2 = 1, ng trn (C ) c tm I(2; 2) ct (C) ti cc im A, B sao cho di on AB =

2.

Vit phng trnh ng thng AB.

TRN ANH TUN - 0974 396 391 - (04) 66 515 343

WWW.VNMATH.COM

Trang 181

CHUYN LUYN THI I HC

www.VNMATH.com

www.luyenthi24h.com
www.luyenthi24h.com
www.VNMATH.com

Bi 9.116 : Vit phng trnh tip tuyn ca ng trn (C) : (x + 2)2 + (y + 2)2 = 25 ti im A(2; 1).
Bi 9.117 : Vit phng trnh tip tuyn vi ng trn (C) : x2 + y2 6x 4y + 11 = 0 ti im M(4; 3).
Bi 9.118 : Vit phng trnh tip tuyn vi ng trn (C) : x2 + y2 x 7y = 0 ti cc giao im ca (C) v ng thng
d : 3x + 4y 3 = 0.

Bi 9.119 : Vit phng trnh tip tuyn vi ng trn (C) : x2 + y2 4x + 6y + 3 = 0, bit tip tuyn c h s gc bng 3.
Bi 9.120 : Vit phng trnh tip tuyn vi ng trn (C) : x2 + y2 2x + 8y + 1 = 0, bit rng song song vi ng thng

d : 5x + 12y 6 = 0. Tm ta cc tip im.

Bi 9.121 : Cho A(3; 4) v ng trn (C) : x2 + y2 4x 2y = 0.


a) Vit phng trnh tip tuyn ca (C), bit rng i qua im A.
b) Gii s cc tip tuyn tip xc vi (C) ti M v N. Hy tnh di on MN.
Bi 9.122 : Cho M(3; 1) v ng trn (C) : x2 + y2 2x 6y + 6 = 0. Gi T 1 , T 2 l cc tip im ca cc tip tuyn k t M n
(C). Vit phng trnh ng thng T 1 T 2 .

Bi 9.123 : Cho ng thng d : x y + 1 = 0 v ng trn (C) : x2 + y2 + 2x 4y = 0. Tm ta im M thuc ng thng tip

= 60 .
xc vi ng trn (C) ti A v B sao cho gc AMB

Bi 9.124 : Xt ng thng d : 2x + my + 1 2 = 0 v hai ng trn

(C1 ) : x2 + y2 2x + 4y 4 = 0 v (C2 ) : x2 + y2 + 4x 4y 56 = 0.
a) Gi I l tm ng trn (C1 ). Tm m sao cho d ct (C1 ) ti hai im phn bit A v B. Vi gi tr no ca m th din tch tam gic
IAB ln nht v tnh gi tr ln nht .
b) Chng minh (C1 ) tip xc vi (C2 ). Vit phng trnh tng qut ca tt c cc tip tuyn chung ca (C1 ) v (C2 ).
Bi 9.125 : Cho hai ng trn
(C1 ) : x2 + y2 4x + 2y 4 = 0 v (C2 ) : x2 + y2 10x 6y + 30 = 0
c tm ln lt l I v J.
a) Chng minh (C1 ) tip xc ngoi vi (C2 ) v tm ta tip im H.
b) Gi d l mt tip tuyn chung khng i qua H ca (C1 ) v (C2 ). Tm ta giao im K ca d v ng thng I, J. Vit phng
trnh ng trn (C) i qua K v tip xc vi hai ng trn (C1 ) v (C2 ) ti H.
Bi 9.126 : Vit phng trnh tip tuyn chung ca hai ng trn
(C1 ) : x2 + y2 = 1 v (C2 ) : x2 + y2 6x + 6y + 17 = 0.
Bi 9.127 : Cho hai ng trn
(C1 ) : x2 + y2 2x 2y 2 = 0 v (C2 ) : x2 + y2 8x 2y + 16 = 0.
a) Chng minh rng (C1 ) v (C2 ) tip xc nhau.
b) Vit phng trnh cc tip tuyn chung ca (C1 ) v (C2 ).
Bi 9.128 : Vit phng trnh cc tip tuyn chung ca hai ng trn
(C1 ) : x2 + y2 6x + 5 = 0 v (C2 ) : x2 + y2 12x 6y + 44 = 0.

TRN ANH TUN - 0974 396 391 - (04) 66 515 343

WWW.VNMATH.COM

Trang 182

www.VNMATH.com

CHUYN LUYN THI I HC

www.luyenthi24h.com
www.luyenthi24h.com
www.VNMATH.com

9.4 ng elip
x2
y2
+
= 1. Xc nh ta cc tiu im, ta cc nh, di cc trc.
25 16
x2 y2
Bi 9.130 : Cho elip (E) : 2 + 2 = 1, vi a > b > 0. Xc nh tm sai ca elip trong mi trng hp sau :
a
b

Bi 9.129 : Cho elip (E) :

a) (E) c di trc ln bng 3 ln trc nh.


3
ln tiu c ca n.
2
c) Mi nh trn trc nh ca elip nhn hai tiu im di mt gc 120 .

b) Khong cch gia hai nh lin tip nhau ca elip bng

Bi 9.131 : Lp phng trnh chnh tc ca elip, bit :


a) cc tiu im F1 (4; 0), F2(4; 0) v di trc ln bng 10.

b) elip i qua cc im M(2 3; 1) v N( 3; 2).

5
c) elip i qua im M
; 15 v c hai tiu im F1 (3; 0) v F2 (3; 0).
4

d) di trc ln bng 4 2, cc nh trn trc nh v cc tiu im ca elip nm trn mt ng trn.

e) elip i qua im M( 5; 2) v khong cch gia hai ng chun l 10.

f) elip i qua im M(2; 2) v phng trnh cc ng chun x = 4.


g) elip i qua im M(8; 12) v MF1 = 20 vi F1 l tiu im bn tri ca elip.


3 5 4 5
;
h) elip i qua im M
v
F1 MF2 = 90 , vi F1 , F2 l cc tiu im ca elip.
5
5
x2 y2
+
= 1.
9
4
1. Tm to cc tiu im, cc nh ; tnh tm sai, tnh din tch hnh ch nht c s.

Bi 9.132 : Cho elip (E) c phng trnh

2. Xc nh m ng thng d : y = x + m v (E) c im chung.


3. Vit phng trnh ng thng i qua M(1; 1) v ct (E) ti hai im A, B sao cho M l trung im ca on AB.
Bi 9.133 : Cho elip (E) : 9x2 + 25y2 = 225. ng thng d vung gc vi trc ln ti tiu im bn phi F2 , ct (E) ti hai im M
v N.
1. Tm ta ca M v N.
2. Tnh di cc on thng MF1 , MF2 v MN.
Bi 9.134 : Cho elip (E) :

x2
+ y2 = 1 c cc tiu im F1 , F2 . Tm ta im M trn elip tha mn :
9

1. MF1 = 3MF2 .
2. im M nhn hai tiu im di mt gc vung.
3. im M nhn hai tiu im di mt 120.
x2 y2
+
= 1 vi tiu im F(c; 0). Tm im M trn elip (E) sao cho di F M l nh nht.
a2 b2
x2 y2
Bi 9.136 : Cho im C(2; 0) v elip (E) :
+
= 1. Tm ta cc im A, B thuc (E), bit rng hai im A, B i xng vi
4
1
nhau qua trc honh v tam gic ABC l tam gic u.

x2 y2
Bi 9.137 : Cho elip (E) :
+
= 1 v ng thng d : x 2y + 2 = 0. ng thng d ct elip (E) ti hai im B v C. Tm ta
8
4
im A trn elip sao cho tam gic ABC c din tch ln nht.
Bi 9.135 : Cho elip (E) :

x2 y2
+
= 1. Xt im M chuyn ng trn tia Ox v im N chuyn ng trn tia Oy sao cho ng thng
16 9
MN lun lun tip xc vi elip (E). Xc nh ta ca M, N on MN c di nh nht. Tnh gi tr nh nht .

Bi 9.138 : Cho elip (E) :

TRN ANH TUN - 0974 396 391 - (04) 66 515 343

WWW.VNMATH.COM

Trang 183

www.VNMATH.com
Bi 9.139 : Cho (E) :

CHUYN LUYN THI I HC

www.luyenthi24h.com
www.luyenthi24h.com
www.VNMATH.com

x2 y2
+
(a > b > 0) vi cc tiu im F1 , F2 .
a2 b2

1. Chng minh rng vi mi im M trn elip (E) ta lun c :


(a) OM 2 + MF1 .MF2 = a2 + b2 .
(b) OM a.
2. Gi A v B l hai im thuc elip (E) sao cho OAOB. Chng minh rng :

1
1
1
1
+
= 2 + 2.
2
2
OA
OB
a
b

Bi 9.140 : Cho hai ng trn C1 (F1 ; R1 ) v C2 (F2 ; R2 ). (C1 ) nm trong (C2 ) v F1 , F2 . ng trn (C ) thay i lun tip xc
ngoi vi (C1 ) v tip xc trong vi (C2 ). Hy chng t rng tm M ca ng trn (C ) di ng trn mt elip.
Bi 9.141 : Trong mt phng ta Oxy cho im M(x; y) di ng c ta lun tha mn
8
< x = 5 cos t
:y = 4 sin t

trong t l tham s thay i.


Hy chng minh im M di ng trn mt elip.
Bi 9.142 : Trong mt phng vi h ta Oxy, cho im A chy trn trc Ox, im B chy trn trc Oy nhng di on AB bng
a khng i. Tm tp hp cc im M thuc on AB sao choMB = 2MA.
Bi 9.143 :

1. Vit phng trnh chnh tc ca elip (E), bit n c mt tiu im F(2; 0) v khong cch t F n nh trc nh

bng 3.
2. Hai ng thng d : mx y = 0 v d : x + my = 0 ln lt ct (E) ti M, P v N, Q. T gic MNPQ l hnh g. Tnh din tch
ca t gic MNPQ theo m.

3. Tm m MNPQ l hnh vung.


Bi 9.144 : Cho elip (E) : 5x2 + 9y2 = 45 c tiu im F1 , F2 . M l im bt k trn (E).
1. Chng minh rng chu vi tam gic F1 MF2 khng i. Tm M din tch tam gic F1 MF2 bng 2.
2. Tm M sao cho : T = F1 M + F2 M +

1
1
+
ln nht.
F1 M F2 M

Bi 9.145 : Cho im M di ng trn elip : 9x2 + 16y2 = 144. H v K l hnh chiu ca im M ln hai trc ta . Tm M din
tch t gic OHMK ln nht.
Bi 9.146 : Cho M, N l hai im bt k trn elip : 4x2 + 9y2 = 36 v khng trng vi cc nh. Gi I l trung im ca MN.
1. Chng minh rng tch h s gc ca ng thng MN v ng thng OI c gi tr khng i.
2. Vit phng trnh ng thng MN, bit trung im I c ta (1; 1).

9.5 ng hypebol
Bi 9.147 : Lp phng trnh chnh tc ca hypebol (H), bit :
1. Mt tiu im l (5; 0), mt nh l (4; 0).
5
2. di trc o bng 12, tm sai bng .
4
3
3. Mt nh l (2; 0), tai sai bng .
2

4. Tm sai bng 2, (H) i qua im A(5; 3).

5. (H) i qua hai im P(6; 1) v Q(8; 2 2).


Bi 9.148 : Lp phng trnh chnh tc ca hypebol (H), bit :

TRN ANH TUN - 0974 396 391 - (04) 66 515 343

WWW.VNMATH.COM

Trang 184

CHUYN LUYN THI I HC

www.VNMATH.com

www.luyenthi24h.com
www.luyenthi24h.com
www.VNMATH.com

1. (H) c di trc thc l 6, tiu im l (4; 0).


2. (H) c mt nh l (5; 0) v tim cn l y = 2x.

3. (H) c tim cn l y = 2x v qua im M(4; 2).


4. (H) qua hai im M(1;

3) v N( 2; 2 2).

5. (H) c tiu im F2 (3; 0) v qua im


4 5
3;
.
5

Bi 9.149 : Lp phng trnh chnh tc ca hypebol (H), bit :


1
1. Phng trnh cc cnh ca hnh ch nht c s l x = , y = 1.
2
2. Mt nh l (3; 0) v phng trnh ng trn ngoi tip hnh ch nht c s l x2 + y2 = 16.
4
3. Mt tiu im l (10; 0) v phng trnh cc ng tim cn l y = x.
3
4. (H) i qua im N(6; 3) v gc gia hai ng tim cn bng 60 .
Bi 9.150 : Cho hypebol (H) :

x2 y2

= 1.
9
3

1. Tm trn (H) im M c tung bng 1.


2. Tm trn (H) im M c gc F1 MF2 bng 90 .
3. Tm trn (H) im M sao cho F1 M = 2F2 M.
Bi 9.151 : Tm cc im trn hypebol (H) : 4x2 y2 = 4 tha mn :
1. Nhn hai tiu im di gc vung.
2. Nhn hai tiu im di gc 120.
3. C ta nguyn.
x2 y2

= 1 c cc tiu im F1 , F2 . M l im bt k trn (H). Chng minh tch khong cch


a2 b2
t M n hai tim cn c gi tr khng i.

Bi 9.152 :

1. Cho hypebol (H) :

x2 y2

= 1. Mt ng thng d bt k c phng trnh : y = x + m ct (H) ti M, N v hai tim cn ti P, Q.


1
2
Chng minh rng MP = NQ.

2. Cho hypebol (H) :

Bi 9.153 : Cho ng trn (C ) di ng, lun chn trn hai trc to hai dy cung c di l 6 v 4. Chng minh rng tm ng
trn di ng trn mt hypebol c nh.
Bi 9.154 : Cho hai im A(1; 0), B(1; 0) v ng thng : x

1
= 0.
4

1. Tm tp hp cc im M sao cho MB = 2MH, vi H l hnh chiu vung gc ca M trn .


2. Tm tp hp cc im N sao cho cc ng thng AN v BN c tch cc h s gc bng 2.
Bi 9.155 : Cho ba im A, B, C thng hng theo th t , AB = 3a, BC = a. im I di ng trn ng thng d vung gc vi AC
ti B. Cc tip tuyn v t A v C n ng trn tm I, bn knh IB, ct nhi ti D. Chng minh rng D di ng trn mt hypebol c
nh.
Bi 9.156 : Tm tp hp tm ng trn chn trn hai trc Ox, Oy hai on thng c di ln lt l 10 v 6.

TRN ANH TUN - 0974 396 391 - (04) 66 515 343

WWW.VNMATH.COM

Trang 185

www.VNMATH.com

CHUYN LUYN THI I HC

www.luyenthi24h.com
www.luyenthi24h.com
www.VNMATH.com

9.6 ng parabol
Bi 9.157 : Lp phng trnh chnh tc ca parabol c nh O v trc i xng Ox, bit :
1. parabol i qua im A(1; 2) ;
2. khong cch t tiu im n ng chun l 3 ;
3. dy cung MN ca parabol vung gc vi trc Ox ti tiu im F c di 4 ;
4. dy cung MN vung gc vi trc Ox c di l 8 v khong cch t nh n dy cung MN bng 2 ;

5. dy cung vung gc vi trc Ox ti trung im I ca on OF c di bng 2 2, vi F l tiu im ca parabol ;

6. ng thng d : 2x y 4 = 0 chn trn (P) mt on c di bng 3 5 ;


Bi 9.158 : Chp parabol (P) : y2 = 8x. Tm im M thuc parabol (P), bit bn knh qua tiu ca M bng 8.
Bi 9.159 : Cho parabol (P) : y2 = 32x. Tm im M trn parabol (P) sao cho khong cch t n ng thng : 4x + 3y + 10 = 0
bng 2.
Bi 9.160 : Cho parabol (P) : y2 = 4x c tiu im F. Tm im M trn parabol (P) sao cho tam gic F MN vung gc ti im F, vi

N(2; 2 2).

Bi 9.161 : Cho parabol (P) : y2 = x v im I(0; 2). Tm ta hai im M, N thuc (P) sao cho I M = 4IN.
Bi 9.162 : Cho parabol (P) : y2 = x. Tm hai im A v B trn parabol (P) i xng nhau qua trc honh sao cho tam gic OAB u.
Bi 9.163 : Cho parabol (P) : y2 = 64x. Tm im M trn parabol (P) sao cho khong cch t n ng thng : 4x + 3y + 86 = 0
l nh nht.
Bi 9.164 : Cho parabol (P) : y2 = x v hai im A(1; 1), B(9; 3) nm trn (P). Gi M l im thuc cung AB ca (P) (phn ca (P)
b chn bi dy AB). Xc nh v tr ca M trn cung AB sao cho tam gic MAB c din tch ln nht.

Bi 9.165 : Cho parabol (P) : y2 = 2x v ng thng d : 2mx 2y m = 0. Gi A v B l cc giao im ca d v (P). chng minh
ng trn ng knh AB lun lun tip xc vi mt ng thng c nh khi m thay i.

Bi 9.166 : Cho parabol (P) : y2 = 4x. Mt ng thng bt k i qua tiu im ca parabol cho v ct parabol ti hai im phn
bit A v B. Chng minh rng tch cc khong cch t A v B n trc ca parabol l mt i lng khng i.
Bi 9.167 : Chp parabol (P) : y2 = 6x. Vit phng trnh ng thng i qua im M(4; 1) v ct parabol (P) ti hai im phn bit
A, B sao cho M l trung im ca on AB.
Bi 9.168 : Cho parabol (P) : y2 = 64x v ng thng : 4x 3y + 46 = 0. Hy vit phng trnh ng trn c tm nm trn ng
thng , tip xc vi parabol (P) v c bn knh nh nht.

Bi 9.169 : Cho parabol (P) : y2 = 8x v im I(2; 4) nm trn parabol. Xt gc vung thay i quay quanh im I v hai cnh gc
vung ct parabol ti hai im M v N (khc vi im I). Chng minh rng ng thng MN lun lun i qua mt im c nh.
Bi 9.170 : Cho im A v ng thng c nh khng qua A. Tm tp hp im M l tm ca ng trn (C) lun qua A v tip
xc .
Bi 9.171 : Cho hnh vung ABCD c E l trung im BC. M l im di ng trn cnh AB. Gi N, P ln lt l giao im ca MD
v MC vi AE. Gi H l giao im ca NC v DP, I l giao im ca ng trung trc ca on thng DH vi ng thng vung
gc vi AH ti H. Chng minh rng khi M di ng trn cnh AB th I di ng trn mt ng c nh.
Bi 9.172 : Cho ng trn (O) tip xc vi ng thng d ti A. Tm qu tch tm I ca cc ng trn tip xc vi (O) v tip xc
vi d ln lt ti hai im M, N phn bit.
Bi 9.173 : Cho ng trn (O) c nh tm O v hai ng knh AB, CD vung gc nhau. M l im ty trn (O), H l hnh chiu
ca M trn CD. Tm tp hp giao im I ca OM v AH khi M di ng trn (O).

TRN ANH TUN - 0974 396 391 - (04) 66 515 343

WWW.VNMATH.COM

Trang 186

www.VNMATH.com

CHUYN LUYN THI I HC

www.luyenthi24h.com
www.luyenthi24h.com
www.VNMATH.com

9.7 Phng php ta trong mt phng qua cc k thi tuyn sinh H


Bi 9.174 (C08) : Tm im A thuc trc honh v im B thuc trc tung sao cho A v B i xng nhau qua ng thng d :
x 2y + 3 = 0.
Bi 9.175 (C09) : Trong mt phng vi h ta Oxy, cho tam gic ABC c C(1; 2), ng trung tuyn k t A v ng cao k

t B ln lt c phng trnh l 5x + y 9 = 0 v x + 3y 5 = 0. Tm ta cc nh A v B.

Bi 9.176 (C09) : Trong mt phng vi h ta Oxy, cho cc ng thng 1 : x 2y 3 = 0 v 2 : x + y + 1 = 0. Tm ta


1
im M thuc ng thng 1 sao cho khong cch t im M n ng thng 2 bng .
2

Bi 9.177 (A02) : Trong mt phng Oxy, xt tam gic ABC vung ti A, phng trnh ng thng BC l 3x y 3 = 0, cc nh

A v B thuc trc honh v bn knh ng trn ni tip bng 2. Tm ta trng tm G ca tam gic ABC.

Bi 9.178 (A04) : Cho hai im A(0; 2), B( 3; 1). Tm to trc tm v tm ng trn ngoi tip tam gic OAB.
Bi 9.179 (A05) : Cho hai ng thng : d1 : x y = 0 v d2 : 2x + y 1 = 0.

Tm to cc nh hnh vung ABCD bit nh A thuc d1 , nh C thuc d2 v cc nh B, D thuc trc honh.

Bi 9.180 (A06) : Cho cc ng thng : d1 : x + y + 3 = 0, d2 : x y 4 = 0, d3 : x 2y = 0.

Tm to im M trn ng thng d3 sao cho khong cch t M n ng thng d1 bng hai ln khong cch t M n ng

thng d2 .
Bi 9.181 (A07) : Cho tam gic ABC c A(0; 2), B(2; 2) v C(4; 2). Gi H l chn ng cao k t B; M v N ln lt l trung

im ca cc cnh AB v BC. Vit phng trnh ng trn i qua cc im H, M, N.

5
v hnh ch nht c s ca (E) c cho vi bng 20.
Bi 9.182 (A08) : Vit phng trnh elp (E), bit rng (E) c tm sai bng
3

Bi 9.183 (A09) : Trong mt phng vi h to Oxy, cho hnh ch nht ABCD c im I(6; 2) l giao im ca hai ng cho AC
v BD. im M(1; 5) thuc ng thng AB v trung im E ca cnh CD thuc ng thng : x + y 5 = 0. Vit phng trnh
ng thng AB.

Bi 9.184 (A09) : Trong mt phng vi h to Oxy, cho ng trn (C) : x2 +y2 +4x+4y+6 = 0 v ng thng : x+my2m+3 =
0, vi m l tham s thc. Gi I l tm ca ng trn (C). Tm m ct (C) ti hai im phn bit A v B sao cho din tch tam gic
IAB ln nht.
Bi 9.185 (A10) : Trong mt phng ta Oxy, cho hai ng thng d1 :

3x + y = 0 v d2 : 3x y = 0. Gi (T ) l ng trn

tip xc vi d1 ti
A, ct d2 ti hai im B v C sao cho tam gic ABC vung ti B. Vit phng trnh ca (T ), bit tam gic ABC c
3
din tch bng
v im A c honh dng.
2
Bi 9.186 (A10) : Trong mt phng to Oxy, cho tam gic ABC cn ti A c nh A(6; 6); ng thng i qua trung im ca cc
cnh AB v AC c phng trnh x + y 4 = 0. Tm to cc nh B v C, bit im E(1; 3) nm trn ng cao i qua nh C ca
tam gic cho.

Bi 9.187 (B02) : Cho hnh ch nht ABCD c tm I


cc nh A, B, C, D bit nh A c honh m.

1
; 0 , phng trnh ng thng AB : x 2y + 2 = 0 v AB = 2AD. Tm to
2

= 90 . Bit M(1; 1) l trung im cnh BC v G


Bi 9.188 (B03) : Cho tam gic ABC c AB = AC, BAC

gic ABC. Tm to cc nh A, B, C.

2
; 0 l trng tm tam
3

Bi 9.189 (B04) : Cho hai im A(1; 1), B(4; 3). Tm im C thuc ng thng x 2y 1 = 0 sao cho khong cch t C n ng
thng AB bng 6.

Bi 9.190 (B05) : Cho hai im A(2; 0) v B(6; 4). Vit phng trnh ng trn (C) tip xc vi trc honh ti im A v khong
cch t tm ca (C) n im B bng 5.
Bi 9.191 (B06) : Cho ng trn (C) : x2 + y2 2x 6y + 6 = 0 v im M(3; 1). Gi T 1 , T 2 l cc tip im ca cc tip tuyn k

t M n (C). Vit phng trnh ng thng T 1 T 2 .

TRN ANH TUN - 0974 396 391 - (04) 66 515 343

WWW.VNMATH.COM

Trang 187

www.VNMATH.com

CHUYN LUYN THI I HC

www.luyenthi24h.com
www.luyenthi24h.com
www.VNMATH.com

Bi 9.192 (B07) : Cho im A(2; 2) v cc ng thng


d1 : x + y 2 = 0 v d2 : x + y 8 = 0.
Tm to cc im B, C ln lt thuc d1 , d2 sao cho tam gic ABC vung cn ti A.
Bi 9.193 (B08) : Tm to nh C ca tam gic ABC bit rng hnh chiu vung gc ca C trn ng thng AB l im H(1; 1),

ng phn gic trong ca gc A c phng trnh x y + 2 = 0 v ng cao k t B c phng trnh 4x + 3y 1 = 0,


4
Bi 9.194 (B09) : Trong mt phng vi h to Oxy, cho ng trn (C) : (x 2)2 + y2 = v hai ng thng 1 : x y = 0,
5
2 : x 7y = 0. Xc nh to tm K v tnh bn knh ca ng trn (C1 ) bit ng trn (C1 ) tip xc vi cc ng thng 1 , 2
v tm K thuc ng trn (C).

Bi 9.195 (B09) : Trong mt phng vi h to Oxy, cho tam gic ABC cn ti A c nh A(1; 4) v cc nh B, C thuc ng
thng : x y 4 = 0. Xc nh to cc im B, C v bit din tch tam gic ABC bng 18.

Bi 9.196 (B10) : Trong mt phng to Oxy, cho tam gic ABC vung ti A, c nh C(4; 1), phn gic trong gc A c phng
trnh x + y 5 = 0. Vit phng trnh ng thng BC, bit din tch tam gic ABC bng 24 v nh A c honh dng.

x2 Y 2
Bi 9.197 (B10) : Trong mt phng to Oxy, cho im A(2; 3) v elip (E) :
+
= 1. Gi F1 v F2 l cc tiu im ca (E)
3
2
(F1 c honh m); M l giao im c tung dng ca ng thng AF1 vi (E); N l im i xng ca F2 qua M. Vit phng
trnh ng trn ngoi tip tam gic ANF2 .
x2 y2
+
= 1. Xt im M chuyn ng trn tia Ox v im N chuyn ng trn tia Oy sao cho ng
16 9
thng MN lun tip xc vi (E). Xc nh to im M, N MN c di nh nht. Tnh gi tr nh nht .
Bi 9.198 (D02) : Cho elp (E) :

Bi 9.199 (D03) : Cho ng trn (C) : (x 1)2 + (y 2)2 = 4 v ng thng d : x y 1 = 0.

Vit phng trnh ng trn (C ) i xng vi ng trn (C) qua ng thng d. Tm to cc giao im ca (C) v (C ).

Bi 9.200 (D04) : Cho tam gic ABC c cc nh A(1; 0), B(4; 0), C(0; m), vi m , 0. Tm to trng tm G ca tam gic ABC
theo m. Xc nh m tam gic GAB vung ti G.
x2 y2
+
= 1. Tm to cc im A, B thuc (E), bit rng hai im A, B i xng
4
1
nhau qua trc honh v tam gic ABC l tam gic u.
Bi 9.201 (D05) : Cho im C(2; 0) v elp (E) :

Bi 9.202 (D06) : Cho ng trn (C) : x2 + y2 2x 2y + 1 = 0 v ng thng d : x y + 3 = 0. Tm to im M nm trn d


sao cho ng trn tm M c bn knh gp i bn knh ng trn (C), tip xc ngoi vi ng trn (C).

Bi 9.203 (D07) : Cho ng trn (C) : (x 1)2 + (y + 2)2 = 9 v ng thng d : 3x 4y + m = 0.

Tm m trn d c duy nht mt im P m t c th k c hai tip tuyn PA, PB ti (C) (A, B l cc tip im) sao cho tam

gic PAB u.
Bi 9.204 (D08) : Cho parabol (P) : y2 = 16x v im A(1; 4). Hai im phn bit B, C (B v C khc A) di ng trn (P) sao cho gc
= 90 . Chng minh rng ng thng BC lun i qua mt im c nh.
BAC

Bi 9.205 (D09) : Trong mt phng vi h to Oxy, cho tam gic ABC c M(2; 0) l trung im ca cnh AB. ng trung tuyn
v ng cao qua nh A ln lt c phng trnh l 7x 2y 3 = 0 v 6x y 4 = 0. Vit phng trnh ng thng AC.

Bi 9.206 (D09) : Trong mt phng vi h to Oxy, cho ng trn (C) : (x 1)2 + y2 = 1. Gi I l tm ca (C). Xc nh to
im M thuc (C) sao cho I
MO = 30 .

Bi 9.207 (D10) : Trong mt phng to Oxy, cho tam gic ABC c nh A(3; 7), trc tm l H(3; 1), tm ng trn ngoi tip
l I(2; 0). Xc nh ta nh C, bit C c honh dng.

Bi 9.208 (D10) : Trong mt phng ta Oxy, cho im A(0; 2) v l ng thng i qua O. Gi H l hnh chiu vung gc ca A
trn . Vit phng trnh ng thng , bit khong cch t H n trc honh bng AH.

9.8 Bi tp tng hp
Bi 9.209 : Lp phng trnh ng thng i qua im M(1; 1) v cng vi cc ng thng 2x 3y + 4 = 0, 3x + 2y + 5 = 0 to thnh
mt tam gic cn.

TRN ANH TUN - 0974 396 391 - (04) 66 515 343

WWW.VNMATH.COM

Trang 188

CHUYN LUYN THI I HC

www.VNMATH.com

www.luyenthi24h.com
www.luyenthi24h.com
www.VNMATH.com

Bi 9.210 : Vit phng trnh cc cnh ca hnh vung ABCD bit A(4; 5) v ng cho BD : 7x y + 8 = 0.

Bi 9.211 : Lp phng trnh cc cnh ca tam gic ABC bit A(1; 6) v hai trung tuyn c phng trnh x2y+1 = 0 v 3xy2 = 0.
Bi 9.212 : Cho ng trn (C) : x2 + y2 2x + 6y 15 = 0 v im A(2; 1).

Vit phng trnh ng thng ct (C) ti hai im M, N sao cho A l trung im ca MN.

Bi 9.213 : Trong mt phng vi h ta Oxy, cho tam gic ABC c A(0; 2), B(2; 2) v C(4; 2). Gi H l chn ng cao k t
B ; M, N ln lt l trung im cc cnh AB v BC. Vit phng trnh ng trn i qua cc im H, M, N.

Bi 9.214 : Trong mt phng vi h ta Oxy, cho ng trn (C) : (x 1)2 + (y 2)2 = 4 v ng thng d : x y 1 = 0.

Vit phng trnh ng trn (C ) i xng vi ng trn (C) qua ng thng d v tm ta cc giao im ca (C) v (C ).

Bi 9.215 : Vit phng trnh ng trn ni tip tam gic ABC c ba nh l A(1; 7), B(4; 3), C(4; 1).

Bi 9.216 : Trong mt phng vi h ta Oxy, cho im A(1; 0) v ng thng d : x 3y + 2 = 0. Tm ta im B nm trn


trc honh v im C nm trn ng thng d sao cho ABC u.

Bi 9.217 : Trong mt phng vi h ta Oxy, cho ng thng dLx + y 3 = 0 v e-lp (E) :


thuc (E) c khong cch n d l ngn nht.

Bi 9.218 : Trong mt phng vi h ta Oxy, cho elp (E) :


(E), bit tip tuyn to vi d mt gc 30 .

x2 y2
+
= 1. Tm ta im M
4
1

x2
+ y2 = 1 v ng thng d : y = 2. Lp phng trnh tip tuyn vi
4

Bi 9.219 : Trong mt phng vi h ta Oxy, cho ng thng : x + y 1 = 0, cc im A(0; 1), B(2; 1). T gic ABCD l hnh

thoi c tm nm trn . Tm ta cc im C, D.

5 1
Bi 9.220 : Trong mt phng vi h ta Oxy, cho tam gic ABC trng tm G
; , ng trn i qua trung im ca cc cnh
3 3
2
2
c phng trnh x + y 2x + 4y = 0. Vit phng trnh ng trn ngoi tip tam gic ABC.
Bi 9.221 : Trong mt phng vi h ta Oxy, cho A(3; 3) v ng thng d : x + y 2 = 0. Lp phng trnh ng trn i qua A

ct d ti B, C sao cho ABAC v AB = AC.

= 90 , ng thng AB c phng trnh


Bi 9.222 : Trong mt phng vi h ta Oxy, cho tam gic ABC, AB = AC, BAC

x y + 1 = 0, trng tm l G(3; 2) v tung ca im A ln hn 3. Tm ta cc nh A, B, C.


Bi 9.223 : Trong mt phng vi h ta Oxy, choi tam gic ABC vi A(4; 2), B(1; 2) v tm ng trn ni tip tam gic l I(2; 3).
Xc nh ta im C.
Bi 9.224 : Trong mt phng vi h ta Oxy, cho tam gic ABC c A(4; 6), phng trnh cc ng thng cha ng cao v trung
tuyn k t C ln lt l 2x y + 13 = 0; 6x 13y + 29 = 0. Vit phng trnh ng trn ni tip tam gic ABC.
Bi 9.225 : Trong mt phng vi h ta Oxy, xt elip (E) i qua im M(2; 3) v c phng trnh mt ng chun l x + 8 = 0.
Vit phng trnh chnh tc ca elip.

Bi 9.226 : Trong mt phng vi h ta Oxy, cho parabol (P) : y2 = 8x. ng thng d i qua tiu im ca (P) v ct (P) ti hai
im A, B. Vit phng trnh ng thng d bit AB = 8.
Bi 9.227 : Trong mt phng vi h ta Oxy, cho cc ng thng 1 : 2x + y + 3 = 0, 2 : 3x 2y 1 = 0, : 7x y + 8 = 0.
Tm im P 1 , Q 2 sao cho l ng trung trc ca on PQ.

Bi 9.228 : Trong mt phng vi h ta Oxy, cho im K(3; 2). ng trn (C) : x2 + y2 2x 4y + 1 = 0 vi tm l I. Tm im


M (C) sao cho I
MK = 60 .

Bi 9.229 : Trong mt phng vi h ta Oxy, cho hai ng trn


(C1 ) : x2 + y2 + 10x 39 = 0, (C2 ) : x2 + y2 10x + 21 = 0.
1. Vit phng trnh ng trn tip xc vi (C1 ) v (C2 ) ng thi c tm thuc ng thng y = 3.
2. Chng minh rng tm cc ng trn ng thi tip xc vi (C1 ) v (C2 ) nm trn mt ng Hypebol. Vit phng trnh
Hypebol .

TRN ANH TUN - 0974 396 391 - (04) 66 515 343

WWW.VNMATH.COM

Trang 189

www.VNMATH.com

CHUYN LUYN THI I HC

www.luyenthi24h.com
www.luyenthi24h.com
www.VNMATH.com

x2 y2
+
= 1 v ng thng d : 5x + 3 2y 3 10 = 0. Gi A, B l
9
5
cc giao im ca (E) v d. Tm ta im C trn (E) sao cho tam gic ABC cn ti C.
Bi 9.230 : Trong mt phng vi h ta Oxy, cho elip (E) :

Bi 9.231 : Trong mt phng vi h ta Oxy, cho hai ng thng 1 : y 2x = 0 v 2 : y + 2x = 0. Gi A 1 , B 2 tha mn



OA.OB = 3. Hy tm tp hp trung im M ca AB.
Bi 9.232 : Trong mt phng vi h ta Oxy, cho tam gic ABC, ng phn gic trong ca gc A c phng trnh x + 2y 5 = 0,

ng cao i qua A c phng trnh 4x + 13y 10 = 0 v im C(4; 3). Tm ta im B.

Bi 9.233 : Trong mt phng vi h ta Oxy, cho ng trn (C) : x2 + y2 + 6x 2y + 6 = 0 v cc im B(2; 3), C(4; 1). Xc
nh ta im A thuc ng trn sao cho tam gic ABC cn ti A v c din tch nh nht.

Bi 9.234 : Trong mt phng vi h ta Oxy, cho ng trn (C) : x2 + y2 = 1. Tm cc gi tr thc ca m trn ng thng
y = m tn ti ng hai tip tuyn vi (C) sao cho gc gia hai tip tuyn bng 60 .
Bi 9.235 : Trong mt phng vi h ta Oxy, cho Hypebol (H) : 4x2 y2 = 4. Tm im N trn (H) sao cho N nhn hai tiu im
gc 120.

Bi 9.236 : Trong mt phng vi h ta Oxy, cho ng trn (C) : x2 + y2 + 4x 6y + 9 = 0, im K(1; 4) v ng thng


: x y 3 = 0. Tm cc im trn ng thng t k c hai tip tuyn n ng trn (C) sao cho ng thng i qua cc
tip im cng i qua K.

Bi 9.237 : Trong mt phng vi h ta Oxy, cho hai ng trn


(C1 ) : x2 + y2 4x 2y + 4 = 0 v (C2 ) : x2 + y2 2x 6y + 6 = 0.
Chng minh rng hai ng trn ct nhau v vit phng trnh cc tip tuyn chung ca chng.
Bi 9.238 : Trong mt phng vi h ta Oxy, cho hai im A(1; 1), B(3; 3). Vit phng trnh ng trn i qua A, B v nhn Ox
lm tip tuyn.
Bi 9.239 : Trong mt phng vi h ta Oxy, cho hnh bnh hnh ABCD c din tch bng 4. Bit A(1; 0), B(0; 2) v giao im I
ca hai ng cho nm trn ng thng y = x. Tm ta cc nh C, D.
Bi 9.240 : Trong mt phng vi h ta Oxy, cho tam gic ABC c nh A(1; 2), ng trung tuyn BM : 2x + y + 1 = 0 v phn
gic trong CD : x + y 1 = 0. Vit phng trnh ng thng BC.

Bi 9.241 : Trong mt phng vi h ta Oxy, cho hnh ch nht c din tch bng 12, tm I thuc ng thng d : x y 3 = 0 v
9
c honh im I bng , trung im ca mt cnh l giao im ca d v trc Ox. Tm ta cc nh ca hnh ch nht.
2
Bi 9.242 : Trong mt phng vi h ta Oxy, cho ng trn (C) : (x 1)2 + (y + 2)2 = 4. Vit phng trnh cc tip tuyn ca (C)

bit tip tuyn i qua im A(1; 2). Tm ta cc tip im tng ng.

Bi 9.243: Trong
mt
phng
ta Oxy, cho hnh vung ABCD c tm I bit A(2; 2) v trng tm cc tam gic ABC v IBC ln
4
7 5
;2 , G
;
. Vit phng trnh ng thng CD.
lt l G
3
3 3

WWW.VNMATH.COM

TRN ANH TUN - 0974 396 391 - (04) 66 515 343

WWW.VNMATH.COM

Trang 190

www.luyenthi24h.com
www.luyenthi24h.com

Chng 10

M u v hnh hc khng gian. Quan h song


song
Sau khi hc xong mc ny, hc sinh cn bit :
1. Mt mt phng c xc nh nu bit mt trong cc iu kin sau y :
(a) Mt phng i qua ba im khng thng hng.
(b) Mt phng i qua mt im v mt ng thng khng cha im .
(c) Mt phng i qua hai ng thng ct nhau.
(d) Mt phng i qua hai ng thng song song.
(e) Mt phng i qua mt ng thng v song song vi mt ng thng cho vi ng thng y.
(f) Mt phng i qua mt im v song song vi mt mt phng khng cha ng thng y.
(g) Mt phng i qua mt im v song song vi hai ng thng cho nhau.
2. Du hiu nhn bit hai ng thng song song :
(a) Hai ng thng ng phng v khng c im chung.
(b) Hai mt phng phn bit ct nhau v ln lt i qua hai ng thng song song th giao tuyn ca chng song song vi
t nht mt trong hai ng thng y.
(c) Hai ng thng phn bit cng song song vi ng thng th ba th song song vi nhau.
(d) Nu ng thng a song song vi mt phng (P) th mi mt phng (Q) cha a m ct (P) th giao tuyn song song vi a.
(e) Hai mt phng phn bit ct nhau v cng song song vi mt ng thng th giao tuyn ca chng song song vi ng
thng .
(f) Nu hai mt phng song song b ct bi mt phng th ba th hai giao tuyn song song vi nhau.
(g) Nu ng thng a song song vi ng thng b v b khng song song vi l th hai hnh chiu a , b ca a v b theo
phng l ln mt phng (P) song song hoc trng nhau.
(h) Nu ng thng a song song vi mt phng (P) th hnh chiu a ca a trn (P) song song vi a.
3. Du hiu nhn bit ng thng song song vi mt phng :
(a) Nu mt ng thng v mt mt phng khng c im chung th chng song song vi nhau.
(b) Nu a b, a 1 (P), b (P) th a (P).
(c) Nu a (P), (P) (Q) th a (Q).

(d) Nu ba ng thng chn trn hai ct tuyn nhng on thng tng ng t l th ba on thang cng song song vi
mt mt phng (mt phng ny cha hai ng thng ct nhau ln lt song song vi hai trong ba ng thng trn).

191

WWW.VNMATH.COM

www.VNMATH.com

CHUYN LUYN THI I HC

www.luyenthi24h.com
www.luyenthi24h.com
www.VNMATH.com

(e) a b, a (P), b 1 (P), b (P).


(f) a (P), (P) (Q), a 1 (Q) a (Q).
4. Du hiu nhn bit hai mt phng song song :
(a) Hai mt phng khng c im chung th song song vi nhau.
(b) Nu mt mt phng cha hai ng thng ct nhau v cng song song vi mt mt phng khc th ahi mt phng sng
song.
(c) Nu hai ng thng ct nhau ca mt mt phng ln lt song song vi hai ng thng ca mt phng khc th hai mt
phng song song.
(d) Hai mt phng phn bit cng song song vi mt phng th ba th song song vi nhau.

10.1 i cng v ng thng v mt phng


Xc nh giao tuyn ca hai mt phng (P) v (Q) ; xc nh giao im ca ng thng a v mt phng (P) ; chng minh ba im
thng hng v ba ng thng ng quy ; tm thit din ca hnh chp ct bi mt phng ; chng minh bn im ng phng.

Vn 1 : Xc nh giao tuyn ca hai mt phng

Giao tuyn ca hai mt phng l mt ng thng. V vy ta cn xc nh c hai giao im ca hai mt phng . Mun xc

nh giao im ca hai mt phng ta chn hai ng thng a (P) v b (Q) sao cho a b = {M}. Khi M l mt giao im
ca hai mt phng .

Bi 10.1 : Trong mt phng () cho t gic ABCD c cc cnh i AB v CD khng song song vi nhau. Gi S l mt im khng
thuc mt phng ().
1. Xc nh giao tuyn hai mt phng (S AC) v (S BD).
2. Xc nh giao tuyn hai mt phng (S AB) v (S CD).

Vn 2 : Xc nh giao im ca ng thng a v mt phng (P)


1. Phng php chung l cn tm ng thng (P) v ct a, giao im chnh l giao im ca a v (P).
2. Cch tm ng thng : Chn mt phng (Q) sao cho a (Q), (Q) (P) = a = a (P). Thng ta chn mt phng
(Q) sao cho d xc nh giao tuyn vi mt phng (P).

Bi 10.2 : Cho tam gic ABC v mt im O nm ngoi mt phng (ABC). Trn cc on thng OA, OB, OC ta ln lt ly cc im
A , B , C khng trng vi u mt cc on thng . Gi M l mt im thuc mt phng (ABC) v nm trong tam gic ABC. Tm
im chung (giao im) ca :
1. ng thng BC vi mt phng (OAM).

TRN ANH TUN - 0974 396 391 - (04) 66 515 343

2. ng thng OM vi mt phng (A BC ).

WWW.VNMATH.COM

Trang 192

www.VNMATH.com

CHUYN LUYN THI I HC

www.luyenthi24h.com
www.luyenthi24h.com
www.VNMATH.com

Vn 3 : Phng php chng minh ba im thng hng v ba ng thng ng quy

Mun chng minh ba im A, B, C thng hng ta chng minh A, B, C cng nm trn hai mt phng phn bit (P), (Q). Khi A, B, C
thuc giao tuyn ca hai mt phng , nn chng thng hng.
Cn nu mun chng minh ba ng thng a, b, c ng quy, ta xc nh giao im ca hai trong ba ng thng ri chng minh giao
im thuc ng thng cn li. Hoc c th dng nh l giao tuyn ca ba mt phng.

Bi 10.3 : Cho tam gic ABC v mt im O nm ngoi mt phng (ABC). Gi A , B , C ln lt l cc im ly trn OA, OB, OC v
khng trng vi u mt cc on thng . Chng minh rng cc cp ng thng A B v AB, BC v BC, C A v CA ct nhau ln
lt ti D, E, F th ba im D, E, F thng hng.
Bi 10.4 : Cho t din ABCD. Gi E, F, G l ba im ln lt nm trn ba cnh AB, AC, BD sao cho EF ct BC ti I, EG ct AD ti
H. Chng minh rng CD, IG, HF ng quy.

Vn 4 : Tm thit din ca hnh chp ct bi mt phng

Mun tm thit din ca hnh chp ct bi mt phng () ta tm cc on giao tuyn ca () giao vi cc mt (bn v y) ca hnh
chp.
Ch : Mt phng () ct mi mt bn ti khng qu hai im trong ca cc cnh ca mt bn .

Bi 10.5 : Cho hnh chp t gic S .ABCD. im C nm trn cnh S C. Tm thit din ca hnh chp vi mt phng (ABC ).
Bi 10.6 : Cho bn im A, B, C, D khng ng phng.
1. im D thuc nhng mt phng no ?
2. Chng minh AC v BD cho nhau.
3. Gi Bx l ng thng i qua B v song song vi AD v M AD. Gi J l trung im on BM. Nu im M di ng trn

ng thng AD, im B di ng trn ng thng Bx, chng minh rng khi ng thng CJ lun lun nm trong mt phng
c nh.

Bi 10.7 : Cho hai ng thng a v b cho nhau. Trn a ta ly hai im phn bit A, B v trn b ly hai im phn bit C, D.
1. Chng minh rng AC v BD cho nhau.
2. Gi M l mt im trn on AC, N l im trn on BD. Khi ng thng MN c th song song vi AB hoc CD c
khng ?
3. Gi O l im trn on MN. Chng minh rng AO ct CN v BO ct DM.
Bi 10.8 : Cho mt phng () xc nh bi ng thng a v im A khng thuc a. Gi a l ng thng i qua A v song song vi
a. Ly mt im M trn a v mt im B nm ngoi mt phng ().
1. Chng minh rng im M thuc mt phng ().
2. Tm im chung ca cc cp mt phng (ABM) v (), (ABM) v (a , B), (ABM) v (a, B).
3. Tm im chung ca ba mt phng (), (a, B), (ABM).
4. Gi I, K ln lt l cc trung im ca cc on thng AB v MB . Chng minh rng IK song song vi mt phng ().

TRN ANH TUN - 0974 396 391 - (04) 66 515 343

WWW.VNMATH.COM

Trang 193

www.VNMATH.com

CHUYN LUYN THI I HC

www.luyenthi24h.com
www.luyenthi24h.com
www.VNMATH.com

Bi 10.9 : Gi () l mt phng xc nh bi hai ng thng a v b ct nhau ti O v c l mt ng thng ct () ti I khc O.


1. Xc nh giao tuyn ca hai mt phng () v (O, c).
2. Gi M l mt im nm trn c v khng trng vi I. Tm giao tuyn m ca hai mt phng (M, a) v (M, b). Chng minh rng
khi M di ng trn ng thng c, giao tuyn m ny lun nm trong mt mt phng c inh.
Bi 10.10 : Cho t din ABCD. Gi M v N ln lt l trung im ca AC v BC. Gi K l mt im ly trn cnh BD sao cho
BK = 3KD.
1. Tm giao tuyn ca hai mt phng (MNK) vi mt phng (BCD).
2. Tm giao tuyn ca hai mt phng (MNK) vi mt phng (ACD).
Bi 10.11 : Cho hnh chp S .ABCD vi y ABCD l t gic c cc cp cnh i khng song song. Tm giao tuyn ca cc cp mt
phng sau:
1. (S AC) v (S BD) ;

2. (S AB) v (S CD) ;

3. (S AD) v (S BC).

Bi 10.12 : Cho hnh chp S .ABCD vi y ABCD l hnh bnh hnh tm O. Gi M, N, P ln lt l trung im ca cc on
BC, CD, S O. Xc nh thit din ca hnh chp to bi mt phng (MNP).
Bi 10.13 : Cho hnh chp S .ABCD c y ABCD l hnh bnh hnh. Gi M, N ln lt l trung im cc cnh S B, S D . Ly mt
im P trn cnh S C sao cho S P = 3PC. Tm giao tuyn ca mt phng (MNP) vi cc mt (S AC), (S AB), (S AD) v (ABCD) ca
hnh chp.
Bi 10.14 : Cho t din ABCD. Gi M, N ln lt ly trn cc cnh AC, BC sao cho MN khng song song vi AB. Gi O l mt im
thuc min trong tam gic ABD. Tm giao im ca AB v AD vi mt phng (OMN).
Bi 10.15 : Cho hnh chp S .ABCD. Gi M l mt im trn cnh S C.
1. Tm giao im ca AM vi mt phng (S BD).
2. Ly mt im N trn cnh BC. Tm giao im ca S D v mt phng (AMN).
Bi 10.16 : Cho hnh chp S .ABCD c y ABCD l hnh bnh hnh. Gi M l trung im ca S C.
1. Tm giao im I ca ng thng AM vi mt phng (S BD). Chng minh rng IA = 2I M.
2. Tm giao im P ca ng thng S D vi mt phng (ABM).
3. Gi N l mt im tu trn cnh AB. Tm giao im ca ng thng MN vi mt phng (S BD).
Bi 10.17 : Cho t din ABCD. Gi M, N l hai im ln lt ly trn AC, AD. Gi G l trng tm tam gic BCD. Tm giao im ca:
1. MN v mt phng (ABG).

2. AG v mt phng (BMN).

Bi 10.18 : Cho hnh chp S .ABCD. Gi I, K l hai im c nh trn S A v S C vi S I = 2IA v S K = 13 KC. Mt mt phng ()
quay quanh IK ct S B ti M v S D ti N. Gi O l giao im ca AC v BD.
1. Chng minh rng ba ng thng IK, MN, S O ng quy.
2. Gi {E} = AD BC v {F} = IN MK. Chng minh rng ba im S , E, F thng hng.
3. Gi {P} = IN AD v {Q} = MK BC. Chng minh rng khi () thay i ng thng PQ lun i qua mt im c inh.
Bi 10.19 : Cho hnh chp S .ABCD. Gi I l mt im trn cnh AD v K l mt im trn cnh S B.
1. Tm giao im E, F ca IK v DK vi mt phng (S AC).
2. Gi {O} = AD BC, {M} = S C OK. Chng minh rng bn im A, E, F, M thng hng.
Bi 10.20 : Cho t din ABCD. Trn on CA, CB, BD cho ln lt cc im M, N, P sao cho MN khng song song vi AB , NP
khng song song vi CD. Gi () l mt phng xc nh bi ba im M, N, P ni trn. Tm thit din to bi mt phng () v t din
ABCD.

TRN ANH TUN - 0974 396 391 - (04) 66 515 343

WWW.VNMATH.COM

Trang 194

www.VNMATH.com

CHUYN LUYN THI I HC

www.luyenthi24h.com
www.luyenthi24h.com
www.VNMATH.com

Bi 10.21 : Cho hnh chp S .ABCD c y l hnh bnh hnh tm O. Gi M, N, E l ba im ln lt ly trn AD, CD, S O. Tm thit
din ca hnh chp to bi mt phng (MNP).
Bi 10.22 : Cho hnh chp S .ABCD. Trong tam gic S BC ly mt im M v trong tam gic S CD ly mt im N.
1. Tm giao im ca ng thng MN vi mt phng (S AC).
2. Tm giao im ca cnh S C vi mt phng (AMN).
3. Tm thit din ca hnh chp ct bi mt phng (AMN).
Bi 10.23 : Cho hnh chp S .ABCD c y ABCD l hnh bnh hnh. Gi M l trung im ca cnh S C.
1. Tm giao im I ca ng thng AM vi mt phng (S BD). Chng minh rng IA = 2I M.
2. Tm giao im F ca ng thng S D vi mt phng (ABM). Chng minh rng F l trung im ca cnh S D v t gic ABMF
l mt hnh thang.
3. Gi N l im tu trn cnh BC. Tm thit din ca hnh chp ct bi mt phng (AMN).
Bi 10.24 : Cho hnh chp S .ABCD. Gi M, N l cc im ln lt trn cc on BC v S D.
1. Tm giao im I ca ng thng BN vi mt phng (S AC) v giao im K ca ng thng MN vi mt phng (S AC).
2. Tm thit din ca hnh chp S .ABCD vi mt phng (BCN).
Bi 10.25 : Cho hnh chp S .ABCD c y l hnh bnh hnh. Gi M, N ln lt l cc im trn cc on S B v AD. ng thng
BN ct CD ti I.
1. Chng minh rng ba im M, I v trng tm G ca tam gic S AD thng hng.
2. Tm thit din ca hnh chp ct bi mt phng (CGM). Chng minh rng trung im ca on S A thuc thit din nay.
3. Tm thit din ca hnh chp ct bi mt phng (AGM).
Bi 10.26 : Cho chp S .ABCD c y ABCD l mt t gic c cc cp cnh i khng song song. Gi M l mt im ty nm trn
cnh S C (M khng trng vi C v S ), mt phng (ABM) ct S D ti N.
1. Gi I l giao im ca BM v AN. Chng minh rng khi im M di ng trn cnh S C th I di ng trn mt on thng c
nh. Hy xc nh on thng .
2. Gi J l giao im ca AM v BN. Chng minh rng khi M di ng trn cnh S C th J di ng trn mt on thng c nh.
Bi 10.27 : Cho t din ABCD. Gi I v J ln lt l trung im ca AB v CD. Cho M v N nm trn cc cnh BC v AD sao cho
AN
BM
=
= 3. Chng minh rng M, N, I, J lun ng phng.
MC
ND

10.2 Hai ng thng song song


Tm giao tuyn ca hai mt phng (dng quan h song song) ; chng minh hai ng thng song song ; chng minh hai ng thng
cho nhau

Vn 1 : Tm giao tuyn ca hai mt phng (dng quan h song song)

Nu hai mt phng () v () c im chung S v ln lt cha hai ng thng song song d v d th giao tuyn c () v () l

ng thng i qua S v song song vi d v d .

Bi 10.28 : Cho hnh chp S .ABCD c y l hnh bnh hnh ABCD. Tm giao tuyn ca cc cp mt phng sau y.

TRN ANH TUN - 0974 396 391 - (04) 66 515 343

WWW.VNMATH.COM

Trang 195

www.VNMATH.com

CHUYN LUYN THI I HC

1. (S AC) v (S BD) ;

2. (S AB) v (S CD) ;

www.luyenthi24h.com
www.luyenthi24h.com
www.VNMATH.com

3. (S AD) v (S BC).

Bi 10.29 : Cho hnh chp S .ABCD c y l hnh thang vi cc cnh y l AB, CD. Gi I, J ln lt l trung im ca AD, BC v
G l trng tm tam gic S AB.
1. Tm giao tuyn ca (S AB) v (I JG).
2. Xc nh thit din ca hnh chp vi mt phng (I JG). Thit din l hnh g? Tm iu kin i vi AB v CD thit din l
hnh bnh hnh.

Vn 2 : Chng minh hai ng thng song song


1. Dng nh ngha ( Chng minh chng cng thuc mt mt phng, ri p dng cc phng php chng minh thng thng,
thng p dng nh l Talet).
2. Dng phn chng
3. Dng nh l giao tuyn ca ba mt phng.
4. Dng tnh cht : Hai mt phng phn bit ln lt i qua hai ng thng song song th giao tuyn ca chng (nu c) song
song vi hai ng thng , hoc trng vi mt trong hai ng thng .
5. Dng tnh cht bc cu

Bi 10.30 : Cho t din ABCD. Gi I, J ln lt l trng tm cc tam gic ABC, ABD. Chng minh rng I J CD.
Bi 10.31 : Cho hnh chp S .ABCD c y ABCD l hnh thang vi hai cnh y AB v CD (AB > CD). Gi M, N ln lt l trung
im S A, S B.
1. Chng minh MN CD ;
2. Gi P l giao im ca S C v mt phng (ADN), I l giao im AN v DP. Chng minh rng S I AB. T gic S ABI l hnh
g.

Vn 3 : Chng minh hai ng thng cho nhau

Chng ta thng dng phng php phn chng.

Bi 10.32 : Cho d1 , d2 l hai ng thng cho nhau. Trn d1 , ly hai im phn bit A, B v trn d2 ly hai im phn bit C, D.
Chng minh rng AC v BD cho nhau.
Bi 10.33 : Cho tam gic ABC nm trong mt phng (). Gi Bx, Cy l hai na ng thng song song v nm v cng mt pha i
vi mt phng (). M, N l hai im ln lt di ng trn Bx, Cy sao cho CN = 2BM.
1. Chng minh rng ng thng MN lun i qua mt im c nh I khi M, N di ng.
1
EA, IE ct AN ti F. Gi Q l giao im ca BE v CF. Chng minh rng AQ song song vi Bx
3
v (QMN) cha mt ng thng c nh khi M, N thay i.

2. E thuc on AM v EM =

Bi 10.34 : Cho hnh chp S .ABCD c y l hnh bnh hnh. Gi M, N, P, Q ln lt l cc im nm trn BC, S C, S D, AD sao cho
MN BS , NP CD, MQ CD.

TRN ANH TUN - 0974 396 391 - (04) 66 515 343

WWW.VNMATH.COM

Trang 196

CHUYN LUYN THI I HC

www.VNMATH.com

www.luyenthi24h.com
www.luyenthi24h.com
www.VNMATH.com

1. Chng minh rng PQ S A.


2. Gi K l giao im ca MN v PQ, chng minh rng S K AD.
3. Qua Q dng cc ng thng Qx S C v Qy S B. Tm giao im ca Qx vi (S AB) v ca Qy vi (S CD).
Bi 10.35 : Cho hnh chp S .ABCD y l hnh thang, cc cnh y AD = a, BC = b. I, J ln lt l trng tm cc tam gic
S AD, S BC.
1. Tm on giao tuyn ca (ADJ) vi mt phng (S BC) v on giao tuyn ca (BCI) vi mt phng (S AD).
2. Tnh di on giao tuyn ca hai mt phng (ADJ) v (BCI) gii hn bi hai mt phng (S AB) v (S CD).
Bi 10.36 : Cho t din ABCD c tt c cc cnh bng a. I, J ln lt l trung im ca AC, BC. Gi K l mt im trn cnh BD vi
KB = 2KD.
1. Xc nh thit din ca t din vi mt phng (I JK). Chng minh thit din l hnh thang cn.
2. Tnh din tch thit din theo a.
Bi 10.37 : Cho hnh chp S .ABCD, y l hnh vung cnh a, tm O, mt bn S AB l tam gic u, S
AD = 900 . Gi Dx l ng
thng qua D v song song vi S C.
1. Tm giao im I ca Dx v mt phng (S AB). Chng minh AI S B.
2. Tm thit din ca hnh chp ct bi mt phng (AIC). Tnh din tch thit din.

10.3 ng thng v mt phng song song


Chng minh ng thng song song vi mt phng ; tm giao tuyn ca hai mt phng, dng thit din song song vi mt ng thng
; dng mt mt phng cha mt ng thng v song song vi ng thng khc, xc nh giao im ca ng thng vi mt phng ;
dng mt phng qua im M v song song vi hai ng thng cho nhau a v b

Vn 1 : Chng minh ng thng song song vi mt phng


1. Dng nh ngha (thng l phn chng).
2. Dng tiu chun : Nu mt ng thng d khng nm trn mt phng () v song song vi mt ng thng a no nm trn
() th ng thng d song song vi mt phng ().
Ch : Nu a khng c sn ta thng chn mt mt phng () cha d v ly a l giao tuyn ca () v ().

Bi 10.38 : Cho hai hnh bnh hnh ABCD v ABEF khng cng nm trong mt mt phng. Gi O, O ln lt l tm ca cc hnh
bnh hnh ABCD v ABEF; G1 , G2 ln lt l trng tm ca tam gic ABD v ABE. Chng minh rng :
1. OO song song vi mt phng (ADF) v (BCE) ;

2. G1G2 song song vi mt phng (CEF).

Vn 2 : Tm giao tuyn ca hai mt phng. Dng thit din song song vi mt ng thng

Ta c th dng nh l sau : Cho ng thng d (). Nu d () v () () = d th d d.

TRN ANH TUN - 0974 396 391 - (04) 66 515 343

WWW.VNMATH.COM

Trang 197

www.VNMATH.com

CHUYN LUYN THI I HC

www.luyenthi24h.com
www.luyenthi24h.com
www.VNMATH.com

Bi 10.39 : Cho hnh chp S .ABCD. M, N l hai im trn AB, CD v () l mt phng qua MN v song song vi S A.
1. Tm cc giao tuyn ca () vi (S AB) v (S AC).
2. Xc nh thit din ca hnh chp vi (). Tm iu kin ca MN thit din l hnh thang.
Bi 10.40 : Cho hnh lng tr ABC.A BC , y l tam gic u cnh a. Cc mt bn ABB A , ACC A l cc hnh vung. Gi I, J l
tm cc mt bn ni trn v O l tm ng trn ngoi tip tam gic ABC.
1. Chng minh rng I J (ABC).
2. Xc nh thit din ca lng tr vi mt phng (OI J). Chng minh rng thit din l hnh thang cn v tnh din tch thit din.
Bi 10.41 : Cho hnh chp S .ABCD, y l hnh bnh hnh tm O. Gi M, N ln lt l trung im S A v CD.
1. Chng minh rng (OMN) (S BC) ;
2. Gi AE, AF l cc ng phn gic trong ca cc tam gic ACD v S AB. Gi I l trung im S E, J l mt im trn (ABCD)
v cch u AB v CD. Chng minh rng I J (S AB).
3. Gi s hai tam gic S AD, ABC u cn ti A. Chng minh rng EF (S AD).

Vn 3 : Dng mt mt phng cha mt ng thng v song song vi ng thng khc


Xc nh giao im ca ng thng vi mt phng

Cho a, b cho nhau. Ta s dng mt phng (P) cha a v b (P) nh sau :

Cch 1 : Xt mt ng thng c ct a v c b. Khi (P) l mt phng cha a v c.


Cch 2 : Xt mt mt phng (Q) cha b, (R) cha a. Ta c (R) (P) = a, (Q) (R) = c v gi s c a = {M} th (P) (Q) l ng
thng d qua M v song song vi b.

Vy (P) l mt phng cha a v d.

Bi 10.42 : Cho hnh chp S .ABCD c y l hnh bnh hnh. Gi M l mt im nm gia hai im S v C ; () l mt phng cha
AM v song song vi BD.
1. Hy xc nh cc giao im E, F ca mt phng () ln lt vi cc cnh S B, S D.
2. Gi I l giao im ca ME v CB, J l giao im ca MF v CD. Chng minh rng ba im I, A, J thng hng.
Bi 10.43 : Cho hnh chp S .ABCD c y ABCD l hnh bnh hnh. Hy xc nh thit din ca hnh chp ct bi mt phng i qua
trung im M ca AB, song song vi cc ng thng BD v S A.
Bi 10.44 : Cho hnh chp S .ABCD c y l hnh bnh hnh. Gi M, N ln lt l trung im cc cnh AB, CD.
1. Chng minh MN song song vi (S BC) v (S AD) ;
2. Gi P l trung im S A. Chng minh S B v S C u song song vi mt phng (MNP).
3. Gi G1 , G2 ln lt l trng tm cc tam gic ABC v S BC. Chng minh G1G2 song song vi mt phng (S AB).
Bi 10.45 : Cho hnh chp S .ABCD, y l hnh thang vi cc cnh y AB, CD. Gi I, J ln lt l trung im AD, BC v G l trng
tm tam gic S AB.
1. Tm giao tuyn ca cp mt phng (S AB) v (I JG) ;
2. Xc nh thit din ca hnh chp ct bi mt phng (I JG). Tm iu kin ca AB, CD thit din l hnh bnh hnh.

TRN ANH TUN - 0974 396 391 - (04) 66 515 343

WWW.VNMATH.COM

Trang 198

www.VNMATH.com

CHUYN LUYN THI I HC

www.luyenthi24h.com
www.luyenthi24h.com
www.VNMATH.com

Bi 10.46 : Cho hnh chp S .ABCD c y l hnh bnh hnh. Gi I, J ln lt l trng tm cc tam gic S AB v S AD, M l trung
im CD. Xc nh thit din ca hnh chp ct bi mt phng (I JM).
Bi 10.47 : Cho hnh chp S .ABCD. Gi M, N l hai im trn AB, CD; () l mt phng qua M v song song vi S A.
1. Tm giao tuyn ca () vi cc mt phng (S AB) v (S AC).
2. Xc nh thit din ca hnh chp ct bi mt phng ();
3. Tm iu kin ca MN thit din l hnh thang.
Bi 10.48 : Cho lng tr ABC.A BC . Gi H l trung im A B .
1. Chng minh CB song song vi mt phng (AHC );
2. Tm giao im ca AC vi (BCH);
3. Mt phng () qua trung im ca CC , song song vi AH v CB . Xc nh thit din v t s m cc nh ca thit din chia
cc cnh tng ng ca lng tr.

10.4 Hai mt phng song song


Chng minh hai mt phng song song ; tm giao tuyn ca hai mt phng, thit din ct bi mt mt phng song song vi mt mt
phng cho trc.

Vn 1 : Chng minh hai mt phng song song


1. Dng nh ngha (thng l phn chng).
2. Chng minh chng cng song song vi mt mt phng th ba.
3. Dng tiu chun : Nu mt phng () cha hai ng thng a v b ct nhau v hai ng thng ny cng song song vi mt
mt phng () cho trc th hai mt phng () v () song song vi nhau.

Bi 10.49 : Cho hnh chp S .ABCD y l hnh bnh hnh tm O. Gi M, N ln lt l trung im ca S A, S D.


1. Chng minh rng (OMN) song song vi (S BC).
2. Gi P l trung im ca AB, Q trn on ON sao cho OQ = 3ON. Chng minh rng PQ (S BC).

Vn 2 : Tm giao tuyn ca hai mt phng

Thit din ct bi mt mt phng song song vi mt mt phng cho trc

Chng ta thng dng nh l : Nu hai mt phng () v () song song th mi mt phng () ct () u phi ct () v cc giao
tuyn ca chng song.

Bi 10.50 : Cho hnh chp S .ABCD c y l hnh bnh hnh tm O vi AC = a, BD = b, tam gic S BD u. Mt mt phng () di
ng song song vi mt phng (S BD) v i qua im I trn on AC khc A v C.
1. Xc nh thit din ca hnh chp vi mt phng ().

TRN ANH TUN - 0974 396 391 - (04) 66 515 343

WWW.VNMATH.COM

Trang 199

www.VNMATH.com

CHUYN LUYN THI I HC

www.luyenthi24h.com
www.luyenthi24h.com
www.VNMATH.com

2. Tnh din tch thit din theo a, b v x = AI.


Bi 10.51 : Cho hnh chp S .ABCD c y l hnh bnh hnh tm O. Gi M, N ln lt l trung im S A, S D.
1. Chng minh (OMN) (S BC) ;
2. Gi P, Q l trung im AB v ON. Chng minh PQ (S BC).
Bi 10.52 : Cho t din ABCD, gi G1 , G2 , G3 ln lt l trng tm cc tam gic ABC, ACD, ADB.
1. Chng minh rng (G1G2G3 ) (BCD).
2. Tm thit din ca t din ABCD vi mt phng (G1G2G3 ). Tnh din tch thit din, bit din tch tam gic l s.
3. M l im di ng trong t din sao cho G1 M lun song song vi mt phng (ACD). Tm tp hp nhng im M.

WWW.VNMATH.COM

TRN ANH TUN - 0974 396 391 - (04) 66 515 343

WWW.VNMATH.COM

Trang 200

www.luyenthi24h.com
www.luyenthi24h.com
www.VNMATH.com

www.VNMATH.com

Chng 11

Vect trong khng gian. Quan h vung gc


Sau khi hc xong chng ny, hc sinh cn bit :
1. c hai ng thng d v d vung gc, c th chng minh :
u .
v = 0,
u v
v ln lt l vect ch phng ca d v d .

Gc gia chng bng 90 .


d song song vi ng thng , cn d vung gc vi ( l ng thng no ).
d() m () cha d , hoc d () m () cha d.

Khi d v d ct nhau, c th s dng cc phng php trong hnh hc phng nh trung tuyn ca tam gic cn, nh l o
ca nh l Pytago, . . .
2. c ng thng d vung gc vi mt phng (), c th chng minh :

d vung gc vi hai ng thng ct nhau trong ().


d d m d ().

d() m () ().
d l trc ca tam gic ABC nm trn mt phng () (ngha l chng minh d cha hai im cch u A, B, C).
d l giao tuyn ca hai mt phng cng vung gc vi ().
S dng tnh cht hai mt phng vung gc : nu ()() m d nm trong () v d vung gc vi giao tuyn ca () v
() th d().
3. c hai mt phng vung gc, c th chng minh :

Gc gia chng bng 90 .


Mt phng ny cha ng thng vung gc vi mt phng kia.
Mt phng ny vung gc vi mt phng song song vi mt phng kia.
4. Ngoi ra, chng ta cn bit xc nh gc, xc nh khong cch gia cc yu t.
H thc lng trong tam gic vung
Cho tam gic ABC vung ti A, AH, AM tng ng l ng cao, trung tuyn xut pht t A.
A

C
H

201

WWW.VNMATH.COM

www.luyenthi24h.com
www.luyenthi24h.com

Chng 11

Vect trong khng gian. Quan h vung gc


Sau khi hc xong chng ny, hc sinh cn bit :
1. c hai ng thng d v d vung gc, c th chng minh :
u .
v = 0,
u v
v ln lt l vect ch phng ca d v d .

Gc gia chng bng 90 .


d song song vi ng thng , cn d vung gc vi ( l ng thng no ).
d() m () cha d , hoc d () m () cha d.

Khi d v d ct nhau, c th s dng cc phng php trong hnh hc phng nh trung tuyn ca tam gic cn, nh l o
ca nh l Pytago, . . .
2. c ng thng d vung gc vi mt phng (), c th chng minh :

d vung gc vi hai ng thng ct nhau trong ().


d d m d ().

d() m () ().
d l trc ca tam gic ABC nm trn mt phng () (ngha l chng minh d cha hai im cch u A, B, C).
d l giao tuyn ca hai mt phng cng vung gc vi ().
S dng tnh cht hai mt phng vung gc : nu ()() m d nm trong () v d vung gc vi giao tuyn ca () v
() th d().
3. c hai mt phng vung gc, c th chng minh :

Gc gia chng bng 90 .


Mt phng ny cha ng thng vung gc vi mt phng kia.
Mt phng ny vung gc vi mt phng song song vi mt phng kia.
4. Ngoi ra, chng ta cn bit xc nh gc, xc nh khong cch gia cc yu t.
H thc lng trong tam gic vung
Cho tam gic ABC vung ti A, AH, AM tng ng l ng cao, trung tuyn xut pht t A.
A

C
H

201

WWW.VNMATH.COM

www.VNMATH.comCHUYN LUYN THI I HC


AB2 + AC 2 = BC 2 (nh l Pytago);
1
1
1
AB.AC

=
+
; AH =
;
AH 2
AB2 AC 2
BC

www.luyenthi24h.com
www.luyenthi24h.com
www.VNMATH.com

AB2 = BH.BC; AC 2 = CH.BC;


BC
BC
, nu Cb = 30 th AB =
.
AM =
2
2

Nhc li mt s h thc lng trong tam gic.


Cho tam gic ABC c AB = c, BC = a, CA = b; ha , hb , hc v ma , mb , mc ln lt l cc ng cao v cc ng trung tuyn xut
a+b+c
pht t A, B, C; R, r tng ng l bn knh ng trn ngoi tip, ni tip tam gic; S l din tch tam gic ABC; v p =
l
2
na chu vi tam gic.
1. nh l hm s cosin :
a2 = b2 + c2 2bc cos A; cos A =
2. nh l hm s sin :

b 2 + c2 a 2
.
2bc

a
b
c
=
=
= 2R a = 2R sin A.
sin A sin B sin C

3. Cng thc trung tuyn :


m2a =

2(b2 + c2 ) a2
.
4

4. Cng thc din tch tam gic:


(a) Tam gic thng
S =

1
1
abc
2S
abc
S
a.ha = b.c. sin A =
= pr =
p(p a)(p b)(p c) ha =
,R =
,r = .
2
2
4R
a
4S
p

1
a2
AB.AC v nu l tam gic vung cn cnh a th S = .
2
2

a2 3
a 3
(c) Tam gic ABC u cnh a th S =
v ng cao bng
;
4
2

(b) Tam gic ABC vung ti A th S =

5. Din tch hnh vung cnh a l S = a2 .


6. Din tch hnh ch nht cnh a, b l S = ab.
=
7. Din tch hnh bnh hnh ABCD l S = y.cao = AB.AD. sin BAD
=
8. Din tch hnh thoi ABCD l S = y.cao = AB.AD. sin BAD

1
AC.BD. sin(AC, BD).
2

1
AC.BD.
2

( y ln + y nh ) cao
.
2
1
10. Din tch t gic c hai ng cho vung gc l S = tch hai ng cho.
2
9. Din tch hnh thang l S =

11.1 Vect trong khng gian. S ng phng ca cc vect


Vn 1 : Biu th mt vect qua ba vect khng ng phng


a ,
a ,
Nu ba vect
b , c khng ng phng th vect d bt k biu th c mt cch duy nht qua ba vect
b , c ; ngha l tn ti duy

nht b ba s m, n, p sao cho d = m a + n b + p c .



Bi 11.1 : Cho hnh hp ABCD.A BC D . t AA =
a , AB = b , AD =
c . Gi I l tm hnh bnh hnh CDD C , J l im trn


a ,

cnh B C sao cho JB = k.JC (k R cho trc). Hy biu th cc vect CB , AI, I J theo ba vect
b , c .


a =
Bi 11.2 : Cho lng tr tam gic ABC.A BC . t
AC , b = BA ,
c = CB . Gi M l trung im AA v G l trong tm tam gic

a ,
ABC. Hy biu din cc vect AA , BG, MN theo ba vect
b , c .

TRN ANH TUN - 0974 396 391 - (04) 66 515 343

WWW.VNMATH.COM

Trang 202

www.VNMATH.com

www.luyenthi24h.com
www.luyenthi24h.com
CHUYN LUYN THI I HCwww.VNMATH.com
Vn 2 : Chng minh cc ng thc vect


1. S dng quy tc ba im, quy tc hnh bnh hnh, quy tc hnh hp bin i v ny thnh v kia v ngc li.
2. S dng cc tnh cht ca cc php ton vect v cc tnh cht hnh hc ca hnh cho.


Bi 11.3 : Cho hnh hp ABCD.EFGH. Chng minh rng AB + AD + AE = AG.

Bi 11.4 : Cho hnh chp S .ABCD c y l hnh bnh hnh ABCD. Chng minh rng S A + S C = S B + S D.


Bi 11.5 : Cho hnh chp S .ABCD c y l hnh ch nht ABCD. Chng minh rng S A2 + S C 2 = S B2 + S D2 .
CA
m
Bi 11.6 : Cho on thng AB. Trn ng thng AB ly im C sao cho
= , vi m, n > 0. Chng minh rng vi S bt k ta
CB
n
n
m

lun c S C =
SA+
S B.
m+n
m+n
Bi 11.7 : Cho hnh lp phng ABCD.A BC D cnh a. Gi O v O theo th t l tm ca hai hnh vung ABCD v A BC D .



1. Hy biu din cc vect AO, AO theo cc vect AA , AB, AD.

2. Chng minh rng AD + D C + D A = AB.
Bi 11.8 : Trong khng gian cho im O v bn im A, B, C, D phn bit v khng thng hng. Chng minh rng iu kin cn v

bn im A, B, C, D to thnh mt hnh bnh hnh l : OA + OC = OB + OD.

Vn 3 : Chng minh cc im thng hng v quan h song song


1. chng minh ba im A, B, C thng hng ta c th

Chng minh vect hai AB v AC cng phng, tc l AB = k AC.

Chn mt im I no v chng minh IC = mOA + nOB vi m + n = 1.



2. Hai ng thng phn bit AB v CD song song vi nhau khi v ch khi hai vect AB v CD cng phng.
3. ng thng AB khng nm trn (P) v AB (P) khi v ch khi c mt ng thng CD (P) sao cho AB CD hoc

u + y
v trong cc vect
u v
v c gi song song hoc nm trn (P).
AB = x

Bi 11.9 : Cho hnh hp ABCD.A BC D . Xt cc im M, N ln lt trn cc ng thng AC v C D sao cho MA = k MC,


NC = lND (k v l u khc 1). t BA =
a , BB = b , BC =
c.

a ,
1. Hy biu th cc vect BM v BN qua cc vect
b , c .
2. Xc nh cc s k, l ng thng MN song song vi ng thng BD .

Bi 11.10 : Cho hnh hp ABCD.A BC D . M l mt im trn ng thng AB sao cho MA = mAB. Tm im N trn ng thng
BC v im P trn ng thng AC sao cho ba im M, N, P thng hng (m , 0).

Bi 11.11 : Cho t din ABCD, M v N l cc im ln lt thuc AB v CD sao cho MA = 2 MB, ND = 2NC. Cc im I, J, K

ln lt thuc AD, MN, BC sao cho IA = k ID, JM = k JN, KB = k KC. Chng minh rng cc im I, J, K thng hng.
Bi 11.12 : Cho hai ng thng , 1 ct ba mt phng song song (), (), () ln lt ti A, B, C v A1 , B1, C1 . Vi im O bt k

trong khng gian, t OI = AA1 , OJ = BB1, OK = CC1 . Chng minh rng ba im I, J, K thng hng.

TRN ANH TUN - 0974 396 391 - (04) 66 515 343

WWW.VNMATH.COM

Trang 203

www.VNMATH.comCHUYN LUYN THI I HC

www.luyenthi24h.com
www.luyenthi24h.com
www.VNMATH.com

Bi 11.13 : Cho t din ABCD. Gi B0 , C0 , D0 ln lt l trng tm cc tam gic ACD, ADB v ABC. Gi G v G0 l trng tm tam
gic BCD v B0C0 D0 . Chng minh rng ba im A, G0 , G thng hng.
1

Bi 11.14 : Cho hnh hp ABCD.A1 B1C1 D1 . M l im trn cnh AD sao cho AM = AD. N l im trn ng thng BD1 , P l
3



MN
im trn ng thng CC1 sao cho ba im M, N, P thng hng. Tnh .
NP
Bi 11.15 : Cho hnh hp ABCD.A BC D . Mt ng thng ct cc ng thng AA , BC, C D ln lt ti M, N, P sao cho
MA

.
N M = 2NP. Tnh
MA
Bi 11.16 : Cho hnh hp ABCD.A1 B1C1 D1 .
1. Chng minh rng nh A, trng tm G ca tam gic BDA1 v nh C1 thuc mt ng thng.
GA
2. Tnh t s
.
GC1
Bi 11.17 : Cho hnh hp ABCD.A BC D . Gi O l giao im ca hai ng cho ca mt phng ABB A . M l mt im trn OB .
Mt phng (MD C) ct BC I v DA J. Chng minh rng ba im I, M, J thng hng.
Bi 11.18 : Cho lng tr tam gic ABC.A BC . Gi G v G ln lt l trng tm cc tam gic ABC v A BC , gi I l giao im ca
hai ng thng AB v A B. Chng minh rng hai ng thng GI v GG song song vi nhau.
Bi 11.19 : Cho hnh lng tr tam gic ABC.A1 B1C1 , gi E, F l nhng im ln lt nm trn cc ng cho CA1 , AB1 ca cc mt
EF
, xc nh v tr ca E, F.
bn sao cho EF BC1 . Tm t s
BC1
Bi 11.20 : Cho hnh lng tr tam gic ABC.A1 B1C1 , im M l trung im cnh bn AA1 . Trn ng cho AB1 , BC1 ca cc mt
EF
, xc nh v tr ca E, F.
bn ln lt ly cc im E, F sao cho EF CM. Tm t s
CM
Bi 11.21 : Cho hnh lng tr tam gic ABC.A1 B1C1 . Gi M, N ln lt l trung im cnh bn AA1 , CC1 . Hai im E, F ln lt trn
EF
cc ng thng CM, AB1 sao cho EF BN. Tm t s
, xc nh v tr ca E, F.
BN
Bi 11.22 : Cho hnh lng tr tam gic ABC.A1 B1C1 . Gi M, N, P ln lt l cc im trn cc cnh bn AA1 , BB1, CC1 sao cho
AM
B1 N C 1 P 3
EF
=
=
= . Hai im E, F ln lt trn cc ng thng CM, A1 N sao cho EF B1 P. Tm t s
.
AA1
BB1 CC1 4
B1 P
Bi 11.23 : Cho hnh hp ABCD.A1 B1C1 D1 . Chng minh rng tn ti im M duy nht thuc ng thng AC v im N duy nht
MN
thuc DC1 sao cho MN BD1 . Tnh t s
.
BD1

Bi 11.24 : Cho hnh lp phng ABCD.A BC D . Gi M, N ln lt l cc im thuc AD v DB sao cho MA = k MD , ND = k NB


(k , 0, k , 1).
1. Chng minh rng MN (A BC) ;
2. Khi ng thng MN AC, chng minh rng MN vung gc vi AD v DB.
Bi 11.25 : Cho hnh hp ABCD.A BC D . Gi M, N ln lt l trung im CD v DD ; G, G ln lt l trng tm ca cc t din
A D MN v BCC D . Chng minh rng ng thng GG v mt phng (ABB A ) song song vi nhau.

Vn 4 : Chng minh cc vect ng phng


a ,
Mun chng minh cc vect
b , c ng phng chng ta c th :

a ,
1. Da vo nh ngha : Chng t cc vect
b , c c gi cng song song vi mt mt phng.

a ,
c = m
a + n
a ,
2. Ba vect
b , c ng phng khi v ch khi c cp s m, n duy nht sao cho
b , trong
b l hai vect khng
cng phng.

Bi 11.26 : Cho hnh hp ABCD.A BC D . Hy xt s ng phng ca cc vect :

TRN ANH TUN - 0974 396 391 - (04) 66 515 343

WWW.VNMATH.COM

Trang 204

www.VNMATH.com

1. AB, AC , B D ;

www.luyenthi24h.com
www.luyenthi24h.com
CHUYN LUYN THI I HCwww.VNMATH.com

2. AB, BB, BC ;


3. AB, B D, C D .

Bi 11.27 : Cho t din ABCD. Trn cnh AD ly im M sao cho AM = 3 MD v trn cnh BC ly im N sao cho NB = 3NC.

Chng minh rng ba vect AB, DC, MN ng phng.
Bi 11.28 : Cho hnh hp ABCD.EFGH. Gi I l giao im hai ng cho ca hnh bnh hnh ABFE v K l giao im hai ng

cho ca hnh bnh hnh BCGF. Chng minh rng ba vect BD, IK, GF ng phng.
Bi 11.29 : Cho t din ABCD. Gi P, Q ln lt l trung im ca cc cnh AB v CD. Trn cc cnh AC v BD ta ln lt ly cc
im M, N sao cho
BN
AM
=
= k (k > 0).
AC
BD

Chng minh rng ba vect PQ, PM, PN ng phng.

Bi 11.30 : Cho hai hnh bnh hnh ABCD v ABC D c chung nh A. Chng minh rng cc vect BB, CC , DD ng phng.
Bi 11.31 : Cho hai ng gic u OABCD v OA BC D c chung nh O v nm trn hai mt phng phn bit. Chng minh rng

cc vect AA , BB, CC , DD ng phng.
Bi 11.32 : Cho hnh lp phng ABCD.A1 B1C1 D1 . Cc im M, N ln lt thuc cc cnh AD v BB1 sao cho AM = BN. Chng

minh rng ba vect MN, AB, B1 D ng phng.
Bi 11.33 : Cho t din OABC. Gi M, N, P l ba im trong khng gian c xc nh t cc h thc vect sau :

OM = OA + OB 2OC; ON = ( + 1)OA + 2OB + OC; OP = ( 2)OB + 2OC



vi l s thc. Tm ba vect OM, ON, OP ng phng.
d zOx
d v phn gic ngoi ca xOy
d thuc
Bi 11.34 : Cho gc tam din Oxyz. Chng minh rng cc phn gic trong ca cc gc yOz,

mt mt phng.
Bi 11.35 : Cho hnh hp ABCD.A1 B1C1 D1 . Gi l mt phng i qua nh D1 song song vi DA1 v AB1 . Mt phng ny ct ng

thng BC1 ti M, v gi s BM = k BC1 . Hy tnh k ?


Bi 11.36 : Cho t din ABCD. Gi P, Q l trung im cc cnh AB v CD. R, S l hai im theo th t thuc hai cnh AC v BD sao
AR
BS
cho
=
. Chng minh rng bn im P, Q, R, S thuc mt mt phng.
AC
BD

Bi 11.37 : Cho lng tr ABC.A BC . Gi I v J ln lt l trung im ca BB v AC . im K thuc BC sao cho KC = 2KB .


Chng minh rng bn im A, I, J, K cng thuc mt mt phng.

Bi 11.38 : Cho t din ABCD ; I v J ln lt l trung im ca AB v CD ; M l im thuc AC sao cho MA = k1 MC ; N l im

thuc BD sao cho NB = k2 ND. Chng minh rng cc im I, J, M, N cng thuc mt mt phng khi v ch khi k1 = k2 .
1
2

Bi 11.39 : Cho t din ABCD. Ly cc im M, N, P, Q ln lt thuc AB, BC, CD, DA sao cho AM = AB, BN = BC, AQ =
3
3
1

AD, DP = k DC. Hy xc nh k bn im P, Q, M, N cng nm trn mt mt phng.


2

11.2 Hai ng thng vung gc


Vn 1 : Tnh gc gia hai vect



a ,
c bit
1. Dng trc tip nh ngha : Nu OA =
a , OB = b th (
b ) = (OA, OB) = AOB.

Gc gia hai vect chung gc hoc chung ngn tnh bi cng thc

(OA, OB) = (AO, BO) = AOB.

TRN ANH TUN - 0974 396 391 - (04) 66 515 343

WWW.VNMATH.COM

Trang 205

www.VNMATH.comCHUYN LUYN THI I HC

www.luyenthi24h.com
www.luyenthi24h.com
www.VNMATH.com

Gc gia hai vect c gc ca vect ny l ngn ca vect kia tnh bi cng thc


(AO, OB) = (OA, BO) = 180 (OA, OB) = 180 AOB.

u ,
v ) = u . v .
2. Dng h qu ca tch v hng : cos(

v |
| u |.|

Bi 11.40 : Cho t din u ABCD, gi H l trung im AB. Tnh gc gia cc cp vect sau:

1. AC v CD;


2. CH v CD.

Bi 11.41 : Cho hnh lp phng ABCD.A BC D . Tnh gc gia cc cp vect sau:



1. A C v AB;


2. AC v AB ;


3. A B v B D .

Bi 11.42 : Cho t din OABC c OA, OB, OC i mt vung gc v OA = OB = OC = 1. Gi M l trung im AB. Tnh gc gia

hai vect OM v BC.


Bi 11.43 : Cho hnh chp tam gic S .ABC c S A = S B = S C = AB = AC = a v BC = a 2. Tnh gc gia hai vect AB v S C.

Vn 2 : Tnh gc gia hai ng thng a v b


1. Dng trc tip nh ngha : Ly hai ng thng a v b cng i qua mt im ln lt song song hoc trng vi a v b. Gc
gia a v b bng gc gia a v b .
2. Tnh qua gc gia hai vect, c th

CD) = (AB, CD).


Nu (AB, CD) 90 th (AB,

Nu (AB, CD) > 90 th (AB,


CD) = 180 (AB, CD).


Nu tnh theo phng php vect th cos(AB, CD) = cos(AB, CD) .

Bi 11.44 : Cho hnh lp phng ABCD.A BC D . Tnh gc gia cc cp ng thng sau:


1. AC v DA ;

2. BD v AC .

Bi 11.45 : Cho t din OABC, c OA = OB = OC = a v OAOB, OBOC, OCOA. Gi M l trung im ca OB. Tnh csin gc
gia cc cp ng thng :
1. AM v BC ;

2. AM v OP, vi P l trung im BC.

Bi 11.46 : Cho hnh chp S .ABCD c y ABCD l hnh thoi, cnh bn S A = AB v S ABC.
1. Tnh gc gia hai ng thng S D v BC.
2. Gi I, J ln lt l cc im thuc S B v S D sao cho I J BD. Chng minh rng gc gia hai ng thng AC v I J khng
ph thuc vo v tr ca I v J.
Bi 11.47 : Cho t din ABCD c tt c cc cnh bng a, gi M l trung im ca BC. Tnh csin gc gia hai ng thng AB v
DM.

TRN ANH TUN - 0974 396 391 - (04) 66 515 343

WWW.VNMATH.COM

Trang 206

www.luyenthi24h.com
www.luyenthi24h.com
CHUYN LUYN THI I HCwww.VNMATH.com

www.VNMATH.com

Bi 11.48 : Cho t din ABCD. Gi M, N ln lt l trung im cc cnh BC, AD. Tnh gc gia hai ng thng AB v CD, bit

AB = CD = 2a v MN = a 3.
Bi 11.49 : Cho t din ABCD. Gi M l mt im trn cnh AB (M khng trng vi A v B). Tm v tr ca M mt phng qua M
v vung gc vi AC, BD ct t din theo thit din c din tch ln nht.

Vn 3 : Chng minh hai ng thng vung gc


Mun chng minh ABCD ta thng chng minh gc gia AB v CD bng 90 hoc chng minh AB.CD = 0.

Bi 11.50 : Cho hnh lp phng ABCD.A BC D . Gi M, N ln lt l trung im ca AD v BB. Chng minh rng MNA C.
Bi 11.51 : Cho t din ABCD c ABD l tam gic u cnh a, BCD l tam gic cn c CB = b, AC = c.

2. Tnh cosin gc gia hai vect AB, CD.

1. Chng minh rng ACBD ;

Bi 11.52 : Trn cc ng cho D1 A, A1 B, B1C, C1 D ca cc mt ca hnh lp phng ABCD.A1 B1C1 D1 ly cc im M, N, P, Q sao


cho :

D1 M = k D1 A; BN = k BA1 ; B1 P = k B1C; DQ = k DC1 .

Tm s thc k MNPQ.
Bi 11.53 : Cho t din u ABCD cnh a. Gi O l tm ng trn ngoi tip tam gic BCD. Chng minh rng OACD.
Bi 11.54 : Cho hnh lp phng ABCD.A BC D c cnh bng a. Trn cc cnh DC v BB ta ln lt ly cc im M, N khng
trng vi u mt sao cho DM = BN. Chng minh rng AC MN.

Bi 11.55 : Cho t gic ABCD. Gi M, N, P, Q ln lt l trung im cc on AC, BD, BC, AD v c MN = PQ. Chng minh rng
ABCD.
Bi 11.56 : Cho hnh hp ABCD.A BC D c tt c cc cnh u bng nhau. Chng minh rng ACB D . Chng minh rng nu
BA = B
BC = 60 th A BCD l hnh vung.
=B

ABC

Bi 11.57 : Cho t din ABCD. Ly im M, N ln lt thuc cc ng thng BC, AD sao cho MB = k MC v NA = k ND, vi k l


s thc khc 0 cho trc. t = ( MN, BA), = ( MN, CD). Tm mi lin h gia AB v CD = = 45 .
Bi 11.58 : Cho hai tam gic cn ABC v DBC c chung cnh y BC v nm trong hai mt phng khc nhau.
1. Chng minh rng ADBC.

2. Gi M, N l cc im ln lt thuc cc ng thng AB, DB sao cho MA = k MB, ND = k NB. Tnh gc gia hai ng thng
MN v BC.
4
5
AB. Gi I, J, K ln lt l trung im ca BC, AC, BD. Bit JK = AB, tnh gc gia cc
3
6
ng thng CD vi cc ng thng I J v AB.
Bi 11.59 : Cho t din ABCD c CD =

Bi 11.60 : Cho t din ABCD c BC = AD = a, AC = BD = b, AB = CD = c. t , , l gc gia BC v AD, AC v BD, AB v


CD. Chng minh rng trong ba s hng a2 cos , b2 cos , c2 cos c mt s hng bng tng hai s hng cn li.

11.3 ng thng vung gc vi mt phng


Vn 1 : Chng minh ng thng a vung gc vi mt phng (P)

1. Chng minh ng thng a vung gc vi hai ng thng phn bit ct nhau v nm trong (P).

TRN ANH TUN - 0974 396 391 - (04) 66 515 343

WWW.VNMATH.COM

Trang 207

www.VNMATH.comCHUYN LUYN THI I HC

www.luyenthi24h.com
www.luyenthi24h.com
www.VNMATH.com

2. Chng minh ng thng a song song vi ng thng b m b vung gc vi (P).


3. Chng minh ng thng a vung gc vi (Q) m (Q) song song vi (P).

Bi 11.61 : Cho hnh chp S .ABC c y l tam gic vung cn ti A, cnh bn S A vung gc vi y. Gi I l trung im BC.
1. K ng thng qua A vung gc vi S I ti H. Chng minh rng AH(S BC).
2. Gi G1 , G2 ln lt l trng tm cc tam gic ABC v S BC. Chng minh rng G1G2 (ABC).
Bi 11.62 : Cho hnh chp S .ABCD c y l hnh thoi v S A = S C.
1. Chng minh rng AC(S BD).
2. K ng thng qua S vung gc vi (ABCD) ti I. Chng minh rng I cch u A v C.
B = 90 , BS

C = 60 , AS
C = 120. Gi O l trung im cnh AC.
Bi 11.63 : Cho hnh chp S .ABC c S A = S B = S C = a, AS

Chng minh rng S O(ABC).


Bi 11.64 : Cho t din ABCD c ABC v DBC l hai tam gic u, gi I l trung im cnh BC.
1. Chng minh rng BC(AID).
2. V ng cao AH ca tam gic AID. Chng minh rng AH(BCD).

Bi 11.65 : Cho hnh chp S .ABCD, y ABCD l hnh ch nht c AB = a, BC = a 3, mt bn S BC vung ti B, mt bn S CD

vung ti D v c S D = a 5.
1. Chng minh rng S A(ABCD) v tnh S A.
2. Mt phng qua A vung gc vi AC, ct cc ng thng CB, CD ti I, J. gi H l hnh chiu vung gc ca A trn S C. Hy
xc nh cc giao im K, L ca S B, S D vi (HI J). Chng minh rng AK(S BC), AL(S CD).
3. Tnh din tch t gic AKHL.
Bi 11.66 : Cho tam gic ABC. Gi () l mt phng vung gc vi ng thng CA ti A v () l mt phng vung gc vi ng
thng CB ti B. Chng minh rng hai mt phng () v () ct nhau v giao tuyn ca chng vung gc vi mt phng (ABC).
Bi 11.67 : Cho hnh chp S .ABC c S A = S B = S C. Gi O l tm ng trn ngoi tip tam gic y ABC. Chng minh rng
S O(ABC). Hy tng qut ha bi ton.
= 120, ng thi S A = S B = S C = 2a.
Bi 11.68 : Cho hnh chp S .ABC c y ABC l tam gic cn, c AB = AC = a v BAC

Gi D l im i xng ca A qua trung im ca BC.


1. Chng minh rng BC(S AD);

2. Tnh gc gia S B v (ABC).

Bi 11.69 : Cho hnh chp S .ABCD c y ABCD l hnh thang vung (Ab = 90 ), y ln AD = 2a v AB = BC = a, ng thi
S A = S C = S D. Gi M l trung im AD. Chng minh rng S M(ABCD) v AC(S BM).

Vn 2 : Chng minh hai ng thng vung gc vi nhau


1. Chng minh ng thng ny vung gc vi mt mt phng cha ng thng kia.
2. Dng nh l ba ng vung gc : Cho ng thng a khng vung gc vi mt phng (P) v ng thng b nm trong (P).
Khi , iu kin cn v b vung gc vi a l b vung gc vi hnh chiu a ca a trn (P).
3. Chng minh ng thng ny vung gc vi mt mt phng song song vi ng thng kia.

TRN ANH TUN - 0974 396 391 - (04) 66 515 343

WWW.VNMATH.COM

Trang 208

www.VNMATH.com

www.luyenthi24h.com
www.luyenthi24h.com
CHUYN LUYN THI I HCwww.VNMATH.com

Bi 11.70 : Cho hnh chp S .ABCD c y l hnh vung tm O v c cnh S A(ABCD). Gi H, I, K ln lt l hnh chiu vung
gc ca im A trn cc cnh S B, S C, S D.
1. Chng minh rng BC(S AB), CD(S AD), BD(S AC).
2. Chng minh rng S C(AHK) v im I (AHK).
3. Chng minh rng HK(S AC), t suy ra HKAI.
Bi 11.71 : Hnh chp S .ABCD c y l hnh thoi ABCD tm O v c S A = S C, S B = S D.
1. Chng minh rng S O(ABCD).
2. Gi I, K ln lt l trung im cc cnh BA, BC. Chng minh rng IK(S BD) v IKS D.
Bi 11.72 : Cho t din ABCD. Chng minh cc cp cnh i din ca t din ny vung gc vi nhau tng i mt.
Bi 11.73 (Bi ton c bn) : Cho t din OABC c ba cnh OA, OB, OC i mt vung gc vi nhau. K OH vung gc vi mt
phng (ABC) ti H. Chng minh rng:
1. OABC, OBCA, OCAB.
2. H l trc tm ca tam gic ABC.
3.

1
1
1
1
=
+
+
.
OH 2 OA2 OB2 OC 2

4. Tam gic ABC nhn


5. sin2 + sin2 + sin2 = 1, trong , , l gc gia cc ng thng OA, OB, OC vi mt phng (ABC).
2
2
2
2
6. S ABC
= S OAB
+ S OBC
+ S OCA
.

Bi 11.74 : Hnh chp S .ABCD c y l hnh ch nht ABCD v c cnh bn S A vung gc vi mt phng y. Chng minh cc
mt bn ca hnh chp cho l cc tam gic vung.
Bi 11.75 : Cho chp S .ABC c tam gic ABC vung ti B, S A(ABC).
1. Chng minh rng BC(S AB).
2. Gi AH l ng cao ca tam gic S AB. Chng minh rng AHS C.
Bi 11.76 : Cho hnh chp S .ABCD, y l hnh vung cnh a. Mt bn S AB l tam gic u, S CD l tam gic vung cn nh S .
Gi I, J ln lt l trung im AB, CD.
1. Tnh cc cnh ca tam gic S I J v chng minh rng S I(S CD), S J(S AB).
2. Gi H l hnh chiu vung gc ca S trn I J. Chng minh rng S HAC.
3. Gi M l mt im thuc ng thng CD sao cho BMS A. Tnh AM theo a.

Bi 11.77 : Cho hnh chp S .ABCD c y ABCD l hnh vung cnh a, mt bn S AB l tam gic u v S C = a 2. Gi H, K l
trung im AB, AD.
1. Chng minh rng S H(ABCD) ;

2. Chng minh rng ACS K, CKS D.

Bi 11.78 : Cho hnh lng tr tam gic ABC.A BC . Gi H l trc tm tam gic ABC v bit rng A H(ABC). Chng minh rng
1. AA BC v AA BC .
2. Gi MM l giao tuyn ca mt phng (AHA ) vi mt bn BCC B , trong M BC v M BC . Chng minh rng t gic
BCC B l hnh ch nht v MM l ng cao ca hnh ch nht .

TRN ANH TUN - 0974 396 391 - (04) 66 515 343

WWW.VNMATH.COM

Trang 209

www.VNMATH.comCHUYN LUYN THI I HC

www.luyenthi24h.com
www.luyenthi24h.com
www.VNMATH.com

Bi 11.79 : Cho hnh chp S .ABC c y ABC l tam gic vung ti A v c cnh bn S A vung gc vi mt y l (ABC). Gi D l
im i xng ca B qua trung im O ca cnh AC. Chng minh rng CDCA, CD(S CA).
Bi 11.80 : Cho hnh chp S .ABCD c y l hnh vung cnh a v S A = a, hnh chiu vung gc ca S xung mt phng (ABCD)
trng vi trung im M ca cnh AB; gi N l trung im AD.
1. Chng minh rng BC(S AB) v CN(S D).
2. Tnh gc gia hai ng thng S D v AC.
Bi 11.81 : Cho hnh chp S .ABCD c ABCD l hnh ch nht v S A = S B. Chng minh rng CD(S I J), trong I, J tng ng l
trung im ca AB v CD.
Bi 11.82 : Cho hnh chp S .ABC c y l tam gic vung ti A; S A(ABC) v H thuc cnh AC v tha mn S H 2 = HA.HC.
Chng minh rng S C(S AB).

A = 60 ; AS
B = 90 v S A = S B = S C. Chng minh rng ABC l tam gic
Bi 11.83 : Cho hnh chp S .ABC c BS
C = 120; CS

vung v S I(ABC), trong I l trung im ca BC.

Vn 3 : Xc nh gc gia ng thng a v mt phng (P)


1. S dng nh ngha : Nu a khng vung gc vi (P) th gc gia a v (P) bng gc gia a v hnh chiu vung gc a ca a
trn mt phng (P).
2. Nu a (P) hoc a (P) th gc gia a v (P) bng 0 .
3. Nu a(P) th gc gia a v (P) bng 90 .
4. Nu a khng vung gc vi (P) v ct (P) ti A, ta chn mt im B trn a (B khng trng vi A) v xc nh hnh chiu vung

gc H ca B ln (P). Khi gc gia a v (P) bng BAH.

a
B

(P)

Bi 11.84 : Cho hnh chp t gic S .ABCD c y l hnh vung cnh a, cnh bn S A vung gc vi y v S A = a. Tnh gc gia
ni cnh bn ca hnh chp vi mt y.

Bi 11.85 : Cho hnh chp t gic S .ABCD c y l hnh vung cnh a, cnh bn S A vung gc vi y v S A = a 6. Tnh gc
gia
1. S C v (ABCD);

2. S C v (S AB);

3. S B v (S AC);

4. AC v (S BC).

Bi 11.86 : Cho lng tr u ABC.A BC c cnh y bng a, cnh bn AA = a 2.


1. Tnh gc gia ng thng BC v (ABB A ).

TRN ANH TUN - 0974 396 391 - (04) 66 515 343

WWW.VNMATH.COM

Trang 210

www.VNMATH.com

www.luyenthi24h.com
www.luyenthi24h.com
CHUYN LUYN THI I HCwww.VNMATH.com

2. Gi M l trung im CC . Tnh tang ca gc gia ng thng BM v (A BC ).

Bi 11.87 : Cho tam gic ABC cn ti A, c Ab = 120, BC = a 3. Ly im D ngoi mt phng cha tam gic sao cho DA = a.
Gi O l tm ng trn ngoi tip tam gic DBC.
1. Chng minh rng AO(DBC).
= 90 .
2. Tnh gc gia ng thng DA v mt phng (BCD) khi BDC

Bi 11.88 : Cho hnh chp S .ABCD c y l hnh vung cnh a v c tm O, bit S A(ABCD). Gi M v N ln lt l trung im
cc cnh S A v BC. Bit rng gc gia MN v (ABCD) bng 60 .
1. Tnh di MN v S O;

2. Tnh gc gia ng thng MN v mt phng (S BD).

= . Bit S A, S B, S C u hp vi mt phng (ABC)


Bi 11.89 : Cho hnh chp S .ABC c ABC l tam gic cn, AB = AC = a, BAC

mt gc .
1. Chng minh rng hnh chiu vung gc ca S trn mt phng (ABC) l tm ng trn ngoi tip tam gic ABC.
2. Tnh khong cch t S n mt phng (ABC).
Bi 11.90 : Cho lng tr u ABC.A BC c cnh y bng a. ng cho BC ca mt bn BCC B hp vi ABB A gc 30 .
1. Tnh AA .
2. Tnh khong cch t trung im M ca AC n mt phng (BAC ).
3. Gi N l trung im ca cnh BB. Tnh gc gia MN v mt phng (BAC ).
Bi 11.91 : Cho lng tr ABC.A BC c y ABC vung cn ti A, AA vung gc vi mt phng (ABC). on ni trung im M ca
AB v trung im N ca BC c di bng a, MN hp vi y gc v mt bn (BCC B ) gc .
1. Tnh cc cnh y v cnh bn ca lng tr theo a v ;

2. Chng minh rng cos =

2 sin .

Bi 11.92 : Cho hnh chp S .ABCD c y l hnh vung cnh a, S A(ABCD) v S A = 2a. Mt phng () qua BC hp vi AC gc
30 , ct S A, S D ln lt ti M v N. Tnh din tch t gic BCN M.
Bi 11.93 : Cho hnh chp S .ABC c cc cnh bn S A, S B, S C cng to vi y mt gc . Gi O l tm ng trn ngoi tip tam
gic y ABC. Chng minh rng S O(ABC). Hy tng qut ha bi ton.

Bi 11.94 : Cho hnh chp S .ABC c y ABC l tam gic vung ti A, c AB = a, AC = a 3. Cc cnh bn S A, S B, S C cng to
vi y mt gc 60 . Tnh gc to bi
1. S A v (S BC);

2. S A v BC.

Bi 11.95 : Cho hnh chp S .ABCD c y ABCD l hnh ch nht, c AB = a, AD = a 2.Cc cnh bn S A, S B, S C, S D cng to
vi y mt gc 45 . Gi M l trung im AD.
1. Chng minh rng BMS A;

2. Tnh gc gia BM v S C.

Vn 4 : Dng mt phng qua im M cho trc v vung gc vi mt ng thng d cho trc

Gi () l mt phng i qua M v vung gc vi d.


1. Dng hai ng thng ct nhau cng vung gc vi d v c t nht mt ng thng qua im M. Mt phng xc nh bi hai
ng thng ni trn chnh l mt phng ().

TRN ANH TUN - 0974 396 391 - (04) 66 515 343

WWW.VNMATH.COM

Trang 211

www.VNMATH.comCHUYN LUYN THI I HC

www.luyenthi24h.com
www.luyenthi24h.com
www.VNMATH.com

2. Nu c sn hai ng thng ct nhau hay cho nhau a, b cng vung gc vi d th chn () a (hoc cha a) v () b (hoc
cha b).

Bi 11.96 : Cho chp S .ABCD c y ABCD l hnh thang vung ti A v B, vi AB = BC = a, AD = 2a, S A(ABCD) v S A = 2a.
Gi M l im trn cnh AB, () l mt phng qua M, vung gc vi AB. t AM = x (0 < x < a).
1. Tm thit din ca hnh chp S .ABCD vi (). Thit din l hnh g?
2. Tnh din tch thit din theo a v x. Tm v tr ca M trn cnh AB thit din c din tch ln nht.
Bi 11.97 : Cho t din S ABC c ABC l tam gic u cnh bng a, S A(ABC) v S A = 2a. Gi () l mt phng qua B v vung
gc vi S C. Tm thit din ca din S ABC vi () v tnh din tch ca thit din ny.
Bi 11.98 : Cho hnh t din S ABC c tam gic ABC l tam gic u cnh a, S A(ABC), S A = a. Tm thit din ca t din S ABC
vi mt phng () v tnh din tch thit din trong cc trng hp sau:
1. () qua S v vung gc vi BC.
2. () qua A v vung gc vi trung tuyn S I ca tam gic S BC.
3. () qua trung im M ca S C v vung gc vi BC.
Bi 11.99 : Cho hnh chp t gic S .ABCD c y l hnh vung cnh a, S AB l tam gic u nm trn mt phng vung gc vi y.
M l trng tm ca tam gic BCD, () i qua M v vung gc vi AB, () i qua M v vung gc vi CJ (J l im gia on AB).
Hy xc nh v tnh din tch cc thit din ca hnh chp ct bi cc mt phng () v ().
Bi 11.100 : Cho hnh chp tam gic S .ABC c y l tam gic u ABC cnh a, S A = 2a. Cc mt phng (S AC) v (S BC) cng
vung gc vi (ABC) v M l trung im ca cc cnh AB. Xc nh v tnh din tch thit din ca hnh chp khi ct bi
1. mt phng qua M v vung gc vi AB.

2. mt phng qua M v vung gc vi S C.

Bi 11.101 : Cho hnh chp tam gic S .ABC c S A = S B = S C = AB = AC = BC = a, M l mt im thuc on AB sao cho
AM = x (vi 0 < x < a). Xc nh v tnh thit din ca hnh chp khi ct bi mt phng qua M v vung gc vi S A. Tm v tr ca
M din tch thit din l ln nht.
Bi 11.102 : Cho hnh lp phng ABCD.A BC D c cnh bng a. Hai im M, N ln lt l trung im ca AB, CC . Hy xc nh
v tnh din tch thit din ca hnh lp phng ct bi mt phng trung trc ca MN.
= 600 . Cnh S C = a v vung gc vi
Bi 11.103 : Cho hnh chp S .ABC, trong ABC l tam gic vung ti A, vi AB = a, ABC

(ABC). Gi s M l mt im trn on S A sao cho AM = x (M khng trng vi A v S ). Xc nh v tnh din tch thit din ca
hnh chp khi ct bi mt phng qua M v vung gc vi S A. Tm v tr ca M thit din c din tch ln nht.

Bi 11.104 : Cho lng tr ng OAB.O A B c y l tam gic vung cn ti O vi OA = OB = a, chiu cao AA = a 2. Gi M l


trung im ca OA, () l mt phng qua M v vung gc vi A B. Hy xc nh v tnh din tch thit din ca lng tr ct bi ().
Bi 11.105 : Cho hnh chp S .ABCD c y ABCD l hnh vung cnh a, S A(ABCD). Qua A xc nh mt phng () vung gc

vi S C ct S B, S C, S D ln lt ti E, K, H. Tnh din tch thit din ca hnh chp ct bi () khi S A = a 2.


Bi 11.106 : Trong mt phng (P) v hnh thoi to bi hai tam gic u ABD v CBD c cnh bng a. V ng thng vung gc vi
mt phng (P) ti A v
ly trn im S sao cho AS = a. T M trn ng cho AC ca hnh thoi, ta v mt phng (Q) vung gc
x 3
vi AC. t CM =
.
2
1. Ty theo x, kho st hnh dng ca thit din ca hnh chp ct bi (Q). Tnh din tch ca thit din.
2. Tm x din tch thit din t gi tr ln nht.
= 600 . Cnh S C = a v vung gc vi
Bi 11.107 : Cho hnh chp S .ABC, trong ABC l tam gic vung ti A, vi AB = a, ABC

(ABC). Xc nh thit din ca hnh chp ct bi mt phng qua M S A v vung gc vi S A. t AM = x. Tnh din tch thit din

v xc nh v tr ca M thit din c din tch ln nht.

TRN ANH TUN - 0974 396 391 - (04) 66 515 343

WWW.VNMATH.COM

Trang 212

www.VNMATH.com

www.luyenthi24h.com
www.luyenthi24h.com
CHUYN LUYN THI I HCwww.VNMATH.com

11.4 Hai mt phng vung gc


Vn 1 : Xc nh gc gia hai mt phng

Gi s cn tnh gc gia hai mt phng (P) v (Q), ta c cc phng php sau :


1. S dng nh ngha : Gc gia hai mt phng l gc gia hai ng thng ln lt vung gc vi hai mt phng . Ngha l,
ly a(P) v b(Q) th gc gia (P) v (Q) l gc gia a v b.
2. Gi s c = (P) (Q). Xt mt phng (R) vung gc vi c, ln lt ct (P) v (Q) theo cc giao tuyn a v b. Lc , gc gia
(P) v (Q) bng gc gia hai ng thng a v b.

Trong nhiu bi ton thng c sn ng thng AB (A (P) v B (Q)) vung gc vi c, ta ch cn k AH vung gc vi c

ti H. Lc ny mt phng (R) chnh l mt phng (ABH) v gc


l gc
AHB (nu AHB 90 ) v l gc 180 AHB (nu

> 90 ). Trong thc hnh thng dng cng thc cos = cos AHB
.
AHB

3. S dng nh l hnh chiu : Gi s a gic H nm trong mt phng (P) c hnh chiu ln mt phng (Q) l a gic H . Khi
S

, cos =
vi S l din tch hnh H v S l din tch hnh H .
S

Bi 11.108 : Cho hnh chp S .ABCD c y l hnh vung cnh a, S A(ABCD) v S A = a 3. Tnh gc gia cc cp mt phng sau
1. (S BC) v (ABCD);

2. (S CD) v (ABCD);

3. (S BC) v (S CD).

= 90 , AB = 2a, BC = a 3, S A = 2a v S A(ABC).
Bi 11.109 : Cho t din S ABC c ABC
1. Tnh gc gia hai mt phng (ABC) v (S BC).
2. Mi M l trung im ca AB. Tnh di ng cao AK ca tam gic AMC.
3. Tnh tan , vi l gc gia hai mt phng (ABC) v (S MC).
Bi 11.110 : Cho hnh lp phng ABCD.A BC D . Tnh gc gia hai mt phng
1. (ABCD) v (A BC D );

2. (ABCD) v (CDDC );

3. (ACC A ) v (ABB A );

4. (A BD) v (ABCD).

Bi 11.111 : Cho hnh chp S .ABCD c y l hnh vung cnh a, S A(ABCD) v S A = x.


1. Xc nh x hai mt phng (S BC) v (S DC) to vi nhau gc 60 .
2. Vi x c xc nh t trn, hy tnh gc gia hai mt phng (S BC) v (S AD).

= 120 . Gi M l trung im cnh CC1 .


Bi 11.112 : Cho lng tr ng ABC.A1 B1C1 c AB = a, AC = 2a, AA1 = 2a 5 v BAC
Chng minh rng MBMA1 v tnh khong cch t im A ti mt phng (A1 BM).
Bi 11.113 : Cho hnh chp S .ABCD c y ABCD l na lc gic u ni tip ng trn ng knh AB = 2a, S A(ABCD) v

S A = a 3. Tnh gc gia cc cp mt phng sau:


1. (S AD) v (S BC);

2. (S CD) v (S BC).

Bi 11.114 : Cho hnh chp S .ABC c y ABC l tam gic vung cn, vi AB = BC = a, S A(ABC), S A = a. Gi E v F ln lt
l trung im cc cnh AB v AC. Tnh gc gia cc cp mt phng sau:
1. (S AC) v (S BC);

TRN ANH TUN - 0974 396 391 - (04) 66 515 343

2. (S EF) v (S BC).

WWW.VNMATH.COM

Trang 213

www.VNMATH.comCHUYN LUYN THI I HC

www.luyenthi24h.com
www.luyenthi24h.com
www.VNMATH.com

d = 90 , yOz
d = zOx
d = 60 . Tnh gc gia hai mt phng (yOz) v
Bi 11.115 : Cho ba tia Ox, Oy, Oz khng ng phng sao cho xOy

(zOx).
Bi 11.116 : Trong mt phng () cho ng trn (C ) tm O bn knh R. Trn ng thng vung gc vi () ti O ly im S sao
cho OS = R. Gi M v N l hai im khc nhau trn (C ), a v b l hai tip tuyn vi (C ) ti M v N. Tnh gc gia hai mt phng
(S , a) v (S , b) trong mi trng hp sau :
1. MN l ng knh ca ng trn;

= 90 .
2. MON

Bi 11.117 : Cho hnh chp S .ABCD c y l hnh bnh hnh tm O. Cc mt phng (S AB) v (S CD) l cc tam gic vung ln lt
= .
ti A v C, cng hp vi y mt gc , bit ABC

1. Chng minh rng S O(ABCD);


2. Chng minh (S BC) v (S AD) cng hp vi y (ABCD) mt gc tha mn cot = cot cos .
= , S A(ABC) v S A = a. Gi l gc gia
Bi 11.118 : Cho hnh chp S .ABC, y ABC l tam gic vung ti B, AB = a, BAC

hai mt bn (S AC) v (S BC).

1 + cos2
1. Chng minh rng tan . tan =
;
cos

2. Tam gic ABC tha mn iu kin g = 60 .

Bi 11.119 : Cho t din OABC c OA, OB, OC i mt vung gc. Gi , , ln lt l gc to bi cc mt phng (OBC), (OCA), (OAB)
vi mt phng (ABC). Chng minh rng cos2 + cos2 + cos2 = 1.
= 60 . Gi M, N ln lt l trung im
Bi 11.120 : Cho lng tr ng ABCD.A BC D c y ABCD l hnh thoi cnh a, gc BAD

cc cnh AA v CC .
1. Chng minh bn im B , M, D, N ng phng. T gic B MDN l hnh g ?
2. Tnh di AA theo a t gic B MDN l hnh vung.
3. Khi t gic B MDN l hnh vung, hy tnh gc gia hai mt phng (B MDN) v (ABCD).

Vn 2 : Chng minh hai mt phng (P) v (Q) vung gc


1. Chng minh gc gia (P) v (Q) bng 90 .
2. Chng minh (P) cha ng thng a, trong a(Q).

Bi 11.121 : Cho tam gic u ABC cnh a, I l trung


im BC, D l im i xng ca A qua I. Trn ng thng vung gc vi
a 6
mt phng (ABCD) ti D, ly im S sao cho S D =
. Chng minh rng (S BC)(S AD) v (S AB)(S AC).
2
Bi 11.122 : Cho hnh chp S .ABCD c y l hnh vung, S A(ABCD).
1. Chng minh rng (S AC)(S BD).
2. Tnh gc gia hai mt phng (S AB) v (S BC).
3. Gi BE, DF l hai ng cao ca tam gic S BD. Chng minh rng (ACF)(S BC), (AEF)(S AC).

a 3
a 6
Bi 11.123 : Cho hnh chp S .ABCD c y ABCD l hnh thoi tm O cnh a, OB =
, S O(ABCD), S O =
.
3
3

1. Chng minh rng AS


C = 90 .

2. Chng minh rng (S AB)(S AD).

TRN ANH TUN - 0974 396 391 - (04) 66 515 343

WWW.VNMATH.COM

Trang 214

www.luyenthi24h.com
www.luyenthi24h.com
CHUYN LUYN THI I HCwww.VNMATH.com

www.VNMATH.com

3. Tnh gc to bi hai mt phng (S BC) v (ABC).


Bi 11.124 : Cho hnh chp S .ABCD c y l hnh vung cnh a, cnh bn S A vung gc vi y. Gi M, N ln lt l hai im
a
3a
nm trn BC, DC sao cho BM = ; DN =
. Chng minh rng (S AM)(S MN).
2
4
Bi 11.125 : Cho hnh chp tam gic u S .ABC c y l tam gic u cnh a. Tnh ng cao S O theo a hai mt phng (S AB)
v (S AC) vung gc vi nhau.
Bi 11.126 : Cho hnh vung ABCD tm O v c cnh bng a. Trn hai tia Bx v Dy cng vung gc vi mt phng (ABCD) cng
a2
= , DON
= .
na mt phng (ABCD) ly hai im M, N sao cho BM.DN = . t BOM
2
1. Chng minh rng tan . tan = 1. C kt lun g v hai gc ny ? Chng minh rng (ACM)(ACN).
2. Gi H l hnh chiu vung gc ca O ln MN. Tnh di on OH. T chng minh AHHC v (AMN)(CMN).

Vn 3 : Chng minh ng thng a vung gc vi mt phng (P)


1. Ly mt phng (Q) cha a m (Q)(P), (P) (Q) = c ri chng minh ac.
2. Chng minh a l giao tuyn ca hai mt phng ct nhau cng vung gc vi (P).

Bi 11.127 : Cho tam gic ABC vung ti B. Mt on AD vung gc vi mt phng (ABC). T im A trong mt phng (ABD) ta v
AH vung gc vi BD, chng minh AH(BCD).
Bi 11.128 : Cho hnh t din ABCD c hai mt (ABC), (ABD) cng vung gc vi mt (DBC). V cc ng cao BE, DF ca tam
gic BCD, v ng cao DK ca tam gic ACD.
1. Chng minh rng AB(BCD).
2. Chng minh rng (ABE)(ADC), (DFK)(ADC).
3. Gi O v H ln lt l trc tm ca hai tam gic BCD v ACD. Chng minh rng OH(ADC).

Bi 11.129 : Cho hnh chp S .ABCD c y l hnh ch nht, cc cnh AB = a 2, AD = a, tam gic S AB u v nm trong mt
phng vung gc vi y. Gi M l trung im AB.
1. Chng minh rng S M(ABCD); BC(S AB) v ACS D.

2. Tnh gc gia hai mt phng (ABCD) v (S CD).

Bi 11.130 : Cho hnh vung ABCD. Gi S l im trong khng gian sao cho S AB l tam gic u v (S AB)(ABCD).
1. Chng minh rng (S AB)(S AD) v (S AB)(S BC).
2. Tnh gc gia hai mt phng (S AD) v (S BC).
3. Gi H v I ln lt l trung im ca AB v BC. Chng minh rng (S HC)(S DI).
Bi 11.131 : Cho hnh chp S .ABC c y ABC l tam gic u c cnh bng a, tam gic S AB vung ti S v c S
AB = 30 . Tnh
gc gia mt phng (ABC) v (S BC).
= 60 , M l trung im AB. Cc mt phng (S AB) v (S CM)
Bi 11.132 : Cho hnh chp S .ABC c tam gic ABC vung ti A, ABC

cng vung gc vi mt phng (ABC). Bit gc gia S C v (ABC) l 60 , tnh gc gia hai mt phng (S BC) v (ABC).
Bi 11.133 : Cho lng tr ABC.A BC c y l tam gic vung cn ti B. Hai mt phng (ABB A ) v (ACB ) cng vung gc vi
(ABC).
1. Chng minh rng BCC B l hnh ch nht.

TRN ANH TUN - 0974 396 391 - (04) 66 515 343

WWW.VNMATH.COM

Trang 215

www.VNMATH.comCHUYN LUYN THI I HC

www.luyenthi24h.com
www.luyenthi24h.com
www.VNMATH.com

2. Bit gc gia hai mt phng (BCC B ) v (A BC ) bng 30 . Tnh gc gia hai mt phng (ABC) v (ACC A ).
=
Bi 11.134 : Cho hnh vung ABCD. Mt phng (P) vung gc vi (ABCD) theo giao tuyn AB. im M di ng sao cho AMB
= 90 .
AMD

1. Chng minh rng M thuc mt phng trung trc ca BD;


2. Gi s MD ct (P) ti M . Chng minh rng AM BM .
Bi 11.135 : Cho hai hnh vung ABCD v ABEF cnh a v nm trong hai mt phng vung gc vi nhau. Trn hai cnh AC, BF ln

lt ly hai im M v N sao cho AM = BN = x (0 < x < a 2).


1. Chng minh rng AF(ABCD).
2. Gi M1 l hnh chiu vung gc ca M trn AB. Chng minh rng MM1 M1 N v MN (CDEF).
3. Tnh MN theo a v x. Tm x MN nh nht.
4. Khi MN nh nht hy chng minh MN vung gc vi AC, BF v MN DE.

Vn 4 : Dng mt phng (Q) cha a v vung gc vi (P) (gi thit a khng vung gc vi (P))

T mt im trn a, dng ng thng b vung gc vi (P). Mt phng (a, b) chnh l mt phng (Q) cn dng.

= 60 . ng thng S O(ABCD) v S O =
Bi 11.136 : Cho hnh chp S .ABCD c y l hnh thoi tm O cnh a, gc BAD

Gi E l trung im ca BC, F l trung im ca BE.

3a
.
4

1. Chng minh rng (S OF)(S BC).


2. Gi O , A ln lt l hnh chiu vung gc ca O, A trn (S BC). Tnh di cc on thng OO , AA .
3. Gi (P) l mt phng qua AD v vung gc vi (S BC). Xc nh thit din ct bi (P) v tnh din tch thit din . Tnh gc
gia (P) v (ABCD).
Bi 11.137 : Cho hnh chp S .ABCD c ABCD l hnh vung cnh a, S A(ABCD) v S A = a. Gi () l mt phng cha AB v
vung gc vi mt (S CD).
1. Dng mt phng (). Mt phng () ct hnh chp theo thit din l hnh g?
2. Tnh din tch thit din .
Bi 11.138 : Cho hnh chp S .ABCD c ABCD l hnh vung cnh a, S A(ABCD) v S A = a. Xc nh v tnh din tch thit din
ca hnh chp S .ABCD vi mt phng () trong cc trng hp sau y:
1. () qua tm O ca y, trung im M ca S D v vung gc vi (ABCD).
2. () qua A, trung im N ca CD v vung gc vi (S BC).

Bi 11.139 : Cho lng tr tam gic u ABC.A BC c cnh y bng a, cnh bn bng a 2. Gi M, N ln lt l trung im ca cc
cnh AB v AC . Xc nh thit din ca lng tr vi mt phng () qua MN v vung gc vi mt phng (BCC B ). Tnh din tch
thit din v tnh gc to bi mt phng () vi mt phng y.
Bi 11.140 : Cho hnh chp S .ABCD c y l hnh thang vung ABCD vung ti A v D, c AB = 2a, AD = DC = a, c cnh
S A(ABCD) v S A = a.
1. Chng minh rng (S AD)(S CD) v (S AC)(S CB).
2. Gi l gc gia hai mt phng (S BC) v (ABCD), tnh tan .

TRN ANH TUN - 0974 396 391 - (04) 66 515 343

WWW.VNMATH.COM

Trang 216

www.luyenthi24h.com
www.luyenthi24h.com
CHUYN LUYN THI I HCwww.VNMATH.com

www.VNMATH.com

3. Gi () l mt phng cha S D v vung gc vi (S AC). Hy xc nh () v tnh din tch thit din ca hnh chp ct bi mt
phng ().
Bi 11.141 : Cho hnh chp S .ABCD c y ABCD l hnh vung cnh a, S A(ABCD). Qua A xc nh mt phng () vung gc

vi S C ct S B, S C, S D ln lt ti E, K, H. Tnh din tch thit din ca hnh chp ct bi () khi S A = a 2.


Bi 11.142 : Cho hnh chp S .ABCD c y ABCD l hnh vung cnh a, S A(ABCD), S A = a. Xc nh v tnh din tch thit din
ca hnh chp ct bi mt phng (P) cha AN v vung gc vi (S BC), trong N l trung im ca CD.

11.5 Khong cch


Vn 1 : Tnh khong cch t im M n ng thng cho trc

1. Trong mt phng xc nh bi im M v ng thng v MH) ti H. Ta c d(M, ) = MH.

2. Trong khng gian dng mt phng () qua M v (), ct ti H. Ta c d(M, ) = MH.

Bi 11.143 : Cho hnh chp S .ABCD c y l hnh vung ABCD tm O, cnh a, S A(ABCD) v S A = a. Gi I l trung im cnh
S C v M l trung im on AB.
1. Chng minh rng OI(ABCD).

2. Tnh d(I, CM).

Bi 11.144 : Cho hnh chp S .ABC; ABC l tam gic vung cn (AB = AC = a); S B(ABC) v S B = a. Tnh khong cch t S n
a
CM, vi M thuc on AB v AM = .
3
= 60 v S A(ABCD).
Bi 11.145 : Cho hnh chp S .ABCD c y ABCD l hnh thoi tm O, S A = AB = 2a, ABC

1. Chng minh : BDS C, t suy ra khong cch t O n S C.


2. Tnh d(O; S B) v d(D; S C).
Bi 11.146 : Cho tam gic ABC vi AB = 7cm, BC = 5cm, CA = 8cm. Trn ng thng vung gc vi mt phng (ABC) ti A ly
im O sao cho AO = 4cm. Tnh d(O, BC).
Bi 11.147 : Cho tam gic ABC vung ti B (AB = a; BC = 2a). Ax v Cy cng vung gc vi (ABC) v v cng mt pha. Ly

M Ax v N Cy vi AM = a, CN = a 5. Chng minh rng AB(BCy). Tnh khong cch t M n BN.

a 7
Bi 11.148 : Cho gc vung xOy v mt im A nm ngoi mt phng xOy. Khong cch t A n Ox, Oy u bng a v AO =
.
2
Tnh khong cch t A n (xOy).

Vn 2 : Dng ng thng i qua mt im A cho trc v vung gc vi mt mt phng (P) cho trc.
Khong cch t im A n mt phng (P)


Bc 1 : Dng mt phng (Q) i qua A v vung gc vi (P).
Bc 2 : Gi c l giao tuyn ca (P) v (Q). Trong (Q) k ng thng qua A vung gc vi c ti H, AH chnh l ng thng cn
dng v AH l khong cch t H n mt phng (P).
Nu bit khong cch t B n (P), tnh khong cch t A n (P) chng ta c th s dng mi quan h sau :

TRN ANH TUN - 0974 396 391 - (04) 66 515 343

WWW.VNMATH.COM

Trang 217

www.VNMATH.comCHUYN LUYN THI I HC

www.luyenthi24h.com
www.luyenthi24h.com
www.VNMATH.com

1. Nu AB (P) th d(A, (P)) = d(B, (P)).


d(A, (P)) OA
=
.
2. Nu AB (P) = {O} th
d(B, (P)) OB

B
A

Bi 11.149 (Bi ton c bn) : Cho hnh chp S .ABC c S A(ABC). Hy dng hnh chiu vung gc ca A trn mt phng (S BC).
Bi 11.150 (Bi ton c bn) : Mt on thng AB khng vung gc vi mt phng () ct mt phng ny ti trung im O ca on
thng . Cc ng thng vung gc vi () qua A v B ln lt ct mt phng () ti A v B . Chng minh rng ba im A , O, B
thng hng v AA = BB.
Nh vy ta c h qu ca bi ton ny l : Hai im A v B phn bit cch u (P) (hoc ) khi v ch khi AB (P) hoc trung im
M ca AB thuc (P) (tng ng ).

Bi 11.151 : Cho hnh chp S .ABC c S A(ABC), tam gic ABC u cnh a v S A = a 2. Xc nh v tnh khong cch t A n
mt phng (S BC).
Bi 11.152 : Cho t din O.ABC c OA, OB, OC i mt vung gc vi nhau v OA = a, OB = b, OC = c. Xc nh v tnh khong
cch t O n mt phng (ABC).
Bi 11.153 : Cho hnh chp S .ABCD c y l hnh vung cnh a, cc cnh bn cng bng 2a. Gi O l giao im hai ng cho
y.
1. Chng minh rng S O(ABCD).
2. Xc nh v tnh khong cch t O n mt phng (S BC).

Bi 11.154 : Cho t din ABCD c AB = CD = 6a; BC = BD = 5a; AC = AD = a 73. Gi H l hnh chiu vung gc ca A xung
(BCD). Chng minh rng H nm trn trung tuyn BI ca tam gic BCD. Tnh d(A, (BCD)).
Bi 11.155 : Cho hnh chp S .ABC c ABC l tam gic vung ti B (AB = 2a, BC = a); S A(ABC). Tnh d(B, (S AC)).
MS
1
Bi 11.156 : Cho hnh chp S .ABC c S A = h, S A(ABC); M l im thuc on S B sao cho
= , I l trung im ca CM.
MB
2
Tnh d(I, (ABC)).
Bi 11.157 : Cho hnh chp S .ABCD c y l hnh vung tm O cnh a, S A(ABCD) v S A = 2a. Xc nh v tnh
1. d(A, (S CD));

2. d(O, (S CD));

3. d(B, (S CD));

4. d(C, (S BD)).

Bi 11.158 : Cho hnh chp S .ABCD c y l hnh vung cnh a, S A(ABCD) v S A = a 3. Gi G l trng tm tam gic S AB.
Tnh d(G, (S AC)).

Bi 11.159 : Cho hnh chp S .ABCD c y ABCD l hnh vung tm O cnh a, , S A(ABCD) v S A = a 3, G l trng tm tam
gic S AB. Tnh

TRN ANH TUN - 0974 396 391 - (04) 66 515 343

WWW.VNMATH.COM

Trang 218

www.luyenthi24h.com
www.luyenthi24h.com
CHUYN LUYN THI I HCwww.VNMATH.com

www.VNMATH.com
1. d(M, (ABCD));

2. d(A, (S BC));

3. d(O, (S BC));

4. d(G, (S AC)).

Bi 11.160 : Cho hnh lp phng ABCD.A BC D cnh a. Xc nh v tnh khong cch t mt im n mt mt phng cho di
y.
1. im A v mt phng (BDB D ) ;

2. im A v mt phng (A BD).

Bi 11.161 : Cho t din u ABCD c tt c cc cnh bng a. Tnh d(B, (ACD)).


Bi 11.162 : Cho hnh lp phng ABCD.A BC D cnh a. Xc nh v tnh khong cch t im A n mt phng (AB D ).
Bi 11.163 : Cho hnh chp u S .ABC cnh a. Xc nh chn ng vung gc h t A n mt phng (S BC) v tnh d(A, (S BC)).
Bi 11.164 : Cho hnh thoi ABCD tm O, cnh bng a v AC = a. T trung im H ca cnh AB dng S H(ABCD) vi S H = a.
1. Tnh d(H, (S CD)). T suy ra khong cch t O n mt phng (S CD).
2. Tnh khong cch t A n mt phng (S BC).
d v mt im M nm ngoi mt phng cha gc vung, OM = 23cm v khong cch t M ti hai
Bi 11.165 : Cho gc vung xOy

cnh Ox, Oy u bng 17cm. Tnh khong cch t M n mt phng cha gc vung.

Bi 11.166 : Cho tam gic ABC vung ti A, c cnh AB = a nm trong mt phng (), cnh AC = a 2 v to vi () mt gc 60 .
1. Tnh khong cch CH t C ti ().

2. Chng minh rng cnh BC to vi () mt gc bng 45 .

Bi 11.167 : Cho hnh chp t gic u S .ABCD c y ABCD l hnh vung tm O, AB = 2a, S A = 4a. Tnh :
1. d(O; (S AB)) ;

2. d(A; (S CD)).

Bi 11.168 : Cho t din DABC, c ABC l tam gic vung ti A, S B = a, AC = 2a. Cc mt (DAB) v (DAC) cng hp vi (ABC)
gc , mt bn (S BC) vung gc vi (ABC).
1. Tnh khong cch d t D n (ABC) theo a v ;

2a
2. Tm s o khi bit d = , khi hy tnh d(C; (DAB)).
3

Bi 11.169 : Cho hnh chp S .ABC c gc to bi hai mt phng (S BC) v (ABC) l 60 . Cc tam gic S BC v ABC u, AB = a.
Tnh theo a khong cch t B n mt phng (S AC) trong mi trng hp sau :
1. Hnh chiu vung gc ca S xung mt phng (ABC) nm min trong tam gic ABC.
2. Hnh chiu vung gc ca S xung mt phng (ABC) nm min ngoi tam gic ABC.
Bi 11.170 : Cho hnh chp S .ABCD c y l hnh vung tm O cnh a, S A = a v S A(ABCD), gi M l trung im S C. Tnh
1. d(A, (S CD));

3. d(O, (S CD));

5. d(M, (ABCDC));

2. d(B, (S CD));

4. d(C, (S BD));

6. d(M, (S AD)).

Bi 11.171 : Cho hnh chp S .ABC c y ABC l tam gic vung ti A, AB = a, AC = a 3 v cc cnh bn cng hp vi y mt
gc 60 .
1. Tnh d(S , (ABC)), d(A, (S BC)), d(C, (S AB));
2. Tnh cosin gc gia S B v AC;
3. Tnh cosin gc gia hai mt phng (S BC) v (S AC).

Vn 3 : on vung gc chung v khong cch gia hai ng thng cho nhau

Ta xt cc trng hp sau y:

TRN ANH TUN - 0974 396 391 - (04) 66 515 343

WWW.VNMATH.COM

Trang 219

www.luyenthi24h.com
www.luyenthi24h.com
www.VNMATH.com

www.VNMATH.comCHUYN LUYN THI I HC


b

A
B
a

B
A

b
M

a)

H
c)

b)

a) Gi s a, b l hai ng thng cho nhau v ab.


- Ta dng mt phng () cha a v vung gc vi b ti B.
- Trong () dng BAa ti A, ta c di on BA l khong cch gia hai ng thng cho nhau a v b (hnh a).
b) Gi s a v b l hai ng thng cho nhau nhng khng vung gc vi nhau.
Cch 1 : - Ta dng mt phng () cha a v song song vi b (hnh b).
- Ly mt im M ty trn b dng MM () ti M .

- T A dng AB MM ct b ti B, di on AB l khong cch gia hai ng thng cho nhau a v b.


Cch 2 : - Ta dng mt phng ()a ti O, () ct b ti I (hnh c).
- Dng hnh chiu vung gc ca b l b trn ().
- Trong mt phng (), v OHb ti H.
- T H dng ng thng song song vi a ct b ti B.
- T B dng ng thng song song vi OH ct a ti A.
di on AB l khong cch gia hai ng thng cho nhau a v b.
Ch : tnh khong cch gia hai ng thng cho nhau a v b ta thng lm nh sau :

Ta dng mt phng () cha a v song song vi b (hnh b).


Ly im M b. Ta c d(a, b) = d(b, ()) = d(M, ()).

Bi 11.172 : Cho t din OABC c OA, OB, OC i mt vung gc vi nhau v OA = OB = OC = a. Gi I l trung im BC. Hy
dng v tnh di on vung gc chung ca cc cp ng thng:
1. OA v BC;

2. AI v OC.

= 60 , S O(ABCD), S O =
Bi 11.173 : Cho hnh chp S .ABCD c y l hnh thoi tm O, cnh a, gc BAD

1. Tnh d(O, (S BC)) v d(A, (S BC));

3a
.
4

2. Tnh d(AD, S B).

Bi 11.174 : Cho lng tr ng ABC.A BC c cc mt bn u l hnh vung cnh a. Gi D, E, F ln lt l trung im cc cnh


AB, AC , BC . Tnh khong cc gia cc cp ng thng sau :
1. DE v AB ;

2. A B v BC .

Bi 11.175 : Cho hnh chp S .ABCD c y ABCD l hnh vung cnh a, c cnh S A = a v vung gc vi mt phng (ABCD).
Dng v tnh di on vung gc chung ca:

TRN ANH TUN - 0974 396 391 - (04) 66 515 343

WWW.VNMATH.COM

Trang 220

www.VNMATH.com
1. S B v CD;

www.luyenthi24h.com
www.luyenthi24h.com
CHUYN LUYN THI I HCwww.VNMATH.com
2. S C v BD;

3. S C v AB;

4. AC v S D.

Bi 11.176 : Cho hnh lp phng ABCD.A BC D cnh a. Gi I, J ln lt l trung im ca BC v AD. Tnh cc khong cch :
1. d(A, (CDDC )) ;

3. d(AA , (BB D D)) ;

5. d(BD, AC) ;

2. d(A, CC ) ;

4. d((AIA ), (CJC )) ;

6. d(AA , BD ) ;

7. d(AI, JC ).

Bi 11.177 : Cho hnh lng tr ng ABC.A BC , mt y ABC l tam gic cn ti nh A c Ab = 120, cnh bn bng a.
1. Tnh d(A, (BBC C)).
2. Xc nh on vung gc chung ca hai ng thng AA v BC.
Bi 11.178 : Cho t din ABCD c AD = BC = a, AC = BD = b, AB = x, CD = y. Tnh khong cch gia hai ng thng AB v
CD.
Bi 11.179 : Cho hnh lp phng ABCD.A BC D cnh a. Tnh d(A, (BDA)), d((A BD), (CB D )), d(A D, DC).

Bi 11.180 : Cho hnh lng tr ng ABC.A BC c cnh bn AA = a, y ABC l tam gic vung ti A c BC = 2a, AB = a 3.
Tnh d(AA , (BCC B )), d(A , (ABC )), d(A, (A BC)).

Bi 11.181 : Cho hnh chp S .ABCD c y l hnh vung cnh a v S A = S B = S C = S D = a 2. Tnh d(S , (ABCD)), d(AD, S B).

Bi 11.182 : Cho hnh chp S .ABC c S A vung gc vi mt phng y v S A = a 2, y ABC l tam gic vung ti B c BA = a.
Gi M l trung im ca AB. Tnh d(S M, BC).
Bi 11.183 : Cho hai tia cho nhau Ax v By hp vi nhau gc 60 , nhn AB = a lm on vung gc chung. Trn By, ly im C sao
cho BC = a. Tnh d(C, (B, Ax)), d(C, Ax) v tm im cch u cc nh A, B, C, D.
Bi 11.184 : Cho t din ABCD c bn mt l bn tam gic c din tch bng nhau. Chng minh rng on ni trung im hai cnh
i din cng l on vung gc chung ca hai cnh .
Bi 11.185 : Cho hnh lng tr tam gic u ABC.A BC c cnh bn bng h. Bit khong cch gia A B v BC bng d. Tnh cnh
y ca hnh lng tr theo d v h.

a 2
Bi 11.186 : Cho hnh chp S .ABC c y ABC l tam gic vung vi AB = AC = a, S A(ABC), S A =
. Tnh gc gia hai mt
2
phng (S AC) v (S BC); tnh khong cch gia hai ng thng AI v S C, vi I l trung im BC.
= BAD
= 90 , S A = AB = BC = a v AD = 2a. Bit hai mt
Bi 11.187 : Cho hnh chp S .ABCD c y ABCD l hnh thang, CBA

phng (S AB) v (S AD) cng vung gc vi y.


1. Tnh d(S , (BCD)); d(A, (S CD)); d(AD, (S BC)).
2. Tnh cosin gc gia hai mt phng (S CD) v (ABCD).
3. Tnh d(S A, CD), d(BC, S D), d(S B, CD).
Bi 11.188 : Cho hnh lng tr tam gic ABC.A BC c ABC v ABB l hai tam gic u cnh a v nm trong hai mt phng vung
gc vi nhau.
1. Tnh d(B, (ABC)); d(A, (BCC B )).
2. Tnh gc v khong cch gia hai ng thng AB v CC .
Bi 11.189 : Cho khi chp S .ABC c tam gic ABC u cnh A, chn ng cao ca hnh chp trng vi trung im ca BC v gc
gia S A v mt phng (ABC) bng 60 .
1. Tnh d(S A, BC); d(B, (S AC)).
2. Gi G l trng tm tam gic S BC. Tnh gc gia (ABC) v (ABG), t suy ra din tch ca tam gic ABG.

TRN ANH TUN - 0974 396 391 - (04) 66 515 343

WWW.VNMATH.COM

Trang 221

www.VNMATH.comCHUYN LUYN THI I HC

www.luyenthi24h.com
www.luyenthi24h.com
www.VNMATH.com

11.6 Khi a din v th tch khi a din


Vn 1 : Phng php trc tip tm th tch khi chp

1
S .h vi hnh chp v V = S .h vi hnh lng tr, trong S l din tch y cn h l di ng cao.
3
1. Phng php xc nh trc tip chn ng cao :

S dng cng thc V =

Di y l mt s c im thng gp ca hnh chp v v tr chn ng cao tng ng.

Hnh chp c cnh bn vung gc vi y th cnh bn chnh l ng cao.


Hnh chp u l hnh chp c y l a gic u v ng i qua tm ca y (cc cnh bn bng nhau).
Hnh chp c hai mt phng (cng cha nh) vung gc vi y th ng cao chnh l giao tuyn ca hai mt phng .
Hnh chp c cc cnh bn bng nhau hoc cc cnh bn cng to vi y nhng gc bng nhau th chn ng cao chnh
l tm ng trn ngoi tip a gic y.

Hnh chp c mt mt (cha nh) vung gc vi y, th ng cao chnh l ng cao (xut pht t nh) ca mt bn
.
Hnh chp c cc mt bn to vi y nhng gc bng nhau hoc cc mt bn c cc ng cao (xut pht t nh) bng
nhau th chn ng cao cch u cc cnh ca y. Nu lc ny y l tam gic th chn ng cao chnh tm ng trn
ni tip hoc ngoi tip tam gic y.
2. Phng php gin tip xc nh di ng cao : Chng ta s dng cc phng php xc nh khong cch t mt im n
mt phng, c th

Nu AB (P) th d(A, (P)) = d(B, (P));


d(A, (P)) OA
Nu AB (P) = {O} th
=
.
d(B, (P)) OB

Bi 11.190 : Cho chp u S .ABCD c cnh y bng a, S


AC = 45 . Tnh th tch hnh chp S .ABCD.
Bi 11.191 : Cho hnh chp S .ABC c S B = S C = BC = CA = a; hai mt bn (ABC) v (AS C) cng vung gc vi (S BC). Tnh th
tch khi chp.
Bi 11.192 : Cho hnh lp phng ABCD.A BC D c cnh a. Ly M AB, N C D . Chng minh rng t din B A MN c th tch
khng i.

Bi 11.193 : Cho hnh lp phng ABCD.A BC D c cnh a. Gi M, N ln lt l trung im ca BC v CD. Tnh th tch khi t
din AB MN.

Bi 11.194 : Cho hnh chp S .ABCD c y l hnh vung cnh a; (S AC)(ABCD); AS


C = 90 v S A to vi y mt gc . Tnh

th tch khi chp.


= 90 , ABC
= ; S BC l tam gic u cnh a, (S BC)(ABC). Tnh th tch khi chp.
Bi 11.195 : Cho hnh chp S .ABC c BAC

Bi 11.196 : Cho t din ABCD c AD = b v 5 cnh cn li u bng a. Tnh th tch khi t din.
Bi
11.197 : Cho hnh chp u S .ABCD c tt c cc cnh u bng nhau. Tnh cnh ca hnh chp bit th tch ca khi chp bng
9a3 2
.
2
Bi 11.198 : Cho hnh chp tam gic u S .ABC. Bit khong cch t A n (S BC) bng d, gc gia AB v mt phng (S BC) l bng
. Tnh th tch v din tch xung quanh ca khi chp.
Bi 11.199 : Cho khi chp tam gic u S .ABC c y l tam gic u cnh bng a. Cc cnh bn to vi y mt gc bng 60 .
Tnh th tch khi chp .

TRN ANH TUN - 0974 396 391 - (04) 66 515 343

WWW.VNMATH.COM

Trang 222

www.VNMATH.com

www.luyenthi24h.com
www.luyenthi24h.com
CHUYN LUYN THI I HCwww.VNMATH.com

Bi 11.200 : Cho khi chp S .ABC c y l tam gic cn, AB = AC = 5a, BC = 6a. Cc mt bn to vi y mt gc 60 , chn
ng cao ca hnh chp nm trong min trong tam gic ABC. Hy tnh th tch khi chp .
Bi 11.201 : Cho khi chp tam gic u S .ABC c chiu cao bng h v gc AS B bng 2. Hy tnh th tch khi chp.
Bi 11.202 : Cho khi chp S .ABC c S A(ABC); y l tam gic ABC cn ti A, di trung tuyn AD bng a, cnh bn S B to
vi y mt gc v to vi mt (S AD) gc . Tnh th tch khi chp.
Bi 11.203 : Cho t din ABCD. Gi d l khong cch gia hai ng thng AB v CD, l gc gia hai ng thng . Chng
minh rng
1
AB.CD.d. sin .
6
Bi 11.204 : Cho khi chp S .ABC c y ABC l tam gic vung cn nh C v S A(ABC), S C = a. Hy tm gc gia hai mt
VABCD =

phng (S CB) v (ABC) th tch khi chp ln nht.


Bi 11.205 : Cho khi chp t gic u S .ABCD m khong cch t A n mt phng (S BC) bng 2a. Vi gi tr no ca gc gia
mt bn v mt y ca khi chp th th tch ca khi chp nh nht.
Bi 11.206 : Tnh th tch khi t din ABCD c cc cp cnh i bng nhau :
AB = CD = a; AC = BD = b; AB = BC = c.
Bi 11.207 : Cho khi lng tr tam gic u ABC.A BC c cnh y bng a, chiu cao bng h. Tnh th tch khi chp A.BC A .
Bi 11.208 : Cho ng trn ng knh AB nm trn mt phng (P) v mt im M di ng trn ng trn. Trn ng thng
vung gc vi mt phng (P) ti A, ly mt im S . Mt phng (Q) qua A vung gc vi S B ti K ct S M ti H. Tm v tr ca M
nh hn cung BM.

th tch khi chp S .AHK ln nht. Chng minh rng khi cung AM

Bi 11.209 : Cho lng tr ABC.A BC c y ABC l tam gic vung cn ti A, cnh BC = 2a. Cnh bn ca lng tr bng a v
vung gc vi y.
1. Chng minh rng 6 nh ca lng tr nm trn mt mt cu. Xc nh tm v bn knh mt cu .
2. Tnh th tch v din tch ton phn ca lng tr .
3. Tnh gc gia mt phng (CA B ) v mt y (ABC).
Bi 11.210 : Cho lng tr ng ABCD.A BC D c y ABCD l hnh thoi cnh a, gc Ab = 60 . ng cho AC ca lng tr hp
vi y mt gc bng 60 .
1. Tnh din tch xung quanh, din tch ton phn, th tch ca khi lng tr .
2. Hai mt phng (P) v (Q) cha DD ct cc cnh AB, A B , BC v BC ln lt ti M, M , N v N . Gi s AM = x, BN = y.
Tm x, y (P) v (Q) chia lng tr thnh ba phn tng ng (c th tch bng nhau).
Bi 11.211 : Cho lng tr ng ABC.A BC c y ABC l tam gic cn ti A. Gc gia AA v BC l 30 v khong cch gia
chng bng a. Gc gia hai mt bn cha AA l 60 . Tnh th tch khi lng tr.

a 3
v hp vi BC mt gc vi sin =
Bi 11.212 : Cho lng tr u ABC.A B C . Mt phng (A BC) cch A mt khong bng
4

15
. Tnh th tch khi lng tr.
10
Bi 11.213 : Cho lng tr ABC.A BC c y l tam gic u cnh a v A A = A B = AC = b.

1. Chng minh rng BCC B l hnh ch nht.


2. Xc nh b theo a mt bn (ABB A ) hp vi y gc 60 .
3. Tnh th tch v din tch ton phn ca khi lng tr theo a vi gi tr b va tm c.
Bi 11.214 : Cho lng tr ng ABC.A BC c y l tam gic u cnh a. nh A c hnh chiu trng vi tm O ca tam gic ABC.
Cnh bn hp vi y mt gc 45 .
1. Tnh th tch ca khi lng tr.

TRN ANH TUN - 0974 396 391 - (04) 66 515 343

WWW.VNMATH.COM

Trang 223

www.VNMATH.comCHUYN LUYN THI I HC

www.luyenthi24h.com
www.luyenthi24h.com
www.VNMATH.com

2. Tnh din tch xung quanh v din tch ton phn ca lng tr .
Bi 11.215 : Cho lng tr ABCD.A BC D c cnh bn bng a hp vi y mt gc , y l hnh thoi gc Ab = v AC = 2a. Mt
cho ACC A vung gc vi y.
1. Chng minh rng BDD B l hnh ch nht v cc mt bn bng nhau.
2. Tnh th tch khi lng tr.
3. Tnh din tch xung quanh v din tch ton phn ca lng tr .
= 2. nh A cch u ba nh A, B, C. Cc
Bi 11.216 : Cho lng tr ABC.A BC c y ABC l tam gic cn ti A, BC = 2a, BAC

cnh bn hp vi y mt gc 60 .
1. Tnh th tch khi lng tr.
2. Tnh din tch xung quanh v din tch ton phn ca lng tr .
Bi 11.217 : Cho lng tr ABCD.A BC D c y ABCD l hnh thoi cnh a, gc Ab = 60 . Hnh chiu ca A xung di mt phng
(ABCD) trng vi tm O ca ng trn ngoi tip tam gic ABD. Cho bit
BAA = 45 .
1. Tnh th tch khi lng tr.
2. Tnh din tch xung quanh v din tch ton phn ca lng tr .
Bi 11.218 : Cho khi lng tr t gic u ABCD.A1 B1C1 D1 c khong cch gia hai ng thng AB v A1 D bng 2 v di dg
cho mt bn bng 5.
1. H AKA1 D (K A1 D). Chng minh rng AK = 2.
2. Tnh th tch khi lng tr ABCD.A1 B1C1 D1 .
Bi 11.219 : y ca khi lng tr ng ABC.A1 B1C1 l mt tam gic u. Mt phng (A1 BC) to vi y mt gc 30 v tam gic
A1 BC c din tch bng 8. Tnh th tch khi lng tr.
= 45 . Cc ng cho AC1 v DB1 ln lt
Bi 11.220 : Cho khi lng tr ng ABCD.A1 B1C1 D1 c y l hnh bnh hnh v BAD

to vi y AC1 v DB1 ln lt to vi y nhng gc 45 v 60 . Hy tnh th tch ca khi lng tr nu bit chiu cao ca n bng 2.

Bi 11.221 : Cho khi hp ABCD.A1 B1C1 D1 c tt c cc cnh bng nhau v bng a, A


1 AB = BAD = A1 AD = (0 < < 90 ).

Hy tnh th tch ca khi hp.

Bi 11.222 : Cho khi hp ABCD.A BC D c y l hnh ch nht vi AB = a 3, AD = a 7. Hai mt bn (ABB A ) v (ADD A )


ln lt to vi y nhng gc 45 v 60 . Hy tnh th tch ca khi lng tr nu bit cnh bn bng 1.
Bi 11.223 : Cho khi lng tr tam gic ABC.A1 B1C1 m mt bn ABB1 A1 c din tch bng 4. Khong cch gia CC1 v mt phng
(ABB1 A1 ) bng 7. Hy tnh th tch khi lng tr.

Cho khi lng tr ABC.A1 B1C1 c y l tam gic vung cn vi cnh huyn AB bng 2. Cho bit mt phng (AA1 B) vung gc vi

mt phng (ABC), AA1 = 3, gc A


1 AB nhn, gc gia mt phng (AA1C) v mt phng (ABC) bng 60 . Hy tnh th tch khi lng
tr.
Bi 11.224 : Cho hai ng thng cho nhau Ax, By. Gi C v D l hai im di ng ln lt trn Ax, By sao cho
l di cho trc, k l s thc dng cho trc).

a
b
+
= k (a, b
AC BD

1. Chng minh rng on CD lun lun ct mt on thng c nh.


2. Xc nh v tr ca C, D th tch t din ABCD nh nht.
Bi 11.225 : Cho tam gic u ABC cnh a. Trn ng thng d vung gc vi mt phng (ABC) ti A, ta ly im M. Gi H l trc
tm ca tam gic ABC, K l trc tm ca tam gic BCM.
1. Chng minh rng MC(BHK) v HK(BCM).
2. Khi M thay i trn d, tm gi tr ln nht v nh nht ca th tch t din KABC.

TRN ANH TUN - 0974 396 391 - (04) 66 515 343

WWW.VNMATH.COM

Trang 224

www.VNMATH.com

www.luyenthi24h.com
www.luyenthi24h.com
CHUYN LUYN THI I HCwww.VNMATH.com

Bi 11.226 : Trn cnh AD ca hnh vung ABCD c di cnh l a, ly im M sao cho AM = x(0 x a). Trn na ng
thng Ax vung gc vi mt phng hnh vung ti im A, ly im S sao cho S A = y(y > 0).
1. Chng minh rng (S AB)(S BC).
2. Tnh khong cch t M n mt phng (S CA).
3. Tnh th tch khi chp S .ABCM theo a, y v x.
4. Bit rng x2 + y2 = a2 . Tm gi tr ln nht ca th tch khi chp S .ABCM.
Bi 11.227 : Hnh lng tr ng ABC.A BC c y ABC l mt tam gic vung ti a v AC = b, Cb = 60 . ng thi ng cho
BC ca mt bn BBC C to vi mt phng (AAC C) mt gc 30 .
1. Tnh di on AC ;

2. Tnh th tch ca khi lng tr.

Bi 11.228 : Cho chp t gic u S .ABCD.


1. Bit AB = a v S A = l, tnh th tch khi chp theo a v l.
2. Bit S A = l v gc gia mt bn v mt y bng . Tnh th tch khi chp theo v l.
Bi 11.229 : Cho hnh chp t gic u S .ABCD.
1. Bit AB = a v gc gia mt bn v y bng , tnh th tch khi chp theo a v .
2. Bit di ca on thng ni nh hnh chp vi trung im ca mt cnh y bng d v gc gia cnh bn v y bng ,
tnh th tch khi chp theo d v .
Bi 11.230 : Cho lng tr tam gic ABC.A BC c tt c cc cnh y u bng a. Hn na gc to thnh bi cnh bn v mt y
bng 60 v hnh chiu H ca nh A ln mt phng (A BC ) trng vi trung im ca cnh BC .
1. Tnh khong cch gia hai mt y.
2. Tnh gc gia hai ng thng BC v AC .
3. Tnh gc gia mt phng (ABB A ) v mt y.
4. Tnh th tch ca khi lng tr.
Bi 11.231 : Cho hnh chp S .ABCD c y l hnh vung cnh a, mt bn S AD l tam gic u v nm trong mt phng vung gc
vi y. Gi M, N, P ln lt l trung im ca cc cnh S B, BC, CD. Chng minh AM vung gc vi BP v tnh th tch ca khi t
din CMNP.
Bi 11.232 : Cho lng tr u ABC.A BC c chiu cao bng h v hai ng thng AB , BC vung gc nhau. Tm th tch khi lng
tr .
Bi 11.233 : Cho hnh chp t gic u S .ABCD c di cnh y AB bng a v gc S AB bng . Tnh th tch khi chp S .ABCD
theo a v .

Bi 11.234 : Cho hnh chp S .ABCD, y l hnh ch nht c AB = 2a, BC = a. Cc cnh bn ca hnh chp u bng a 2. Tnh th
tch khi chp S .ABCD theo a.
Bi 11.235 : Cho hnh lp phng OBCD.O1 B1C1 D1 c di mi cnh bng a.
1. Tnh khong cch gia hai ng thng O1 B v B1C.
2. Gi N l trung im ca BD1 . Tnh th tch khi chp ONBB1.
3. Gi M l mt im bt k thuc OO1 . Chng minh rng t s th tch khi chp MBCC1 B1 v hnh lng tr OCBO1 B1C1 khng
phc thuc vo v tr im M.
1
Bi 11.236 : Chng minh rng nu mt t din ch c ng mt cnh c di ln hn 1 th th tch ca t din ln nht l .
8
Bi 11.237 : K hiu S v V ln lt l din tch ton phn v th tch ca hnh chp u n - gic.

TRN ANH TUN - 0974 396 391 - (04) 66 515 343

WWW.VNMATH.COM

Trang 225

www.VNMATH.comCHUYN LUYN THI I HC

www.luyenthi24h.com
www.luyenthi24h.com
www.VNMATH.com

1. Vi cc gi tr cho trc n v S , hy tm gi tr ln nht ca th tch V.


2. Tnh cc cnh y v ng cao ca tt c cc hnh chp vi n = 4, S = 114, V = 64.

Bi 11.238 : Cho hnh chp S .ABCD c y ABCD l hnh ch nht vi AB = a, AD = a 2, S A = a v S A vung gc vi mt phng
(ABCD). Gi M v N ln lt l trung im ca AD v S C; I l giao im ca BM v AC. Chng minh rng mt phng (S AC) vung
gc vi mt phng (S MB). Tnh th tch ca khi t din ANIB.
Bi 11.239 : Cho hnh chp tam gic S .ABC c y ABC l tam gic u cnh a, S A = 2a v S A vung gc vi mt phng (ABC).
Gi M v N ln lt l hnh chiu vung gc ca A trn cc ng thng S B v S C. Tnh th tch ca khi chp A.BCN M.
Bi 11.240 : Cho hnh chp S .ABCD u c cnh y bng a, gc gia cnh bn v mt y bng (0 < < 90 ). Tnh tan ca gc
gia hai mt phng (S AB), (ABCD). Tnh th tch khi chp theo a, .
= 60 , BC = a, S A = a.
Bi 11.241 : Cho hnh chp S .ABC. y ABC l tam gic vung ti B, cnh S A vung gc vi y, gc ACB

Gi M l trung im cnh S B. Chng minh mt phng (S AB) vung gc vi mt phng (S BC). Tnh th tch khi t din MABC.
= ABC
= 90 , AB = BC = a, AD = 2a, S A vung gc vi y
Bi 11.242 : Cho hnh chp S .ABCD c y ABCD l hnh thang, BAD

v S A = 2a. Gi M, N ln lt l trung im S A, S D. Chng minh rng BCN M l hnh ch nht v tnh th tch khi chp S .BCN M
theo a.

Bi 11.243 : Cho lng tr ng ABC.A BC c y ABC l tam gic vung, AB = BC = a, cnh bn AA = a 2. Gi M l trung
im cnh BC. Tnh theo a th tch khi lng tr ABC.A BC v khong cch gia hai ng thng AM, BC.

Bi 11.244 : Cho hnh chp S .ABCD c y ABCD l hnh vung cnh 2a, S A = a, S B = a 3 v mt phng (S AB) vung gc vi
mt y. Gi M, N ln lt l trung im cc cnh AB, BC. Tnh theo a th tch khi chp S .BMDN v tnh cosin gc gia hai ng
thng S M, DN.

Bi 11.245 : Cho lng tr ABC.A BC c di cnh bn bng 2a, y ABC l tam gic vung ti A, AB = a, AC = a 3 v hnh
chiu vung gc ca nh A trn mt phng (ABC) l trung im ca cnh BC. Tnh theo a th tch khi chp A .ABC v tnh cosin
ca gc gia hai ng thng AA , BC .

Bi 11.246 : Cho t din ABCD c BC = CD = a, BC l on vung gc chung gia AB v CD, gc gia BA v CD bng 60 v
ADAB. Tnh th tch khi t din ABCD theo a.
Bi 11.247 : Cho hnh chp u S .ABCD c cnh y bng a, cc cnh bn to vi y mt gc . Mt phng qua AC, vung gc vi
(S AD) ct cnh S D ti E. Tnh th tch khi a din S BCEA theo a v .
Bi 11.248 : Cho hnh chp S .ABCD c y ABCD l hnh vung cnh a, tam gic S AB u, gc gia mt bn (S AB) v y bng
60 , I l trung im S C. Tnh th tch khi chp S AIB.

Bi 11.249 : Trong
khng gian cho hnh chp S .ABCD vi ABCD l hnh thoi tm O, c cnh a, gc ABC = 60 , chiu cao S O ca
a 3
hnh chp bng
. Gi M l trung im AD, (P) l mt phng qua BM v song song vi S A, ct S C ti K.
2
Tnh th tch khi chp K.BCDM.

Bi 11.250 : Cho hnh chp S .ABCD c y l hnh ch nht ABCD, AD = a 2, CD = 2a. Cnh S A vung gc vi y v S A =

3a 2. Gi K l trung im AB. Chng minh rng mt phng (S AC) vung gc vi mt phng (S DK) v tnh th tch khi chp S .CDK

theo a.
Bi 11.251 : Cho hnh lng tr tam gic ABC.A BC c y l tam gic u cnh a, cc cnh AA = A B = CA , bit gc gia mt
bn ABB A to vi y ca lng tr mt gc bng 60 . Chng minh rng BCC B l hnh ch nht v tnh th tch khi lng tr theo a.
Bi 11.252 : Cho hnh vung ABCD cnh a nm trn mt phng (P). Dng hai na ng thng Bx v Dy cng vung gc vi mt
phng (P) v nm cng pha vi mt phng (P). Trn hai na ng thng Bx, Dy ln lt ly hai im M Bx, N Dy sao cho

BM = b, DN = c (b, c > 0). Tnh th tch khi t din ACMN theo a, b, c. Khi M, N thay i trn Bx, Dy sao cho (ACM)(ACN), hy

xc nh b, c theo a th tch khi t din ACMN l nh nht.


Bi 11.253 : Trong mt phng (p) cho na ng trn ng knh AB = 2R v im C thuc na ng trn sao chi AC = R. Trn
ng thng vung gc vi (P) ti A ly im S sao cho ((S
AB), (S BC)) = 60 . Gi H, K ln lt l hnh chiu ca A trn S B, S C.
Chng minh rng tam gic AHK vung v tnh th tch khi chp S .ABC.

TRN ANH TUN - 0974 396 391 - (04) 66 515 343

WWW.VNMATH.COM

Trang 226

www.VNMATH.com

www.luyenthi24h.com
www.luyenthi24h.com
CHUYN LUYN THI I HCwww.VNMATH.com

Bi 11.254 : Cho hnh chp S .ABCD c y l hnh ch nht, viAB = a, AD = 2a, cnh S A vung gc vi y, cnh S B to vi
a 3
. Mt phng (BCM) ct cnh S D ti N. Tnh th tch khi chp
mt y mt gc 60 . Trn cnh S A ly im M sao cho AM =
3
S .BCN M.

a 3

Bi 11.255 : Cho hnh hp ng ABCD.A B C D c cc cnh AB = AD = a, AA =


, BAD = 60 . Gi M, N ln lt l trung
2
im ca A D , A B . Chng minh rng AC (BDMN) v tnh th tch A.BDMN.
= 90 .
Bi 11.256 : Cho t din ABCD vi AB = AC = a, BC = b. Hai mt phng (BCD) v (ABC) vung gc vi nhau v gc BDC

Xc nh tm v tnh bn knh mt cu ngoi tip t din ABCD theo a v b.


= 120 , cnh bn BB = a.
Bi 11.257 : Cho lng tr ng ABC.A BC c y ABC l tam gic cn vi AB = AC = a v gc BAC

Gi I l trung im CC . Chng minh rng tam gic AB I vung A. Tnh cosin ca gc gia hai mt phng (ABC) v (AB I).
Bi 11.258 : Cho hnh lng tr ABC.A BC c A .ABC l hnh chp tam gic u, cnh y AB = a, cnh bn AA = b. Gi l gc
gia hai mt phng (ABC) v (A BC). Tnh tan v th tch khi a din A BBC C.

Vn 2 : Tnh th tch hnh chp mt cch gin tip


1. Ta thng phi so snh ng cao, din tch y ca hnh chp, din tch y ca hnh chp phi tnh vi mt hnh chp bit
(hay d tnh ton hn).
2. Dng phng php chia tch khi a din.
3. S dng kt qu sau: Cho khi chp S .ABC. Trn tia S A, S B, S C ln lt ly ba im A , B , C khc vi S . Khi
S A S B SC
VS .ABC
=
.
.
.
VS .A B C
S A S B S C
Ch : Cng thc t s th tch trn ch p dng cho hai hnh chp tam gic c chung nh v chung cc cnh bn.

Bi 11.259 : Cho t din ABCD c th tch bng V. Gi M, P ln lt l trung im ca AB v CD, N thuc cnh AD sao cho
DA = 3NA. Tnh th tch t din BMNP theo P.
Bi 11.260 : Cho hnh chp S .ABCD c th tch l V; ABCD l hnh bnh hnh. Gi M, N, P ln lt l trung im ca cc cnh
BC, CD, S D. Tnh th tch t din AMNP theo V.
Bi 11.261 : Cho hnh chp S .ABC c y l tam gic vung B. Cnh S A vung gc vi y. T A k ng AD vung gc vi S B
v AE vung gc vi S C. Bit AB = a, BC = b, S A = c. Hy tnh th tch hnh chp S .ADE.
Bi 11.262 : Cho hnh chp u S .ABCD. Mt phng (P) qua A v vung gc vi S C ct S B, S C, S D ln lt ti B , C , D . Bit rng
S B 2
= .
AB = a,
SB
3
1. Tnh t s th tch ca hai khi chp S .ABC D v S .ABCD.
2. Tnh th tch khi chp S .ABC D .
Bi 11.263 : Cho hnh chp u S .ABCD c cnh y v ng cao cng bng a. Gi M, N ln lt l trung im S B, S D ; P l giao
im ca mt phng (AMN) vi S C. Tnh th tch khi chp S .ABC D .
Bi 11.264 : Cho chp S .ABCD y l hnh vung, cnh a, c S A(ABCD). Mt phng () qua A v vung gc vi S C, ct
S B 2
S B, S C, S D ln lt ti B , C , D v bit
= . Tnh th tch hnh chp S .ABC D .
SB
3

Bi 11.265 : Cho chp S .ABCD y l hnh vung tm O, cnh a, c S A(ABCD) v S A = a 2. Gi H v K ln lt l hnh chiu
ca A ln S B v S D. Chng minh rng S C(AHK). Tnh th tch khi chp OAHK.
= 60 , S A(ABCD) v S A = a. Gi C l trung im ca S C.
Bi 11.266 : Cho hnh chp S .ACBD c y l hnh thoi cnh a, BAD

Mt phng (P) i qua AC v song song vi BD ct cc cnh S B, S D ln lt ti B , D . Tnh th tch khi chp S .ABC D .

TRN ANH TUN - 0974 396 391 - (04) 66 515 343

WWW.VNMATH.COM

Trang 227

www.VNMATH.comCHUYN LUYN THI I HC

www.luyenthi24h.com
www.luyenthi24h.com
www.VNMATH.com

Bi 11.267 : Cho khi chp S .ABCD c y l hnh vung cnh a, S A vung gc vi mt phng y v S A = 2a. Gi B , D ln lt
l hnh chiu vung gc ca A trn S B v S D. Mt phng (AB D ) ct S C ti C . Tnh th tch khi chp S .ABC D .
2a
Bi 11.268 : Cho hnh lp phng ABCD.A BC D c cnh bng a v im K thuc CC sao cho CK =
. Mt phng () i qua
3
A, K v song song vi BD chia khi lp phng thnh hai khi a din. Tnh th tch hai khi a din .
= 60 , S A(ABCD), S A = a. Gi C l trung im S C. Mt
Bi 11.269 : Cho hnh chp S .ABCD c y l hnh thoi cnh a, BAD

phng (P) i qua AC v song song vi BD, ct cc cnh S B, S D ca hnh chp ti B , D . Tnh th tch khi chp S .ABC D .
Bi 11.270 : Cho hnh chp S .ABC, c S A = a, S B = b, S C = c. Tnh th tch khi chp S .ABC, bit :
B = BS

A = 60 ;
1. AS
C = CS

B = 90 ; BS

A = 60 .
2. AS
C = 120 ; CS

Bi 11.271 : Bit th tch khi hp ABCD.A BC D l V. Tnh th tch khi t din ACB D .
Bi 11.272 : Cho hnh hp ch nht ABCD.A BC D c AB = a, BC = b, AA = c. Gi M, N ln lt l trung im A B , BC . Tnh
t s gia th tch khi chp D .DMN v th tch khi hp ch nht ABCD.A BC D .
Bi 11.273 : Cho hnh hp ch nht ABCD.A BC D c AB = a, BC = b, BB = c. Gi E v F ln lt l nhng im thuc cc cnh
1
1
BB v DD sao cho BE = EB , DF = FD . Mt phng (AEF) chia khi hp ch nht ABCD.A BC D thnh hai khi a din (H)
2
2
v (H ). Gi (H ) l khi a din cha nh A . Hy tnh th tch ca (H).
Bi 11.274 : Cho hnh hp ch nht ABCD.A BC D c AB = a, BC = b, AA = c. Gi E v F ln lt l trung im ca BC v
C D . Mt phng (AEF) chia hnh hp thnh hai hnh a din (H) v (H ), trong (H) l hnh a din cha nh A . Tnh th tch
ca (H) v (H ).
Bi 11.275 : Cho hnh hp ABCD.A BC D . Gi E, F ln lt l trung im BC , C D . Mt phng (AEF) chia hnh hp thnh
hai hnh a din (H) v (H ), trong (H) l hnh a din cha nh A . Tnh t s s gia th tch hnh a din (H) v th tch hnh a
din (H ).
Bi 11.276 : Cho khi hp MNPQ.M N P Q c th tch V. Tnh th tch khi t din P MNP theo V.
1
Bi 11.277 : Trn cnh PQ ca t din MNPQ ly im I sao cho PI = PQ. Tnh t s th tch ca hai t din MNIQ v MNIP.
3

Bi 11.278 : Cho hnh chp u S ABCD. y l ABCD l hnh vung c cnh bng 2a. Cnh bn S A = a 5. Mt mt phng (P) i
qua AB v vung gc vi mt phng (S CD), v (P) ln lt ct C v S D ti C v D .
1. Tnh din tch t gic ABC D .
2. Tnh th tch ca khi a din ABCDDC .
Bi 11.279 : Cho hnh chp t gic u S .ABCD c cnh y bng a, cnh bn bng a.
1. Tnh th tch khi chp.
2. Gi M, N, P l trung im ca AB, AD, S C. Chng minh rng mt phng (MNP) chia hnh chp thnh hai phn c th tch bng
nhau.

Vn 3 : Dng cng thc th tch gii mt s bi ton hnh hc


1. Chng ta c th dng cng thc t s th tch chng minh mt s h thc.
2. Hoc dng cng thc th tch tnh khong cch t mt im n mt phng hoc tnh din tch a gic y, c th
d(A, (S BC)) =

TRN ANH TUN - 0974 396 391 - (04) 66 515 343

3.VS ABC
3V
v tng qut h =
.
S S BC
S y

WWW.VNMATH.COM

Trang 228

www.VNMATH.com

www.luyenthi24h.com
www.luyenthi24h.com
CHUYN LUYN THI I HCwww.VNMATH.com

Bi 11.280 : Cho t din ABCD v im M min trong ca t din. ng thng AM ct mt phng (BCD) ti A , BM ct mt
phng (ACD) ti B , CM ct mt phng (ABD) ti C , DM ct mt phng (ABC) ti D . Chng minh rng :
V M.BCD
MA
=
VABCD
AA
MA MB MC MD
+
+
+
= 1.
AA
BB
CC
DD
Bi 11.281 : Cho hnh chp t gic u S .ABCD. Mt mt phng ct cc cnh S A, S B, S C, S D ln lt ti A , B , C , D . Chng minh
v suy ra

rng
1
1
1
1
+
=
+
.
S A S C S B S D
Bi 11.282 : Cho t din ABCD v M l mt im nm min trong tam gic BCD. V MB AB (B (ACD)); MC AC
V MACD
MB
(C (ABD)); MD AD (D (ABC)). Chng minh rng BM v AB ct nhau trn CD v
. T suy ra
=
VBACD
BA
MB MC MD
+
+
= 1.
BA
CA
DA
Bi 11.283 : Cho t din ABCD cso im O nm trong t din v cch u cc mt ca t din mt khong bng r. Gi hA , hB , hC , hD
ln lt l khong cch t cc im A, B, C, D n cc cnh i din. Chng minh rng
1
1
1
1
1
=
+
+
+
.
r hA hB hC hD
Bi 11.284 : Cho hnh chp tam gic S .ABC c y l tam gic vung B. Cnh S A vung gc vi y. Bit AB = a, BC = b, S A = c.
Hy tnh khong cch t A n mt phng (S BC).
Bi 11.285 : Cho hai ng thng cho nhau d v d . Trn d ly hai im A v B, trn d ly hai im C v D sao cho AB = a, CD = c.
Bit gc gia d v d bng 60 v khong cch gia d v d bng h.
1. Tnh th tch t din ABCD.
2. Gi s BC vung gc vi CD v BC = b. Tnh khong cch t A n mt phng (BCD).
Bi 11.286 : Cho hnh hp ch nht ABCD.A BC D c AB = a, BC = 2a, AA = a. Ly im M trn cnh AD sao cho AM = 3MD.
1. Tnh th tch hnh chp M.ABC.
2. Tnh khong cch t M n mt phng (ABC).
Bi 11.287 : Cho hnh lp phng ABCD.A BC D c cnh bng a; M, N ln lt l trung im ca AB v C D . Chng minh rng
A MCN l hnh thoi v tnh khong cch t B n (A MCN).

Bi 11.288 : Cho hnh chp S .ABC, y l tam gic u cnh a 3, ng cao S A = a. Mt phng qua A v vung gc vi S B ti H
ct S C ti K. Tnh S K v din tch tam gic AHK.
Bi 11.289 : Cho hnh lp phng ABCD.A BC D c cnh bng a. Tnh th tch khi t din BDC B v suy ra khong cch t B
n (BDC ).
CM
DN
Bi 11.290 : Cho t din ABCD c th tch bng V; M, N, P ln lt l cc im trn cc cnh AC, AD, BD sao cho
=
=
CA
DA
DP 2
= . Bit d(D, (MNP)) = h, tnh din tch tam gic MNP.
DB 3
Bi 11.291 : Cho hnh lp phng ABCD.A BC D c cnh bng a. Gi K l trung im DD . Tnh khong cch gia CK v A D.
Bi 11.292 : Trong khng gian, cho cc im A, B, C theo th t thuc cc tia Ox, Oy, Oz vung gc vi nhau tng i mt sao cho

OA = a(a > 0), OB = a 2, OC = c(c > 0). Gi D l nh i din vi O ca hnh ch nht AOBD v M l trung im ca on BC,
(P) l mt phng i qua A, M v ct mt phng (OCD) theo mt ng thng vung gc vi ng thng AM.
1. Gi E l giao im ca (P) vi ng thng OC, tnh di ca on thng OE.
2. Tnh t s th tch ca hai khi a din c to thnh khi ct khi chp C.AOBD bi mt phng (P).
3. Tnh khong cch t C n mt phng (P).

TRN ANH TUN - 0974 396 391 - (04) 66 515 343

WWW.VNMATH.COM

Trang 229

www.VNMATH.comCHUYN LUYN THI I HC

www.luyenthi24h.com
www.luyenthi24h.com
www.VNMATH.com

11.7 Phn loi mt s hnh khi a din


11.7.1 Hnh chp c cnh bn vung gc vi y

Bi 11.293 : Cho hnh chp S .ABCD c y ABCD l hnh thang vung ti A, hai y l AD = 2a, BC = a. Bit AB = a, S A = a 2
v S A(ABCD).
1. Chng minh cc tam gic S BC v S DC l cc tam gic vung.
2. K AJS B, AHS C. Chng minh rng : (JAH)(S DC).
3. Tnh gc gia cc cp mt phng sau : (S DC) v (ABCD) ; (S DC) v (S AD).
4. Xc nh v tnh di on vung gc chung ca cc cp ng thng AD v S B ; AD v S C.
Bi 11.294 : Cho hnh chp S .ABCD c y ABCD l hnh vung cnh a, S A vung gc vi mt phng (ABCD) v S A = a. Gi E
trung im ca cnh CD. Tnh theo a khong cch t im S n BE.
Bi 11.295 : Cho tam gic vung cn ABC c cnh huyn BC = a. Trn ng thng vung gc vi mt phng (ABC) ti im A ly
im S sao cho gc gia hai mt phng (ABC) v (S BC) bng 60 . Tnh di on thng S A theo a.
Bi 11.296 : Cho hnh chp O.ABC, vi OA, OB, OC i mt vung gc v OA = a, OB = b, OC = c.
1. Tnh khong cch t O n mt phng (ABC).
2. Chng minh rng tam gic ABC nhn.
3. Gi , , ln lt l gc to bi cc mt (OBC), (OCA), (OAB) vi mt (ABC). Chng minh rng cos2 + cos2 + cos2 = 1.

Bi 11.297 (B06) : Cho hnh chp S .ABCD c y ABCD l hnh ch nht vi AB = a, AD = a 2, S A = a v S A vung gc vi mt
phng (ABCD). Gi M v N ln lt l trung im ca AD v S C; I l giao im ca BM v AC. Chng minh rng mt phng (S AC)
vung gc vi mt phng (S MB). Tnh th tch ca khi t din ANIB.
Bi 11.298 (D02) : Cho hnh t din ABCD c cnh AD vung gc vi mt phng (ABC); AC = AD = 4cm ; AB = 3cm ; BC = 5cm.
Tnh khong cch t A ti mt phng (BCD).
Bi 11.299 (D06) : Cho hnh chp tam gic S .ABC c y ABC l tam gic u cnh a, S A = 2a v S A vung gc vi mt phng
(ABC). Gi M v N ln lt l hnh chiu vung gc ca A trn cc ng thng S B v S C. Tnh th tch ca khi chp A.BCN M.
= BAD
= 90 , BA = BC = a, AD = 2a. Cnh bn S A vung
Bi 11.300 (D07) : Cho hnh chp S .ABCD c y l hnh thang, ABC

gc vi y v S A = a 2. Gi H l hnh chiu ca A trn S B. Chng minh rng tam gic S CD vung v tnh theo a khong cch t

H n mt phng (S CD).
Bi 11.301 : Cho hnh chp S .ABC c y ABC l tam gic u cnha v cnh bn S A vung gc vi mt phng y (ABC). Tnh
a 6
khong cch t im A ti mt phng (S BC) theo a, bit rng S A =
.
2
Bi 11.302 : Cho t din OABC c ba cnh OA, OB, OC i mt vung gc v , , ln lt l cc gc gia mt phng (ABC) vi
cc mt phng (OBC), (OCA), (OAB). Chng minh rng :
cos + cos + cos

3.

Bi 11.303 : Cho hnh chp S .ABC c y ABC l tam gic vung ti B, AB = a, BC = 2a, cnh S A vung gc vi y v S A = 2a.
Gi M l trung im ca S C. Chng minh rng tam gic AMB cn ti M v tnh din tch tam gic AMB theo a.
Bi 11.304 : Cho t din ABCD c AD vung gc vi mt phng (ABC) v tam gic ABC vung ti A, AD = a, AC = b, AB = c.
Tnh din tch S ca tam gic BCD theo a, b, c v chng minh rng :
2S

abc(a + b + c).

Bi 11.305 : Cho hnh chp S .ABCD c y ABCD l hnh ch nht vi AB


= a, AD = 2a, cnh S A vung gc vi y, cnh S B to
a 3

vi mt phng y mt gc 60 . Trn cnh S A ly im M sao cho AM =


. Mt phng (BCM) ct S D ti N. Tnh th tch khi
2
chp S .BCN M.

TRN ANH TUN - 0974 396 391 - (04) 66 515 343

WWW.VNMATH.COM

Trang 230

www.luyenthi24h.com
www.luyenthi24h.com
CHUYN LUYN THI I HCwww.VNMATH.com

www.VNMATH.com

= 60 , S A vung gc vi mt phng (ABCD), S A = a.


Bi 11.306 : Cho hnh chp S .ABCD c y ABCD l hnh thoi cnh a, BAD

Gi C l trung im ca cnh S C. Mt phng (P) i qua AC v song song vi BD, ct cc cnh S B, S D ca hnh chp ln lt ti
B , D . Tnh th tch khi chp S .ABC D .

Bi 11.307 : Cho hnh chp S .ABCD c y ABCD l hnh vung tm O, S A vung gc vi y. Cho AB = a, S A = a 2. Gi H v
K ln lt l hnh chiu ca A ln S B, S D. Chng minh S C(AHK) v tnh th tch hnh chp O.AHK.
Bi 11.308 : Trong mt phng (P) cho na ng trn ng knh AB = 2R v im C thuc na ng trn sao cho AC = R. Trn
ng thng vung gc vi (P) ti A ly im S sao cho gc gia hai mt phng (S AB) v (S BC) bng 60 . Gi H, K l lt l hnh
chiu vung gc ca A trn S B, S C. Chng minh tam gic AHK vung v tnh VS .ABC .

11.7.2 Hnh chp u


NH NGHA : Hnh chp u l hnh chp c y l a gic u v cc cnh bn bng nhau (ng cao vung gc vi y ti tm
y).
Chp t gic th chn gc ta l tm y ; chp tam gic th chn gc ta l trung im M ca BC (hnh di) :
z
z
S
S

y
D

C
y

B
x
x
Cu hi : Hy tm ta cc nh ca hnh chp trong mi trng hp trn, bit di cnh y bng a v chiu cao bng h.
B

Cc yu t xc nh hnh chp u : bit cnh dy l a v


1. di ng cao l h ;
2. di cnh bn bng b, khi s tnh c chiu cao ;
3. gc gia cnh bn v mt y bng , khi s tnh c chiu cao ;
4. gc gia mt bn v mt y bng , khi s tnh c chiu cao.
Cu hi 1 : Cho hnh chp u S .ABCD c cnh y bng a. Hy gn hnh chp vo h trc ta Oxyz nh hnh trn v tm ta
cc nh ca hnh chp trong mi trng hp sau :

1. ng cao bng a 2 ;

3. Gc gia cnh bn v mt y bng 30 ;

2. Cnh bn bng a 3 ;

2
4. Csin gc gia mt bn v mt y bng .
3

Cu hi 2 : Hy lm bi ton trn vi gi thit l hnh chp u S .ABC c di cnh y bng a.


Ch : T din u l t din c tt c cc cch bng nhau.

Bi 11.309 : Cho hnh t din u c tt c cc cnh bng a = 6 2. Tnh khong cch gia hai ng thng AD v BC.

Bi 11.310 : Cho khi chp u S .ABCD c AB = a, gc gia mt bn v mt y bng 60 . Tnh th tch khi chp S .ABCD theo a.
Bi 11.311 (B07) : Cho hnh chp t gic u S .ABCD c y l hnh vung cnh a. Gi E l im i xng ca D qua trung im
ca S A, M l trung im ca AE, N l trung im ca BC. Chng minh MN vung gc vi BD v tnh (theo a) khong cch gia hai
ng thng MN v AC.

TRN ANH TUN - 0974 396 391 - (04) 66 515 343

WWW.VNMATH.COM

Trang 231

www.VNMATH.comCHUYN LUYN THI I HC

www.luyenthi24h.com
www.luyenthi24h.com
www.VNMATH.com

Bi 11.312 (A02) : Cho hnh chp tam gic u S .ABC c nh S , c di cnh y bng a. Gi M, N ln lt l cc trung im cc
cnh S B v S C. Tnh theo a din tch tam gic AMN, bit rng mt phng (AMN) vung gc vi mt phng (S BC).
Bi 11.313 (B04) : Cho hnh chp t gic u S .ABCD c cnh y bng a, gc gia cnh bn v mt y bng (0 < < 90 ).
Tnh tang ca gc gia hai mt phng (S AB) v (ABCD) theo . Tnh th tch khi chp S .ABCD theo a v .
Bi 11.314 : Cho hnh chp u S .ABC, y c cnh bng a, mt bn to vi y mt gc bng , vi (0 < < 90 ). Tnh th tch
khi chp S .ABC v tnh khong cch t A n mt phng (S BC).
Bi 11.315 : Cho hnh chp t gic u S .ABCD c cnh y bng a. Gi S H l ng cao ca hnh chp. Khong cch t trung
im I ca S H n mt phng (S BC) bng b. Tnh th tch khi chp S .ABCD.
Bi 11.316 : Tnh th tch khi t din ABCD, bit AB = a; AC = b; AD = c v cc gc BAC, CAD, DAB u bng 60 .

11.7.3 Hnh chp c mt bn vung gc vi y


Gii s mt bn (S AB) vung gc vi y (AB l cnh y v y c th l tam gic hoc t gic), ta c quy trnh v hnh nh sau :
Bc 1 : V a gic y ;
Bc 2 : V ng S H ca hnh chp, H AB. ty thuc vo tnh cht ca tam gic S AB m ta c v tr ca H, chng hn S AB l
tam gic cn ti S th H l trung im ca AB. Da vo cc yu t c th tnh c chiu cao S H.

Bi 11.317 : Cho hnh chp S .ABC vi tam gic S AB cn ti S , tam gic ABC vung cn ti C v (S AB)(ABC).
1. K S H(ABC). Chng minh H l trung im cnh AB v CH(S AB).
2. Gi M, N ln lt l trung im ca cc cnh AC v BC. Chng minh rng :
(a) (S HM)(S AC) v (S HN)(ABC).
(b) Hai mt bn (S AC) v (S BC) cng to vi y (ABC) hai gc bng nhau.
(c) d(H, (S AC)) = d(H, (S BC)).
3. Gi D l im i xng ca C qua H. Chng minh rng S .ADBC l hnh chp t gic u.
Bi 11.318 (A07) : Cho hnh chp S .ABCD c y l hnh vung cnh a, mt bn S AD l tam gic u v nm trong mt phng
vung gc vi y. Gi M, N, P ln lt l trung im cc cnh S B, BC, CD. Chng minh AM vung gc vi BP v tnh th tch khi
t din CMNP.

Bi 11.319 (B08) : Cho hnh chp S .ABCD c y ABCD l hnh vung cnh 2a, S A = a, S B = a 3 v mt phng (S AB) vung
gc vi mt y. Gi M, N ln lt l trung im cc cnh AB, BC. Tnh theo a th tch khi chp S .BMDN v tnh cosin gc gia hai
ng thng S M, DN.
= 90 .
Bi 11.320 : Cho t din ABCD vi AB = AC = a, BC = b. Hai mt phng (BCD) v (ABC) vung gc vi nhau v gc BDC

Xc nh v tnh bn knh mt cu ngoi tip t din ABCD theo a v b.

Bi 11.321 : Cho hnh chp S .ABC c y ABC l tam gic vung ti A, AB = a, AC = a 3, mt bn S BC l tam gic u v vung
gc vi mt phng y. Tnh theo a th tch khi chp S .ABC.
Bi 11.322 : Cho hnh chp S .ABCD c y l hnh vung cnh a, S A = AB = a, mt phng (S AB) vung gc vi mt phng y
(ABCD), tam gic S AB vung. Tnh bn knh mt cu ngai tip hnh chp S .ABD.
Bi 11.323 : y ca mt hnh chp l mt tam gic vung c cnh huyn bng a v mt gc nhn bng 60 . Mt bn cha cnh
huyn vung gc vi y, cc mt cn li cng hp vi y mt gc .
1. Tnh th tch khi chp ny.
2. Mt mt phng qua cnh huyn ca tam gic y v ct cnh i din ti trung im. Tnh t s th tch hai phn ca hnh chp
ct bi mt phng .
Bi 11.324 : Cho hnh chp S .ABC c bng bn (S BC) vung gc vi y, hai mt bn (S AB) v (S AC) cng lp vi y gc 45 ,
y ABC l tam gic vung cn ti A v c AB = a ng thi S BC l tam gic nhn. Tnh th tch khi chp.

TRN ANH TUN - 0974 396 391 - (04) 66 515 343

WWW.VNMATH.COM

Trang 232

www.VNMATH.com

www.luyenthi24h.com
www.luyenthi24h.com
CHUYN LUYN THI I HCwww.VNMATH.com

Bi 11.325 : Cho hnh chp S .ABC c hai tam gic ABC v S BC l hai tam gic u cnh a v (S BC) vung gc vi y. Tnh th
tch khi chp.
Bi 11.326 : Cho hnh chp S .ABCD c y ABCD l hnh vung cnh a. Mt bn S AB l tam gic u v vung gc vi y. Gi H
l trung im ca AB v M l mt im di ng trn ng thng BC.
1. Chng minh rng : S H(ABCD). Tnh th tch hnh chp S .ABCD.
2. Tm tp hp hnh chiu vung gc ca S ln DM.
3. Tnh khong cch t S n DM theo a v x = CM.

11.7.4 Hnh chp c hai mt vung gc vi y


Nu hai mt vung gc vi y th giao tuyn ca hai mt phng ny s vung gc vi y. V vy ta cng s xc nh c ngay
phng ca ng cao.

Bi 11.327 : Cho
hnh chp S .ABC, tam gic y ABC c AB = a, B = 45 , C = 30 , hai mt bn (S AB) v (S AC) vung gc vi y
a 6
. Gi H l hnh chiu vung gc ca S trn BC.
(ABC), S A =
2

1. Chng minh rng : (S AH)(S BC) v tnh khong cch t H n mt phng (S AC).
2. Tnh gc gia hai mt phng (ABC) v (S BC).
Bi 11.328 : Cho hnh chp S .ABC c y l tam gic ABC cn ti A v c trung tuyn AD = a. Hai mt bn S AB v S AC vung gc
vi y. Cnh bn S B hp vi y mt gc v hp vi mt phng (S AD) mt gc .
1. Chng minh rng : S B2 = S A2 + AD2 + BD2 .
2. Tnh th tch khi chp.
3. Tnh khong cch t A n (S BC).
Bi 11.329 : Cho hnh chp S .ABCD c y l hnh thoi, gc nhn A = . Hai mt bn (S AB) v (S AD) vung gc vi y v hai
mt bn cn li hp vi y gc , bit S A = a.
1. Tnh th tch v din tch xung quanh khi chp.
2. Tnh cosin gc gia S B v mt phng (S AC).

11.7.5 Hnh chp c cc cnh bn bng nhau hoc cc cnh bn cng to vi y nhng gc bng nhau
Nu cc cnh bn ca hnh chp l bng nhau hoc cc cnh bn cng to vi y nhng gc bng nhau th ng cao ca hnh chp
s i qua tm (ng trn ngoi tip) ca a gic y.
B = 60 , BS

Bi 11.330 : Xc nh tm v tnh bn knh mt cu ngoi tip hnh chp S .ABC, bit S A = S B = S C = a, AS


C = 90 ,
A = 120 .
CS

Bi 11.331 : Cho hnh chp S .ABCD c y ABCD l hnh ch nht vi AB = 2a, BC = a. Cc cnh bn ca hnh chp bng nhau v

cng bng a 2.
1. Tnh th tch khi chp.
2. Gi M, N, E, F ln lt l trung im ca cc cnh AB, CD, S C, S D. Chng minh S N vung gc vi (MEF).
3. Tnh khong cch t A n mt phng (S CD).
= 60 . Bit S O =
Bi 11.332 : Cho hnh chp S .ABCD c y ABCD l hnh thoi tm O, cnh bng a, BAD

Gi H v K ln lt l trung im BC v BH.

3a
v S O(ABCD).
4

1. Chng minh rng : (S OK)(S BC).


2. Tnh gc gia hai mt phng (S BC) v (ABCD).

TRN ANH TUN - 0974 396 391 - (04) 66 515 343

WWW.VNMATH.COM

Trang 233

www.VNMATH.comCHUYN LUYN THI I HC

www.luyenthi24h.com
www.luyenthi24h.com
www.VNMATH.com

3. Tnh khong cch t A n mt phng (S BC).


4. Cho (P) l mt phng qua AD v vung gc vi mt phng (S BC). Xc nh thit din ca hnh chp khi ct bi mt phng (P).
Tnh din tch thit din .

= 60 , cc cnh S A = S B = S D = a 3.
Bi 11.333 : Cho hnh chp S .ABCD c y ABCD l hnh thoi cnh a, vi BAD
1. Chng minh tam gic S BC vung ;

2. Tnh khong cch gia S C v AD.

11.7.6 Hnh hp - Hnh lng tr


Bi 11.334 : Cho hnh hp ch nht ABCD.A BC D vi AB = a, BC = b, CC = c.
1. Tnh khong cch t im A n (A BD).
2. Tnh khong cch t im A ti ng thng C D.
3. Tnh khong cch gia hai ng thng BC v CD .
Bi 11.335 : Cho khi lng tr tam gic ABC.A BC c y l tam gic u cnh a, im A cch u ba im A, B, C v cnh bn
AA to vi mt y gc 60 .
1. Tnh th tch khi lng tr .
2. Chng minh mt bn BCC B l hnh ch nht.
3. Tnh din tch xung quanh ca khi lng tr .
Bi 11.336 : Cho hnh lp phng ABCD.A BC D cnh a. Trn cc cnh AA , BC, C D ln lt ly cc im M, N, P sao cho
AM = CN = D P = t, vi 0 < t < a. Chng minh rng (MNP) (ACD ) v tnh khong cch gia hai mt phng .
Bi 11.337 (A03) : Cho hnh lp phng ABCD.A BC D . Tnh s o gc gia hai mt phng (A BC) v (ACD).

Bi 11.338 (D08) : Cho lng tr ng ABC.A BC c y ABC l tam gic vung, AB = BC = a, cnh bn AA = a 2. Gi M l
trung im cnh BC. Tnh theo a th tch khi lng tr ABC.A BC v khong cch gia hai ng thng AM, BC.

Bi 11.339 (A08) : Cho lng tr ABC.A BC c di cnh bn bng 2a, y ABC l tam gic vung ti A, AB = a, AC = a 3 v
hnh chiu vung gc ca nh A trn mt phng (ABC) l trung im ca cnh BC. Tnh theo a th tch khi chp A .ABC v tnh
cosin ca gc gia hai ng thng AA , BC .
= 60 . Gi M l trung
Bi 11.340 (B03) : Cho hnh lng tr ng ABCD.A BC D c y ABCD l mt hnh thoi cnh a, gc BAD

im cnh AA v N l trung im cnh CC . Chng minh rng bn im B , M, D, N cng thuc mt mt phng. Tnh di cnh
AA theo a t gic B MDN l hnh vung.
Bi 11.341 : Cho lng tr ng ABC.A1 B1C1 c tt c cc cnh u bng a. M l trung im ca on AA1 . Chng minh BMB1C
v tnh khong cch gia hai ng thng BM v B1C.
= 120, cnh bn BB = a.
Bi 11.342 : Cho lng tr ng ABC.A BC c y ABC l tam gic cn, vi AB = AC = a v gc BAC

Gi I l trung im CC . Chng minh rng tam gic AB I vung A. Tnh cosin ca gc gia hai mt phng (ABC) v (AB I).
Bi 11.343 : Cho hnh lp phng ABCD.A BC D . Tm im M thuc cnh AA sao cho mt phng (BD M) ct hnh lp phng
theo mt thit din c din tch nh nht.

a 3
= 60 . Gi M, N ln lt l
Bi 11.344 : Cho hnh hp ng ABCD.A B C D c cc cnh AB = AD = a AA =
v gc BAD
2
trung im cc cnh A D , A B . Chng minh rng AC vung gc vi mt phng (BDMN). Tnh th tch khi chp A.BDMN.

Bi 11.345 : Cho hnh lng tr ABC.A BC c A ABC l hnh chp u, cnh y AB = a, cnh bn AA = b. Gi l gc gia hai
mt phng (ABC) v (A BC). Tnh tan v th tch khi a din A BBC C.
2a
. Mt phng () i
3
qua A, K v song song vi BD, chia khi lp phng thnh hai khi a din. Tnh th tch ca hai khi a din .

Bi 11.346 : Cho hnh lp phng ABCD.A BC D c cnh bng a v im K thuc cnh CC sao cho CK =

TRN ANH TUN - 0974 396 391 - (04) 66 515 343

WWW.VNMATH.COM

Trang 234

www.VNMATH.com

www.luyenthi24h.com
www.luyenthi24h.com
CHUYN LUYN THI I HCwww.VNMATH.com

= 120. Gi M l trung im ca cnh


Bi 11.347 : Cho hnh lng tr ng ABC.A1 B1C1 c AB = a, AC = 2a, AA1 = 2a 5 v BAC
CC1 . Chng minh MBMA1 v tnh khong cch d t im A ti mt phng (A1 BM).

Bi 11.348 : Cho lng tr ng ABC.A1 B1C1 c y ABC l tam gic vung AB = AC = a, AA1 = a 2. Gi M, N ln lt l trung
im ca on AA1 v BC1 . Chng minh MN l ng vung gc chung ca cc ng thng AA1 v BC1 . Tnh V M.A1 BC1 .

a 2

.
Bi 11.349 : Cho hnh lp phng ABCD.A B C D c on ni hai tm ca hai mt bn k nhau l
2
1. Tnh th tch hnh lp phng.
2. Ly im M trn BC. Mt phng (MB D) ct A D ti N. Chng minh rng MNC D.
3. Tnh gc gia hai mt phng (A BD) v mt phng (ABCD).
Bi 11.350 : Cho lng tr ABC.A BC c y ABC l tam gic u tm O v hnh chiu ca C trn y (ABC) trng vi O. Bit
= 120 .
khong cch t O n CC bng a. Gi E l hnh chiu ca A ln CC v gc AEB

1. Chng minh mt bn ABB A l hnh ch nht.


2. Tnh th tch lng tr.
3. Tnh gc gia mt bn BCC B v mt y ABC.
Bi 11.351 : Cho hnh hp ABCD.A BC D c cc mt u l hnh thoi cnh a. Ba cnh xut pht t nh A to vi nhau cc gc
nhn bng nhau v cng bng .
1. Chng minh rng hnh chiu H ca A trn (ABCD) nm trn ng cho AC.
2. Tnh th tch hnh hp.
3. Tnh gc ca ng cho CA v mt y ca hnh hp.

11.8 Bi tp tng hp
Bi 11.352 : Cho hnh vung ABCD v tam gic S AB u cnh a trong hai mt phng vung gc vi nhau. Gi I, J, K ln lt l
trung im cc cnh AB, CD, BC.
1. Chng minh rng S I(ABCD).
2. Tnh gc gia S A, S B, S C v (ABCD).
3. Gi H l hnh chiu vung gc ca I trn S J. Chng minh rng IH(S CD). T suy ra gc gia S I v (S CD).
4. Chng minh rng S AD v S BC l cc tam gic vung. Tnh khong cch t I n (S KD).
5. Chng minh (S AD), (S BC) cng vung gc vi (S AB). Tnh gc gia S C, S D v (S AB).
Bi 11.353 : Cho hnh chp S .ABCD c y l hnh ch nht ABCD tm O, cnh bn S A vung gc vi mt phng y. Gi E, F, G, H
ln lt l hnh chiu vung gc ca A trn S B, S C, S D, S O. Chng minh rng
1. Cc mt bn ca hnh chp l cc tam gic vung.
2. (S BC)(S AB) v (S CD)(S AD).
3. Nu ABCD l hnh vung th AH(S BD) v H l trc tm tam gic S BD.
4. Cc im A, E, F, G ng phng v (S AC)(AEFG).

5. T gic AEFG ni tip v S E.vecS B = S F.S C = S G.S I.


6. Nu ABCD l hnh vung th hai ng cho ca t gic AEFG vung gc vi nhau.
Bi 11.354 : Cho hnh chp S .ABCD c y l hnh vung cnh a, tm O, cnh bn S A vung gc vi mt phng y. Gc gia S C
v (S AB) bng 30 . Gi E, F, G, H ln lt l hnh chiu vung gc ca A trn S B, S C, S D, S O.

TRN ANH TUN - 0974 396 391 - (04) 66 515 343

WWW.VNMATH.COM

Trang 235

www.VNMATH.comCHUYN LUYN THI I HC

www.luyenthi24h.com
www.luyenthi24h.com
www.VNMATH.com

1. Tnh gc gia
(a) S B v (ABCD), (S AD), (S CD), (S AC), (AEFG).
(b) (S AB) v (S CD); (S AD) v (S BC); (S BC) v (S CD).
(c) (AEFG) v cc mt phng ca hnh chp.
2. Tnh khong cch theo a
(a) T A n (S BC), (S CD), (S BD).
(b) Gia BD v (AEFG).
(c) Gia cc cnh i din ca t din S BCD.
3. Trn cnh AB ly mt im M v t AM = x (0 < x < a). Mt phng (Q) qua M v vung gc vi AB ct CD, S C, S B theo th
t N, P, Q.
(a) Xc nh hnh dng ca thit din MNPQ. Tnh theo a v x chu vi v din tch ca thit din .
(b) Gi I l trung im ca S C, J l hnh chiu vung gc ca I trn CM. Tm tp hp ca J khi x bin thin trong khong
(0; a).
Bi 11.355 : Cho hnh chp S .ABCD c y l hnh thang vung c ng cao AD = a v AB CD, vi AB = 2a, CD = a,

S A(ABCD), S A = a 2.
1. Tnh khong cch
(a) T im A n cc mt phng (S CD) v (S BC).
(b) T cc im B, C, D n cc mt phng ca hnh chp khng cha n.
(c) T CD n (S AB); AB n (S CD); DE n (S BC) vi E l trung im AB.
(d) Gia S A v BC; S B v C; S D v AB; S D v BC; S C v AB; S C v AD.
2. Tnh gc gia hai mt phng (S BC) v (S CD).
3. Gi M l im di ng trn cnh AD vi AM = x (M khng trng vi A v D). Mt phng (Q) qua M song song vi (S CD) ct
BC, S B, S A theo th t ti N, P, Q.
(a) Hi t gic MNPQ l hnh g? Tnh din tch MNPQ theo a v x.
(b) Tm qu tch giao im I ca MQ v NP khi M chy trn AD.
Bi 11.356 : Trong mt phng () cho ng trn tm O, ng knh AB = 2R. Trn ng thng d vung gc vi () ti A ly im

S . Gi M l mt im thuc ng trn tm O; D, E theo th t l hnh chiu vung gc ca A trn S B, S M. Gi s S A = R 3, gc


gia S M v () bng 60 . Tnh
1. Gc gia S A v (S BM); S B v (S AM); S M v (S AB); S M v (ADE).
2. Gc gia (S BM) v (); (S BM) v (S AB); (ADE) v (S AM).
3. Khong cch t M n (S AB); t S n (ADE); t A n (S BM). Khong cch gia cc cnh i nhau ca hnh t din S ABM.
4. Trn on thng AB ly im I, t AI = x (0 < x < 2R). Mt phng qua I vung gc vi AM ct AM, S M, S B ln lt ti
J, K, L. Xc nh hnh dng v tnh din tch thit din I JKL theo R, x. Tm x I JKL c din tch ln nht.
Bi 11.357 : Cho ba na ng thng S x, S y, S z khng ng phng v vung gc vi nhau tng i mt. Trn S x, S y, S z ln lt ly
cc im A, B, C khc im S . t S A = a, S B = b, S C = c. Gi , , l gc gia mt phng (ABC) vi cc mt phng (S BC),
(S CA), (S AB). Ly P, Q, R ln lt l trung im cc cnh BC, CA, AB. Gi G, H, O, r ln lt l trng tm, trc tm, tm ng trn
ngoi tip v bn knh ng trn ngoi tip tam gic ABC. Gi D l im i xng ca H qua S . Chng minh rng
1. Cc cp i ca t din S ABC vung gc vi nhau tng i mt v cos2 + cos2 + cos2 = 1.

TRN ANH TUN - 0974 396 391 - (04) 66 515 343

WWW.VNMATH.COM

Trang 236

www.luyenthi24h.com
www.luyenthi24h.com
CHUYN LUYN THI I HCwww.VNMATH.com

www.VNMATH.com
2. S H(ABC) v

1
1
1
1
=
+
+ .
S H 2 a 2 b 2 c2

3. S 2ABC = S S2 BC + S S2 AC + S S2 AB , S S2 BC = S HBC .S ABC .


4.

9
3S ABC S S BC + S S CA + S S AB S H 2 , (BC + CA + AB)2 6(a2 + b2 + c2 ).
2

5. Tam gic ABC c c ba gc u nhn v a2 tan A = b2 tan B = c2 tan C = 2S ABC .

a 3
, S B = S D = a.
Bi 11.358 : Hnh chp S .ABCD c y ABCD l hnh thoi tm O cnh a, S O(ABCD). Gi s OB =
2
1. Chng minh rng tam gic S AC vung ti S , S CBD, (S AC)(S BD).
= 60 , S O =
2. Gi s BDA

3a
. Gi E l trung im BC, F l trung im BE. Chng minh rng (S OF)(S BC).
4

3. Tnh khong cch t O v A n mt phng (S BC).


4. Gi () l mt phng qua AD v vung gc vi mt phng (S BC). Xc nh thit din ca hnh chp ct bi (). Tnh din tch
thit din ny v gc gia () v mt phng (ABCD).
5. Tm tp hp hnh chiu vung gc ca O trn (S CD) khi S di ng trn ng thng qua O v vung gc vi (ABCD).

Bi 11.359 : Cho hnh chp t gic t u S .ABCD, c AB = a, S A = a 2. Qua im A dng mt phng () vung gc vi S C.
1. Dng thit din to bi ()vi hnh chp.
2. Mt phng () chia khi chp trn thnh 2 phn c t s th tch bng bao nhiu?
Bi 11.360 : Cho hnh chp S .ABCD c y ABCD l hnh vung, mt bn S AB l tam gic u v vung gc vi y. Tinh th tch
khi chp S .ABCD bit khong cch gia hai ng thng AB v S C bng a.

Bi 11.361 : Cho t din ABCD c ABC l tam gic u v BCD l tam gic cn ti D. Cho bit AB = a, CD = a 5, gc gia hai
mt phng (ABC) v (BCD) bng 30 . Tnh khong cch gia AD v BC.
Bi 11.362 : Cho lng tr tam gic u ABC.A BC c AB = 1, CC = m. Tm m bit rng gc gia hai ng thng AB v BC bng
60 .
= (0 < < 90 ). Gi H, K ln lt l hnh chiu
Bi 11.363 : Cho t din ABCD c AD(ABC), AB = AD = 1, AC = 2, BAC

vung g ca B ln AC v CD. ng thng HK ct tia i ca tia AD ti E. Chng minh BECD v tnh th tch khi t din BCDE.

Bi 11.364 : Cho hnh chp S .ABC


c S C (ABC) v tam gic ABC vung ti B. Bit AB = a, AC = a 3. Gc gia 2 mt phng
13
(S AC) v (S AB) bng vi tan =
. Tnh th tch khi chp S .ABC theo a.
6
Bi 11.365 : Cho t din OABC c cc gc phng nh O u bng 90 . Bit rng din tch tam gic ABC bng 1 v tng din tch

cc tam gic OAB, OBC, OCA bng 3. Tnh th tch khi t din OABC.

Bi 11.366 : Cho hnh chp S .ABCD c y ABCD l hnh ch nht, AB = a 2, CD = 2a, S A(ABCD), S A = 3a 2. Gi K l
trung im ca AB. Chng minh rng (S AC)(S KD) v tnh th tch khi chp S .CDK.

a 3
= 60 . Gi M v N l lt l trung im
Bi 11.367 : Cho hnh hp ng ABCD.A BC D c AB = AD = a, AA =
v BAD
2
A D v A B . Chng minh rng AC vung gc vi mt phng (BDMN). Tnh th tch khi a din ABDMN.

Bi
11.368 : Cho hnh lng tr tam gic u ABC.A B C c cnh y bng a. Bit khong cch gia hai ng thng AB v A C bng
a 15
. Tnh th tch ca khi lng tr.
5
Bi 11.369 : Cho hnh chp S .ABCD c y ABCD l hnh thang vung ti A v D, AB = AD = a, CD = 2a. Cnh bn S D vung

gc vi mt phng (ABCD), S D = a 3.

1. Tnh th tch khi chp S .ABCD theo a.


2. Gi G l trng tm tam gic BCD. Tnh khong cch t G n mt phng (S BC).
= 60 , ng cho BC ca mt bn
Bi 11.370 : Cho lng tr ng ABC.A BC c y ABC l tam gic vung ti A, AC = a, ACB

BBC C to vi mt bn (AAC C) mt gc 30 .

TRN ANH TUN - 0974 396 391 - (04) 66 515 343

WWW.VNMATH.COM

Trang 237

www.VNMATH.comCHUYN LUYN THI I HC

www.luyenthi24h.com
www.luyenthi24h.com
www.VNMATH.com

1. Tnh th tch khi t din C ABC.


2. Tnh din tch mt cu ngoi tip hnh lng tr.
Bi 11.371 : Trong mt phng (P) cho ng trn (C ) tm O ng knh AB = 2R. Trn ng thng vung gc vi (P) ti O ly

2R
im S sao cho OS = R 3. I l im thuc on OS vi S I = . M l mt im thuc (C ) (M khng trng vi A v B). H l hnh
3
chiu ca I trn S M. Tm v tr ca M trn (C ) t din ABHM c th tch ln nht. Tm gi tr ln nht .
b=C
b = . Cc cnh bn S A, S B, S C cng
Bi 11.372 : Cho hnh chp S .ABC c y l tam gic nhn v cn A, c AB = AC = a; B

to vi y mt gc (0 < < 90 ). Tnh th tch khi chp S .ABC.

Bi 11.373 : Cho lng tr tam gic u ABC.A BC c AA = a 2 v A BBC. Gi M l trung im ca on AB. Chng minh
rng A BB M. Tnh th tch khi chp A .ABC.
Bi 11.374 : Cho hnh chp S .ABCD c S A = x v tt c cc cnh cn li c di bng a. Chng minh rng BD(S AC). Tm x
a3 2
theo a th th tch khi chp S .ABCD bng
.
6

Bi 11.375 : Cho lng tr ng ABC.A BC c y ABC l tam gic vung vi AB = BC = a, cnh bn AA = a 2. M l im trn
1
AA sao cho AM = AA . Tnh th tch khi t din MA BC .
3
a2
Bi 11.376 : Cho hnh chp S .ABC c y ABC l tam gic u cnh bng a v S A.S B = S B.S C = S C.S A = . Tnh th tch khi
2
chp S .ABC theo a.
Bi 11.377 : Trong khng gian cho tam gic vung cn ABC c cnh huyn AB = 2a. Trn ng thng d i qua A v vung gc vi
mt phng (ABC) ly im S sao cho mt phng (S BC) to vi mt phng (ABC) mt gc 60 . Tnh din tch mt cu ngoi tip t
din S ABC.
Bi 11.378 : Cho hnh chp S .ABC c y l tam gic u cnh a, mt bn (S BC) vung gc vi mt y, hai mt bn cn li to vi
y mt gc . Tnh th tch khi chp S .ABC.
Bi 11.379 : Hnh chp t gic u S .ABCD c khong cch t A n mt phng (S BC) bng 2. Vi gi tr no ca gc gia mt
bn v mt y ca chp th th tch ca chp nh nht?

Bi 11.380 : Co lng tr ng ABC.A BC c y l tam gic vung cn vi AB = AC = a, cnh bn AA = a 2. M l trung im


ca A B . Dng v tnh din tch ca thit din ca lng tr ct bi mt phng (P) i qua M v vung gc vi BC .
Bi 11.381 : Cho t din ABCD c ba cnh AB, BC, CD i mt vung gc vi nhau v AB = BC = CD = a. Gi C , D ln lt l
hnh chiu vung gc ca B trn AC v AD. Tnh th tch t din ABC D .
Bi 11.382 : Cho hnh chp S .ABC c y l tam gic ABC vung cn ti nh B, BA = BC = 2a, hnh chiu vung gc ca S trn
mt phng y (ABC) l trung im E ca AB v S E = 2a. Gi I, J ln lt l trung im ca EC, S C; M l im di ng trn tia i
= (0 < < 90 ) v H l hnh chiu vung gc ca S trn MC. Tnh th tch ca khi t din EHI J
ca tia BA sao cho gc ECM

theo a, v tm th tch ln nht.

Bi 11.383 : Cho hnh chp S .ABCD c y l hnh vung cnh a, S A = a 3 v S A vung gc vi mt phng y. Tnh th tch khi
t din S ACD v tnh cosin ca gc gia hai ng thng S B v AC.
Bi 11.384 : Cho t din ABCD v cc im M, N, P ln lt thuc cc cnh BC, BD, AC sao cho BC = 4BM, BD = 2BN, AC = 3AP.
AQ
Mt phng (MNP) ct AD ti Q. Tnh t s
v t s th tch hai phn ca khi t din ABCD c phn chia bi mt phng (MNP).
AD
Bi 11.385 : Cho hnh chp S .ABC m mi mt bn l mt tam gic vung, S A = S B = S C = a. Gi N, M, E ln lt l trung im
ca cc cnh AB, AC, BC ; D l im i xng ca S qua E ; I l giao im ca ng thng AD vi mt phng (S MN). Chng minh
rng AD vung gc vi S I v tnh theo a th tch ca khi t din MBS I.
Bi 11.386 : Cho t din ABCD c cc mt ABC v ABD l cc tam gic u cnh a, cc mt ACD v BCD vung gc vi nhau. Hy
tnh theo a th tch khi t din ABCD v tnh s o ca gc gia hai ng thng AD, BC.
Bi 11.387 : Cho hnh lp phng ABCD.A BC D c cnh bng a. K l giao im ca AC v mt phng (A BD). Tnh th tch t
din KCC D v khong cch t C n mt phng (KC D ).

TRN ANH TUN - 0974 396 391 - (04) 66 515 343

WWW.VNMATH.COM

Trang 238

www.luyenthi24h.com
www.luyenthi24h.com
www.VNMATH.com

www.VNMATH.com

Chng 12

Mt cu v khi trn xoay


12.1 Mt cu, khi cu

iu kin cn v mt hnh chp c mt cu ngoi tip l y ca hnh chp c ng trn ngoi tip; iu kin cn v mt
hnh lng tr c mt cu ngoi tip l hnh lng tr phi l mt hnh lng tr ng v c y l mt a gic c ng trn ngoi tip.
Phng php chung xc nh tm mt cu ngoi tip khi chp v khi lng tr l :
(i) Xc nh tm (ng trn ngoi tip) ca a gic y.
(ii) Xc nh trc ca y (l ng thng qua tm y v vung gc vi y).
(iii) Xc nh mt trung trc ca mt cnh bn. Mt trung trc ny ct trc ca y ti u th l tm ca mt cu ngoi tip.
Ch :
1. Nu c mt cnh bn no vung gc vi y (tng qut l ng phng vi trc ca y), ta thay vic dng mt phng trung
trc bi dng ng trung trc ca cnh bn ny trong mt phng to bi ng trung trc v trc ca y.
2. Tm mt cu ngoi tip cng c th xc nh l giao ca trc ca a gic y v trc ca mt mt bn.

WWW.VNMATH.COM

= 120 v ng cao AH = a 2. Trn ng thng vung gc vi mt phng (ABC) ti


Bi 12.1 : Cho tam gic cn ABC c BAC
A ly hai im I v J v hai pha ca im A sao cho IBC l tam gic u v JBC l tam gic vung cn.
1. Tnh theo a di cc cnh ca tam gic ABC.
2. Tnh theo a di AI, AJ.
3. Chng minh rng BI J, CI J l cc tam gic vung.
4. Xc nh tm v tnh theo a bn knh ca mt cu ngoi tip t din I JBC.
5. Xc nh tm v tnh theo a bn knh mt cu ngoi tip t din IABC.
Bi 12.2 : Cho hnh chp S .ABCD c y l hnh ch nht v S A vung gc vi mt phng y. Gi B , C , D ln lt l hnh chiu
vung gc ca A trn S B, S C, S D. Chng minh rng
1. Cc im A, B, C , D ng phng.
2. By im A, B, C, D, B, C , D nm trn mt mt cu.
3. Hnh chp S .ABCD ni tip mt mt cu.

239

WWW.VNMATH.COM

www.luyenthi24h.com
www.luyenthi24h.com

Chng 12

Mt cu v khi trn xoay


12.1 Mt cu, khi cu

iu kin cn v mt hnh chp c mt cu ngoi tip l y ca hnh chp c ng trn ngoi tip; iu kin cn v mt
hnh lng tr c mt cu ngoi tip l hnh lng tr phi l mt hnh lng tr ng v c y l mt a gic c ng trn ngoi tip.
Phng php chung xc nh tm mt cu ngoi tip khi chp v khi lng tr l :
(i) Xc nh tm (ng trn ngoi tip) ca a gic y.
(ii) Xc nh trc ca y (l ng thng qua tm y v vung gc vi y).
(iii) Xc nh mt trung trc ca mt cnh bn. Mt trung trc ny ct trc ca y ti u th l tm ca mt cu ngoi tip.
Ch :
1. Nu c mt cnh bn no vung gc vi y (tng qut l ng phng vi trc ca y), ta thay vic dng mt phng trung
trc bi dng ng trung trc ca cnh bn ny trong mt phng to bi ng trung trc v trc ca y.
2. Tm mt cu ngoi tip cng c th xc nh l giao ca trc ca a gic y v trc ca mt mt bn.

= 120 v ng cao AH = a 2. Trn ng thng vung gc vi mt phng (ABC) ti


Bi 12.1 : Cho tam gic cn ABC c BAC
A ly hai im I v J v hai pha ca im A sao cho IBC l tam gic u v JBC l tam gic vung cn.
1. Tnh theo a di cc cnh ca tam gic ABC.
2. Tnh theo a di AI, AJ.
3. Chng minh rng BI J, CI J l cc tam gic vung.
4. Xc nh tm v tnh theo a bn knh ca mt cu ngoi tip t din I JBC.
5. Xc nh tm v tnh theo a bn knh mt cu ngoi tip t din IABC.
Bi 12.2 : Cho hnh chp S .ABCD c y l hnh ch nht v S A vung gc vi mt phng y. Gi B , C , D ln lt l hnh chiu
vung gc ca A trn S B, S C, S D. Chng minh rng
1. Cc im A, B, C , D ng phng.
2. By im A, B, C, D, B, C , D nm trn mt mt cu.
3. Hnh chp S .ABCD ni tip mt mt cu.

239

WWW.VNMATH.COM

www.VNMATH.com

CHUYN LUYN THI I HC

www.luyenthi24h.com
www.luyenthi24h.com
www.VNMATH.com

Bi 12.3 : Cho tam gic ABC vung ti C. ng thng vung gc vi mt phng (ABC) ti A. im S thay i trn (S khc A).
H ADS C v AES B. Chng minh rng
1. Cc im A, B, C, D, E thuc cng mt mt cu.
2. Bn im B, C, D, E cng mt ng trn.
Bi 12.4 : Cho hnh chp tam gic u S .ABC c y l tam gic u cnh a, mt bn to vi y gc . Tnh bn knh ca mt cu
ngoi tip hnh chp.
Bi 12.5 : Cho t din ABCD c bn knh mt cu ni tip r. Gi S tp l tng din tch cc mt ca t din; hA , hB, hC , hD ln lt l
di ng cao xut pht t A, B, C, D ca t din. Chng minh rng
1
r.S tp .
3
1
1
1
1
1
2.
=
+
+
+
.
r
hA hB hC hD
1. VABCD =

Bi 12.6 : Cho hnh chp u S .ABCD c cnh y bng a. Gi M v N ln lt l trung im ca S B v S D. Bit AMCN. Tnh
bn knh mt cu ngoi tip hnh chp S .ABCD.

Bi 12.7 : Cho hnh chp S .ABCD c y ABCD l hnh vung cnh a, cnh bn S A vung gc vi y, cnh bn S B = a 3.
1. Tnh th tch khi chp S .ABCD.
2. Chng minh rng trung im ca cnh S C l tm mt cu ngoi tip hnh chp S .ABCD.
Bi 12.8 : Cho hnh chp S .ABCD c y l na lc gic u, AB = 2a, BC = CD = DA = a, S A vung gc vi y, S A = h. Mt
phng qua A vung gc vi S B, ct S B, S C, S D ln lt ti B , C , D .
1. Chng minh rng t gic A, B, C , D ni tip mt ng trn.
2. Chng minh rng cc im A, B, C, D, B, C , D thuc cng mt mt cu.
3. Tnh th tch khi chp S .ABC D .
4. Tnh din tch t gic ABC D .
Bi 12.9 : Cho t din ABCD c AB = CD = c, AD = BC = a, AC = BD = b. Tnh bn knh mt cu ngoi tip t din.
Bi 12.10 : Cho t din OABC vung ti O, OA = a, OB = b, OC = c. Tnh bn knh mt cu ngoi tip t din.
Bi 12.11 : Cho t din OABC vung ti O. Gi R, r, h ln lt l bn knh mt cu ngoi tip, ni tip, chiu cao k t O ca t din.
Chng minh rng
1.

h
1 + 3.
r

2.

R 3+3 3

.
r
2

Bi 12.12 : Cho hnh chp S .ABCD c y l hnh ch nht, AB = a, S A vung gc vi y, S C to vi y gc 45 v to vi mt


phng (S AB) gc 30 . Tnh bn knh mt cu ngoi tip hnh chp.
Bi 12.13 : Cho tam gic u ABC. ng thng vung gc vi mt phng (ABC) ti A. im M thay i trn . K BEAC, BFMC
(E AC, F MC). ng thng EF ct ng thng ti N. Chng minh rng
1. AM.AN khng i.
2. Mt cu ngoi tip t din MNBC c tm thuc ng thng c nh.
Bi 12.14 :

1. Cho hnh lp phng ABCD.A BC D c cnh bng a. Hy xc nh tm v tnh bn knh mt cu (tnh theo a)

trong cc trng hp sau :


(a) Mt cu i qua 8 nh ca hnh lp phng ;
(b) Mt cu tip xc vi 12 cnh ca hnh lp phng ;
(c) Mt cu tip xc vi 6 mt bn ca hnh lp phng.

TRN ANH TUN - 0974 396 391 - (04) 66 515 343

WWW.VNMATH.COM

Trang 240

www.VNMATH.com

CHUYN LUYN THI I HC

www.luyenthi24h.com
www.luyenthi24h.com
www.VNMATH.com

2. Chng minh rng : c v s mt cu i qua hai im c nh A v B cho trc. Hy tm tp hp tm cc mt cu .


Bi 12.15 :

1. Cho hai ng trn C1 v C2 c tm O1 v O2 . Hai ng trn ny nm trong hai mt phng khc nhau v c chung

nhau dy cung AB. Chng minh rng c duy nht mt mt cu i qua c hai ng trn C1 v C2 .
2. Cho ng thng a c nh v mt im M c nh khng thuc ng thng a. Gi O l mt im di ng trn ng thng a.
V mt cu (S ) c tm O v bn knh R = OM. Chng minh rng : mt cu (S ) lun lun i qua mt ng trn c nh.
Bi 12.16 :

1. Cho mt cu S (O; R) v mt im A sao cho OA = 2R. Qua A k mt tip tuyn ti mt cu (v gi s B l tip

im) v k mt ct tuyn ct mt cu ti C v D, bit : CD = R 3.


(a) Tnh di AB theo R ;
(b) Tnh khong cch t O ti ng thng CD.

2. Cho mt cu S (O; R) v mt im A thuc mt cu ny. Gi (Q) l mt phng i qua A sao cho gc gia ng thng OA v
mt phng (Q) l 30 .
(a) Tnh din tch thit din (theo R) ca mt cu vi mt phng (Q).
(b) K ng thng i qua A v vung gc vi mt phng (Q). ng thng ct mt cu ti B. Tnh di cnh AB theo
R.
Bi 12.17 :

1. Cho mt cu S (O; R) tip xc vi mt phng (Q) ti im I. Gi M l im thuc mt cu ny. T M k hai

tip tuyn vi mt cu S (O; R) sao cho hai tip tuyn ny ct mt phng (Q) ti A v B. Bit MAMB. Chng minh rng :
AB2 = IA2 + IB2 .
2. Cho mt phng (Q) v hai im A v B c nh nm v mt pha ca mt phng (Q) sao cho ng thng AB ct mt phng (Q)
ti im I. Gi mt cu S (O; R) l mt cu thay i nhng lun i qua A v B ng thi tip xc vi mt phng (Q). Gi M l
tip im ca mt cu S (O; R) vi mt phng (Q). Chng minh rng : im M di ng trn mt ng trn c nh C no .
Bi 12.18 :

1. Cho hnh chp tam gic u S .ABC c cnh y bng a v cnh bn bng b. Hy xc nh tm v tnh bn knh ca

mt cu ngoi tip hnh chp theo a v b.


2. Cho ba on thng S A, S B, S C i mt vung gc vi nhau to thnh mt t din S ABC, vi S A = a, S B = b, S C = c. Xc
nh tm v tnh bn knh mt cu ngoi tip t din theo a, b, c.
Bi 12.19 : Cho hnh lng tr tam gic u ABC.A BC c 9 cnh u bng a.
1. Xc nh tm v tnh bn knh mt cu ngaoij tip hnh lng tr cho ;
2. Tnh din tch mt cu v th tch khi cu ni trn (tnh theo a).
Bi 12.20 : Cho hnh chp S .ABC c S A = S B = S C = a v c chiu cao bng h. Xc nh tm v tnh bn knh mt cu ngoi tip
hnh chp (tnh theo a v h). Tnh din tch ca mt cu .
Bi 12.21 : Trong mt phng (Q) cho hnh vung ABCD c cnh bng a. Ly mt im S thuc ng thng Ax vung gc vi (Q).
Gi (P) l mt phng qua A v vung gc vi S C. Mt phng (P) ct S B, S C, S D ln lt ti M, N, E.
1. Chng minh rng : by im A, B, C, D, M, N, E l cng thuc mt mt cu.
2. Tnh din tch ca mt cu theo a v th tch ca khi cu .
Bi 12.22 : Cho t din S ABC c S A(ABC) v S A = a, S B = b, S C = c. Xc nh tm v tnh bn knh mt cu ngoi tip t din
cho trong cc trng hp sau :
= 90 ;
1. c BAC

= 60 v b = c ;
2. c BAC

= 120 v b = c.
3. c BAC

Bi 12.23 : Cho mt cu S (O; R) v mt phng (Q) cch tm O mt khong bng h (0 < h < R). Mt phng ct mt cu (Q) theo
ng trn C . V ng thng d i qua im A c nh thuc ng trn C v d(Q). ng thng d ct mt cu S (O; R) ti B. Gi
CD l ng knh di ng ca ng trn C .
1. Chng minh rng : AD2 + BC 2 v AC 2 + BD2 l khng i.

TRN ANH TUN - 0974 396 391 - (04) 66 515 343

WWW.VNMATH.COM

Trang 241

www.VNMATH.com

CHUYN LUYN THI I HC

www.luyenthi24h.com
www.luyenthi24h.com
www.VNMATH.com

2. Tm v tr ca ng knh CD din tch tam gic BCD l ln nht.


3. K BHCD vi H CD. Tm tp hp im H khi CD di ng.
Bi 12.24 : Cho hnh t din u ABCD c cnh bng a v c ng cao AH. Gi O l trung im ca AH. Xc nh tm v tnh bn
knh mt cu ngoi tip t din OBCD.
Bi 12.25 : Cho hnh chp S .ABCD c S A = a l chiu cao ca hnh chp v y ABCD l hnh thang vung ti A v B, c
AB = BC = a v AD = 2a. Gi E l trung im ca cnh AD. Xc nh tm v tnh bn knh mt cu ngoi tip hnh chp S .CDE.
4R
Bi 12.26 : Cho hnh cu ng knh AA = 2R. Gi H l im trn on AA sao cho AH =
, mt phng (Q) i qua H v vung
3
gc vi AA ct hnh cu theo ng trn C .
1. Tnh din tch ng trn C ;
2. Gi s tam gic BCD l tam gic u ni tip ng trn C . Hy tnh th tch hnh t din ABCD v A BCD theo R.
Bi 12.27 :

1. Chng minh rng : Nu t din ABCD c mt mt cu tip xc vi 6 cnh ca n th t din c tng cc cp

cnh i din l bng nhau.

2. Cho hnh chp tam gic u S .ABC c cnh y bng a v cnh bn bng a 2. Mt mt cu i qua nh A v tip xc vi hai
cnh S B, S C ti E v F l trung im ca mi cnh.
(a) Chng minh rng : mt cu i qua M v N l trung im ca AB v AC.
(b) Gi giao im th hai ca mt cu vi ng thng S A l D. Tnh di ca AD v S D.
Bi 12.28 : Cho t din ABCD c AB = BC = CA = BD = a, AD = b v mt phng (ACD)(BCD).
1. Chng minh rng : ACD l tam gic vung ;
2. Tm tm mt cu ngoi tip t din ABCD ;
3. Tnh din tch mt cu ngoi tip t din ABCD.
Bi 12.29 : Cho tam gic u ABC c cnh bng a. V ng thng d i qua A v vung gc vi mt phng (ABC). Gi S l mt im
bt k trn d vi S . A.
1. Tm tm mt cu ngoi tip t din S ABC ;
2. Cho S A = h cho trc. Hy tnh din tch v th tch ca hnh cu ngoi tip t din S ABC ;
3. Gi A l im i xng ca A qua tm mt cu ni trn. Chng minh rng : khi S thay i trn ng thng d th A lun thuc
mt ng thng c nh.
Bi 12.30 : Cho tam gic u ABC cnh a. Gi (P) l mt phng i qua cnh BC v (P)(ABC). Gi (C ) l ng trn ng knh
BC v ng trn ny nm trong mt phng (P). Gi S l im di ng trn ng trn (C ). Chng minh rng :
1. Tng T = S A2 + S B2 + S C 2 l mt s khng i ;
2. Tm mt cu ngoi tip t din S ABC l mt im c nh (nu S . B v C).
Bi 12.31 : Cho hnh chp t gic u S .ABCD c cnh y bng a v chiu cao S H =

a
.
2

1. Chng minh rng : tn ti mt cu tm H tip xc vi tt c cc mt bn ca hnh chp. Tnh bn knh R ca mt cu theo a.


2. Gi (Q) l mt phng song song vi (ABCD) v khong cch gia hai mt phng ny l x, vi 0 < x < R. Gi S td l din tch
thit din to bi mt phng (Q) vi hnh chp nhng n b i phn nm trong mt cu. Hy xc nh x S td = R2 .
Bi 12.32 : Cho hnh chp S .ABCD vi y ABCD l t gic c ACBD ti H v S H l ng cao ca hnh chp cho.
1. Chng minh rng : bn tm mt cu ngoi tip cc hnh chp S .HAB, S .HBC, S .HCD, S .HDA l bn im O1 , O2 , O3 , O4 s
to thnh hnh ch nht.

TRN ANH TUN - 0974 396 391 - (04) 66 515 343

WWW.VNMATH.COM

Trang 242

www.VNMATH.com

CHUYN LUYN THI I HC

www.luyenthi24h.com
www.luyenthi24h.com
www.VNMATH.com

2. Gi M, N, E, F l hnh chiu ca im H ln lt trn AB, BC, CD, DA. Chng minh rng : hnh chp S .MNEF c mt cu
ngoi tip.
= v BDC
= .
Tnh din tch thit din ca mt cu y khi ct bi (ABCD), nu bit ME = a, BAC
= . Gi M, N ln lt l hnh chiu vung gc ca A
Bi 12.33 : Cho hnh chp S .ABC c S A(ABC); AB = c, AC = b, BAC

trnS B, S C. Chng minh rng : cc im A, B, C, M, N cng thuc mt mt cu v tnh bn knh ca mt cu theo b, c, .


Bi 12.34 : Cho hai tia Ax, By cho nhau v AxBy. Bit AB l on vung gc chung ca Ax v By. Ly mt im C thuc tia Ax v
im D thuc tia By.
1. Tm tm v tnh bn knh ca mt cu ngoi tip t din ABCD theo a, b, c b = AC v c = BD.
2. Cho C v D di ng trn Ax v By sao cho AC + BD = CD. Chng minh rng : ng thng CD lun tip xc vi mt cu
ng knh AB.
Bi 12.35 : Cho trc mt cu tm O bn knh R v mt im A c nh thuc mt cu. Ba tia At1 , At2 , At3 l ba tia thay i, i mt
vung gc vi nhau v ct mt cu ti cc im B, C, D.
1. Chng minh rng : hnh hp dng trn ba cnh AB, AC, AD c mt ng cho c nh v mt phng (BCD) lun lun i qua
mt im c nh.
2. Chng minh rng : hnh chiu H ca im D trn ng thng BC lun thuc mt mt cu c nh.
Bi 12.36 : Cho ng trn (O) ng knh AB = 2R nm trong mt phng (P). Gi C l im i xng vi O qua A. Ly im S sao
cho : S C(P) v S C = 2R. Tnh th tch ca khi cu i qua ng trn cho v i qua im S .
= 30 . Xt hai tia Bx, Cy cng hng v vung gc vi mt phng (ABC).
Bi 12.37 : Cho tam gic ABC vung A c BC = 2a, ACB

1. Hy xc nh v tr ca im E trn tia Bx sao cho mt cu ng knh BE tip xc vi Cy.


2. Hy xc nh v tr im F thuc tia Cy sao cho mt cu ng knh tip xc vi Bx.
3. Vi cc im E, F tm c trn, hi a din ABCFE c mt cu ngoi tip khng ? Hy tnh th tch ca khi a din .
Bi 12.38 : Trong v s cc hnh hp ni tip mt mt cu bn knh R cho trc. Hy tm hnh hp tha mn mt trong cc tnh cht
sau :
1. Th tch ca hnh hp t gi tr ln nht.
2. Tng di cc cnh ca hnh hp t gi tr ln nht.
Bi 12.39 :

1. Trong v s cc hnh chp tam gic u ni tip mt mt cu bn knh R cho trc. Hy tm hnh chp ch s th

tch ln nht.
2. Hy m rng bi ton cho hnh chp n-gic u.

12.2 Mt trn xoay. Mt tr, hnh tr v khi tr


Bi 12.40 : Cho trc mt phng (P) c im c nh A thuc mt phng (P) v im c nh B < (P), hnh chiu vung gc H
= BMH.
Chng
ca im B ln mt phng (P) l khng trng vi A. Gi M l mt im di ng trn mt phng (P) sao cho ABM

minh rng : im M lun nm trn mt mt tr trn xoay c trc l ng thng AB.


Bi 12.41 : Cho mt tr trn xoay (T ) v mt im S c nh nm ngoi (T ). Gi d l mt ng thng thay i nhng lun i qua
S v ct mt tr (T ) ti A v B. Gi I l trung im ca AB. Chng minh rng : trung im I lun lun nm trn mt mt tr c
nh no .

Bi 12.42 : Mt khi tr c bn knh y bng R v chiu cao h = R 3. Ly im A thuc ng trn (O) v im B thuc ng
trn (O ) l hai ng trn y ca khi tr cho, sao cho gc to bi ng thng AB v trc ca khi tr l 30 .
1. Gi (Q) l mt phng i qua AB v mt phng (Q) song song vi trc ca khi tr. Hy tnh din tch thit din ca mt phng
(Q) vi khi tr (tnh theo R).

TRN ANH TUN - 0974 396 391 - (04) 66 515 343

WWW.VNMATH.COM

Trang 243

www.VNMATH.com

CHUYN LUYN THI I HC

www.luyenthi24h.com
www.luyenthi24h.com
www.VNMATH.com

2. Tnh gc gia hai bn knh OA v O B.


3. Xc nh v tnh di on vung gc chung ca AB vi trc ca khi tr.

Bi 12.43 : Mt hnh tr c cc y l hai hnh trn tm O v tm O v bn knh R. Chiu cao ca hnh tr h = R 2. Ly im A


thuc ng trn (O) v im B thuc ng trn (O ) sao cho OAO B.
1. Chng minh rng : cc mt ca t din OABO l nhng tam gic vung. Tnh th tch ca t din ny theo R.
2. Gi (Q) l mt phng i qua AB v (Q) OO . Tnh khong cch gia ng thng OO v mt phng (Q) theo R.

R 2

dc theo mt ng sinh.
3. Chng minh rng : (Q) tip xc vi mt tr (T ) c trc OO , bn knh bng
2
Bi 12.44 : Cho mt hnh tr c y l hai ng trn (O) v (O ) c bn knh R = 50cm, chiu cao hnh tr l h = 50cm.
1. Tnh din tch xung quanh ca hnh tr v th tch ca khi tr c to nn.
2. Ly im A thuc ng trn (O) v im B thuc ng trn (O ) l hai ng trn y ca hnh tr. Bit AB = 100cm. Tnh
khong cch gia ng thng AB v trc ca hnh tr.
Bi 12.45 : Cho hnh chp tam gic u S .ABC c S A = a v gc gia mt bn v y l . Gi (T ) l hnh tr c ng trn y
l ng trn ni tip tam gic ABC v chiu cao ca hnh tr l bng chiu cao ca hnh chp S .ABC. Tnh din tch xung quanh ca
hnh tr (T ). Hi cc mt bn : S AB, S BC, S CA ct hnh tr (T ) theo giao tuyn th no ?
Bi 12.46 : Cho mt khi tr c chiu cao bng 20cm v bn knh y bng 10cm. K hai bn knh OA v O B ln lt nm trn hai
y ca khi tr sao cho gc gia chng bng 30 . Gi (Q) l mt phng i qua AB v (Q) song song vi trc ca khi tr. Hy tnh
din tch thit din gia khi tr v mt phng (Q).
Bi 12.47 : Cho mt khi tr c bn knh bng R v c thit din qua trc l mt hnh vung.
1. Tnh din tch xung quanh ca khi tr theo R.
2. V hnh tr t gic u ni tip trong hnh tr cho. Hy tnh th tch ca lng tr theo R.
3. Gi V l th tch ca khi lng tr t gic u ni trn v V l th tch ca khi tr. Hy tnh t s :

V
.
V

Bi 12.48 : Mt hnh tr c din tch xung quanh l S , din tch y bng din tch mt cu bn knh a. Hy tnh :
1. Th tch ca hnh tr cho theo a v S .
2. Din tch thit din i qua trc ca hnh tr.
Bi 12.49 : Cho hnh tr c bn knh y bng R v thit din qua trc hnh tr l mt hnh vung.
1. Tnh th tch v din tch ca hnh cu ngoi tip hnh tr theo R.
2. Mt mt phng (P) song song vi trc ca hnh tr, ct y ca hnh tr theo mt dy cung c di bng bn knh y ca
hnh tr. Tnh din tch cc thit din ca hnh tr cho v hnh cu ngoi tip hnh tr khi b ct bi mt phng (P) (tnh theo
R).
Bi 12.50 : Cho hnh tr c bn knh y bng R v chiu cao OO = h. Gi A v B l hai im thay i trn hai ng trn (O) v

(O ) l hai ng trn y sao cho AB = a khng i (h < a < h2 + 4R2 ).


1. Chng minh rng : Gc gia hai ng thng AB v OO l khng i.
2. Chng minh rng : Khong cch gia hai ng thng AB v OO l khng i.
Bi 12.51 : Cho hnh tr c bn knh y bng R, c trc OO = h. Mt mt phng (P) thay i i qua tm O, to vi y hnh tr gc
cho trc v ct hai y hnh tr theo cc dy cung AB v CD (dy AB i qua O).
1. Tnh din tch t gic ABCD theo R v h.
2. Chng minh rng : hnh chiu vung gc K ca im O ln mt phng (P) lun lun thuc mt ng trn c nh.

TRN ANH TUN - 0974 396 391 - (04) 66 515 343

WWW.VNMATH.COM

Trang 244

www.VNMATH.com

CHUYN LUYN THI I HC

www.luyenthi24h.com
www.luyenthi24h.com
www.VNMATH.com

Bi 12.52 : Cho hnh lng tr lc gic u ABCDEF.A BC D E F c cnh y bng a v chiu cao bng h. Tnh theo a v h din
tch :
1. xung quanh v th tch hnh tr ngoi tip lng tr .
2. ton phn v th tch hnh tr ni tip hnh lng tr cho.
Bi 12.53 : Cho hnh lng tr ng ABCD.A BC D c y ABCD l hnh thang cn vi y nh AB = a, y ln CD = 4a, hai cnh
5a
bn bng , chiu cao lng tr bng h.
2
1. Chng minh rng : tn ti mt hnh tr ni tip hnh lng tr cho.
2. Tnh din tch ton phn v th tch hnh lng tr theo a v h.
Bi 12.54 : Cho hnh tr c trc O1 O2 . Mt mt phng () O1 O2 ct hnh tr theo thit din l mt hnh ch nht ABCD. Gi O l
tm ca thit din . Bit bn knh ng trn ngoi tip hnh ch nht ABCD l bn knh ng trn y ca hnh tr cho. Hy

tnh gc O
1 OO2 .

Bi 12.55 : Cho mt hnh tr c thit din qua trc l mt hnh vung, din tch xung quanh bng 4.
1. Tnh din tch ton phn ca hnh tr ;
2. Tnh th tch ca khi tr ;
3. Tnh th tch ca khi lng tr n-gic u ni tip hnh tr ;
4. Tnh th tch ca khi cu ngoi tip hnh tr ;
5. Mt mt phng () song song vi trc ca hnh tr v ct hnh tr theo thit din ABB1 A1 . Bit mt cnh ca thit din l
mt dy cung ca ng trn y v chn mt cung trn 120. Hy tnh din tch thit din .
Bi 12.56 : Xt nt hnh tr ni tip mt mt cu tm O c bn knh R cho trc. Bit rng din tch thit din qua trc ca hnh tr
ny l ln nht (so vi cc hnh tr khc cng ni tip mt cu cho).
1. Tnh th tch V v din tch ton phn S tp ca hnh tr cho theo R.
2. Tnh th tch ca lng tr n-gic u ngoi tip hnh tr cho.
3. Tnh din tch thit din ca hnh tr khi ct bi mt mt phng song song vi trc hnh tr v cch trc mt khong bng

R
.
2

Bi 12.57 : Cho hnh hp ABCD.A1 B1C1 D1 ni tip trong mt hnh tr cho trc, gc gia ng thng B1 D v (ABB1 A1 ) l 30 .
3a
Khong cch t trc hnh tr n mt phng (ABB1 A1 ) l
. Tnh th tch hnh hp cho v th tch hnh cu ngoi tip hnh hp,
2
bit ng cho ca y hnh hp l 5a.
Bi 12.58 : Cho hai im A, B c nh. Gi d l mt ng thng di ng lun lun i qua A v d lun cch im B mt khong bng
AB
. Chng minh rng : ng thng d lun lun nm trn mt mt nn trn xoay.
BH = a =
2
Bi 12.59 : Cho khi nn trn xoay c ng cao h = 20cm, bn knh y r = 25cm. Mt mt phng (P) i qua nh ca khi nn v
c khong cch ti tm O ca y l 12cm. Hy xc nh thit din ca mt phng (P) vi khi nn cho v tnh din tch thit din
.
Bi 12.60 : Cho hnh lp phng ABCD.A BC D c cnh bng a. Gi O l tm hnh vung ABCD. Hy tnh din tch xung quanh
v th tch ca khi nn N c nh l tm O v y l hnh trn ni tip hnh vung A BC D .
Bi 12.61 : Cho mt hnh nn N c nh l im D, c O l tm ng trn y, c di ng sinh bng l v c gc gia ng
sinh vi mt y bng .
1. Tnh din tch xung quanh v th tch ca khi nn N theo l v .
2. Gi I l mt im trn ng cao DO ca hnh nn sao cho
i qua I v vung gc vi trc hnh nn (tnh theo l, , k).

DI
= k. Tnh din tch thit din ca hnh nn vi mt phng (Q)
DO

Bi 12.62 : Cho mt hnh nn N c thit din qua trc l mt tam gic vung cn c cnh bng a.

TRN ANH TUN - 0974 396 391 - (04) 66 515 343

WWW.VNMATH.COM

Trang 245

www.VNMATH.com

CHUYN LUYN THI I HC

www.luyenthi24h.com
www.luyenthi24h.com
www.VNMATH.com

1. Tnh din tch ton phn v th tch ca hnh nn N cho (tnh theo a).
2. Mt mt phng (Q) i qua nh ca hnh nn v to vi mt phng y gc 60 . Tnh din tch thit din c to nn bi hnh
nn N v mt phng (Q).
Bi 12.63 : Cho hnh chp tam gic u S .ABC c cc cnh bn bng a v c gc gia mt bn vi mt y bng . V hnh nn N
c nh S v c ng trn y ni tip tam gic u ABC - gi l hnh nn ni tip hnh chp cho. Hy tnh din tch xung quanh
ca hnh nn N theo a v .
Bi 12.64 : Cho hnh chp t gic u S .ABCD c chiu cao S O = h v S
AB = ( > 45 ). V hnh nn N c nh S v c ng
trn y ngoi tip hnh vung ABCD. Hy tnh din tch xung quanh ca hnh nn N theo h v .
Bi 12.65 : Cho khi nn N c bn knh y R = 12cm v c gc nh bng 120. Hy tnh din tch ca thit din i qua hai
ng sinh vung gc vi nhau.
Bi 12.66 : Cho hnh nn N c bn knh y bng R, ng cao S O. Mt mt phng (P) c nh vung gc vi S O ti A, ct hnh
nn N theo ng trn c bn knh bng R1 . Gi mt phng (Q) l mt phng thay i v vung gc vi S O ti B (im B nm gia
O v A). Mt phng (Q) ct hnh nn theo thit din l mt ng trn c bn knh bng x.
Hy tnh x theo R v R1 nu bit mt phng (Q) chia khi trn xoay trong hnh nn nm gia (P) v ay hnh nn thnh hai phn
c th tch bng nhau.
Bi 12.67 : Cho hnh nn N c bn knh y bng R, gc gia ng sinh v y ca hnh nn bng . Mt mt phng (P) song
song vi y hnh nn v cch y hnh nn mt khong bng h, v ct hnh nn N theo mt ng trn (C ).
1. Tnh bn knh ca ng trn (C ) theo R, h, ;
2. Tnh din tch v thit din phn hnh nn nm gia y hnh nn N v mt phng (P).
1
S O. Gi
3
(P) l mt phng vung gc vi S O ti A. Mt mt phng (Q) qua trc hnh nn ct khi trn xoay nm trong khi nn N - khi
Bi 12.68 : Cho hnh nn N c bn knh y bng R, ng cao S O. Ly mt im A thuc ng cao S O sao cho S A =

nm gia mt phng (P) v y hnh nn - theo thit din l hnh t gic c hai ng cho vung gc vi nhau. Hy tnh th tch khi
trn xoay ca khi nn N nm gia (P) v mt phng cha y hnh nn (tnh theo R).
b = 60 . Bit rng c mt hnh nn ni tip hnh chp
Bi 12.69 : Cho hnh chp S , ABC c y ABC l tam gic vung ti A, c B

cho vi bn knh y l r, gc gia ng sinh v y hnh nn l .


1. Tnh din tch xung quanh v th tch hnh nn theo r v ;
2. Tnh din tch xung quanh v th tch hnh chp theo r v .
Bi 12.70 : Gi (C ) l ng trn cha cc im tip xc ca mt xung quanh hnh nn vi mt cu ni tip hnh nn . ng trn
(C ) chia mt xung quanh ca hnh nn thnh hai phn. Hy tnh t s din tch ca hai phn , bit din tch hnh cu bng din tch
y hnh nn.
Bi 12.71 : Cho hnh nn N c bn knh y R v chiu cao bng 4R.
1. Tnh din tch ton phn ca hnh tr ni tip hnh nn, bit bn knh y hnh tr bng r - tnh theo R v r (Hnh tr c gi
l ni tip hnh nn nu c mt ng trn y ca hnh tr nm trn mt xung quanh ca hnh nn v y cn li ca hnh tr
nm trong mt y ca hnh nn).
2. Tnh bn knh y r v chiu cao h ca hnh tr ni tip hnh nn theo R din tch ton phn ca hnh tr t gi tr ln nht.
Bi 12.72 :

1. Tm hnh nn c th tch ln nht sao cho hnh nn phi ni tip mt mt cu bn knh R cho trc.

2. Tm hnh nn c th tch nh nht sao cho hnh nn phi ngoi tip mt mt cu bn knh r cho trc.
Bi 12.73 : Tm hnh trn c th tch ln nht bit din tch ton phn ca n bng din tch hnh trn c bn knh bng a cho trc.
Bi 12.74 : ng cao ca hnh nn gp hai ln bn knh y ca n. Tnh t s th tch ca hnh cu ngoi tip v ni tip hnh nn
.
Bi 12.75 : Trong tt c cc hnh nn ni tip hnh cu bn knh R cho trc, tm hnh nn c din tch xung quanh ln nht. Vi hnh
nn y, xt hnh tr ni tip hnh nn. Tm chiu cao ca hnh tr , bit thit din qua trc hnh tr l hnh vung.

TRN ANH TUN - 0974 396 391 - (04) 66 515 343

WWW.VNMATH.COM

Trang 246

www.VNMATH.com

CHUYN LUYN THI I HC

www.luyenthi24h.com
www.luyenthi24h.com
www.VNMATH.com

Bi 12.76 : Mt mt phng (Q) i qua hai ng sinh ca hnh nn, ct mt y ca hnh nn theo mt dy cung c di gp k ln
1
ng cao hnh nn. Gi l gc gia mt phng (Q) v mt y ca hnh nn. Bit = gc to bi hai ng sinh ca hnh nn
2
m hai ng sinh nm trong (Q). Hy tnh cos theo k.
Bi 12.77 : Cho hnh nn N c nh S , ng cao S O. Gi A v B l hai im thuc ng trn y ca hnh nn sao cho khong
cch t im O ti AB l bng a. Bit S
AO = 30 v S
AB = 60 . Tnh din tch xung quanh ca hnh nn N theo a.
Bi 12.78 : Cho hai im c nh A v B, AB = a. Vi mi im C trong khng gian sao cho tam gic ABC u, k hiu AE l
ng cao ca tam gic ABC v d l trc ca ng trn ngoi tip tam gic ABC. Trong mt phng cha d v AE, xt ng trn
ng knh AE. Gi S l mt giao im ca ng trn ny v ng thng d.
1. Tnh din tch xung quanh v th tch ca hnh nn ngoi tip hnh chp S .ABC (tnh theo a).
2. Chng minh rng : khi im C thay i th im S lun thuc mt ng trn c nh v mi ng thng S A, S B lun thuc
mt mt nn c nh.

WWW.VNMATH.COM

TRN ANH TUN - 0974 396 391 - (04) 66 515 343

WWW.VNMATH.COM

Trang 247

www.VNMATH.com

CHUYN LUYN THI I HC

TRN ANH TUN - 0974 396 391 - (04) 66 515 343

www.luyenthi24h.com
www.luyenthi24h.com
www.VNMATH.com

WWW.VNMATH.COM

Trang 248

www.luyenthi24h.com
www.luyenthi24h.com
www.VNMATH.com

www.VNMATH.com

Chng 13

Phng php khng gian to trong khng gian


13.1 H to trong khng gian
Vn 1 : Tm ta ca mt vect v cc yu t lin quan n vect tha mn mt s iu kin cho trc

S dng cc nh ngha c tnh lin quan n vect : ta ca vect, di ca vect, tng (hiu) ca hai vect, ta trung im,
ta trng tm, . . .

Bi 13.1 : Vit to ca cc vect sau y :

a = 4
j ; b = i + 2 j ;
c = 3 i +2 j k.

a = (3; 1; 2),
c = (3; 2; 0).
Bi 13.2 : Cho cc vect
b = (1; 3; 4),

a , 3
a 2
c .
1. Hy xc nh to cc vect 3
b , a 3 b + 2


a ,
2. Hy biu din vect d = (1; 0; 2) theo ba vect
b , c .

a v
a +
a
a | = 3, |
Bi 13.3 : Cho hai vect
b to vi nhau mt gc 120 . Tm |
b | v |
b | bit |
b | = 5.
a = (1; 3; 4).
Bi 13.4 : Cho vect

a .
1. Tm y0 v z0 cho vect b = (2; y0 ; z0 ) cng phng vi

c bit rng
a v
c ngc hng v |
c | = 2|
a |.
2. Tm ta ca vect

Bi 13.5 : Cho hnh hp ABCD.A BC D , bit A(1; 0; 1), B(2; 1; 2), D(1; 1; 1), C (4; 5; 5). Tm to cc nh cn li ca hnh
hp.

Bi 13.6 : Trong khng gian Oxyz, xt hnh hp ch nht ABCD.A BC D , cnh y bng a, cnh bn AA = 2a, A(0; 0; 0), B(a; 0; 0),
D(0; a; 0), A (0; 0; 2a).
1. Xc nh to cc nh cn li.

2. Xc nh to DB .

3. Xc nh to trung im M ca on BA.
4. Xc nh to trng tm tam gic BCD.

Vn 2 : ng dng ca tch v hng v tch c hng


1. S dng cc cng thc

249

WWW.VNMATH.COM

www.luyenthi24h.com
www.luyenthi24h.com

Chng 13

Phng php khng gian to trong khng gian


13.1 H to trong khng gian
Vn 1 : Tm ta ca mt vect v cc yu t lin quan n vect tha mn mt s iu kin cho trc

S dng cc nh ngha c tnh lin quan n vect : ta ca vect, di ca vect, tng (hiu) ca hai vect, ta trung im,
ta trng tm, . . .

Bi 13.1 : Vit to ca cc vect sau y :

a = 4
j ; b = i + 2 j ;
c = 3 i +2 j k.

a = (3; 1; 2),
c = (3; 2; 0).
Bi 13.2 : Cho cc vect
b = (1; 3; 4),

a , 3
a 2
c .
1. Hy xc nh to cc vect 3
b , a 3 b + 2


a ,
2. Hy biu din vect d = (1; 0; 2) theo ba vect
b , c .

a v
a +
a
a | = 3, |
Bi 13.3 : Cho hai vect
b to vi nhau mt gc 120 . Tm |
b | v |
b | bit |
b | = 5.
a = (1; 3; 4).
Bi 13.4 : Cho vect

a .
1. Tm y0 v z0 cho vect b = (2; y0 ; z0 ) cng phng vi

c bit rng
a v
c ngc hng v |
c | = 2|
a |.
2. Tm ta ca vect

Bi 13.5 : Cho hnh hp ABCD.A BC D , bit A(1; 0; 1), B(2; 1; 2), D(1; 1; 1), C (4; 5; 5). Tm to cc nh cn li ca hnh
hp.

Bi 13.6 : Trong khng gian Oxyz, xt hnh hp ch nht ABCD.A BC D , cnh y bng a, cnh bn AA = 2a, A(0; 0; 0), B(a; 0; 0),
D(0; a; 0), A (0; 0; 2a).
1. Xc nh to cc nh cn li.

2. Xc nh to DB .

3. Xc nh to trung im M ca on BA.
4. Xc nh to trng tm tam gic BCD.

Vn 2 : ng dng ca tch v hng v tch c hng


1. S dng cc cng thc

249

WWW.VNMATH.COM

www.VNMATH.comCHUYN LUYN THI I HC


S ABC =

1
[ AB, AC] ;
2





V h.hp ABCD.A BC D = [AB, AD].AA ;

VABCD =

1
[ AB, AC].AD ;
6




[ AB, CD].AC
d(AB, CD) = ;
[ AB, CD]

www.luyenthi24h.com
www.luyenthi24h.com
www.VNMATH.com



|[ MA, MB]| |[ MA, AB]|
d(M, AB) =
=
;

|AB|
|AB|

u ,
v ) = u . v ;
cos(
u |.|
v |
|

v ]
[
u ,

sin( u , v ) =
v | ;
|u |.|

cos A = cos(AB, AC) ;




cos(AB, CD) = cos(AB, CD) .

u v
v cng phng khi v ch khi [
u ,
v ] =
0 (tng ng vi ta tng ng t l).
2. Hai vect

3. Ba im A, B, C thng hng khi v ch khi hai vect AB v AC cng phng.
u
v khi v ch khi
u .
v = 0.
4.


5. Bn im A, B, C, D ng phng khi v ch khi [AB, AC].AD = 0.

a = (2; 4; 0),
c = (1; 2 1).
Bi 13.7 : Cho vect
b = (3; 2; 1),


a ,
a ).
1. Tnh cosin ca cc gc sau : (
b ), ( b ,
c ), (c ,


a .
a .
2. Tnh cc tch v hng
b , b .
c , c .

v sao cho
v
a ,
v
v | = |
c |.
3. Tm to ca vect
b v |
Bi 13.8 : Cho ba im A(2; 3; 1), B(1; 1; 3), C(2; 4; 1).
1. Chng minh rng ba im A, B, C to thnh mt tam gic. Tm to trng tm ca tam gic ABC.
2. Tm to im D sao cho ABCD l hnh bnh hnh.
3. Tm im E trn mt phng (Oxy) sao cho ABCE l hnh thang vi hai y l AB v CE.
Bi 13.9 : Cho tam gic ABC vi A(1; 2; 1), B(2; 1; 3), C(4; 7; 5).
1. Tm im D sao cho tam gic ABD nhn C lm trng tm.
2. Tm di ng phn gic trong ca tam gic ABC v t B.
Bi 13.10 : Trong khng gian Oxyz cho tam gic ABC c im C(2; 2; 2) v trng tm G(1; 1; 2).
1. Tm to cc nh A, B ca tam gic ABC bit A nm trn mt phng (Oxy) v B thuc Oz.
2. Gi H l trung im BC, E l im i xng ca H qua A. Tm to im K trn ng thng AC B, E, K thng hng.
Bi 13.11 : Cho tam gic ABC c A(2; 0; 1), B(1; 1; 2), C(2; 3; 1).
1. Chng minh tam gic ABC c Ab l gc t.
2. Tnh chu vi tam gic ABC.
3. Tm im M trn Oy sao cho tam gic MBC vung ti M.
Bi 13.12 : Cho tam gic ABC c A(2; 3; 1), B(0; 1; 2), C(1; 0; 3).
1. Tm to chn ng cao H h t nh A ca tam gic ABC.
2. Tm to giao im D ca ng thng AH vi ng trn ngoi tip tam gic ABC.
Bi 13.13 : Cho hai im A(3; 0; 1), B(1; 3; 2), C(3; 4; 1).
1. Tm im M trn trc Ox sao cho MA = MB.

TRN ANH TUN - 0974 396 391 - (04) 66 515 343

WWW.VNMATH.COM

Trang 250

www.VNMATH.com

www.luyenthi24h.com
www.luyenthi24h.com
CHUYN LUYN THI I HCwww.VNMATH.com

2. Tm im N trn mt phng (Oyz) sao cho NA = NB = NC.



3. Tm im P trn mt phng Oxy sao cho |PA + PB + PC| t gi tr nh nht.
Bi 13.14 : Tm ta im M trong mi trng hp sau y
1. M trn trc Oy v cch u hai im A(3; 1; 4), B(2; 3; 0).
2. M trn mt phng (Oxz) v cch u ba im A(3; 1; 4), B(2; 1; 0), C(4; 5; 2).
Bi 13.15 : Trong khng gian cho 4 im A(4; 2; 2), B(1; 2; 5).C(0; 1; 1), D(2; 0; 3). Chng minh rng :
1. Bn im A, B, C, D khng ng phng.
2. T din ABCD c cc cnh i din vung gc vi nhau.
Bi 13.16 : Cho hnh lng tr tam gic ABC.A1 B1C1 bit :
A(2; 4; 1), B(1; 1; 2), A1(5; 1; 0), C1(2; 0; 1).
1. Tm to cc nh cn li ca lng tr.
2. Mt mt phng (P) qua A, trung im M ca BC v trung im N ca A1 B1 . Tm to giao im ca (P) vi B1C1 .
Bi 13.17 : Cho hnh hp ABCD.A1 B1C1 D1 . Bit A(3; 2; 1), C(4; 2; 0), B1(2; 1; 1), D1(3; 5; 4).
1. Xc nh to cc nh A1 , C1 , B, D v tm K ca hnh hp.

59
.
2. Tm im M trn ng thng AA1 sao cho K M =
2
Bi 13.18 : Cho hnh chp S .ABCD c :

13
, A(1; 2; 3), B(1; 4; 6), C(2; 1; 10), D(4; 1; 7).
S 3; 3;
2
1. Chng minh rng ABCD l hnh ch nht v S I(ABCD), trong I l giao im ca AC v BD.
2. Tnh th tch hnh chp.
Bi 13.19 : Tnh tch c hng ca cc cp vect sau y :

a = (1; 1; 2),
1.
b = (3; 3; 6)

a = (2; 1; 3),
2.
b = (1; 3; 4)

a = (1; 1; 2),
3.
b = (2; 3; 7)

a = (1; 1; 0),
4.
b = (0; 0; 1)


a ,
Bi 13.20 : Xt s ng phng ca b ba vect
b , c sau y :

a = (3; 1; 1),
c = (4; 1; 0).
1.
b = (2; 3; 5),

a = (2; 1; 1),
c = (2; 1; 1).
2.
b = (3; 1; 2),

Bi 13.21 : Cho hai im A(3; 2; 1), B(1; 3; 4). Tm im C trn mt phng (Oxy) sao cho cc iu kin sau c tha mn OC = 1

v cc vect OA, OB, OC ng phng.
Bi 13.22 : Cho hai im A(1; 2; 1), B(2; 1; 3). Tm im M thuc Ox sao cho tam gic AMB c din tch nh nht.

Bi 13.23 : Cho cc im A(1; 0; 1), B(0; 0; 2), C(0; 1; 1), D(2; 1; 0).
1. Chng minh A, B, C, D l cc nh ca mt t din.

2. Tm to trng tm G ca t din ABCD v gc to bi hai ng thng AC v BD.


3. Tnh th tch ca t din ABCD v khong cch t A n mt phng (BCD).
Bi 13.24 : Cho tam gic ABC c A(2; 0; 1), B(0; 1; 1), C(0; 0; 1).
1. Tm to tm ng trn ngoi tip ca tam gic ABC v tnh bn knh ca ng trn .
2. Tm to trc tm H ca tam gic ABC.

TRN ANH TUN - 0974 396 391 - (04) 66 515 343

WWW.VNMATH.COM

Trang 251

www.VNMATH.comCHUYN LUYN THI I HC

www.luyenthi24h.com
www.luyenthi24h.com
www.VNMATH.com

Bi 13.25 : Cho tam gic ABC c A(0; 0; 2), B(0; 1; 0), C(1; 2; 3).
1. Tm to im S trn Oy t din S ABC c th tch bng 8.
2. Tm to hnh chiu H ca O trn mt phng (ABC).
Bi 13.26 : Cho hai im A(3; 2; 1), B(1; 3; 4). Tm im C trn mt phng (Oyz) sao cho cc iu kin sau c tho mn :

OC = 1 v cc vect OA, OB, OC ng phng.
Bi 13.27 : Cho ba im A(2; 1; 3), B(1; 1; 1), C(4; 3; 2).
1. Chng minh A, B, C khng thng hng. Tnh din tch tam gic ABC.
1
2. Tm im D trn trc Oy sao cho t din ABCD c th tch bng .
2

Vn 3 : Lp phng trnh ca mt cu


1. Mun vit c phng trnh mt cu cn bit tm I(a; b; c) v bn knh R ca mt cu . Khi , phng trnh mt cu l
(S ) : (x a)2 + (y b)2 + (z c)2 = R2 .
2. Ta c A (S ) khi v ch khi IA = R.
3. (S ) tip xc vi khi v ch khi d(I, ) = R.
4. (S ) tip xc vi mt phng (P) khi v ch khi d(I, (P)) = R.
5. Nu M(x M ; y M ; z M ) th
(a) d(M, (Oxy)) = |z M |, d(M, (Oyz)) = |x M |, d(M, (Ozx)) = |y M |.
(b) d(M, Ox) =

y2M + z2M , d(M, Oy) =

x2M + z2M , d(M, Oz) =

x2M + y2M .

(c) Hnh chiu vung gc ca M trn trc Ox c ta (x M ; 0; 0).


(d) Hnh chiu vung gc ca M trn mt phng (Oxy) c ta (x M ; y M ; 0).

Bi 13.28 : Lp phng trnh mt cu trong cc trng hp sau y :


1. Nhn I(2; 0; 3) l tm v bn knh R = 4.
2. Nhn AB lm ng knh vi A(2; 3; 5), B(0; 1; 1).
3. Nhn I(3; 4; 1) lm tm v tip xc vi mt phng (Oyz).
4. Nhn I(6; 3; 4) lm tm v tip xc vi trc Oz.
Bi 13.29 : Lp phng trnh mt cu trong mi trng hp sau y :
1. C tm trn trc honh v i qua hai im A(2; 4; 1), B(1; 4; 5).
2. C tm nm trn mt phng (Oyz) v i qua ba im A(2; 1; 5), B(2; 1; 1), C(3; 0; 2).
3. i qua bn im A(1; 3; 4), B(3; 1; 5), C(2; 1; 2), D(0; 2; 3).
Bi 13.30 : Cho mt cu (S ) : x2 + y2 + z2 4x + 2y 4z = 0.
1. Xc nh to tm v tnh bn knh ca (S ).
2. Tm to giao im A, B, C (khc gc O) ca (S ) vi cc trc to . Tnh th tch t din OABC.

TRN ANH TUN - 0974 396 391 - (04) 66 515 343

WWW.VNMATH.COM

Trang 252

www.luyenthi24h.com
www.luyenthi24h.com
CHUYN LUYN THI I HCwww.VNMATH.com

www.VNMATH.com

Bi 13.31 : Cho mt cu (S ) c phng trnh x2 + y2 + z2 + x y + z 1 = 0.


1. Chng minh rng (Oxy) ct mt cu (S ) theo mt ng trn. Tm tm v bn knh ca ng trn ny.
2. Trc Oz ct mt cu ti hai im A, B. Tnh di on AB.
1
Bi 13.32 : Cho mt cu (S ) : x2 + y2 + z2 3x y + z + = 0.
2
1. Chng minh rng mt cu (S ) tip xc vi mt phng (Oyz). Tm to tip im A.
2. Chng minh rng mt cu (S ) tip xc vi Ox ti B. Tm to im B.
Bi 13.33 : Cho S (2; 2; 3), A(2; 2; 1), B(2; 4; 1), C(4; 0; 1), D(0; 2; 1).
1. Chng minh rng ABCD l hnh vung v S A l ng cao ca hnh chp S .ABCD. Tnh th tch hnh chp .
2. Vit phng trnh mt cu ngoi tip hnh chp S .ABCD.
Bi 13.34 : Cho mt cu (S m ) : x2 + y2 + z2 4mx + 4y + 2mz + m2 + 4m = 0. Tm m (S m ) l mt cu c bn knh nh nht.

Bi 13.35 : Cho mt cu (S m ) : x2 + y2 + z2 2mx + 2my 4mz + 5m2 + 2m + 3 = 0. Xc nh tham s m (S m ) l mt mt cu. Tm


tp hp tm I ca mt cu (S m ) khi m thay i.

Vn 4 : Phng php ta gii hnh hc khng gian


Bc 1 : To mt gc tam din (c chung nh v ba cnh i mt vung gc). Gc tam din ny c hai trc Ox, Oy thng nm trn
mt y v trc Oz vung gc vi y.
Bc 2 : Tm ta ca bn im : gc, cc im nm trn cc trc Ox, Oy, Oz.
Bc 3 : Tm ta cc im, cc vect c lin quan, v a bi ton v hnh hc gii tch thng thng.

Bi 13.36 : Cho hnh lp phng ABCD.A BC D c cnh bng a.


1. Gi I l trung im AC, J l trung im AB. Chng minh rng AJA I.
2. Gi G l trng tm tam gic BAC . Chng minh rng B , G, D thng hng.
Bi 13.37 : Cho hnh lp phng ABCD.A1 B1C1 D1 . Gi M, N, P ln lt l trung im ca BB1, CD, A1 D1 . Tnh gc v khong cch
gia C1 N v MP.
a
Bi 13.38 : Cho hnh chp S .ABCD, y ABCD l hnh vung cnh a, S A vung gc vi y, M thuc cnh CD sao cho DM = v
2
3a
N thuc cnh BC sao cho BN =
. Chng minh rng MN(S AN) t suy ra mt phng (S AN)(S MN).
4
Bi 13.39 : Cho hnh chp t gic u S .ABCD c cnh y bng a, chiu cao 2a. Gi O l giao im ca AC v BD, M v N ln lt
l trung im ca S A v BC.
1. Tnh th tch t din OS MN.
2. ng thng MN ct (S BD) ti im P. Tnh OP.
3. Gi K l trung im cnh CD, I l im thay i trn cnh S O vi OI = m. Xc nh m sao cho cc ng thng AB, S C, KI
cng song song vi mt mt phng.
Bi 13.40 : Cho hnh chp S .ABCD c y ABCD l hnh ch nht vi AB = a, AD = b, S A(ABCD) v S C = c. Gi E l im i
xng ca C qua B.

1. Gi M, N ln lt l trung im ca S B, S D. Chng minh rng cc vect AE, AM, AN ng phng.
SM
SN

2. Cho M, N thay i ln lt trn cc tia S B, S D sao cho
= x,
= y. Tm iu kin ca x, y sao cho cc vect AC, AM, AN
SD
SB
ng phng.

TRN ANH TUN - 0974 396 391 - (04) 66 515 343

WWW.VNMATH.COM

Trang 253

www.VNMATH.comCHUYN LUYN THI I HC

www.luyenthi24h.com
www.luyenthi24h.com
www.VNMATH.com

13.2 Phng trnh mt phng


Vn 1 : Vit phng trnh mt phng i qua mt im v c mt vect php tuyn cho trc

1. Vect php tuyn ca mt phng

n ,
(a) Vect
0 c gi l vect php tuyn ca mt phng () nu n nm trn ng thng vung gc vi mt phng ().
Mt mt phng c v s vect php tuyn, cc vect php tuyn lun cng phng.1

u ,
v khng cng phng v c gi ca chng song song (hoc nm trn) () th vect
n = [
u ,
v ] l mt
(b) Nu hai vect
vect php tuyn ca mt phng ().
2. Phng trnh tng qut ca mt phng
Ax + By + Cz + D = 0 vi A2 + B2 + C 2 , 0.
n = (A; B; C) l mt vect php tuyn ca (). Phng trnh mt phng i qua im M(x ; y ; z ) v c vect php tuyn
Khi
0 0 0

n = (A; B; C) c phng trnh


A(x x0 ) + B(y y0 ) + C(z z0 ) = 0.
3. Phng trnh on chn ca mt phng :
Gi s mt phng (P) ct ba trc ta ln lt ti ba im A(a; 0; 0), B(0; b; 0), C(0; 0; c). Khi phng trnh ca mt phng
(P) l
x y x
+ + = 1.
a b c
4. Phng trnh cc mt phng ta
(Oxy) : z = 0; (Oyz) : x = 0; (Ozx) : y = 0.

Bi 13.41 : Cho hai im A(2; 3; 4), B(4; 1; 0). Vit phng trnh ca mt phng trung trc ca on thng AB.
Bi 13.42 : Cho tam gic ABC c : A(1; 2; 3), B(2; 4; 3), C(4; 5; 6).

1. Vit phng trnh mt phng qua A v vung gc vi ng thng BC.


2. Vit phng trnh mt phng qua A, B, C.
Bi 13.43 : Trong khng gian Oxyz cho im M(30; 15; 6).
1. Hy vit phng trnh mt phng () i qua cc hnh chiu vung gc ca M trn cc trc ta .
2. Tm ta hnh chiu H ca O trn mt phng ().
Bi 13.44 : Cho im A(2; 3; 4). Vit phng trnh mt phng () qua cc hnh chiu ca im A trn cc trc to .

Bi 13.45 : Vit phng trnh mt phng i qua im G(1; 2; 3) v ct cc trc to ti cc im A, B, C sao cho G l trng tm ca
tam gic ABC.
Bi 13.46 : Vit phng trnh mt phng trong mi trng hp sau y
1. Ct cc trc ta ti cc im A(3; 0; 0), B(0; 2; 0), C(0; 0; 5).
2. Qua im H(2; 3; 1) v ct cc trc Ox, Oy, Oz ti ba im A, B, C sao cho H l trc tm tam gic ABC.
3. Qua im M(2; 3; 1) v ct cc trc Ox, Oy, Oz ti ba im A, B, C sao cho tam gic ABC l tam gic u.
1

n = (a; b; c) c a , 0 l mt vect php tuyn th ta lun c th chn a = 1 hay mt gi tr khc 0 bt k


Nu

TRN ANH TUN - 0974 396 391 - (04) 66 515 343

WWW.VNMATH.COM

Trang 254

www.luyenthi24h.com
www.luyenthi24h.com
CHUYN LUYN THI I HCwww.VNMATH.com

www.VNMATH.com

4. Qua im G(2; 3; 1) v ct cc trc Ox, Oy, Oz ti ba im A, B, C sao cho G l trng tm tam gic ABC.
5. Qua im N(1; 1; 1) v ct ba tia Ox, Oy, Oz ti ba im A, B, C sao cho OA + OB + OC l nh nht.
Bi 13.47 : Lp phng trnh mt phng () i qua im M(1; 2; 3) v ct ba tia Ox, Oy, Oz ln lt ti A, B, C sao cho th tch ca t
din OABC nh nht.
Bi 13.48 : Vit phng trnh mt phng () qua hai im A(4; 2; 1), B(1; 1; 2) v song song vi trc Ox.

Vn 2 : V tr tng i ca hai mt phng

n = (A; B; C) v
Nu () : Ax + By + Cz + D = 0 v ( ) : A x + B y + C z + D = 0 c cc vect php tuyn tng ng l

n = (A ; B ; C ) th

n v
n khng cng phng.
1. () v ( ) ct nhau khi v ch khi

n v
n cng phng v c im M () th M < ( ).
2. () v ( ) song song khi v ch khi

n v
n cng phng v c im M () th M ( ).
3. () v ( ) trng nhau khi v ch khi

n .

4. () v ( ) vung gc vi nhau khi v ch khi


n = 0.

Ch :

Nu (P) : Ax + By + Cz + D = 0 v (Q) (P) th (Q) c dng Ax + By + Cz + D = 0 vi D , D.


n s c gi song song vi (hoc nm trn) mt phng ().
Nu ()( ) khi

Bi 13.49 : Xt v tr tng i ca mi cp mt phng cho bi mi phng trnh


1. x + 2y z + 5 = 0 v 2x + 3y 7z + 10 = 0;
2. 3x + 2y z + 5 = 0 v 6x + 4y 2z + 10 = 0;
3. x + 2y z + 5 = 0 v x 2y + z + 10 = 0;
Bi 13.50 : Cho hai mt phng
() : 2x my + 3z 6 + m = 0 v ( ) : (m + 3)x 2y + (5m + 1)z 10 = 0.
Vi gi tr no ca m, hai mt phng
1. Song song vi nhau.

2. Trng nhau.

3. Ct nhau.

4. Vung gc vi nhau.

Bi 13.51 : Vn hi nh bi tp 13.50 vi hai mt phng


() : 2x my + 10z + m + 1 = 0 v ( ) : x 2y + (3m + 1)z 10 = 0.
Bi 13.52 : Tm m hai mt phng
() : (m + 2)x + (2m + 1)y + 3z + 2 = 0 v ( ) : (m + 1)x + 2y + (m + 1)z 1 = 0.
1. song song.

2. vung gc.

3. ct nhau.

Bi 13.53 : Cho ng thng A(1; 1; 2) v mt phng () : 2x y + z 1 = 0. Vit phng trnh mt phng (P) qua A v song song
vi ().

TRN ANH TUN - 0974 396 391 - (04) 66 515 343

WWW.VNMATH.COM

Trang 255

www.VNMATH.comCHUYN LUYN THI I HC

www.luyenthi24h.com
www.luyenthi24h.com
www.VNMATH.com

Bi 13.54 : Cho hai im P(3; 1; 1), Q(2; 1; 4) v () : 2x y + 3z 1 = 0. Vit phng trnh mt phng (R) qua hai im P, Q v
vung gc vi mt phng ().

Bi 13.55 : Vit phng trnh mt phng (P) qua A(1; 3; 2) v vung gc vi hai mt phng
() : x 3y + 2z + 5 = 0 v ( ) : 3x 2y + 5z + 4 = 0.
Bi 13.56 : Vit phng trnh mt phng qua im M(2; 1; 2), song song vi trc Oy v vung gc vi mt phng 2x y + 3z + 4 = 0.

Bi 13.57 : Vit phng trnh mt phng (), bit mt phng ()

1. qua im M(1; 1; 5), N(0; 0; 1) v cng phng vi trc Oz.


2. qua im M(1; 1; 5) v song song vi mt phng (Oxy).
Bi 13.58 : Cho ba mt phng
(1 ) : 2x z = 0; (2 ) : x + y z + 5 = 0; (3 ) : 7x y + 4z 3 = 0.
Vit phng trnh mt phng () i qua giao tuyn ca hai mt phng (1 ) v (2 ) ng thi vung gc vi mt phng (3 ).
Bi 13.59 : Cho hai mt phng
(P) : 2x y + 3z + 1 = 0 v (Q) : x + y z + 5 = 0.
Vit phng trnh mt phng () i qua giao tuyn ca hai mt phng (P) v (Q) ng thi vung gc vi mt phng (R) : 3xy+1 = 0.
Bi 13.60 : Lp phng trnh mt phng () i qua im M(3; 4; 1) v giao tuyn ca hai mt phng
(P) : 19x 6y 4z + 27 = 0 v (Q) : 42x 8y + 3z + 11 = 0.
Bi 13.61 : Vit phng trnh mt phng () cha giao tuyn ca hai mt phng
() : x + y z + 1 = 0 v () : y + z = 0
ng thi
1. vung gc vi mt phng (P) : 2x + 3y z = 0.

2. to vi trc Oy mt gc 45 .

Vn 3 : Khong cch t mt im n mt phng

Khong cch t im M(x0 ; y0 ; z0 ) n mt phng () : Ax + By + Cz + D = 0 l


d(M, ()) =

|Ax0 + By0 + Cz0 + D|

.
A2 + B2 + C 2

Ch :

Nu (P) (Q) th d((Q), (P)) = d(M, (P)) vi M l mt im trn (Q).


Nu AB (P) th d(A, (P)) = d(B, (P)).
Vi bi ton vit phng trnh mt phng khi bit khong cch ta thng lm nh sau :

n = (a; b; c) ,
Gi s
0 l mt vect php tuyn ca mt phng.
T cc d kin bi ton ta tm c 2 phng trnh cha a, b, c.
Xt hai trng hp
Nu a = 0, thay vo cc iu kin ta tm c b, c.

Nu a , 0, chn a = 1 (hoc mt gi tr khc 0 bt k), thay vo cc iu kin ta tm c b, c.

TRN ANH TUN - 0974 396 391 - (04) 66 515 343

WWW.VNMATH.COM

Trang 256

www.VNMATH.com

www.luyenthi24h.com
www.luyenthi24h.com
CHUYN LUYN THI I HCwww.VNMATH.com

Bi 13.62 : Tm trn trc Oz cc im cch u A(2; 3; 4) v mt phng () : 2x + 3y + z 17 = 0.


Bi 13.63 : Tm trn trc Oy im M cch u hai mt phng
() : x + y z + 1 = 0 v ( ) : x y + z 5 = 0.
Bi 13.64 : Cho (P) : 2x 3y + 6z + 19 = 0 v im A(2; 4; 3). Vit phng trnh mt phng (Q) qua A v song song vi (P). Tnh

khong cch gia hai mt phng (P) v (Q).


Bi 13.65 : Tm trn trc tung cc im :
1. Cch u hai mt phng

() : x + y z 1 = 0 v ( ) : x y + z 5 = 0.
2. Cch u im A(1; 2; 3) v mt phng () : x + y z + 3 = 0.

Bi 13.66 : Lp phng trnh mt phng () i qua hai im A(2; 1; 0), B(5; 1; 1) v khong cch t im M 0; 0;
phng () bng

1
2

n mt

7
.
6 3

Bi 13.67 : Lp phng trnh ca mt phng () i qua giao tuyn ca hai mt phng


(P) : x 3y + 7z + 36 = 0 v (Q) : 2x + y z 15 = 0
ng thi () cch gc ta mt khong bng 3.
Bi 13.68 : Cho hai mt phng (P) : 3x + 2y 2 = 0 v (Q) : 2x + y 2z 2 = 0.
1. Tm trn giao tuyn ca (P) v mt phng (Oxy) mt im M cch u (Q) v (Oxz).
2. Tm tp hp nhng im cch u hai mt phng (P) v (Q).
Bi 13.69 : Cho hai mt phng (P) : 3x + y + z 1 = 0, (Q) : 4x + 3y z 5 = 0 v hai im A(1; 2; 4), B(3; 2; 2).
1. Chng minh rng hai mt phng (P) v (Q) ct nhau theo giao tuyn . Tm ta giao im ca vi ba mt phng ta .
2. Tm im M trn sao cho M cch u A v B.
1
3. Tm im N trn sao cho t din OABN c th tch bng .
3
Bi 13.70 : Cho mt phng (P) : 2x + 3y z + 3 = 0 v im A(1; 1; 1).
1. Chng minh rng im A khng nm trn (P). Tm ta hnh chiu vung gc ca A trn (P).
2. Tm ta im B i xng vi im A qua mt phng (P).
3. Tm trn trc Ox im M, trn mt phng (P) im N sao cho A l trung im ca on MN.
Bi 13.71 : Tm im M trn trc Oy trong mi trng hp sau y :
1. M cch u im A v mt phng 3x + 4y z = 0.
2. M cch u hai mt phng 3x 2y + 2z 1 = 0 v 4x + y 1 = 0.
3. Khong cch t M n mt phng 2x + y + 2z 3 = 0 gp hai ln khong cch t M n mt phng (Oxy).

Vn 4 : Gc gia hai mt phng


TRN ANH TUN - 0974 396 391 - (04) 66 515 343

WWW.VNMATH.COM

Trang 257

www.VNMATH.comCHUYN LUYN THI I HC

www.luyenthi24h.com
www.luyenthi24h.com
www.VNMATH.com

n = (A; B; C) v
Nu () : Ax + By + Cz + D = 0 v ( ) : A x + B y + C z + D = 0 c cc vect php tuyn tng ng l

n = (A ; B ; C ) th gc gia hai mt phng () v ( ) c tnh theo cng thc


n



n .

cos = cos( n , n ) =

.

| n |.| n |

Ch : Vi bi ton vit phng trnh mt phng khi bit gc gia hai mt phng ta thng lm nh sau :

n = (a; b; c) ,
Gi s
0 l mt vect php tuyn ca mt phng.

T cc d kin bi ton ta tm c 2 phng trnh cha a, b, c.


Xt hai trng hp
Nu a = 0, thay vo cc iu kin ta tm c b, c.
Nu a , 0, chn a = 1 (hoc mt gi tr khc 0 bt k), thay vo cc iu kin ta tm c b, c.

Bi 13.72 : Vit phng trnh mt phng () cha trc Oz v to vi mt phng (P) : x + 2y

5z = 0 mt gc bng 60 .

Bi 13.73 : Vit phng trnh mt phng () qua A(2; 0; 0), B(0; 2; 0) v to vi mt phng (Oyz) mt gc 60 .

Bi 13.74 : Vit phng trnh mt phng () i qua im A(1; 2; 1) v :


1. vung gc vi cc mt phng
() : 2x y + 3z 1 = 0 v () : x + y + z 2 = 0.
x1 y+1 z
=
= .
2
1
2
3. qua im B(2; 0; 1) v vung gc vi mt phng (Q) : x + 2y z + 1 = 0.
2. vung gc vi (P) : x y + 2z = 0 v song song vi ng thng d :
4. qua im C(2; 1; 0) v to vi (Oxy) mt gc 60 .
Bi 13.75 : Trong khng gian vi h to Oxyz, cho
() : mx + 2y + mz 12 = 0 v () : x + my + z + 7 = 0.
Tm tham s m gc gia hai mt phng () v () bng 45 .
Bi 13.76 : Trong khng gian vi h to Oxyz, lp phng trnh mt phng () i qua hai im M(3; 0; 0) v N(0; 0; 1) ng thi

to vi mt phng (Oxy) mt gc .
3
Bi 13.77 : Trong khng gian vi h to Oxyz, cho
(P) : 5x 2y + 5z 1 = 0 v (Q) : x 4y 8z + 12 = 0.
Lp phng trnh mt phng () i qua gc ta O, vung gc vi mt phng (P) v hp vi mt phng (Q) mt gc 45 .
Bi 13.78 : Trong khng gian vi h to Oxyz, cho hnh lp phng ABCD.A BC D , bit A(0; 0; 0), B(1; 0; 0), D(0; 1; 0), A (0; 0; 1).
Lp phng trnh mt phng () cha ng thng CD v to vi mt phng (BB D D) mt gc nh nht.

Vn 5 : V tr tng i gia mt phng v mt cu

Cho mt phng (P) : Ax + By + Cz + D = 0 v mt cu (S ) tm I(a; b; c), bn knh R.


1. Nu d(I, (P)) > R th mt cu (S ) v mt phng (P) khng c im chung.

TRN ANH TUN - 0974 396 391 - (04) 66 515 343

WWW.VNMATH.COM

Trang 258

www.VNMATH.com

www.luyenthi24h.com
www.luyenthi24h.com
CHUYN LUYN THI I HCwww.VNMATH.com

2. Nu d(I, (P)) = R th mt cu (S ) v mt phng (P) c mt im A chung. Khi (P) c gi l mt phng tip din v A c
gi l tip im, ng thi IA(P).
3. Nu d(I, (P)) < R th mt cu (S ) v mt phng (P) ct nhau theo giao tuyn l mt ng trn. Khi tm J ca ng trn l
hnh chiu vung gc ca I ln mt phng (P), v bn knh r ca ng trn c tnh theo cng thc
r2 = R2 d2 (I, (P)).
Mt phng (P) ct mt cu theo mt ng trn c bn knh ln nht khi v ch khi (P) i qua tm I.

Bi 13.79 : Cho bn im A(3; 6; 2), B(6; 0; 1), C(1; 2; 0), D(0; 4; 1).
1. Vit phng trnh mt cu (S ) i qua bn im A, B, C, D.
2. Vit phng trnh tip din ca mt cu (S ) ti im A.
Bi 13.80 :

1. Vit phng trnh mt cu c tm I(2; 1; 1) v tip xc vi mt phng () : x + 2y 2z + 5 = 0.

2. Vit phng trnh tip din ca mt cu (S ) : x2 + y2 + z2 6x 2y + 4z + 5 = 0 ti im M(4; 3; 0).


1
Bi 13.81 : Cho mt cu (S ) : x2 + y2 + z2 x y z + = 0.
2
1. Vit phng trnh mt phng cha trc Oy v tip xc vi mt cu.
2. Vit phng trnh tip din ca mt cu bit tip din ct ba tia Ox, Oy, Oz ln lt ti A, B, C sao cho OA = OB = OC.
Bi 13.82 : Trong khng gian vi h ta Oxyz, cho im A(1; 4; 2) v mt phng (P) c phng trnh : x + 2y + z 1 = 0.
1. Hy tm ta hnh chiu vung gc ca A trn (P).

2. Vit phng trnh mt cu tm A, tip xc vi (P).

Bi 13.83 : Cho ba im A(2; 0; 1), B(1; 0; 0), C(1; 1; 1) v mt phng (P) : x + y + z 2 = 0.


Vit phng trnh mt cu i qua ba im A, B, C v c tm thuc mt phng (P).

Bi 13.84 : Cho mt phng (P) : 2x + 2y + z m2 3m = 0 v mt cu (S ) : (x 1)2 + (y + 1)2 + (z 1)2 = 9. Tm m mt phng


(P) tip xc vi mt cu (S ).

Vi m va tm c, hy xc nh ta tip im ca mt phng (P) v mt cu (S ).


Bi 13.85 : Cho mt cu (S ) : x2 + y2 + z2 2x 2z m2 = 0 v mt phng (P) : 3x + 6y 2z 22 = 0.
Xc nh tham s m (P) ct (S ) theo giao tuyn l ng trn (C) c din tch bng 2.

Bi 13.86 : Cho mt cu (S ) : (x + 1)2 + (y + 2)2 + (z + 3)2 = 14 v im M(1; 3; 2). Lp phng trnh mt phng () i qua M

v ct (S ) theo giao tuyn l mt ng trn c bn knh nh nht.

Bi 13.87 : Cho mt cu (S ) : (x 3)2 + (y + 2)2 + (z 1)2 = 9 v mt phng (P) : x + 2y + 2z + 11 = 0.


Tm im M trn mt cu (S ) sao cho khong cch t M n mt phng (P) l ngn nht.

Bi 13.88 : Xc nh tm v bn knh ng trn l giao tuyn ca mt cu (S ) v mt phng (P), vi


(S ) : x2 + y2 + z2 2(x + y + z) 22 = 0 v (P) : 3x 2y + 6z + 14 = 0.
Bi 13.89 : Cho hai mt phng song song (P1 ) v (P2 ) c phng trnh
(P1 ) : 2x y + 2z 1 = 0 v (P2 ) : 2x y + 2z + 5 = 0
v im A(1; 1; 1) nm trong khong gia hai mt phng . Gi (S ) l mt cu bt k i qua A v tip xc c hai mt phng (P1 ) v
(P2 ).
1. Chng minh rng bn knh mt cu (S ) l mt hng s v tnh bn knh .
2. Gi I l tm mt cu (S ). Chng minh rng I lun thuc mt ng trn c nh. Xc nh tm v bn knh ng trn .
Bi 13.90 : Vit phng trnh mt phng () tip xc vi mt cu (S ) : (x 1)2 + (y + 3)2 + (z 2)2 = 49 ti im M(7; 1; 5).

Bi 13.91 : Vit phng trnh mt phng () tip xc vi mt cu (S ) : x2 + y2 + z2 2x 4y 6z 2 = 0 v song song vi mt phng


(P) : 4x + 3y 12z + 1 = 0.

TRN ANH TUN - 0974 396 391 - (04) 66 515 343

WWW.VNMATH.COM

Trang 259

www.luyenthi24h.com
www.luyenthi24h.com
www.VNMATH.com

www.VNMATH.comCHUYN LUYN THI I HC

13.3 Phng trnh ng thng


Trong chng trnh ton
8 12, chng ta khng xt dng tng qut ca ng thng, tuy nhin trong ti liu ny khi chng ta vit :
Cho ng thng :

<Ax + By + Cz + D = 0

th chng ta hiu rng ng thng l giao tuyn ca hai mt phng

: A x + B y + C z + D = 0

(P) : Ax + By + Cz + D = 0 v (Q) : A x + B y + C z + D = 0.

Vn 1 : Phng trnh tham s v phng trnh chnh tc ca ng thng

Cho ng thng d i qua im M(x0 ; y0 ; z0 ).

u = (a; b; c) ,
1. Xc nh vect ch phng
0 ca ng thng :

u ,
u l mt vect ch phng ca ng thng d.
(a) Nu
0 c gi song song hoc trng vi ng thng d th

n v
n cng vung gc vi d th vect
u = [
n ,

(b) Nu c
1
2
1 n 2 ] , 0 l mt vect ch phng ca ng thng d.
2. Phng trnh tham s v phng trnh chnh tc ca ng thng d ln lt c dng
8
>
x = x0 + at
>
<

d:

>
>
:

y = y0 + bt

z = z0 + ct
x x0 y y0 z z0
hoc d :
=
=
( nu a, b, c u khc 0).
a
b
c

Bi 13.92 : Vit phng trnh tham s v chnh tc ca cc ng thng sau :


u = (1; 3; 5).
1. i qua im A(2; 0; 1) v c vect ch phng
2. i qua hai im A(2; 3; 1) v B(1; 2; 4).
Bi 13.93 : Vit phng trnh tham s ca cc ng thng sau, tm mt im m ng thng i qua v tm mt vect ch phng
ca ng thng , bit :
1. d :

x2
y
z+3
=
=
;
3
3
1

2. d : x =

y1 z
= .
2
3

3. d :

x2 y+1
=
= z + 1.
3
2

Bi 13.94 : Vit phng trnh chnh tc ca cc ng thng sau, tm mt im m ng thng i qua v tm mt vect ch phng
ca ng thng , bit :

1. d :

8
>
x = 1 + 2t
>
<

y = 3 + t

>
>
:

z = 5 3t.

2. d :

8
>
x=5
>
<

y = 2 + 3t

>
>
:

z = 1 + t.

3. d :

8
>
x=t
>
<

y = 1 + 2t

>
>
:

z = 5 3t.

Vn 2 : Tm im trn ng thng tha mn iu kin cho trc


TRN ANH TUN - 0974 396 391 - (04) 66 515 343

WWW.VNMATH.COM

Trang 260

www.luyenthi24h.com
www.luyenthi24h.com
CHUYN LUYN THI I HCwww.VNMATH.com

www.VNMATH.com

1. Chuyn ng thng v dng tham s

8
>
x = x0 + at
>
<

y = y0 + bt

>
>
:

z = z0 + ct.

2. im M nm trn ng thng nn M(x0 + at; y0 + bt; z0 + ct).


3. Chuyn cc c trng hnh hc ca M sang iu kin v vect.

Bi 13.95 : Cho ng thng d c phng trnh

8
>
x = 2t
>
<

y = 1 + 3t

>
>
:

z = 2 + 2t.

Tm im M trn ng thng d tha mn


1. Khong cch t M n mt phng 2x y z 3 = 0 l

6.

2. M cch u hai mt phng (Oxy) v (Oyz).

3. Hnh chiu vung gc ca M ln mt phng (Oxy) nm trn mt cu tm O bn knh l 2 2.

Bi 13.96 : Cho hai ng thng d :

8
>
x = 1 + 2t
>
<

x y z1
= =
, d : y = 1 3t
>
2 3
1
>
:
z = 1 + t

v mt phng (P) : 3x y z = 0.

1. Tm im A trn d, im B trn d sao cho AB vung gc vi mt phng (P).


2. Tm im C trn d, im D trn d sao cho ng thng CD song song vi mt phng (P) v trng tm tam gic OCD nm trn
mt phng (Oxz).
x1 y
= = z v mt phng (P) : 2x 3y 2z 6 = 0. Xc nh cc im A, B, C, D sao cho A, B
2
3

196 10
nm trn d; S nm trn (P) v S .ABCD l hnh chp t gic u nhn gc ta O lm tm ca y c th tch bng
.
3
Bi 13.97 : Cho ng thng d :

Vn 3 : V tr tng i ca hai ng thng v trong khng gian

u v ng thng i qua M v c mt vect ch phng


Gi s i qua M0 v c mt vect ch phng
u.
0
1. v trng nhau khi v ch khi

2. v song song khi v ch khi

3. v ct nhau khi v ch khi

u ,
<[
u]= 0

u ,
:[
M0 M0 ] = 0 M0 th M0 cng thuc .
8

u ,
<[
u]= 0

u ,
:[
M0 M0 ] , 0 M0 th M0 khng thuc .
8

u ,
<[
u ] =, 0

u ,
:[
u ]. M0 M0 = 0.


u ,
4. v cho nhau khi v ch khi [
u ]. M0 M0 , 0.

Bi 13.98 : Xt v tr tng i ca mi cp ng thng sau. Tm ta giao im ca chng nu c. Vit phng trnh mt phng
cha hai ng thng nu chng ng phng.

TRN ANH TUN - 0974 396 391 - (04) 66 515 343

WWW.VNMATH.COM

Trang 261

www.VNMATH.comCHUYN LUYN THI I HC


x1
2
x1
2. d :
2
x2
3. d :
4
x1
4. d :
9

1. d :

5. d :

y7 z3 x6 y+1 z+2
=
;d :
=
=
;
1
4
3
2
1
y2 z x
y+8 z4
=
= ;d :
=
=
;
2
1
2
3
1
y
z+1 x7 y2
z
=
=
;d :
=
=
;
6
8
6
9
12
y6 z3 x7 y6 z5
=
=
;d :
=
=
;
6
3
6
4
2
=

8
>
x = 9t
>
<

y = 5t

>
>
:

www.luyenthi24h.com
www.luyenthi24h.com
www.VNMATH.com

; d l giao tuyn ca hai mt phng :

z = 3 + t
() : 2x 3y 3z 9 = 0 v ( ) : x 2y + z + 3 = 0.

Bi 13.99 : Vi cc ng thng cho trong bi tp 13.98, trong trng hp d v d cho nhau hy vit phng trnh mt phng (Q)
cha d v (Q) song song vi d v vit phng trnh mt phng (R) qua A(1; 2; 3) ng thi (R) song song vi c d v d .
Bi 13.100 : Xt v tr tng i ca cp ng thng cho bi cc phng trnh sau :
8
>
x = 2t
>
<

d1 :

x1 y2 z
=
= ; v d2 : y = 5 + 3t
>
2
2
1
>
:
z = 4.

Bi 13.101 : Cho hai ng thng

8
>
x = 1 + 2t
>
<

>
>
:

y = 1 + t

v :

z = t

1. Xc nh v tr tng i gia v .

8
>
x = 3 t
>
<
>
>
:

y = 2t
z = 1 + t .

2. Tm giao im (nu c) ca v .

Vn 4 : V tr tng i gia ng thng v mt phng (P)

u v mt phng (P) i qua M v c mt vect php tuyn


n .
Gi s i qua M0 v c mt vect ch phng
1
1. nm trn (P) khi v ch khi

8
u .
n = 0
<

: M th M cng thuc (P).


0
0
8
u .
n = 0
<

2. song song vi (P) khi v ch khi

: M th M khng thuc (P).


0
0

u .
n , 0.
3. v (P) ct nhau khi v ch khi

Bi 13.102 : Xt v tr tng i gia ng thng d v mt phng (), tm giao im ca chng nu c, bit :


x 12 y 9 z 1
=
=
, () : 3x + 5y z 2 = 0;
4
3
1
x+1 y3 z
2. d :
=
= , () : 3x 3y + 2z 5 = 0;
2
4
3
x9 y1 z3
3. d :
=
=
, () : x + 2y 4z + 1 = 0;
8
2
3

1. d :

TRN ANH TUN - 0974 396 391 - (04) 66 515 343

WWW.VNMATH.COM

Trang 262

www.VNMATH.com
4. d :

www.luyenthi24h.com
www.luyenthi24h.com
CHUYN LUYN THI I HCwww.VNMATH.com

x7 y1 z5
=
=
, () : 3x y + 2z 5 = 0;
5
1
4

5. d l giao tuyn ca hai mt phng :


(P) : 3x + 5y + 7z + 16 = 0 v (Q) : 2x y + z 6 = 0,
() : 5x z 4 = 0.
Bi 13.103 : Xc nh giao im ca ng thng d v mt phng (P) trong nhng trng hp sau :

1. d :

8
>
x = 12 + 4t
>
<
>
>
:

y = 9 + 3t

v (P) : 3x + 5y z 2 = 0.

z=1+t

2. d l giao tuyn hai mt phng : x + y + z 2 = 0; x + 2y z 1 = 0 v (P) : x + y + 2z 1 = 0.


Bi 13.104 : Cho ng thng l giao tuyn ca hai mt phng
() : 2kx + y z + 1 = 0 v ( ) : x ky + z 1 = 0.
Vi gi tr no ca k th ng thng nm trong mt phng (Oyz).
x 12 y 9 z 1
=
=
v mt phng () : 3x + 5y z 2 = 0.
Bi 13.105 : Xt v tr tng i ca ng thng d :
4
3
1

Vn 5 : Khong cch t mt im n ng thng

u . Khong cch t im M(x ; y ; z ) n ng thng l


Gi s i qua M0 v c mt vect ch phng
0 0 0
d(M, ) =

h
i


M0 M1 , u

u |
|

Vi bi ton vit phng trnh ng thng khi bit khong cch ta thng lm nh sau :

u = (a; b; c) ,
Gi s
0 l mt vect ch phng ca ng thng.

T cc d kin bi ton ta tm c 2 phng trnh cha a, b, c.


Xt hai trng hp
Nu a = 0, thay vo cc iu kin ta tm c b, c.
Nu a , 0, chn a = 1 (hoc mt gi tr khc 0 bt k), thay vo cc iu kin ta tm c b, c.
Ch : Vi hai ng thng , v mt phng (P), ta c :
1. Nu ct th khong cch gia chng bng 0.
2. Nu song song vi th khong cch gia chng bng khong cch t mt im M n .
3. Nu v cho nhau th khong h cch gia chng tnh theo cng thc




[ u , u ]. M0 M0

.
h=



[ u , u ]

4. Nu (P) th khong cch gia v (P) bng khong cch t mt im thuc n (P).

Bi 13.106 : Tnh khong cch gia hai ng thng cho trong bi tp 13.98.

TRN ANH TUN - 0974 396 391 - (04) 66 515 343

WWW.VNMATH.COM

Trang 263

www.luyenthi24h.com
www.luyenthi24h.com
www.VNMATH.com

www.VNMATH.comCHUYN LUYN THI I HC


Bi 13.107 : Cho ng thng :
A(1; 0; 2).

y
z+1 x6
y+1
z+2
x1
= =
, :
=
=
, mt phng (P) : x 2y + 2z 5 = 0 v im
2
1
3
2
2
1

1. Tnh cc khong cch t A v O n cc ng thng v .

2. Tm im M trn Ox sao cho khong cch t M n bng 2 2.


3. Tm im M trn Oy sao cho M cch u v A.
4. Tm im M trn Oz sao cho M cch u v (P).
5. Tm im M trn Ox sao cho M cch u v .
6. Tm im M thuc sao cho khong cch t M n bng 10.
7. Tm im M thuc sao cho M cch u v (P).
Bi 13.108 : Cho ng thng d :

8
>
x = 3 + 2t
>
<

y = 2 + t

>
>
:

v mt phng (P) : x + y + z + 2 = 0.

z = 1 t

Gi A l giao im ca d v (P). Vit phng trnh nm trong (P) sao cho d v d(A, ) =
Bi 13.109 : Cho ng thng d c phng trnh

8
>
x = 1 t
>
<

42.

y = 3 + 2t

>
>
:

z = 4 + 3t.

21
1. Tm im M trn trc Ox cch d mt khong bng
.
7

x y z1
13 42
2. Tm im N trn ng thng = =
cch d mt khong bng
.
2 3
1
14
x+1 y2 z
=
= v mt phng (P) : 2x + y + mz 1 = 0.
Bi 13.110 : Cho ng thng d :
1
2
3

26
1. Tm im M nm trn giao tuyn ca (P) vi (Oxy) v c khong cch n d bng
.
2
2. Tm m sao cho d (P). Khi hy tnh khong cch t d n (P).
Bi 13.111 : Cho ng thng d :

x1 y+3 z+2
=
=
.
1
2
1

5 2
1. Tm im M trn trc Ox sao cho khong cch t M n d l
.
2
3
2. Tm im N trn mt phng (Oyz) sao cho d(N, Oy) = d(N, Oz) = d(N, d).
15

Bi 13.112 : Cho ng thng d :

8
>
x = 2 + 4t
>
<

y = 3 + 2t

>
>
:

v mt phng (P) : x + y + 2z + 5 = 0.

z = 3 + t

1. Chng minh rng d nm trn (P).


2. Vit phng trnh ng thng d nm trong (P), song song vi d v cch d mt khong

14.

Vn 6 : V tr tng i gia ng thng v mt cu

u v mt cu (S ) tm I(a; b), bn knh R.


Gi s i qua M0 v c mt vect ch phng
1. Nu d(I, ) > R th mt cu (S ) v khng c im chung.

TRN ANH TUN - 0974 396 391 - (04) 66 515 343

WWW.VNMATH.COM

Trang 264

www.VNMATH.com

www.luyenthi24h.com
www.luyenthi24h.com
CHUYN LUYN THI I HCwww.VNMATH.com

2. Nu d(I, ) = R th mt cu (S ) v c mt im A chung. Khi c gi l tip tuyn v A c gi l tip im, ng


thi IA.
3. Nu d(I, ) < R th mt cu (S ) v ct nhau ti hai im A v B. Khi di AB c tnh theo cng thc
AB2
= R2 d2 (I, (P)).
4

Bi 13.113 : Cho mt cu (S ) : (x + 2)2 + (y 1)2 + (z + 5)2 = 109 v ng thng d :

8
>
x = 5 + 3t
>
<

y = 1 + 5t

>
>
:

z = 9 4t.

Tm giao im ca ng thng v mt cu. Vit phng trnh cc mt phng tip xc vi mt cu ti cc giao im trn.
Bi 13.114 : Cho ng thng d l giao tuyn ca hai mt phng
() : 5x 4y + 3z + 20 = 0 v ( ) : 3x 4y + z 8 = 0.
Vit phng trnh mt cu c tm I(2; 3; 1) v ct d ti hai im A, B sao cho AB = 16.
Bi 13.115 : Vit phng trnh mt phng tip xc vi mt cu

(S ) : x2 + y2 + z2 10x + 2y + 26z 113 = 0


v song song vi hai ng thng :

8
>
x = 1 + 3t
>
<

d1 :

Bi 13.116 : Cho ng thng d :

x + 5 y 1 z + 13
=
=
; d1 : y = 1 2t
>
2
3
2
>
:
z = 8.

x y1 z+1
=
=
v hai mt phng
2
1
2
(P) : x + y 2z + 5 = 0; (Q) : 2x y + z + 2 = 0.

Vit phng trnh mt cu c tm thuc ng thng d v tip xc vi c hai mt phng (P) v (Q).
Bi 13.117 : Vit phng trnh mt cu (S ) trong mi trng hp sau :
1. c tm I(1; 4; 7) v tip xc vi mt phng () : 6x + 6y 7z + 42 = 0.
2. c tm H(6; 8; 3) v tip xc vi trc Oz.
Bi 13.118 : Vit phng trnh mt cu (S ) c tm nm trn ng thng d l giao tuyn ca hai mt phng x + y + z + 1 = 0 v
x y + z 1 = 0 ng thi tip xc vi hai mt phng () : x + 2y + 2z + 3 = 0 v () : x + 2y + 2z + 7 = 0.
x y1 z+1
Bi 13.119 : Cho ng thng d : =
=
v hai mt phng
2
1
2
() : x + y 2z + 5 = 0 v () : 2x y + z + 2 = 0.
1. Gi A v B ln lt l giao im ca d vi hai mt phng () v (). Tnh di on thng AB.
2. Vit phng trnh mt cu (S ) c tm thuc ng thng d v tip xc vi hai mt phng () v ().
Bi 13.120 : Cho im I(2; 3; 1) v ng thng d l giao tuyn ca hai mt phng 5x 4y + 3z + 20 = 0 v 3x 4y + z 8 = 0.
1. Xc nh ta hnh chiu vung gc H ca im I trn ng thng d.
2. Vit phng trnh mt cu (S ) c tm I v ct ng thng d ti hai im A v B sao cho di on AB = 10.
Bi 13.121 : Cho mt cu (S ) : x2 + y2 + z2 4x + 6y + 6z + 17 = 0 v ng thng d l giao tuyn ca hai mt phng x + y = 0 v
3y 2z 1 = 0.

Vit phng trnh mt phng () cha ng thng d v ct mt cu (S ) theo giao tuyn l mt ng trn c bn knh bng 2.

TRN ANH TUN - 0974 396 391 - (04) 66 515 343

WWW.VNMATH.COM

Trang 265

www.VNMATH.comCHUYN LUYN THI I HC

www.luyenthi24h.com
www.luyenthi24h.com
www.VNMATH.com

Bi 13.122 : Cho ng thng d l giao tuyn ca hai mt phng 2x 2y z + 1 = 0 v x + 2y 2z 4 = 0 v mt cu (S ) :


x2 + y2 + z2 + 4x 6y + m = 0.

Tm m ng thng d ct mt cu (S ) ti hai im M, N sao cho khong cch gia hai im bng 9.

Bi 13.123 : Cho ba im A(2; 4; 1), B(1; 4; 0), C(0; 0; 3).


1. Xc nh tm v bn knh ng trn ngoi tip tam gic ABC.

2. Cho ng thng d :

8
>
x = 2 5t
>
<

y = 4 + 2t

>
>
:

z = 1.

Chng minh rng d ct ng trn ngoi tip tam gic ABC ti hai im phn bit. Tm ta hai giao im .
Bi 13.124 : Cho ng thng d l giao tuyn ca hai mt phng 8x 11y + 8z 30 = 0 v x y 2z = 0, mt cu (S ) : x2 + y2 + z2 +
2x 6y + 4z 15 = 0.

Vit phng trnh mt phng () cha d v tip xc vi mt cu (S ).

Vn 7 : Gc gia hai ng thng ; gc gia ng thng v mt phng

u , ng thng i qua M v c mt vect ch phng


Gi s i qua M0 v c mt vect ch phng
u v mt phng (P) :
0

Ax + By + Cz + D = 0 c vect php tuyn l n = (A; B; C) th gc gia hai ng thng v ; gc gia v (P) tnh theo
1

cng thc.





u .
n


u




u .



cos 1 = cos( u , u ) =
v sin 2 = cos( u , n ) = |
u |.|
n | .
u |.|
|
u|

Ch : Vi bi ton vit phng trnh ng thng khi bit gc ta thng lm nh sau :

u = (a; b; c) ,
Gi s
0 l mt vect ch phng ca ng thng.

T cc d kin bi ton ta tm c 2 phng trnh cha a, b, c.


Xt hai trng hp
Nu a = 0, thay vo cc iu kin ta tm c b, c.
Nu a , 0, chn a = 1 (hoc mt gi tr khc 0 bt k), thay vo cc iu kin ta tm c b, c.

x+1 y1 z2
=
=
vi trc Ox.
2
1
1
x+3 y+1 z3
Bi 13.126 : Tm gc to bi gia ng thng d :
=
=
v mt phng () : x + 2y z + 5 = 0.
2
1
1
Bi 13.127 : Trong khng gian vi h to Oxyz, cho ng thng l giao tuyn ca hai mt phng x z sin + cos = 0 v
Bi 13.125 : Tm gc to bi ng thng d :

y z cos sin , vi l s thc. Tnh gc to bi ng thng v trc Oz.

Bi 13.128 : Trong khng gian vi h to Oxyz, cho hai ng thng : 1 l giao tuyn ca hai mt phng x ay z 1 = 0 v

y 2 = 0 ; 2 l giao tuyn ca hai mt phng ax + 3y a 3 = 0 v z 1 = 0.


Xc nh a 1 v 2 hp vi nhau mt gc 45 .

Bi 13.129 : Lp phng trnh ng thng i qua im M(1; 5; 3) v to vi hai trc ta Ox, Oy cc gc bng nhau v bng
60 .

Bi 13.130 : Trong khng gian vi h to Oxyz, cho


() : x y + z 5 = 0 v :

x y2 z
=
= .
1
2
2

Vit phng trnh ng thng d i qua im A(3; 1; 1), nm trong mt phng () v hp vi ng thng mt gc 45 .

TRN ANH TUN - 0974 396 391 - (04) 66 515 343

WWW.VNMATH.COM

Trang 266

www.VNMATH.com

www.luyenthi24h.com
www.luyenthi24h.com
CHUYN LUYN THI I HCwww.VNMATH.com

Bi 13.131 : Trong khng gian vi h to Oxyz, cho ng thng :


vi cc trc ta Ox, Oy, Oz.

x+4 y3 z+1
=
=
. Gi , , ln lt l gc hp bi
2
1
1

Chng minh rng cos2 + cos2 + cos2 = 1.


Ch :

Khng nh ny cn ng vi ng thng bt k.
Mt khng nh tng t l thay ng thng bi mt phng (P) bt k v gc , , l gc hp bi (P) vi cc mt phng ta
.
Bi 13.132 : Trong khng gian vi h to Oxyz, cho
d:

x3 y4 z+3
=
=
v () : 2x + y + z 1 = 0.
1
2
1

Tnh s o gc to bi ng thng d v mt phng ().


Bi 13.133 : Trong khng gian vi h to Oxyz, cho ng thng d1 l giao tuyn ca hai mt phng x + y 2 = 0 v y + z 2 = 0
x2 y3 z+5
; d2 :
=
=
.
2
1
1
Vit phng trnh mt phng cha ng thng d1 v to vi ng thng d2 mt gc bng 60 .
Bi 13.134 : Trong khng gian vi h to Oxyz, cho
8
>
x = t
>
<

v () : 2x y 2z 2 = 0.

y = 1 + 2t

>
>
:

z =2+t

Vit phng trnh mt phng (P) cha ng thng v to vi mt phng () mt gc nh nht.

Vn 8 : Phng trnh ng thng bit ng thng song song, hoc vung gc vi ng thng hoc mt phng
khc, hoc nm trn mt phng khc

Da vo cc quan h song song, vung gc, nm trong xc nh vect ch phng ca ng thng nh trong vn 1.
Ch :

Nu th u cng l mt vect ch phng ca .


n cng l mt vect ch phng ca .
Nu (P) th
P

Nu th u vung gc vi .
n vung gc vi .
Nu (P) hoc (P) th
P

Khi vit phng trnh theo trng hp ny phi kim tra tnh song song hoc nm trong ca ng thng cn vit.

Bi 13.135 : Vit phng trnh ng thng d trong cc trng hp sau :

1. i qua M(4; 3; 1) v song song vi ng thng :

8
>
x = 1 + 2t
>
<

y = 3t

>
>
:

z = 3 + 2t.

2. i qua im M(2; 1; 0) v vung gc vi mt phng (P) : x + 2y 2z + 1 = 0.

TRN ANH TUN - 0974 396 391 - (04) 66 515 343

WWW.VNMATH.COM

Trang 267

www.VNMATH.comCHUYN LUYN THI I HC


3. i qua im M(2; 1; 1) v vung gc vi hai ng thng

1 :

www.luyenthi24h.com
www.luyenthi24h.com
www.VNMATH.com

8
>
x=t
>
<

x
y+1 z+6
=
=
v 2 : y = 1 2t
>
1
1
2
>
:
z = 0.

4. i qua im M(1; 4; 2) v song song vi cc mt phng


() : 6x + 2y + 2z + 3 = 0 v () : 3x 5y 2z 1 = 0.
5. i qua im A(1; 1; 2), song song vi (P) : x y z 1 = 0 v vung gc vi d :

x+1 y1 z2
=
=
2
1
3

Bi 13.136 : Vit phng trnh ng thng i qua im M(1; 1; 1), song song vi mt phng (P) : x + 2y z + 1 = 0 v vung gc
y
z+1
x+2
=
=
.
vi ng thng d :
1
2
3
Bi 13.137 : Tm tp hp nhng im M trong khng gian cch u ba im A(1; 1; 1), B(1; 2; 0), C(2; 3; 2).
x+1 y1 z2
Bi 13.138 : Cho ng thng d :
=
=
v mt phng () : x y z 1 = 0. Vit phng trnh ng thng qua
2
1
3
A(1; 1; 2), song song vi () v vung gc vi d.

Bi 13.139 : Vit phng trnh tham s ca ng thng l giao tuyn ca hai mt phng () : 2xy+z+5 = 0 v ( ) : 2xz+3 = 0.
Bi 13.140 : Cho ng thng

8
>
x = 2 + 2t
>
<
>
>
:

y = 1 + 3t
z = 4 + 3t.

Vit phng trnh ng thng di dng giao tuyn ca hai mt phng ln lt song song vi Ox v Oy.

Vn 9 : Phng trnh ng thng bit ct


1. Vit theo tham s t hoc t .
2. Gi s ct ti A. Do A nn A c ta theo tham s t hoc t .
3. T cc d kin bi ton ta thit lp c phng trnh tm c cc tham s t v t . T vit c ng thng .
Ch :
n

1.
1 n 2 khi v ch khi n 1 . n 2 = 0.

n cng phng
n khi v ch khi [
n ,

2.
1
2
1 n 2 ] = 0 (hoc ta tng ng t l).

3. Ba im A, B, C thng hng khi v ch khi AB v AC cng phng.
4. Cc bi ton dng ny c th s dng phng php giao tuyn ca hai mt phng.

Bi 13.141 : Cho hai ng thng

8
>
x =8+t
>
<

1 :

y = 5 + 2t

>
>
:

z =8t

v 2 :

8
>
x = 3 7t
>
<

y = 1 + 2t

>
>
:

z = 1 + 3t .

1. Chng minh rng hai ng thng cho nhau.


2. Vit phng trnh ng vung gc chung ca hai ng thng .

TRN ANH TUN - 0974 396 391 - (04) 66 515 343

WWW.VNMATH.COM

Trang 268

www.luyenthi24h.com
www.luyenthi24h.com
CHUYN LUYN THI I HCwww.VNMATH.com

www.VNMATH.com

Bi 13.142 : Trong khng gian Oxyz cho bn im A(4; 1; 4), B(3; 3; 1), C(1; 5; 5; ) v D(1; 1; 1).
1. Chng minh rng A, B, C, D l bn nh ca mt t din.
2. Vit phng trnh ng vung gc chung gia AC v BD.
Bi 13.143 : Cho hai ng thng d1 v d2 c phng trnh :
8
>
x = 2 + 3t
>
<

d1 :

>
>
:

y=t
z = 1 2t

8
>
x = 2 + 2t
>
<

y = t

>
>
:

z = 2 + t .

Vit phng trnh ng thng d i qua im M(1; 1; 1) v ct ng thi d1 , d2 .


Bi 13.144 : Cho ba ng thng
d1 :

x2 y+2 z1
x7 y3 z9
x+1 y+3 z2
=
=
; d2 :
=
=
; d3 :
=
=
.
3
4
1
1
2
1
3
2
1

Vit phng trnh ng thng ct ng thi d1 v d2 ng thi song song vi d3 .


Bi 13.145 : Cho hai ng thng

8
>
x = 1
>
<

d1 :

x1 y2 z
=
= v d2 : y = t
>
3
1
1
>
:
z = 1 + t.

Vit phng trnh ng thng i qua im A(0; 1; 1), vung gc vi d1 v ct d2 .


x+1 y
z
Bi 13.146 : Vit phng trnh ng thng i qua im M(1; 1; 1), ct d :
= =
v song song vi (P) : x+3y+z1 = 0.
2
1 1
Bi 13.147 : Vit phng trnh ng thng vung gc vi mt phng () : x + 2y z + 1 = 0 v ct cc ng thng
1 :

x1 y+1 z2
x2
y
z+1
=
=
v 2 :
=
=
.
2
1
1
1
1
2

x+1 y2 z+2
=
=
.
2
1
3
Vit phng trnh ng thng nm trong mt phng (), vung gc vi d v ct d.

Bi 13.148 : Cho mt phng () : x + 2y z 2 = 0 v ng thng d :

Bi 13.149 : Vit phng trnh ng thng nm trong mt phng (P) : x + 2z = 0 v ct hai ng thng
8
>
x=1t
>
<

d1 :

y=t

>
>
:

z = 4t

Bi 13.150 : Cho hai ng thng

8
>
x =1+t
>
<

d1 :

y = 1 t

>
>
:

z=2

v d2 :

8
>
x = 2 t
>
<

y = 4 + 2t

>
>
:

z = 1.

v d2 :

8
>
x = 3 t
>
<

y = 1 + 2t

>
>
:

z = t .

Lp phng trnh mt cu (S ) c ng knh l on vung gc chung ca hai ng thng d1 v d2 .


x1 y1 z1
x
y+1 z3
Bi 13.151 : Cho hai ng thng d1 :
=
=
v d2 :
=
=
.
1
2
2
1
2
2
1. Tm ta giao im I ca d1 v d2 .

2. Vit phng trnh mt phng (P) qua d1 v d2 .


3. Vit phng trnh ng thng d qua M(0; 1; 2) ct d1 v d2 ln lt ti A v B khc I sao cho AI = AB.

TRN ANH TUN - 0974 396 391 - (04) 66 515 343

WWW.VNMATH.COM

Trang 269

www.luyenthi24h.com
www.luyenthi24h.com
www.VNMATH.com

www.VNMATH.comCHUYN LUYN THI I HC


Vn 10 : Hnh chiu v tnh i xng


1. Hnh chiu H ca A xung . C mt s cch sau :
(a) Gi (P) l mt phng cha A v vung gc vi . Khi H l giao im ca (P) v .

(b) Gi s H , nn H c ta tha mn (theo t). Do AH nn AH.
u = 0. Tm c t, t suy ra H.
2. Hnh chiu H ca A xung (P). C mt s cch sau :
(a) Gi l mt phng cha A v vung gc vi (P). Khi H l giao im ca (P) v .


(b) Gi s H , nn H c ta tha mn (P). Do AH(P) nn [AH,
n (P) ] = 0 . Tm c H.
3. Hnh chiu vung gc ca xung (P).
(a) Gi (Q) l mt phng cha v (Q)(P).
(b) Khi hnh chiu vung gc l giao tuyn ca (P) v (Q).
4. Hnh chiu theo phng ng thng d ca xung (P).
(a) Gi (Q) l mt phng cha v (Q) cng phng vi d ((Q) song song hoc (Q) cha d).
(b) Khi hnh chiu vung gc l giao tuyn ca (P) v (Q).
5. Tm im i xng A ca A qua d hoc (P).
(a) Gi H l hnh chiu vung gc ca A trn d hoc (P).
(b) Khi H l trung im ca AA , tc l

8
<x
:y

= 2xH xA

= 2yH yA .

Bi 13.152 : Vit phng trnh hnh chiu ca ng thng d :

x1 y2 z+3
=
=
trn cc mt phng ta .
2
3
4

Bi 13.153 : Vit phng trnh tham s hnh chiu vung gc ca ng thng d :

8
>
x =2t
>
<

y = 2t

>
>
:

trn () : x + y + z 3 = 0.

z = 1 + 2t

Bi 13.154 : Tm im i xng ca im M(1; 3; 7) i vi mt phng () : 2x + 5y 2z 6 = 0.


Bi 13.155 : Tnh khong cch t im M(2; 3; 1) n ng thng d :
Bi 13.156 : Tnh khong cc gia cc cp ng thng
d1 :

x+2 y1 z+1
=
=
.
1
2
2

x1 y+3 z4
x+2 y1 z+1
=
=
v d2 :
=
=
.
2
1
2
4
2
4

Bi 13.157 : Cho mt phng (P) : 6x + 3y + 2z 6 = 0 v im A(0; 0; 1).


1. Tm ta hnh chiu H ca im A trn (P).
2. Tm ta im A l im i xng ca A qua mt phng (P).

Bi 13.158 : Cho t din ABCD c A(4; 1; 4), B(3; 3; 1), C(1; 5; 5), D(1; 1; 1). Tm ta hnh chiu vung gc ca D trn mt phng
(ABC).

TRN ANH TUN - 0974 396 391 - (04) 66 515 343

WWW.VNMATH.COM

Trang 270

www.luyenthi24h.com
www.luyenthi24h.com
CHUYN LUYN THI I HCwww.VNMATH.com

www.VNMATH.com

Bi 13.159 : Cho mt phng (P) : 3x + 6y z 2 = 0 v ng thng d :

8
>
x = 8 + 4t
>
<

y = 6 + 3t

>
>
:

z = t.

Vit phng trnh ng thng d i xng vi d qua mt phng (P).


z
x+2 y+2
=
=
v im M(4; 3; 2). Tm ta hnh chiu H ca M trn ng thng d.
Bi 13.160 : Cho ng thng d :
3
2
1

Bi 13.161 : Cho ng thng :

8
>
x = 1 + 2t
>
<
>
>
:

v im M(2; 1; 3).

y=2t
z = 3t

Tm im M i xng vi im M qua ng thng .

Bi 13.162 : Cho hai ng thng


1 :

x7 y3 z9
x3 y1 z1
=
=
v 2 :
=
=
.
7
2
3
1
2
1

Vit phng trnh chnh tc ng thng 3 i xng vi 2 qua 1 .


Bi 13.163 : Trong khng gian Oxyz cho ng thng
d:

x+1 y1 z3
=
=
v (P) : 2x 2y + z 3 = 0
1
2
2

Vit phng trnh hnh chiu vung gc d ca d trn (P).

Bi 13.164 : Cho ng thng d l giao tuyn ca hai mt phng :


() : 5x 4y 2z 5 = 0 v ( ) : x + 2z 2 = 0
v mt phng (P) : 2x y + z 1 = 0. Vit phng trnh hnh chiu vung gc d ca d trn mt phng (P).

Vn 11 : Bi ton cc tr


1. Mi quan h ng vung gc v ng xin.
2. Bt ng thc tam gic v cc bt ng thc kinh in.
3. Hoc a v bi ton tm gi tr ln nht, gi tr nh nht ca hm mt bin.
4. Nu qua M c nh v song song vi mt phng (P) c nh th nm trn mt phng (Q) qua M v song song vi (P).
5. Nu qua M c nh v vung gc vi c nh th nm trn mt phng (P) qua M v vung gc vi .

Bi 13.165 : Cho im A(1; 2; 3), ng thng d v mt phng (P) ln lt c phng trnh


d:

x1 y z+2
= =
v (P) : x y + 5z 6 = 0.
2
1
2

1. Trong cc im thuc d, tm im cch A mt khong ngn nht.


2. Tm im M thuc (P) sao cho AM ngn nht.

Bi 13.166 : Vn hi nh bi 13.165 vi gi tr lc ny l A(0; 2; 0), ng thng d v mt phng (P) ln lt c phng trnh


8
>
x = 1+t
>
<

d:

y = 2 + 3t

>
>
:

v (P) : 3y 4z + 1 = 0.

z=t

TRN ANH TUN - 0974 396 391 - (04) 66 515 343

WWW.VNMATH.COM

Trang 271

www.VNMATH.comCHUYN LUYN THI I HC


Bi 13.167 : Cho hai ng thng

www.luyenthi24h.com
www.luyenthi24h.com
www.VNMATH.com

8
>
x = 1 + 2t
>
<

x y2 z+1
=
=
v : y = t
>
2
1
2
>
:
z = 5 t.

1. Chng minh rng v cho nhau.


2. Tm hai im A v B ln lt thuc v sao cho AB t gi tr nh nht.
3. Vit phng trnh mt phng (P) qua O, (P) song song vi c hai ng thng v .
Bi 13.168 : Cho hai ng thng

8
>
x = 2t
>
<

x1 y+2 z+1
=
=
v : y = 1 t
>
1
2
2
>
:
z = 2 2t.

1. Chng minh rng v cho nhau.


2. Tm hai im A v B ln lt thuc v sao cho AB t gi tr nh nht.
3. Vit phng trnh mt phng (P) cha v (P) song song vi .
Bi 13.169 : Cho A(0; 2; 1), B(2; 1; 1). Tm im M trn ng thng :
1. Tam gic ABM c chu vi nh nht.

x1
y
z+2
=
=
sao cho
2
1
3

2. Tam gic ABM c din tch nh nht.

x y1 z+1
=
=
v hai im A(0; 1; 2), B(1; 2; 3). Tm im M thuc sao cho
1
2
2

1. MA2 + MB2 t gi tr nh nht.
3. |2 MA MB| t gi tr nh nht.

Bi 13.170 : Cho ng thng :


4. |2 MA + MB| t gi tr nh nht.

2. MA + MB t gi tr nh nht.

Bi 13.171 : Cho mt phng (P) : x 2y + 2z 3 = 0 v hai im A(0; 1; 2), B(1; 2; 3). Tm im M thuc (P) sao cho

3. |2 MA MB| t gi tr nh nht.

1. MA2 + MB2 t gi tr nh nht.


4. |2 MA + MB| t gi tr nh nht.

2. MA + MB t gi tr nh nht.

Bi 13.172 : Cho mt phng (P) : 2x y + 2z 5 = 0 v mt cu (S ) : (x 1)2 + y2 + (z 2)2 = 9.


1. Tm im M thuc (S ) sao cho M cch (P) mt khong
(a) ln nht;

(b) nh nht.

2. Chng minh rng (P) ct (S ) theo giao tuyn l mt ng trn. Xc nh tm v bn knh ca ng trn .
3. Vit phng trnh mt phng tip din ca (S ), bit tip din song song vi (P).
Bi 13.173 : Cho A(1; 2; 3), B(0; 2; 1) v ng thng :

x y+2 z2
=
=
. Vit phng trnh mt phng (P), bit
1
1
3

1. (P) qua B v cch A mt khong ln nht;


2. (P) cha B, (P) v (P) cch mt khong ln nht.
3. (P) cha v cch A mt khong
(a) ln nht;

(b) nh nht.

4. (P) cha B, O v cch A mt khong

TRN ANH TUN - 0974 396 391 - (04) 66 515 343

WWW.VNMATH.COM

Trang 272

www.VNMATH.com

www.luyenthi24h.com
www.luyenthi24h.com
CHUYN LUYN THI I HCwww.VNMATH.com

(a) ln nht;

(b) nh nht.

5. (P) cha B, (P) v cch A mt khong


(a) ln nht;

(b) nh nht.

6. (P) cha v to vi mt phng (Oxy) mt gc


(a) ln nht;

(b) nh nht.

7. (P) cha A, (P) v to vi mt phng (Oxy) mt gc


(a) ln nht;

(b) nh nht.

8. (P) cha v to vi trc Ox mt gc


(a) ln nht;

(b) nh nht.

Bi 13.174 : Cho A(1; 0; 2), B(0; 0; 1), mt phng (P) : x 2y + 2z 5 = 0 v ng thng :

8
>
x=1
>
<

y=1+t

>
>
:

z = t.

Vit phng trnh ng thng d, bit


1. d i qua A, ct v cch B mt khong
(a) ln nht;

(b) nh nht.

2. d qua A, d (P) v d cch B mt khong


(a) ln nht;

(b) nh nht.

3. d qua A, d v d cch B mt khong


(a) ln nht;

(b) nh nht.

4. d nm trong (P), d v B cch d mt khong nh nht.


5. d nm trong (P), d(Q) v B cch d mt khong nh nht, vi (Q) : 4x + y z 3 = 0.

13.4 Hnh hc khng gian trong cc k thi tuyn sinh H


Bi 13.175 (C08) : Trong khng gian vi h to Oxyz, cho im A(1; 1; 3) v ng thng d c phng trnh
x
y
z1
=
=
.
1 1
2

1. Vit phng trnh mt phng (P) i qua A v vung gc vi ng thng d.


2. Tm ton im M thuc ng thng d sao cho tam gic MOA cn ti nh O.
= ABC
= 90 , AB = BC = a, AD = 2a, S A vung
Bi 13.176 (C08) : Cho hnh chp S .ABCD c y ABCD l hnh thang, BAD

gc vi y v S A = 2a. Gi M v N ln lt l trung im S A v S D. Chng minh rng BCN M l hnh ch nht v tnh th tch khi
chp S .BCN M theo a.

TRN ANH TUN - 0974 396 391 - (04) 66 515 343

WWW.VNMATH.COM

Trang 273

www.VNMATH.comCHUYN LUYN THI I HC

www.luyenthi24h.com
www.luyenthi24h.com
www.VNMATH.com

Bi 13.177 (C09) : Trong khng gian vi h to Oxyz, cho cc mt phng (P1 ) : x + 2y + 3z + 4 = 0 v (P2 ) : 3x + 2y z + 1 = 0.
Vit phng trnh mt phng (P) i qua im A(1; 1; 1), vung gc vi hai mt phng (P1 ) v (P2 ).

Bi 13.178 (C09) : Trong khng gian vi h to Oxyz, cho tam gic ABC c A(1; 1; 0), B(0; 2; 1) v trng tm G(0; 2; 1). Vit

phng trnh ng thng i qua C v vung gc vi mt phng (ABC).

Bi 13.179 (C09) : Cho t gic u S .ABCD c AB = a, S A = a 2. Gi M, N v P ln lt l trung im ca cc cnh S A, S B v


CD. Chng minh rng ng thng MN vung gc vi ng thng S P. Tnh theo a th tch khi t din AMNP.
Bi 13.180 (C10) : Cho hnh chp S .ABCD c y ABCD l hnh vung cnh a, mt phng (S AB) vung gc vi mt phng y,
S A = S B, gc gia ng thng S C v mt phng y bng 45 . Tnh theo a th tch ca khi chp S .ABCD.
Bi 13.181 (C10) : Trong khng gian vi h ta Oxyz, cho hai im A(1; 2; 3), B(1; 0; 1) v mt phng (P) : x + y + z + 4 = 0.
1. Tm ta hnh chiu vung gc ca A trn (P).
AB
c tm thuc ng thng AB v (S ) tip xc vi (P).
6
x
y1 z
Bi 13.182 (C10) : Trong khng gian vi h ta Oxyz, cho ng thng
=
= v mt phng (P) : 2x y + 2z 2 = 0.
2
1
1
1. Vit phng trnh mt phng cha d v vung gc vi (P).
2. Vit phng trnh mt cu (S ) c bn knh bng

2. Tm ta im M thuc d sao cho M cch u gc ta O v mt phng (P).


Bi 13.183 (A02) : Cho hnh chp tam gic u S .ABC c nh S , c di cnh y bng a. Gi M, N ln lt l cc trung im cc
cnh S B v S C. Tnh theo a din tch tam gic AMN, bit rng mt phng (AMN) vung gc vi mt phng (S BC).
Bi 13.184 (A02) : Trong khng gian vi h to Oxyz, cho hai ng thng :

1 :

8
< x 2y + z 4 = 0

: x + 2y 2z + 4 = 0

v 2 :

8
>
x=1+t
>
<

y=2+t

>
>
:

z = 1 + 2t.

1. Vit phng trnh mt phng (P) cha ng thng 1 v song song vi ng thng 2 .
2. Cho im M(2; 1; 4). Tm to im H thuc ng thng 2 sao cho on thng MH c di nh nht.
Bi 13.185 (A03) : Cho hnh lp phng ABCD.A BC D . Tnh s o gc gia hai mt phng (A BC) v (ACD).
Bi 13.186 (A03) : Trong khng gian vi h to Oxyz, cho hnh hp ch nht ABCD.A BC D c A trng vi gc to ,
B(a; 0; 0), D(0; a; 0), A(0; 0; b) vi (a > 0, b > 0). Gi M l trung im cnh CC .
1. Tnh th tch khi t din BDA M theo a v b.
a
2. Xc nh t s hai mt phng (A BD) v (MBD) vung gc vi nhau.
b
Bi 13.187 (A04) : Trong khng gian vi h to Oxyz, cho hnh chp S .ABCD c y l hnh thoi, AC ct BD ti gc to O.

Bit A(2; 0; 0), B(0; 1; 0), S (0; 0; 2 2). Gi M l trung im ca cnh S C.


1. Tnh gc v khong cch gia hai ng thng S A, BM.
2. Gi s mt phng (ABM) ct ng thng S D ti im N. Tnh th tch khi chp S .ABMN.
Bi 13.188 (A04) : Trong khng gian vi h to Oxyz, cho ng thng d v mt phng (P) ln lt c phng trnh
d:

x1 y+3 z3
=
=
v (P) : 2x + y 2z + 9 = 0.
1
2
1

1. Tm to im I thuc d sao cho khong cch t I n mt phng (P) bng 2.

2. Tm to giao im A ca ng thng d v mt phng (P). Vit phng trnh tham s ca ng thng nm trong mt
phng (P), bit i qua A v vung gc vi d.
Bi 13.189 (A06) : Trong khng gian vi h to Oxyz, cho hnh lp phng ABCD.A BC D vi A(0; 0; 0), B(1; 0; 0), D(0; 1; 0),
A (0; 0; 1). Gi M, N l trung im AB, CD.

TRN ANH TUN - 0974 396 391 - (04) 66 515 343

WWW.VNMATH.COM

Trang 274

www.VNMATH.com

www.luyenthi24h.com
www.luyenthi24h.com
CHUYN LUYN THI I HCwww.VNMATH.com

1. Tnh khong cch gia hai ng thng AC v MN.


1
2. Vit phng trnh mt phng cha AC v to vi mt phng (Oxy) mt gc bit cos = .
6
Bi 13.190 (A06) : Cho hnh tr c cc y l hai hnh trn tm O v O , bn knh y bng chiu cao v bng a. Trn ng trn y
tm O ly im A, trn ng trn y tm O ly im B sao cho AB = 2a. Tnh th tch ca khi t din OO AB.
Bi 13.191 (A07) : Trong khng gian vi h to Oxyz, cho hai ng thng
8
>
x = 1 + 2t
>
<

d1 :

x y1 z+2
=
=
v d2 : y = 1 + t
>
2
1
1
>
:
z = 3.

1. Chng minh rng d1 v d2 cho nhau.


2. Vit phng trnh ng thng d vung gc vi mt phng (P) : 7x + y 4z = 0 v ct hai ng thng d1 , d2 .
Bi 13.192 (A07) : Cho hnh chp S .ABCD c y l hnh vung cnh a, mt bn S AD l tam gic u v nm trong mt phng
vung gc vi y. Gi M, N, P ln lt l trung im cc cnh S B, BC, CD. Chng minh AM vung gc vi BP v tnh th tch khi
t din CMNP.
Bi 13.193 (A08) : Trong khng gian vi h to Oxyz, cho im A(2; 5; 3) v ng thng d :

x1 y z2
= =
.
2
1
2

1. Tm to hnh chiu vung gc ca im A trn ng thng d.


2. Vit phng trnh mt phng () cha d sao cho khong cch t A n () ln nht.

Bi 13.194 (A08) : Cho lng tr ABC.A BC c di cnh bn bng 2a, y ABC l tam gic vung ti A, AB = a, AC = a 3 v
hnh chiu vung gc ca nh A trn mt phng (ABC) l trung im ca cnh BC. Tnh theo a th tch khi chp A .ABC v tnh
cosin ca gc gia hai ng thng AA , BC .
Bi 13.195 (A09) : Cho hnh chp S .ABCD c y ABCD l hnh thang vung ti A v D; AB = AD = 2a, CD = a; gc gia hai
mt phng (S BC) v (ABCD) bng 60 . Gi I l trung im ca cnh AD. Bit hai mt phng (S BI) v (S CI) cng vung gc vi mt
phng (ABCD), tnh th tch ca khi chp S .ABCD theo a.
Bi 13.196 (A09) : Trong khng gian vi h to Oxyz, cho mt phng
(P) : 2x 2y z 4 = 0 v mt cu (S ) : x2 + y2 + z2 2x 4y 6z 11 = 0.
Chng minh rng mt phng (P) ct mt cu (S ) theo mt ng trn. Xc nh ta tm v tnh bn knh ca ng trn .
Bi 13.197 (A09) : Trong khng gian vi h to Oxyz, cho mt phng
(P) : x 2y + 2z 1 = 0 v hai ng thng 1 :

x1 y3 z+1
x+1 y z+9
= =
, 2 :
=
=
.
1
1
6
2
1
2

Xc nh ta im M thuc ng thng 1 sao cho khong cch t M n ng thng 2 v khong cch t M n mt phng
(P) bng nhau.
Bi 13.198 (A10) : Cho hnh chp S .ABCD c y ABCD l hnh vung cnh a. Gi M v N ln lt l trung im ca cc cnh AB

v AD; H l giao im ca CN vi DM. Bit S H vung gc vi mt phng (ABCD) v S H = a 3. Tnh th tch khi chp S .CDN M
v tnh khong cch gia hai ng thng DM v S C theo a.
x1 y z+2
= =
v mt phng (P) : x 2y + z = 0. Gi C
2
1
1
l giao im ca vi (P), M l im thuc . Tnh khong cch t M n (P), bit MC = 6.
x+2
y2
z+3
Bi 13.200 (A10) : Trong khng gian to Oxyz, cho im A(0; 0; 2) v ng thng :
=
=
. Tnh khong
2
3
2
cch t A n . Vit phng trnh mt cu tm A, ct ti hai im B v C sao cho BC = 8.

Bi 13.199 (A10) : Trong khng gian to Oxyz, cho ng thng :

Bi 13.201 (B02) : Cho hnh lp phng ABCD.A1 B1C1 D1 c cnh bng a.


1. Tnh theo a khong cch gia hai ng thng A1 B v B1 D.

TRN ANH TUN - 0974 396 391 - (04) 66 515 343

WWW.VNMATH.COM

Trang 275

www.VNMATH.comCHUYN LUYN THI I HC

www.luyenthi24h.com
www.luyenthi24h.com
www.VNMATH.com

2. Gi M, N, P ln lt l cc trung im ca cc cnh BB1, CD, A1 D1 . Tnh gc gia hai ng thng MP v C1 N.


= 60 . Gi M l trung
Bi 13.202 (B03) : Cho hnh lng tr ng ABCD.A BC D c y ABCD l mt hnh thoi cnh a, gc BAD

im cnh AA v N l trung im cnh CC . Chng minh rng bn im B , M, D, N cng thuc mt mt phng. Tnh di cnh
AA theo a t gic B MDN l hnh vung.
Bi 13.203 (B04) : Cho hnh chp t gic u S .ABCD c cnh y bng a, gc gia cnh bn v mt y bng (0 < < 90 ).
Tnh tang ca gc gia hai mt phng (S AB) v (ABCD) theo . Tnh th tch khi chp S .ABCD theo a v .
Bi 13.204 (B04) : Trong khng gian vi h to Oxyz, cho im A(4; 2; 4) v ng thng d :

8
>
x = 3 + 2t
>
<

y=1t

>
>
:

z = 1 + 4t.

Vit phng trnh ng thng i qua im A, ct v vung gc vi ng thng d.

Bi 13.205 (B05) : Trong khng gian vi h to Oxyz cho lng tr ng ABC.A1 B1C1 vi A(0; 3; 0), B(4; 0; 0), C(0; 3; 0),
B1 (4; 0; 4).

1. Tm to cc nh A1 , C1 . Vit phng trnh mt cu c tm A v tip xc vi mt phng (BCC1 B1 ).


2. Gi M l trung im A1 B1 . Vit phng trnh mt phng (P) i qua hai im A, M v song song vi BC1 . Mt phng (P) ct
ng thng A1C1 ti im N. Tnh di MN.
Bi 13.206 (B06) : Trong khng gian vi h to Oxyz, cho im A(0; 1; 2) v hai ng thng
8
>
x =1+t
>
<

d1 :

x y1 z+1
=
=
v d2 : y = 1 2t
>
2
1
1
>
:
z = 2 + t.

1. Vit phng trnh mt phng (P) qua A, ng thi song song vi d1 v d2 .


2. Tm to im M d1 , N d2 sao cho ba im A, M, N thng hng.

Bi 13.207 (B06) : Cho hnh chp S .ABCD c y ABCD l hnh ch nht vi AB = a, AD = a 2, S A = a v S A vung gc vi mt
phng (ABCD). Gi M v N ln lt l trung im ca AD v S C; I l giao im ca BM v AC. Chng minh rng mt phng (S AC)
vung gc vi mt phng (S MB). Tnh th tch ca khi t din ANIB.
Bi 13.208 (B07) : Trong khng gian vi h to Oxyz, cho mt cu (S ) : x2 + y2 + z2 2x + 4y + 2z 3 = 0 v mt phng
(P) : 2x y + 2z 14 = 0.

1. Vit phng trnh mt phng (Q) cha trc Ox v ct (S ) theo mt ng trn c bn knh bng 3.
2. Tm to im M thuc mt cu (S ) sao cho khong cch t M n (P) l ln nht.
Bi 13.209 (B07) : Cho hnh chp t gic u S .ABCD c y l hnh vung cnh a. Gi E l im i xng ca D qua trung im
ca S A, M l trung im ca AE, N l trung im ca BC. Chng minh MN vung gc vi BD v tnh (theo a) khong cch gia hai
ng thng MN v AC.
Bi 13.210 (B08) : Trong khng gian vi h to Oxyz, cho ba im A(0; 1; 2), B(2; 2; 1), C(2; 0; 1).
1. Vit phng trnh mt phng i qua ba im A, B, C.
2. Tm to im M thuc mt phng 2x + 2y + z 3 = 0 sao cho MA = MB = MC.

Bi 13.211 (B08) : Cho hnh chp S .ABCD c y ABCD l hnh vung cnh 2a, S A = a, S B = a 3 v mt phng (S AB) vung
gc vi mt y. Gi M, N ln lt l trung im cc cnh AB, BC. Tnh theo a th tch khi chp S .BMDN v tnh cosin gc gia hai
ng thng S M, DN.
Bi 13.212 (B09) : Cho lng tr tam gic ABC.A BC c BB = a. gc gia ng thng BB v mt phng (ABC) bng 60 ; tam gic
= 60 . Hnh chiu vung gc ca im B ln mt phng (ABC) trng vi trng tm tam gic ABC. Tnh th
ABC vung ti C v BAC

tch khi chp A .ABC theo a.

TRN ANH TUN - 0974 396 391 - (04) 66 515 343

WWW.VNMATH.COM

Trang 276

www.VNMATH.com

www.luyenthi24h.com
www.luyenthi24h.com
CHUYN LUYN THI I HCwww.VNMATH.com

Bi 13.213 (B09) : Trong khng gian vi h to Oxyz, cho t din ABCD c cc nh A(1; 2; 1), B(2; 1; 3), C(2; 1; 1) v
D(0; 3; 1). Vit phng trnh mt phng (P) i qua A, B sao cho khong cch t C n (P) bng khong cch t D n (P).

Bi 13.214 (B09) : Trong khng gian vi h to Oxyz, cho mt phng (P) : x 2y + 2z 5 = 0 v hai im A(3; 0; 1), B(1; 1; 3).
Trong cc ng thng i qua A v song song vi (P), hy vit phng trnh ng thng m khong cch t B n ng thng l

nh nht.
Bi 13.215 (B10) : Cho hnh lng tr tam gic u ABC.A BC c AB = a, gc gia hai mt phng (A BC) v (ABC) bng 60 . Gi
G l trng tm tam gic A BC. Tnh th tch khi lng tr cho v tnh bn knh mt cu ngoi tip t din GABC theo a.
Bi 13.216 (B10) : Trong khng gian to Oxyz, cho cc im A(1; 0; 0), B(0; b; 0), C(0; 0; c), trong b, c dng v mt phng
(P) : y z + 1 = 0. Xc nh b v c, bit mt phng (ABC) vung gc vi mt phng (P) v khong cch t im O n mt phng
(ABC) bng 13.

Bi 13.217 (B10) : Trong khng gian to Oxyz, cho ng thng :


cho khong cch t M n bng OM.

x y1 z
=
= . Xc nh ta im M trn trc honh sao
2
1
2

Bi 13.218 (D02) : Cho hnh t din ABCD c cnh AD vung gc vi mt phng (ABC); AC = AD = 4cm ; AB = 3cm ; BC = 5cm.
Tnh khong cch t A ti mt phng (BCD).
Bi 13.219 (D02) : Trong khng gian vi h to Oxyz, cho mt phng
(P) : 2x y + 2 = 0v ng thng dm :

8
<(2m + 1)x + (1 m)y + m 1 = 0
:mx + (2m + 1)z + 4m + 2 = 0

(m l tham s ).

Xc nh m ng thng dm song song vi mt phng (P).


Bi 13.220 (D03) : Trong khng gian vi h to Oxyz, cho ng thng :

dk :

8
< x + 3ky z + 2 = 0
:kx y + z + 1 = 0.

Tm k ng thng dk vung gc vi mt phng (P) : x y 2z + 5 = 0.

Bi 13.221 (D03) : Cho hai mt phng (P) v (Q) vung gc vi nhau, c giao tuyn l ng thng . Trn ly hai im A, B vi
AB = a. Trong mt phng (P) ly im C, trong mt phng (Q) ly im D sao cho AC, BD vung vung gc vi v AC = BD = AB.
Tnh bn knh mt cu ngoi tip t din ABCD v tnh khong cch t A n mt phng (BCD).
Bi 13.222 (D04) : Trong khng gian vi h to Oxyz, cho hnh lng tr ng ABC.A1 B1C1 . Bit A(a; 0; 0), B(a; 0; 0), C(0; 1; 0),
B1 (a; 0; b), vi a > 0, b > 0.
1. Tnh khong cch gia hai ng thng B1C v AC1 theo a, b.
2. Cho a, b thay i, nhng lun tho mn : a + b = 4. Tm a, b khong cch gia hai ng thng B1C v AC1 ln nht.
Bi 13.223 (D04) : Trong khng gian vi h to Oxyz, cho ba im A(2; 0; 1), B(1; 0; 0), C(1; 1; 1) v mt phng (P) : x+y+z2 = 0.

Vit phng trnh mt cu i qua ba im A, B, C v c tm thuc mt phng (P).


Bi 13.224 (D05) : Trong khng gian vi h to Oxyz, cho hai ng thng :

d1 :

8
<x + y z 2 = 0

x1 y+2 z+1
=
=
v d2 :
: x + 3y 12 = 0.
3
1
2

1. Chng minh rng d1 v d2 song song vi nhau. Vit phng trnh mt phng (P) cha c hai ng thng d1 v d2 .
2. Mt phng to (Oxz) ct hai ng thng d1 , d2 ln lt ti cc im A, B. Tnh din tch tam gic OAB (O l gc to ).
Bi 13.225 (D06) : Trong khng gian vi h to Oxyz, im A(1; 2; 3) v cho hai ng thng :
d1 :

x1 y1 z+1
x2 y+2 z3
=
=
; d2 :
=
=
.
2
1
1
1
2
1

TRN ANH TUN - 0974 396 391 - (04) 66 515 343

WWW.VNMATH.COM

Trang 277

www.VNMATH.comCHUYN LUYN THI I HC

www.luyenthi24h.com
www.luyenthi24h.com
www.VNMATH.com

1. Tm to im A i xng vi im A qua ng thng d1 .


2. Vit phng trnh ng thng i qua im A, vung gc vi d1 v ct d2 .
Bi 13.226 (D06) : Cho hnh chp tam gic S .ABC c y ABC l tam gic u cnh a, S A = 2a v S A vung gc vi mt phng
(ABC). Gi M v N ln lt l hnh chiu vung gc ca A trn cc ng thng S B v S C. Tnh th tch ca khi chp A.BCN M.
Bi 13.227 (D07) : Trong khng gian vi h to Oxyz, cho hai im A(1; 4; 2), B(1; 2; 4) v ng thng
:

x1 y+2 z
=
= .
1
1
2

1. Vit phng trnh ng thng d i qua trng tm G ca tam gic OAB v vung gc vi mt phng (OAB).
2. Tm to im M thuc ng thng sao cho MA2 + MB2 nh nht.
= BAD
= 90 , BA = BC = a, AD = 2a. Cnh bn S A vung
Bi 13.228 (D07) : Cho hnh chp S .ABCD c y l hnh thang, ABC

gc vi y v S A = a 2. Gi H l hnh chiu ca A trn S B. Chng minh rng tam gic S CD vung v tnh theo a khong cch t

H n mt phng (S CD).
Bi 13.229 (D08) : Trong khng gian vi h to Oxyz, cho bn im A(3; 3; 0), B(3; 0; 3), C(0; 3; 3), D(3; 3; 3).
1. Vit phng trnh mt cu i qua bn im A, B, C, D.
2. Tm to tm ng trn ngoi tip tam gic ABC.

Bi 13.230 (D08) : Cho lng tr ng ABC.A BC c y ABC l tam gic vung, AB = BC = a, cnh bn AA = a 2. Gi M l
trung im cnh BC. Tnh theo a th tch khi lng tr ABC.A BC v khong cch gia hai ng thng AM, BC.
Bi 13.231 (D09) : Cho hnh lng tr ng ABC.A BC c y ABC l tam gic vung ti B, AB = a, AA = 2a, AC = 3a. Gi M l
trung im ca on thng AC , I l giao im ca AM v AC. Tnh theo a th tch khi t din IABC v khong cch t im A n
mt phng (IBC).
Bi 13.232 (D09) : Trong khng gian vi h to Oxyz, cho cc im A(2; 1; 0), B(1; 2; 2), C(1; 1; 0) v mt phng (P) : x+y+z20 =
0. Xc nh ta im D thuc ng thng AB sao cho ng thng CD song song vi mt phng (P).
z
x+2 y2
=
=
v mt phng (P) : x+2y3z+4 =
Bi 13.233 (D09) : Trong khng gian vi h to Oxyz, cho ng thng :
1
1
1
0. Vit phng trnh ng thng d nm trong (P) sao cho d ct v vung gc vi ng thng .
Bi 13.234 (D10) : Cho hnh chp S .ABCD c y ABCD l hnh vung cnh a, cnh bn S A = a ; hnh chiu vung gc ca nh
AC
S trn mt phng (ABCD) l im H thuc on AC, AH =
. Gi CM l ng cao ca tam gic S AC. Chng minh M l trung
4
im ca S A v tnh th tch khi t din S MBC theo a.
Bi 13.235 (D10) : Trong khng gian to Oxyz, cho hai mt phng (P) : x + y + z 3 = 0 v (Q) : x y + z 1 = 0. Vit phng
trnh mt phng (R) vung gc vi (P) v (Q) sao cho khong cch t O n (R) bng 2.

Bi 13.236 (D10) : Trong khng gian to Oxyz, cho hai ng thng 1 :

8
>
x =3+t
>
<

y=t

>
>
:

v 2 :

x2 y1
z
=
= . Xc nh ta
2
1
2

z=t

im M thuc 1 sao cho khong cch t M n 2 bng 1.

13.5 Bi tp tng hp
Bi 13.237 : Cho hnh chp S .ABC c y ABC l tam gic u cnha v cnh bn S A vung gc vi mt phng y (ABC). Tnh
a 6
khong cch t im A ti mt phng (S BC) theo a, bit rng S A =
.
2
Bi 13.238 : Cho t din OABC c ba cnh OA, OB, OC i mt vung gc v , , ln lt l cc gc gia mt phng (ABC) vi
cc mt phng (OBC), (OCA), (OAB). Chng minh rng :
cos + cos + cos

TRN ANH TUN - 0974 396 391 - (04) 66 515 343

3.

WWW.VNMATH.COM

Trang 278

www.luyenthi24h.com
www.luyenthi24h.com
CHUYN LUYN THI I HCwww.VNMATH.com

www.VNMATH.com

Bi 13.239 : Trong khng gian vi h to Oxyz, cho mt phng (P) : x y + z + 3 = 0 v hai im A(1; 3; 2), B(5; 7; 12).
1. Tm to im A l im i xng vi im A qua mt phng (P).
2. Gi s M l mt im chy trn mt phng (P), tm gi tr nh nht ca biu thc : MA + MB.
Bi 13.240 : Cho hnh chp S .ABCD c y ABCD l hnh vung cnh a, S A vung gc vi mt phng (ABCD) v S A = a. Gi E
trung im ca cnh CD. Tnh theo a khong cch t im S n BE.
Bi 13.241 : Trong khng gian vi h to Oxyz, cho ng thng :

8
<2x + y + z + 1 = 0
:x + y + z + 2 = 0

v mt phng (P) : 4x 2y + z 1 = 0.

Vit phng trnh hnh chiu vung gc ca ng thng trn mt phng (P).
Bi 13.242 : Cho tam gic vung cn ABC c cnh huyn BC = a. Trn ng thng vung gc vi mt phng (ABC) ti im A ly
im S sao cho gc gia hai mt phng (ABC) v (S BC) bng 60 . Tnh di on thng S A theo a.
Bi 13.243 : Trong khng gian vi h to Oxyz, cho hai ng thng :

d1 :

8
< x az a = 0
:y z + 1 = 0

v d2 :

8
<ax + 3y 3 = 0
: x 3z 6 = 0.

1. Tm a hai ng thng d1 v d2 ct nhau.


2. Vi a = 2, vit phng trnh mt phng (P) cha ng thng d2 v song song vi ng thng d1 . Tnh khong cch gia d1 v
d2 khi a = 2.
Bi 13.244 : Trong khng gian vi h to Oxyz, cho ng thng

d:

8
<2x 2y z + 1 = 0

: x + 2y 2z 4 = 0

v mt cu (S ) : x2 + y2 + z2 + 4x 6y + m = 0.

Tm m ng thng d ct mt cu (S ) ti hai im M v N sao cho khong cch gia hai im bng 9.


Bi 13.245 : Tnh th tch khi t din ABCD, bit AB = a; AC = b; AD = c v cc gc BAC, CAD, DAB u bng 60 .

Bi 13.246 : Cho hnh t din u c tt c cc cnh bng a = 6 2. Hy xc nh v tnh di on vung gc chung ca hai ng
thng AD v BC.
Bi 13.247 : Trong khng gian vi h to Oxyz, cho t din ABCD vi A(2; 3; 2), B(6; 1; 2), C(1; 4; 3), D(1; 6; 5). Tnh gc

gia hai ng thng AB v CD. Tm to im M thuc ng thng CD sao cho tam gic ABM co chu vi nh nht.

= 120 , cnh bn BB = a.
Bi 13.248 : Cho lng tr ng ABC.A BC c y ABC l tam gic cn, vi AB = AC = a v gc BAC

Gi I l trung im CC . Chng minh rng tam gic AB I vung A. Tnh cosin ca gc gia hai mt phng (ABC) v (AB I).
= 90 .
Bi 13.249 : Cho t din ABCD vi AB = AC = a, BC = b. Hai mt phng (BCD) v (ABC) vung gc vi nhau v gc BDC

Xc nh v tnh bn knh mt cu ngoi tip t din ABCD theo a v b.


Bi 13.250 : Trong khng gian vi h to Oxyz, cho hai ng thng

d1 :

8
<3x z + 1 = 0

x y+1 z
=
= v d2 :
:2x + y 1 = 0.
1
2
1

1. Chng minh rng d1 , d2 cho nhau v vung gc vi nhau.


x4 y7 z3
=
=
.
1
4
2
Bi 13.251 : Cho hnh lp phng ABCD.A BC D . Tm im M thuc cnh AA sao cho mt phng (BD M) ct hnh lp phng
2. Vit phng trnh ng thng d ct c hai ng thng d1 , d2 v song song vi ng thng :

theo mt thit din c din tch nh nht.

TRN ANH TUN - 0974 396 391 - (04) 66 515 343

WWW.VNMATH.COM

Trang 279

www.VNMATH.comCHUYN LUYN THI I HC

www.luyenthi24h.com
www.luyenthi24h.com
www.VNMATH.com

Bi 13.252 : Trong khng gian vi h to Oxyz, cho t din OABC vi A(0; 0; a 3), B(a; 0; 0), C(0; a 3; 0), vi (a > 0). Gi M
l trung im BC. Tnh khong cch gia hai ng thng AB v OM.
Bi 13.253 : Cho hnh chp u S .ABC, y c cnh bng a, mt bn to vi y mt gc bng , vi (0 < < 90 ). Tnh th tch
khi chp S .ABC v tnh khong cch t A n mt phng (S BC).
Bi 13.254 : Trong khng gian vi h to Oxyz, cho hai im I(0; 0; 1), K(3; 0; 0). Vit phng trnh mt phng i qua hai im
I, K v to vi mt phng (Oxy) mt gc bng 30 .
Bi 13.255 : Trong khng gian vi h to Oxyz, cho mt phng (P) : 2x + 2y + z m2 3m = 0 (m l tham s) v mt cu
(S ) : (x 1)2 + (y + 1)2 + (z 1)2 = 9.

Tm m mt phng (P) tip xc vi mt cu (S ). Vi m tm c, hy xc nh to tip im ca mt phng (P) v mt cu (S ).


Bi 13.256 : Cho hnh chp S .ABC c y ABC l tam gic vung ti B, AB = a, BC = 2a, cnh S A vung gc vi y v S A = 2a.
Gi M l trung im ca S C. Chng minh rng tam gic AMB cn ti M v tnh din tch tam gic AMB theo a.
Bi 13.257 : Trong khng gian vi h to Oxyz, cho hai im A(2; 1; 1), B(0; 1; 3) v ng thng d :

8
<3x 2y 11 = 0
:y + 3z 8.

1. Vit phng trnh mt phng (P) i qua trung im I ca on AB v vung gc vi AB. Gi K l giao im ca ng thng d
v mt phng (P), chng minh rng d vung gc vi IK.
2. Vit phng trnh hnh chiu vung gc ca d trn mt phng c phng trnh x + y z + 1 = 0.
Bi 13.258 : Cho t din ABCD c AD vung gc vi mt phng (ABC) v tam gic ABC vung ti A, AD = a, AC = b, AB = c.
Tnh din tch S ca tam gic BCD theo a, b, c v chng minh rng :
2S

abc(a + b + c).

Bi 13.259 : Trong khng gian vi h to Oxyz, cho ba im A(1; 1; 0), B(0; 2; 0), C(0; 0; 2).
1. Vit phng trnh mt phng (P) qua gc to O v vung gc vi BC. Tm to giao im ca AC vi mt phng (P).
2. Chng minh rng tam gic ABC l tam gic vung. Vit phng trnh mt cu ngoi tip t din OABC.
Bi 13.260 : Trong khng gian vi h to Oxyz, cho ba im A(2; 0; 0), C(0; 4; 0), S (0; 0; 4).
1. Tm to im B thuc mt phng (Oxy) sao cho t gic OABC l hnh ch nht. Vit phng trnh mt cu qua bn im
O, B, C, S .
2. Tm to im A1 i xng vi im A qua ng thng S C.
Bi 13.261 : Trong khng gian vi h to Oxyz, cho hai ng thng
8
>
x = 1 2t
>
<

d1 :

z
x y
= = v d2 : y = t
>
1 1 2
>
:
z=1+t

(t l tham s )

1. Xt v tr tng i ca d1 v d2 .
2. Tm to im M thuc d1 v im N thuc d2 sao cho ng thng MN song song vi mt phng (P) : x y + z = 0 v

di on MN = 2.
Bi 13.262 : Trong khng gian vi h ta Oxyz, cho im M(5; 2; 3) v mt phng (P) : 2x + 2y z + 1 = 0.
1. Gi M1 l hnh chiu ca M ln mt phng (P). Xc nh to im M1 v tnh di on MM1 .
x1 y1 z5
=
=
.
2
1
6
Bi 13.263 : Trong khng gian vi h to Oxyz, cho lng tr ng OAB.O1 A1 B1 vi A(2; 0; 0), B(0; 4; 0), O1 (0; 0; 4).
2. Vit phng trnh mt phng (Q) i qua M v cha ng thng :

1. Tm to cc im A1 , B1 . Vit phng trnh mt cu qua bn im O, A, B, O1.

TRN ANH TUN - 0974 396 391 - (04) 66 515 343

WWW.VNMATH.COM

Trang 280

www.VNMATH.com

www.luyenthi24h.com
www.luyenthi24h.com
CHUYN LUYN THI I HCwww.VNMATH.com

2. Gi M ls trung im ca AB. Mt phng (P) qua M vung gc vi O1 A v ct OA, OA1 ln lt ti N, K. Tnh di on NK.
Bi 13.264 : Trong khng gian vi h to Oxyz, cho hnh lp phng ABCD.A1 B1C1 D1 vi A(0; 0; 0), B(2; 0; 0), D1 (0; 2; 2).
1. Xc nh to cc im cn li ca hnh lp phng ABCD.A1 B1C1 D1 . Gi M l trung im BC. Chng minh rng hai mt
phng (AB1 D1 ) v (AMB1 ) vung gc nhau.
2. Chng minh rng t s khong cch t im N thuc ng thng AC1 (N . A) ti 2 mt phng (AB1 D1 ) v (AMB1 ) khng ph
thuc vo v tr ca im N.
Bi 13.265 : Trong khng gian vi h to Oxyz, cho hnh lng tr ng ABC.A BC c A(0; 0; 0), B(2; 0; 0), C(0; 2; 0), A (0; 0; 2).
1. Chng minh AC vung gc vi BC. Vit phng trnh mt phng (ABC ).
2. Vit phng trnh hnh chiu vung gc ca BC trn mt phng (ABC ).

a 3
= 60 . Gi M, N ln lt l trung
v gc BAD
2

im cc cnh A D , A B . Chng minh rng AC vung gc vi mt phng (BDMN). Tnh th tch khi chp A.BDMN.

Bi 13.266 : Cho hnh hp ABCD.A BC D c cc cnh AB = AD = a, AA =

Bi 13.267 : Trong khng gian vi h to Oxyz, cho mt phng () : 3x + 2y z + 4 = 0 v hai im A(4; 0; 0), B(0; 4; 0). Gi I l
trung im ca on thng AB.

1. Tm to giao im ca ng thng AB vi mt phng ().


2. Xc nh to im K sao cho KI vung gc vi mt phng () ng thi K cch u gc to v mt phng ().
Bi 13.268 : Cho hnh chp S .ABCD c y ABCD l hnh ch nht vi AB
= a, AD = 2a, cnh S A vung gc vi y, cnh S B to
a
3
vi mt phng y mt gc 60 . Trn cnh S A ly im M sao cho AM =
. Mt phng (BCM) ct S D ti N. Tnh th tch khi
2
chp S .BCN M.
Bi 13.269 : Trong khng gian vi h to Oxyz, cho mt phng (P) : 2x + y z + 5 = 0 v cc im A(0; 0; 4), B(2; 0; 0).
1. Vit phng trnh hnh chiu vung gc ca ng thng AB trn mt phng (P).
2. Vit phng trnh mt cu i qua O, A, B v tip xc vi mt phng (P).
Bi 13.270 : Cho hnh lng tr ABC.A BC c A ABC l hnh chp u, cnh y AB = a, cnh bn AA = b. Gi l gc gia hai
mt phng (ABC) v (A BC). Tnh tan v th tch khi a din A BBC C.
Bi 13.271 : Trong khng gian vi h to Oxyz, cho hai ng thng
8
>
x =1+t
>
<

1 :

y = 1 t

>
>
:

z=2

v 2 :

x3 y1 z
=
= .
1
2
1

1. Vit phng trnh mt phng cha ng thng 1 v song song vi ng thng 2 .


2. Xc nh im A trn 1 v im B trn 2 sao cho on AB c di nh nht.
= 60 , S A vung gc vi mt phng (ABCD), S A = a.
Bi 13.272 : Cho hnh chp S .ABCD c y ABCD l hnh thoi cnh a, BAD

Gi C l trung im ca cnh S C. Mt phng (P) i qua AC v song song vi BD, ct cc cnh S B, S D ca hnh chp ln lt ti
B , D . Tnh th tch khi chp S .ABC D .
Bi 13.273 : Trong khng gian vi h to Oxyz, cho mt phng (P) : 4x 3y + 11z 26 = 0 v hai ng thng
d1 :
1. Chng minh rng d1 v d2 cho nhau.

x
y3 z+1
x4 y z3
=
=
; d2 :
= =
.
1
2
3
1
1
2

2. Vit phng trnh ng thng nm trn (P), ng thi ct c d1 , d2 .


Bi 13.274 : Cho hnh chp t gic u S .ABCD c cnh y bng a. Gi S H l ng cao ca hnh chp. Khong cch t trung
im I ca S H n mt phng (S BC) bng b. Tnh th tch khi chp S .ABCD.

TRN ANH TUN - 0974 396 391 - (04) 66 515 343

WWW.VNMATH.COM

Trang 281

www.VNMATH.comCHUYN LUYN THI I HC

www.luyenthi24h.com
www.luyenthi24h.com
www.VNMATH.com

Bi 13.275 : Trong khng gian vi h to Oxyz, cho A(1; 2; 0), B(0; 4; 0), C(0; 0; 3).
1. Vit phng trnh ng thng qua O v vung gc vi mt phng (ABC).
2. Vit phng trnh mt phng (P) cha OA, sao cho khong cch t B n (P) bng khong cch t C n (P).
2a
. Mt phng () i
3
qua A, K v song song vi BD, chia khi lp phng thnh hai khi a din. Tnh th tch ca hai khi a din .
Bi 13.276 : Cho hnh lp phng ABCD.A BC D c cnh bng a v im K thuc cnh CC sao cho CK =

Bi 13.277 : Trong khng gian vi h to Oxyz, cho hai im A(1; 3; 2), B(3; 7; 18) v mt phng (P) : 2x y + z + 1 = 0.
1. Vit phng trnh mt phng cha AB v vung gc vi mt phng (P).
2. Tm to im M thuc (P) sao cho MA + MB nh nht.

= 120 . Gi M l trung im ca cnh


Bi 13.278 : Cho hnh lng tr ng ABC.A1 B1C1 c AB = a, AC = 2a, AA1 = 2a 5 v BAC
CC1 . Chng minh MBMA1 v tnh khong cch d t im A ti mt phng (A1 BM).
Bi 13.279 : Trong khng gian vi h to Oxyz, cho cc im A(2; 0; 0), B(0; 4; 0), C(2; 4; 6) v ng thng d :

8
<6x 3y + 2z = 0

:6x + 3y + 2z 24 = 0.

1. Chng minh cc ng thng AB v OC cho nhau.


2. Vit phng trnh ng thng song song vi d v ct cc ng thng AB, OC.
Bi 13.280 : Cho hnh chp S .ABC, gc gia hai mt phng (S BC) v (ABC) bng 60 ; cc tam gic ABC v S BC l cc tam gic
u cnh a. Tnh khong cch t B n mt phng (S AC).
Bi 13.281 : Trong khng gian vi h to Oxyz, cho cc im A(3; 5; 5), B(5; 3; 7) v mt phng (P) : x + y + z = 0.
1. Tm giao im I ca ng thng AB vi mt phng (P).
2. Tm im M (P) sao cho MA2 + MB2 nh nht.

Bi 13.282 : Cho hnh chp S .ABCD c y ABCD l hnh vung tm O, S A vung gc vi y. Cho AB = a, S A = a 2. Gi H v
K ln lt l hnh chiu ca A ln S B, S D. Chng minh S C(AHK) v tnh th tch khi t din OAHK.
Bi 13.283 : Trong khng gian vi h to Oxyz, cho cc im A(2; 0; 0), M(0; 3; 6).
1. Chng minh rng mt phng (P) : x + 2y 9 = 0 tip xc vi mt cu tm M, bn knh OM. Tm to tip im.
2. Vit phng trnh mt phng (Q) cha A, M v ct cc trc Oy, OZ ti cc im tng ng B, C sao cho VOABC = 3.
Bi 13.284 : Trong mt phng (P) cho na ng trn ng knh AB = 2R v im C thuc na ng trn sao cho AC = R. Trn
ng thng vung gc vi (P) ti A ly im S sao cho gc gia hai mt phng (S AB) v (S BC) bng 60 . Gi H, K l lt l hnh
chiu vung gc ca A trn S B, S C. Chng minh tam gic AHK vung v tnh VS .ABC .
x3 y+2 z+1
Bi 13.285 : Trong khng gian vi h to Oxyz, cho ng thng d :
=
=
v mt phng (P) : x + y + z + 2 = 0.
2
1
1
1. Tm giao im M ca d v (P).

2. Vit phng trnh ng thng nm trong (P) sao cho d v khong cch t M n bng 42.

Bi 13.286 : Cho lng tr ng ABC.A1 B1C1 c y ABC l tam gic vung AB = AC = a, AA1 = a 2. Gi M, N ln lt l trung
im ca on AA1 v BC1 . Chng minh MN l ng vung gc chung ca cc ng thng AA1 v BC1 . Tnh V M.A1 BC1 .
Bi 13.287 : Trong khng gian vi h to Oxyz, cho mt phng (P) : x 2y + 2z 1 = 0 v cc ng thng
d1 :

x1 y3 z
x5 y z+5
=
= v d2 :
= =
.
2
3
2
6
4
5

1. Vit phng trnh mt phng (Q) cha d1 v (Q)(P).

2. Tm cc im M d1 , N d2 sao cho MN (P) v cch (P) mt khong bng 2.


Bi 13.288 : Cho lng tr ng ABC.A1 B1C1 c tt c cc cnh u bng a. M l trung im ca on AA1 . Chng minh BMB1C
v tnh khong cch gia hai ng thng BM v B1C.

TRN ANH TUN - 0974 396 391 - (04) 66 515 343

WWW.VNMATH.COM

Trang 282

www.VNMATH.com

www.luyenthi24h.com
www.luyenthi24h.com
CHUYN LUYN THI I HCwww.VNMATH.com

Bi 13.289 : Cho khi chp u S .ABCD c AB = a, gc gia mt bn v mt y bng 60 . Tnh th tch khi chp S .ABCD theo a.
x2 y1 z
Bi 13.290 : Trong khng gian vi h ta Oxyz, cho im A(1; 2; 3) v ng thng d :
=
= .
1
2
1
1. Hy tm ta ca hnh chiu vung gc ca A trn d.
2. Vit phng trnh mt cu tm A, tip xc vi d.
Bi 13.291 : Trong khng gian vi h ta Oxyz, cho im A(1; 2; 3) v mt phng (P) : 2x + 2y z + 9 = 0.
1. Vit phng trnh ng thng d i qua A v vung gc vi (P).
2. Tm ta im A i xng vi im A qua mt phng (P).

Bi 13.292 : Cho hnh chp S .ABC c y ABC l tam gic vung ti A, AB = a, AC = a 3, mt bn S BC l tam gic u v vung
gc vi mt phng y. Tnh theo A th tch khi chp S .ABC.
Bi 13.293 :

1. Trong khng gian Oxyz tm phng trnh mt phng (P) qua hai im M(4; 9; 12), A(2; 0; 0) v ct trc Oy, Oz

ln lt ti B, C sao cho OB = 1 + OC (B, C khng trng vi gc O).

2. Tm phng trnh ca mt phng (Q) qua im M(4; 9; 12) v ct trc Ox, Oy, Oz ln lt ti A0 , B0, C0 sao cho
8
<OC0 = OA0 + OB0
: 4

OC0

1
1
+
.
OA0 OB0

Bi 13.294 : Tm phng trnh mt phng i qua im (1; 0; 2) v vung gc vi c hai mt phng


(P1 ) : 2x + y z 2 = 0 v (P2 ) : x y z 3 = 0.
Bi 13.295 : Tm phng trnh tng qut ca mt phng i qua hai im A(2; 1; 1), B(3; 2; 2) v vung gc vi mt phng x + 2y
5z 3 = 0.

v = (2; 3; 1) v vung gc vi mt phng


Bi 13.296 : Vit phng trnh ca mt phng i qua im A(1; 0; 2), song song vi vect
2x y 5z + 1 = 0.

Bi 13.297 : Vit phng trnh mt phng qua hai im (0; 2; 0), (2; 0; 0) v to vi mt phng (Oyz) mt gc 60 .
Bi 13.298 :

1. Vit phng trnh mt phng i qua im (3; 4; 1) v qua giao tuyn ca hai mt phng 19x 6y 4z + 27 = 0 v

42x 8y + 3z + 11 = 0.

2. Vit phng trnh mt phng i qua im (4; 3; 2) v vung gc vi giao tuyn ca hai mt phng
x y + 2z 3 = 0
2x y 3z = 0.
Bi 13.299 : Tm phng trnh chnh tc ca ng thng i qua im (1; 4; 2) v song song vi ng thng
8
<6x + 2y + 2z + 3 = 0

:3x 5y 2z 1 = 0.

Bi 13.300 : Tm phng trnh tham s ca ng thng nm trong mt phng x + 3y z + 4 = 0 v vung gc vi ng thng


8
< x 2z 3
:y 2z = 0

ti giao im ca ng thng v mt phng.

Bi 13.301 : Lp phng trnh ca ng thng i qua im (3; 2; 1) vung gc vi ng thng


.

x
y
z+3
= =
v ct ng thng
2 4
1

Bi 13.302 : Tm phng trnh tng qut ca ng thng i qua im (4; 5; 3) v ct c hai ng thng
x2 y+1 z1
=
=
.
2
3
5

TRN ANH TUN - 0974 396 391 - (04) 66 515 343

WWW.VNMATH.COM

x+1 y+3 z2
=
=
,
3
3
1

Trang 283

www.VNMATH.comCHUYN LUYN THI I HC

www.luyenthi24h.com
www.luyenthi24h.com
www.VNMATH.com

Bi 13.303 : Vit phng trnh tham s ca ng thng i qua im (0; 1; 1) vung gc vi ng thng
ng thng

8
<x + y z + 2 = 0

x1
y+2
z
=
= v ct
3
1
1

: x + 1 = 0.

Bi 13.304 : Tm phng trnh ca ng thng qua im (3; 1; 4) ct trc Oy v song song vi mt phng 2x + y = 0.
x+1 y1 z2
Bi 13.305 : Cho ng thng d :
=
=
v mt phng (P) : x y z 1 = 0.
2
1
3
1. Tm phng trnh chnh tc ca ng thng i qua im M(1; 1; 2), song song vi (P) v vung gc vi d.
2. Gi N l giao im ca d v (P). Tm im K trn d sao cho K M = KN.
Bi 13.306 : Cho ng thng d c phng trnh
1. Xc nh giao im A ca d v (P).

x3
y
z1
=
=
m mt phng (P) c phng trnh x + y + z = 0.
2
1
3

2. Vit phng trnh ca ng thng qua A vung gc vi d v nm trong (P).


Bi 13.307 : Trong khng gian vi h to cc vung gc Oxyz cho :
8
>
x = 1 + 2t
>
<

d:

>
>
:

y=2t

v (P) : 2x y 2z + 1 = 0.

z = 3t;

1. Tm to cc im thuc ng thng d sao cho khong cch t mi im n mt phng (P) bng 1.


2. Gi K l im i xng ca im I(2; 1; 3) qua ng thng d. Hy xc nh to im K.

x1 y2 z1
=
=
. Vit phng trnh hnh chiu vung
1
2
3

Bi 13.308 : Trong khng gian Oxyz, cho (P) : x + y + z + 1 = 0 v d :


gc ca d trn mt phng (P).
Bi 13.309 : Cho hai ng thng :
8
<

d:

2x 3y 2 = 0

8
<

2x 3y + 9 = 0

x + 3z + 2 = 0,

y + 2z + 1 = 0.

1. Chng minh d//. Vit phng trnh mt phng (P) cha v d.


2. Tm to im N i xng vi im M(2; 3; 4) qua d.
Bi 13.310 : Cho A(0; 1; 1) v hai ng thng :

d1 :

x1 y+2 z
=
= ,
3
1
1

d2 :

8
<x + y z + 2 = 0
: x + 1 = 0.

Lp phng trnh ng thng qua A, vung gc vi d1 v ct d2 .


Bi 13.311 : Trong khng gian vi h to Oxyz cho hai ng thng :
8
>
x = 2 + 2t
>
<

1 :

>
>
:

y = 1 + t
z = 1,

v 2 :

8
>
x=1
>
<

y = 1 + t

>
>
:

z = 3 t .

1. Chng t 1 v 2 cho nhau. Vit phng trnh mt phng () cha 1 v song song vi 2 .
2. Tnh khong cch gia 1 v 2 .
Bi 13.312 : Trong khng gian vi h to Oxyz cho im A(2; 1; 1) v ng thng :

TRN ANH TUN - 0974 396 391 - (04) 66 515 343

8
<y + z 4 = 0

:2x y z + 2 = 0.

WWW.VNMATH.COM

Trang 284

www.VNMATH.com

www.luyenthi24h.com
www.luyenthi24h.com
CHUYN LUYN THI I HCwww.VNMATH.com

1. Vit phng trnh mt phng () i qua A v vung gc vi .


2. Xc nh to im B i xng vi A qua .
Bi 13.313 : Trong khng gian vi h to trc chun Oxyz, cho t din S ABC vi cc nh
S (2; 2; 4), A(2; 2; 0), B(5; 2; 0), C(2; 1; 1).
Tnh khong cch gia 2 cnh i S A v BC.
Bi 13.314 : Trong khng gian vi h to Cc vung gc Oxyz cho hai ng thng song song :
d1 :

x y + 4 z + 18
x+7 y5 z9
=
=
v d1 : =
=
.
3
1
4
3
1
4

1. Vit phng trnh mt phng cha d1 v d2 .

2. Tnh khong cch gia d1 v d2 .

Bi 13.315 : Trong khng gian vi h to trc chun Oxyz xt ng thng c phng trnh :
:

x y4 z+1
=
=
4
3
2

v mt phng c phng trnh (P) : x y + 3z + 8 = 0. Vit phng trnh chnh tc ca hnh chiu vung gc ca ng thng trn
mt phng (P).

Bi 13.316 : Trong khng gian Oxyz cho ba im A(a; 0; 0), B(0; b; 0), C(0; 0; c) vi a, b, c > 0.
1. Chng t rng tam gic ABC khng th l tam gic vung.
2. Tnh th tch hnh chp OABC v din tch tam gic ABC theo a, b, c.
Bi 13.317 : Cho hnh chp S .ABC c y ABC l tam gic cn ti nh A. Cc cnh bn ca hnh chp to vi y cc gc bng .
1. Chng minh hnh chp c cc cnh bn bng nhau.
2. Gi I l trung im ca BC. Chng minh rng mt phng (S AI) vung gc vi mt phng ABC.
3. Gi K l hnh chiu vung gc ca A ln S I. Chng minh rng AK vung gc vi mt phng (S BC).
= , khong cch t S n mt phng ABC l d. Tnh din tch ca tam gic ABC theo d, , .
4. Cho bit gc BAC

Bi 13.318 : Cho gc tam din ba mt vung Oxyz. Trn Ox, Oy, Oz ln lt ly cc im A, B, C.


1. Tnh din tch tam gic ABC v khong cch t O n mt phng (ABC) theo OA = a, OB = b, OC = c.
2. Gi s A, B, C thay i nhng lun c : OA + OB + OC + AB + BC + CA = k khng i. Hy xc nh gi tr ln nht ca th
tch t din OABC.
3. Gi s im A c nh cn B v C thay i sao cho OB + OC = OA. Hy xc nh v tr ca B v C sao cho th tch t din
OABC l ln nht. Chng minh rng khi bn knh mt cu ngoi tip t din OABC li l nh nht.
Bi 13.319 : Trong khng gian vi h to Oxyz, cho hai ng thng
d1 :

8
<5x 6y 6z + 13 = 0

x3 y3 z3
=
=
v d2 :
: x 6y + 6z 7 = 0.
2
2
1

1. Chng minh rng d1 v d2 cho nhau.


2. Gi I l
giao im ca d1 v d2 . Tm ta cc im A, B ln lt thuc d1 , d2 sao cho tam gic IAB cn ti I v c din tch
41
.
bng
42
Bi 13.320 : Trong khng gian vi h to Oxyz, cho mt phng
(P) : 2x + 3y 3z + 1 = 0 v ng thng d :

x3 y z+5
= =
2
9
1

v ba im A(4; 0; 3), B(1; 1; 3), C(3; 2; 6).

TRN ANH TUN - 0974 396 391 - (04) 66 515 343

WWW.VNMATH.COM

Trang 285

www.VNMATH.comCHUYN LUYN THI I HC

www.luyenthi24h.com
www.luyenthi24h.com
www.VNMATH.com

1. Vit phng trnh mt cu (S ) i qua ba im A, B, C v c tm thuc mt phng (P).


2. Vit phng trnh mt phng (Q) cha ng thng d v ct mt cu (S ) theo mt ng trn c bn knh ln nht.
Bi 13.321 : Trong khng gian vi h to Oxyz, cho cc im A(1; 1; 1), B(2a; 0; 0), C(a; b; 0), D(a; 0; c), E(0; 0; 2), F(a; b; c),
G(5; 1; 1) v H(18; 0; 0), trong a, b, c l cc s thc dng. Bit bn im A, B, C, D nm trn mt phng (P) v bn im E, F, G, H
nm trn mt phng (Q). Xc nh a, b, c v vit phng trnh mt phng (P).
Bi 13.322 : Trong khng gian vi h to Oxyz, cho cc im A(1; 4; 2), B(1; 2; 4) v ng thng d :
1. Tm im M thuc ng thng d sao cho

x1 y+2 z
=
= .
1
1
2


(a) | MA + MB| nh nht;

(c) MA + MB nh nht ;

(b) MA2 + MB2 nh nht ;

(d) Din tch tam gic AMB nh nht.

2. Vit phng trnh mt phng (P) cha d sao cho khong cch t A n (P) ln nht.
3. Vit phng trnh mt phng (Q) cha d v to vi mt phng (Oxy) mt gc nh nht.
4. Vit phng trnh mt phng (R) cha d v to vi trc Oy mt gc ln nht.
5. Vit phng trnh ng thng i qua A v ct d sao cho khong cch t B n l
(a) ln nht ;

(b) nh nht.

Bi 13.323 : Cho mt phng () : x y + 2z = 0 v cc im A(1; 2; 1), B(3; 1; 2), C(1; 1; 1). Tm im M thuc () sao cho :
1. MA + MB nh nht ;
2. |MA MB| nh nht ;

TRN ANH TUN - 0974 396 391 - (04) 66 515 343

3. MA2 MB2 MC 2 ln nht ;



4. MA + MB + MC nh nht.

WWW.VNMATH.COM

Trang 286

You might also like